You are on page 1of 190

G.R. No.

79974 December 17, 1987 Section 16, Article VII of the 1987 Constitution says:

ULPIANO P. SARMIENTO III AND JUANITO G. ARCILLA, petitioners, The President shall nominate and, with the consent of the Commission on
vs. Appointments, appoint the heads of the executive departments, ambassadors,
SALVADOR MISON, in his capacity as COMMISSIONER OF THE BUREAU OF CUSTOMS, AND other public ministers and consuls, or officers of the armed forces from the
GUILLERMO CARAGUE, in his capacity as SECRETARY OF THE DEPARTMENT OF BUDGET, rank of colonel or naval captain, and other officers whose appointments are
respondents, COMMISSION ON APPOINTMENTS, intervenor. vested in him in this Constitution. He shall also appoint all other officers of the
Government whose appointments are not otherwise provided for by law, and
PADILLA, J.: those whom he may be authorized by law to appoint. The Congress may, by
law, vest the appointment of other officers lower in rank in the President alone,
in the courts, or in the heads of the departments, agencies, commissions or
Once more the Court is called upon to delineate constitutional boundaries. In this petition for boards.
prohibition, the petitioners, who are taxpayers, lawyers, members of the Integrated Bar of the
Philippines and professors of Constitutional Law, seek to enjoin the respondent Salvador Mison from
performing the functions of the Office of Commissioner of the Bureau of Customs and the The President shall have the power to make appointments during the recess of
respondent Guillermo Carague, as Secretary of the Department of Budget, from effecting the Congress, whether voluntary or compulsory, but such appointments shall
disbursements in payment of Mison's salaries and emoluments, on the ground that Mison's be effective only until disapproval by the Commission on Appointments or until
appointment as Commissioner of the Bureau of Customs is unconstitutional by reason of its not the next adjournment of the Congress.
having been confirmed by the Commission on Appointments. The respondents, on the other hand,
maintain the constitutionality of respondent Mison's appointment without the confirmation of the It is readily apparent that under the provisions of the 1987 Constitution, just quoted, there are four
Commission on Appointments. (4) groups of officers whom the President shall appoint. These four (4) groups, to which we will
hereafter refer from time to time, are:
Because of the demands of public interest, including the need for stability in the public service, the
Court resolved to give due course to the petition and decide, setting aside the finer procedural First, the heads of the executive departments, ambassadors, other public
questions of whether prohibition is the proper remedy to test respondent Mison's right to the Office ministers and consuls, officers of the armed forces from the rank of colonel or
of Commissioner of the Bureau of Customs and of whether the petitioners have a standing to bring naval captain, and other officers whose appointments are vested in him in this
this suit. Constitution; 2

By the same token, and for the same purpose, the Court allowed the Commission on Appointments Second, all other officers of the Government whose appointments are not
to intervene and file a petition in intervention. Comment was required of respondents on said otherwise provided for by law; 3
petition. The comment was filed, followed by intervenor's reply thereto. The parties were also heard
in oral argument on 8 December 1987. Third, those whom the President may be authorized by law to appoint;

This case assumes added significance because, at bottom line, it involves a conflict between two (2) Fourth, officers lower in rank 4 whose appointments the Congress may by law
great departments of government, the Executive and Legislative Departments. It also occurs early in vest in the President alone.
the life of the 1987 Constitution.

The first group of officers is clearly appointed with the consent of the Commission on Appointments.
The task of the Court is rendered lighter by the existence of relatively clear provisions in the Appointments of such officers are initiated by nomination and, if the nomination is confirmed by the
Constitution. In cases like this, we follow what the Court, speaking through Mr. Justice (later, Chief Commission on Appointments, the President appoints. 5
Justice) Jose Abad Santos stated in Gold Creek Mining Corp. vs. Rodriguez, 1 that:

The second, third and fourth groups of officers are the present bone of contention. Should they be
The fundamental principle of constitutional construction is to give effect to the appointed by the President with or without the consent (confirmation) of the Commission on
intent of the framers of the organic law and of the people adopting it. The Appointments? By following the accepted rule in constitutional and statutory construction that an
intention to which force is to be given is that which is embodied and expressed express enumeration of subjects excludes others not enumerated, it would follow that only those
in the constitutional provisions themselves. appointments to positions expressly stated in the first group require the consent (confirmation) of
the Commission on Appointments. But we need not rely solely on this basic rule of constitutional
The Court will thus construe the applicable constitutional provisions, not in accordance with how construction. We can refer to historical background as well as to the records of the 1986
the executive or the legislative department may want them construed, but in accordance with what Constitutional Commission to determine, with more accuracy, if not precision, the intention of the
they say and provide. framers of the 1987 Constitution and the people adopting it, on whether the appointments by the
President, under the second, third and fourth groups, require the consent (confirmation) of the the power of confirmation by the Commission on Appointments, under the 1935 Constitution,
Commission on Appointments. Again, in this task, the following advice of Mr. Chief Justice J. Abad transformed that commission, many times, into a venue of "horse-trading" and similar malpractices.
Santos in Gold Creek is apropos:
On the other hand, the 1973 Constitution, consistent with the authoritarian pattern in which it was
In deciding this point, it should be borne in mind that a constitutional provision molded and remolded by successive amendments, placed the absolute power of appointment in the
must be presumed to have been framed and adopted in the light and President with hardly any check on the part of the legislature.
understanding of prior and existing laws and with reference to them. "Courts
are bound to presume that the people adopting a constitution are familiar with Given the above two (2) extremes, one, in the 1935 Constitution and the other, in the 1973
the previous and existing laws upon the subjects to which its provisions relate, Constitution, it is not difficult for the Court to state that the framers of the 1987 Constitution and
and upon which they express their judgment and opinion in its adoption." the people adopting it, struck a "middle ground" by requiring the consent (confirmation) of the
(Barry vs. Truax 13 N.D., 131; 99 N.W., 769,65 L. R. A., 762.) 6 Commission on Appointments for the first group of appointments and leaving to the President,
without such confirmation, the appointment of other officers, i.e., those in the second and third
It will be recalled that, under Sec. 10, Article VII of the 1935 Constitution, it is provided that groups as well as those in the fourth group, i.e., officers of lower rank.

xxx xxx xxx The proceedings in the 1986 Constitutional Commission support this conclusion. The original text of
Section 16, Article VII, as proposed by the Committee on the Executive of the 1986 Constitutional
(3) The President shall nominate and with the consent of the Commission on Commission, read as follows:
Appointments, shall appoint the heads of the executive departments and
bureaus, officers of the army from the rank of colonel, of the Navy and Air Section 16. The president shall nominate and, with the consent of a
Forces from the rank of captain or commander, and all other officers of the Commission on Appointment, shall appoint the heads of the executive
Government whose appointments are not herein otherwise provided for, and departments and bureaus, ambassadors, other public ministers and consuls, or
those whom he may be authorized by law to appoint; but the Congress may by officers of the armed forces from the rank of colonel or naval captain and all
law vest the appointment of inferior officers, in the President alone, in the other officers of the Government whose appointments are not otherwise
courts, or in the heads of departments. provided for by law, and those whom he may be authorized by law to appoint.
The Congress may by law vest the appointment of inferior officers in the
(4) The President shall havethe power to make appointments during the recess President alone, in the courts, or in the heads of departments 7 [Emphasis
of the Congress, but such appointments shall be effective only until disapproval supplied].
by the Commission on Appointments or until the next adjournment of the
Congress. The above text is almost a verbatim copy of its counterpart provision in the 1935 Constitution. When
the frames discussed on the floor of the Commission the proposed text of Section 16, Article VII, a
xxx xxx xxx feeling was manifestly expressed to make the power of the Commission on Appointments over
presidential appointments more limited than that held by the Commission in the 1935 Constitution.
Thus-
(7) ..., and with the consent of the Commission on Appointments, shall appoint
ambassadors, other public ministers and consuls ...
Mr. Rama: ... May I ask that Commissioner Monsod be
recognized
Upon the other hand, the 1973 Constitution provides that-

The President: We will call Commissioner Davide later.


Section 10. The President shall appoint the heads of bureaus and offices, the
officers of the Armed Forces of the Philippines from the rank of Brigadier
General or Commodore, and all other officers of The government whose Mr. Monsod: With the Chair's indulgence, I just want to
appointments are not herein otherwise provided for, and those whom he may take a few minutes of our time to lay the basis for some
be authorized by law to appoint. However, the Batasang Pambansa may by law of the amendments that I would like to propose to the
vest in the Prime Minister, members of the Cabinet, the Executive Committee, Committee this morning.
Courts, Heads of Agencies, Commissions, and Boards the power to appoint
inferior officers in their respective offices. xxx xxx xxx

Thus, in the 1935 Constitution, almost all presidential appointments required the consent
(confirmation) of the Commission on Appointments. It is now a sad part of our political history that
On Section 16, I would like to suggest that the power of the Commission on MR. FOZ: Yes, but the regional directors are under the
Appointments be limited to the department heads, ambassadors, generals and supervisiopn of the staff bureau directors.
so on but not to the levels of bureau heads and colonels.
xxx xxx xxx
xxx xxx xxx 8 (Emphasis supplied.)
MR. MAAMBONG: May I direct a question to
In the course of the debates on the text of Section 16, there were two (2) major changes proposed Commissioner Foz? The Commissioner proposed an
and approved by the Commission. These were (1) the exclusion of the appointments of heads of amendment to delete 'and bureaus on Section 16. Who
bureaus from the requirement of confirmation by the Commission on Appointments; and (2) the will then appoint the bureau directors if it is not the
exclusion of appointments made under the second sentence 9 of the section from the same President?
requirement. The records of the deliberations of the Constitutional Commission show the following:
MR. FOZ: It is still the President who will appoint them but
MR. ROMULO: I ask that Commissioner Foz be recognized their appointment shall no longer be subject to
confirmation by the Commission on Appointments.
THE PRESIDENT: Commissioner Foz is recognized
MR. MAAMBONG: In other words, it is in line with the
MR. FOZ: Madam President, my proposed amendment is same answer of Commissioner de Castro?
on page 7, Section 16, line 26 which is to delete the words
"and bureaus," and on line 28 of the same page, to change MR. FOZ: Yes.
the phrase 'colonel or naval captain to MAJOR GENERAL
OR REAR ADMIRAL. This last amendment which is co- MR. MAAMBONG: Thank you.
authored by Commissioner de Castro is to put a period (.)
after the word ADMIRAL, and on line 29 of the same page,
start a new sentence with: HE SHALL ALSO APPOINT, et THE PRESIDENT: Is this clear now? What is the reaction of
cetera. the Committee?

MR. REGALADO: May we have the amendments one by xxx xxx xxx
one. The first proposed amendment is to delete the words
"and bureaus" on line 26. MR. REGALADO: Madam President, the Committee feels
that this matter should be submitted to the body for a
MR. FOZ: That is correct. vote.

MR. REGALADO: For the benefit of the other MR. DE CASTRO: Thank you.
Commissioners, what would be the justification of the
proponent for such a deletion? MR. REGALADO: We will take the amendments one by
one. We will first vote on the deletion of the phrase 'and
MR. FOZ: The position of bureau director is actually quite bureaus on line 26, such that appointments of bureau
low in the executive department, and to require further directors no longer need confirmation by the Commission
confirmation of presidential appointment of heads of on Appointment.
bureaus would subject them to political influence.
Section 16, therefore, would read: 'The President shall nominate, and with the
MR. REGALADO: The Commissioner's proposed consent of a Commission on Appointments, shall appoint the heads of the
amendment by deletion also includes regional directors as executive departments, ambassadors. . . .
distinguished from merely staff directors, because the
regional directors have quite a plenitude of powers within THE PRESIDENT: Is there any objection to delete the
the regions as distinguished from staff directors who only phrase 'and bureaus' on page 7, line 26? (Silence) The
stay in the office. Chair hears none; the amendments is approved.
xxx xxx xxx FR. BERNAS: Will Commissioner Davide restate his
proposed amendment?
MR. ROMULO: Madam President.
MR. DAVIDE: After 'captain,' add the following: AND
THE PRESIDENT: The Acting Floor Leader is recognized. OTHER OFFICERS WHOSE APPOINTMENTS ARE VESTED IN
HIM IN THIS CONSTITUTION.

THE PRESIDENT: Commissioner Foz is recognized


FR. BERNAS: How about:"AND OTHER OFFICERS WHOSE
APPOINTMENTS REQUIRE CONFIRMATION UNDER THIS
MR. FOZ: Madam President, this is the third proposed CONSTITUTION"?
amendment on page 7, line 28. 1 propose to put a period
(.) after 'captain' and on line 29, delete 'and all' and
substitute it with HE SHALL ALSO APPOINT ANY. MR. DAVIDE: Yes, Madam President, that is modified by
the Committee.

MR. REGALADO: Madam President, the Committee


accepts the proposed amendment because it makes it FR. BERNAS: That will clarify things.
clear that those other officers mentioned therein do not
have to be confirmed by the Commission on THE PRESIDENT: Does the Committee accept?
Appointments.
MR. REGALADO: Just for the record, of course, that
MR. DAVIDE: Madam President. excludes those officers which the Constitution does not
require confirmation by the Commission on
THE PRESIDENT: Commissioner Davide is recognized. Appointments, like the members of the judiciary and the
Ombudsman.

xxx xxx xxx


MR. DAVIDE: That is correct. That is very clear from the
modification made by Commissioner Bernas.
MR. DAVIDE: So would the proponent accept an
amendment to his amendment, so that after "captain" we
insert the following words: AND OTHER OFFICERS WHOSE THE PRESIDENT: So we have now this proposed
APPOINTMENTS ARE VESTED IN HIM IN THIS amendment of Commissioners Foz and Davide.
CONSTITUTION?
xxx xxx xxx
FR. BERNAS: It is a little vague.
THE PRESIDENT: Is there any objection to this proposed
MR. DAVIDE: In other words, there are positions provided amendment of Commissioners Foz and Davide as
for in the Constitution whose appointments are vested in accepted by the Committee? (Silence) The Chair hears
the President, as a matter of fact like those of the different none; the amendment, as amended, is
constitutional commissions. approved 10 (Emphasis supplied).

FR. BERNAS: That is correct. This list of officials found in It is, therefore, clear that appointments to the second and third groups of
Section 16 is not an exclusive list of those appointments officers can be made by the President without the consent (confirmation) of
which constitutionally require confirmation of the the Commission on Appointments.
Commission on Appointments,
It is contended by amicus curiae, Senator Neptali Gonzales, that the second
MR. DAVIDE: That is the reason I seek the incorporation of sentence of Sec. 16, Article VII reading-
the words I proposed.
He (the President) shall also appoint all other officers of the Government As to the fourth group of officers whom the President can appoint, the intervenor Commission on
whose appointments are not otherwise provided for by law and those whom Appointments underscores the third sentence in Sec. 16, Article VII of the 1987 Constitution, which
he may be authorized by law to appoint . . . . (Emphasis supplied) reads:

with particular reference to the word "also," implies that the President shall "in like manner" appoint The Congress may, by law, vest the appointment of other officers lower in rank
the officers mentioned in said second sentence. In other words, the President shall appoint the in the President alone, in the courts, or in the heads of departments, agencies,
officers mentioned in said second sentence in the same manner as he appoints officers mentioned commissions, or boards. [Emphasis supplied].
in the first sentence, that is, by nomination and with the consent (confirmation) of the Commission
on Appointments. and argues that, since a law is needed to vest the appointment of lower-ranked officers in the
President alone, this implies that, in the absence of such a law, lower-ranked officers have to be
Amicus curiae's reliance on the word "also" in said second sentence is not necessarily supportive of appointed by the President subject to confirmation by the Commission on Appointments; and, if this
the conclusion he arrives at. For, as the Solicitor General argues, the word "also" could mean "in is so, as to lower-ranked officers, it follows that higher-ranked officers should be appointed by the
addition; as well; besides, too" (Webster's International Dictionary, p. 62, 1981 edition) which President, subject also to confirmation by the Commission on Appointments.
meanings could, on the contrary, stress that the word "also" in said second sentence means that the
President, in addition to nominating and, with the consent of the Commission on Appointments, The respondents, on the other hand, submit that the third sentence of Sec. 16, Article VII,
appointing the officers enumerated in the first sentence, can appoint (without such consent abovequoted, merely declares that, as to lower-ranked officers, the Congress may by law vest their
(confirmation) the officers mentioned in the second sentence- appointment in the President, in the courts, or in the heads of the various departments, agencies,
commissions, or boards in the government. No reason however is submitted for the use of the word
Rather than limit the area of consideration to the possible meanings of the word "also" as used in "alone" in said third sentence.
the context of said second sentence, the Court has chosen to derive significance from the fact that
the first sentence speaks of nomination by the President and appointment by the President with the The Court is not impressed by both arguments. It is of the considered opinion, after a careful study
consent of the Commission on Appointments, whereas, the second sentence speaks only of of the deliberations of the 1986 Constitutional Commission, that the use of the word alone" after
appointment by the President. And, this use of different language in two (2) sentences proximate to the word "President" in said third sentence of Sec. 16, Article VII is, more than anything else, a slip
each other underscores a difference in message conveyed and perceptions established, in line with or lapsus in draftmanship. It will be recalled that, in the 1935 Constitution, the following provision
Judge Learned Hand's observation that "words are not pebbles in alien juxtaposition" but, more so, appears at the end of par. 3, section 1 0, Article VII thereof
because the recorded proceedings of the 1986 Constitutional Commission clearly and expressly
justify such differences.
...; but the Congress may by law vest the appointment of inferior officers, in the
President alone, in the courts, or in the heads of departments. [Emphasis
As a result of the innovations introduced in Sec. 16, Article VII of the 1987 Constitution, there are supplied].
officers whose appointments require no confirmation of the Commission on Appointments, even if
such officers may be higher in rank, compared to some officers whose appointments have to be
confirmed by the Commission on Appointments under the first sentence of the same Sec. 16, Art. The above provision in the 1935 Constitution appears immediately after the provision which makes
VII. Thus, to illustrate, the appointment of the Central Bank Governor requires no confirmation by practically all presidential appointments subject to confirmation by the Commission on
the Commission on Appointments, even if he is higher in rank than a colonel in the Armed Forces of Appointments, thus-
the Philippines or a consul in the Consular Service.
3. The President shall nominate and with the consent of the Commission on
But these contrasts, while initially impressive, merely underscore the purposive intention and Appointments, shall appoint the heads of the executive departments and
deliberate judgment of the framers of the 1987 Constitution that, except as to those officers whose bureaus, officers of the Army from the rank of colonel, of the Navy and Air
appointments require the consent of the Commission on Appointments by express mandate of the Forces from the rank of captain or commander, and all other officers of the
first sentence in Sec. 16, Art. VII, appointments of other officers are left to the President without Government whose appointments are not herein provided for, and those
need of confirmation by the Commission on Appointments. This conclusion is inevitable, if we are whom he may be authorized by law to appoint; ...
to presume, as we must, that the framers of the 1987 Constitution were knowledgeable of what
they were doing and of the foreseable effects thereof. In other words, since the 1935 Constitution subjects, as a general rule, presidential appointments to
confirmation by the Commission on Appointments, the same 1935 Constitution saw fit, by way of
Besides, the power to appoint is fundamentally executive or presidential in character. Limitations an exception to such rule, to provide that Congress may, however, by law vest the appointment of
on or qualifications of such power should be strictly construed against them. Such limitations or inferior officers (equivalent to 11 officers lower in rank" referred to in the 1987 Constitution) in the
qualifications must be clearly stated in order to be recognized. But, it is only in the first sentence of President alone, in the courts, or in the heads of departments,
Sec. 16, Art. VII where it is clearly stated that appointments by the President to the positions therein
enumerated require the consent of the Commission on Appointments.
In the 1987 Constitution, however, as already pointed out, the clear and expressed intent of its the Deputy Commissioner of Customs shall be appointed by the President of the
framers was to exclude presidential appointments from confirmation by the Commission on Philippines (Emphasis supplied.)
Appointments, except appointments to offices expressly mentioned in the first sentence of Sec. 16,
Article VII. Consequently, there was no reason to use in the third sentence of Sec. 16, Article VII the Of course, these laws (Rep. Act No. 1937 and PD No. 34) were approved during the effectivity of the
word "alone" after the word "President" in providing that Congress may by law vest the appointment 1935 Constitution, under which the President may nominate and, with the consent of the
of lower-ranked officers in the President alone, or in the courts, or in the heads of departments, Commission on Appointments, appoint the heads of bureaus, like the Commissioner of the Bureau
because the power to appoint officers whom he (the President) may be authorized by law to appoint of Customs.
is already vested in the President, without need of confirmation by the Commission on
Appointments, in the second sentence of the same Sec. 16, Article VII.
After the effectivity of the 1987 Constitution, however, Rep. Act No. 1937 and PD No. 34 have to be
read in harmony with Sec. 16, Art. VII, with the result that, while the appointment of the
Therefore, the third sentence of Sec. 16, Article VII could have stated merely that, in the case of Commissioner of the Bureau of Customs is one that devolves on the President, as an appointment
lower-ranked officers, the Congress may by law vest their appointment in the President, in the he is authorizedby law to make, such appointment, however, no longer needs the confirmation of
courts, or in the heads of various departments of the government. In short, the word "alone" in the the Commission on Appointments.
third sentence of Sec. 16, Article VII of the 1987 Constitution, as a literal import from the last part of
par. 3, section 10, Article VII of the 1935 Constitution, appears to be redundant in the light of the
second sentence of Sec. 16, Article VII. And, this redundancy cannot prevail over the clear and Consequently, we rule that the President of the Philippines acted within her constitutional authority
positive intent of the framers of the 1987 Constitution that presidential appointments, except those and power in appointing respondent Salvador Mison, Commissioner of the Bureau of Customs,
mentioned in the first sentence of Sec. 16, Article VII, are not subject to confirmation by the without submitting his nomination to the Commission on Appointments for confirmation. He is thus
Commission on Appointments. entitled to exercise the full authority and functions of the office and to receive all the salaries and
emoluments pertaining thereto.

Coming now to the immediate question before the Court, it is evident that the position of
Commissioner of the Bureau of Customs (a bureau head) is not one of those within the first group WHEREFORE, the petition and petition in intervention should be, as they are, hereby DISMISSED.
of appointments where the consent of the Commission on Appointments is required. As a matter of Without costs.
fact, as already pointed out, while the 1935 Constitution includes "heads of bureaus" among those
officers whose appointments need the consent of the Commission on Appointments, the 1987 SO ORDERED.
Constitution on the other hand, deliberately excluded the position of "heads of bureaus" from
appointments that need the consent (confirmation) of the Commission on Appointments. Yap, Fernan, Narvasa, Paras, Feliciano, Gancayco, Bidin and Cortes, JJ., concur.

Moreover, the President is expressly authorized by law to appoint the Commissioner of the Bureau
of Customs. The original text of Sec. 601 of Republic Act No. 1937, otherwise known as the Tariff
and Customs Code of the Philippines, which was enacted by the Congress of the Philippines on 22
June 1957, reads as follows:

601. Chief Officials of the Bureau.-The Bureau of Customs shall have one chief
and one assistant chief, to be known respectively as the Commissioner
(hereinafter known as the 'Commissioner') and Assistant Commissioner of
Customs, who shall each receive an annual compensation in accordance with
the rates prescribed by existing laws. The Assistant Commissioner of Customs
shall be appointed by the proper department head.

Sec. 601 of Republic Act No. 1937, was amended on 27 October 1972 by Presidential Decree No. 34,
amending the Tariff and Customs Code of the Philippines. Sec. 601, as thus amended, now reads as
follows:

Sec. 601. Chief Officials of the Bureau of Customs.-The Bureau of Customs shall
have one chief and one assistant chief, to be known respectively as the
Commissioner (hereinafter known as Commissioner) and Deputy
Commissioner of Customs, who shall each receive an annual compensation in
accordance with the rates prescribed by existing law. The Commissioner and
MA. J. ANGELINA G. MATIBAG, petitioner, vs. ALFREDO L. BENIPAYO, RESURRECCION Z. BORRA, reassignment in a Memorandum dated April 14, 2001[12]addressed to the COMELEC en
FLORENTINO A. TUASON, JR., VELMA J. CINCO, and GIDEON C. DE GUZMAN in his banc. Specifically, Commissioner Sadain questioned Benipayos failure to consult the Commissioner-
capacity as Officer-In-Charge, Finance Services Department of the Commission on in-Charge of the EID in the reassignment of petitioner.
Elections, respondents.
On April 16, 2001, petitioner requested Benipayo to reconsider her relief as Director IV of the
GR 149036 April 2 2002 EID and her reassignment to the Law Department.[13] Petitioner cited Civil Service Commission
Memorandum Circular No. 7 dated April 10, 2001, reminding heads of government offices that
CARPIO, J.: transfer and detail of employees are prohibited during the election period beginning January 2 until
June 13, 2001. Benipayo denied her request for reconsideration on April 18, 2001,[14] citing COMELEC
The Case Resolution No. 3300 dated November 6, 2000, which states in part:
Before us is an original Petition for Prohibition with prayer for the issuance of a writ of preliminary
injunction and a temporary restraining order under Rule 65 of the 1997 Rules of Civil NOW, THEREFORE, the Commission on Elections by virtue of the powers conferred upon it by the
Procedure. Petitioner Ma. J. Angelina G. Matibag (Petitioner for brevity) questions the Constitution, the Omnibus Election Code and other election laws, as an exception to the foregoing
constitutionality of the appointment and the right to hold office of the following: (1) Alfredo L. prohibitions, has RESOLVED, as it is hereby RESOLVED, to appoint, hire new employees or fill new
Benipayo (Benipayo for brevity) as Chairman of the Commission on Elections (COMELEC for brevity); positions and transfer or reassign its personnel, when necessary in the effective performance of its
and (2) Resurreccion Z. Borra (Borra for brevity) and Florentino A. Tuason, Jr. (Tuason for brevity) as mandated functions during the prohibited period, provided that the changes in the assignment of
COMELEC Commissioners. Petitioner also questions the legality of the appointment of Velma J. its field personnel within the thirty-day period before election day shall be effected after due notice
Cinco[1] (Cinco for brevity) as Director IV of the COMELECs Education and Information Department and hearing.
(EID for brevity).
Petitioner appealed the denial of her request for reconsideration to the COMELEC en banc in a
The Facts Memorandum dated April 23, 2001.[15] Petitioner also filed an administrative and criminal
complaint[16] with the Law Department[17] against Benipayo, alleging that her reassignment violated
Section 261 (h) of the Omnibus Election Code, COMELEC Resolution No. 3258, Civil Service
On February 2, 1999, the COMELEC en banc appointed petitioner as Acting Director IV of the
Memorandum Circular No. 07, s. 001, and other pertinent administrative and civil service laws, rules
EID. On February 15, 2000, then Chairperson Harriet O. Demetriou renewed the appointment of
and regulations.
petitioner as Director IV of EID in a Temporary capacity. On February 15, 2001, Commissioner Rufino
S.B. Javier renewed again the appointment of petitioner to the same position in a Temporary During the pendency of her complaint before the Law Department, petitioner filed the instant
capacity.[2] petition questioning the appointment and the right to remain in office of Benipayo, Borra and
Tuason, as Chairman and Commissioners of the COMELEC, respectively. Petitioner claims that the ad
On March 22, 2001, President Gloria Macapagal Arroyo appointed, ad interim, Benipayo as
interim appointments of Benipayo, Borra and Tuason violate the constitutional provisions on the
COMELEC Chairman,[3] and Borra[4] and Tuason[5] as COMELEC Commissioners, each for a term of
independence of the COMELEC, as well as on the prohibitions on temporary appointments and
seven years and all expiring on February 2, 2008. Benipayo took his oath of office and assumed the
reappointments of its Chairman and members. Petitioner also assails as illegal her removal as
position of COMELEC Chairman. Borra and Tuason likewise took their oaths of office and assumed
Director IV of the EID and her reassignment to the Law Department.Simultaneously, petitioner
their positions as COMELEC Commissioners. The Office of the President submitted to the
challenges the designation of Cinco as Officer-in-Charge of the EID. Petitioner, moreover, questions
Commission on Appointments on May 22, 2001 the ad interim appointments of Benipayo, Borra and
the legality of the disbursements made by COMELEC Finance Services Department Officer-in-Charge
Tuason for confirmation.[6] However, the Commission on Appointments did not act on said
Gideon C. De Guzman to Benipayo, Borra and Tuason by way of salaries and other emoluments.
appointments.
In the meantime, on September 6, 2001, President Macapagal Arroyo renewed once again
On June 1, 2001, President Arroyo renewed the ad interim appointments of Benipayo, Borra
the ad interim appointments of Benipayo as COMELEC Chairman and Borra and Tuason as
and Tuason to the same positions and for the same term of seven years, expiring on February 2,
Commissioners, respectively, for a term of seven years expiring on February 2, 2008.[18] They all took
2008.[7] They took their oaths of office for a second time. The Office of the President transmitted on
their oaths of office anew.
June 5, 2001 their appointments to the Commission on Appointments for confirmation. [8]
The Issues
Congress adjourned before the Commission on Appointments could act on their
appointments. Thus, on June 8, 2001, President Macapagal Arroyo renewed again the ad The issues for resolution of this Court are as follows:
interim appointments of Benipayo, Borra and Tuason to the same positions. [9] The Office of the
President submitted their appointments for confirmation to the Commission on 1. Whether or not the instant petition satisfies all the requirements before this Court
Appointments.[10] They took their oaths of office anew. may exercise its power of judicial review in constitutional cases;

In his capacity as COMELEC Chairman, Benipayo issued a Memorandum dated April 11, 2. Whether or not the assumption of office by Benipayo, Borra and Tuason on the basis
2001[11] addressed to petitioner as Director IV of the EID and to Cinco as Director III also of the EID, of the ad interim appointments issued by the President amounts to a temporary
designating Cinco Officer-in-Charge of the EID and reassigning petitioner to the Law appointment prohibited by Section 1 (2), Article IX-C of the Constitution;
Department. COMELEC EID Commissioner-in-Charge Mehol K. Sadain objected to petitioners
3. Assuming that the first ad interim appointments and the first assumption of office by On the other hand, if Benipayo is the lawful COMELEC Chairman because he assumed office
Benipayo, Borra and Tuason are legal, whether or not the renewal of their ad in accordance with the Constitution, then petitioners reassignment is legal and she has no cause to
interim appointments and subsequent assumption of office to the same positions complain provided the reassignment is in accordance with the Civil Service Law. Clearly, petitioner
violate the prohibition on reappointment under Section 1 (2), Article IX-C of the has a personal and material stake in the resolution of the constitutionality of Benipayos assumption
Constitution; of office. Petitioners personal and substantial injury, if Benipayo is not the lawful COMELEC
Chairman, clothes her with the requisite locus standi to raise the constitutional issue in this petition.
4. Whether or not Benipayos removal of petitioner from her position as Director IV of
the EID and her reassignment to the Law Department is illegal and without Respondents harp on petitioners belated act of questioning the constitutionality of the ad
authority, having been done without the approval of the COMELEC as a collegial interim appointments of Benipayo, Borra and Tuason. Petitioner filed the instant petition only on
body; August 3, 2001, when the first ad interim appointments were issued as early as March 22, 2001.
However, it is not the date of filing of the petition that determines whether the constitutional issue
5. Whether or not the Officer-in-Charge of the COMELECs Finance Services Department, was raised at the earliest opportunity. The earliest opportunity to raise a constitutional issue is to
in continuing to make disbursements in favor of Benipayo, Borra, Tuason and Cinco, raise it in the pleadings before a competent court that can resolve the same, such that, if it is not
is acting in excess of jurisdiction. raised in the pleadings, it cannot be considered at the trial, and, if not considered at the trial, it
First Issue: Propriety of Judicial Review cannot be considered on appeal.[22] Petitioner questioned the constitutionality of the ad
interim appointments of Benipayo, Borra and Tuason when she filed her petition before this Court,
Respondents assert that the petition fails to satisfy all the four requisites before this Court which is the earliest opportunity for pleading the constitutional issue before a competent
may exercise its power of judicial review in constitutional cases. Out of respect for the acts of the body. Furthermore, this Court may determine, in the exercise of sound discretion, the time when a
Executive department, which is co-equal with this Court, respondents urge this Court to refrain from constitutional issue may be passed upon.[23] There is no doubt petitioner raised the constitutional
reviewing the constitutionality of the ad interim appointments issued by the President to Benipayo, issue on time.
Borra and Tuason unless all the four requisites are present. These are: (1) the existence of an actual
and appropriate controversy; (2) a personal and substantial interest of the party raising the Moreover, the legality of petitioners reassignment hinges on the constitutionality of
constitutional issue; (3) the exercise of the judicial review is pleaded at the earliest opportunity; and Benipayos ad interim appointment and assumption of office. Unless the constitutionality of
(4) the constitutional issue is the lis mota of the case.[19] Benipayos ad interim appointment and assumption of office is resolved, the legality of petitioners
reassignment from the EID to the Law Department cannot be determined. Clearly, the lis mota of
Respondents argue that the second, third and fourth requisites are absent in this this case is the very constitutional issue raised by petitioner.
case. Respondents maintain that petitioner does not have a personal and substantial interest in the
case because she has not sustained a direct injury as a result of the ad interim appointments of In any event, the issue raised by petitioner is of paramount importance to the public. The
Benipayo, Borra and Tuason and their assumption of office. Respondents point out that petitioner legality of the directives and decisions made by the COMELEC in the conduct of the May 14, 2001
does not claim to be lawfully entitled to any of the positions assumed by Benipayo, Borra or national elections may be put in doubt if the constitutional issue raised by petitioner is left
Tuason. Neither does petitioner claim to be directly injured by the appointments of these three unresolved. In keeping with this Courts duty to determine whether other agencies of government
respondents. have remained within the limits of the Constitution and have not abused the discretion given them,
this Court may even brush aside technicalities of procedure and resolve any constitutional issue
Respondents also contend that petitioner failed to question the constitutionality of the ad raised.[24] Here the petitioner has complied with all the requisite technicalities. Moreover, public
interim appointments at the earliest opportunity. Petitioner filed the petition only on August 3, 2001 interest requires the resolution of the constitutional issue raised by petitioner.
despite the fact that the ad interim appointments of Benipayo, Borra and Tuason were issued as
early as March 22, 2001. Moreover, the petition was filed after the third time that these three Second Issue: The Nature of an Ad Interim Appointment
respondents were issued ad interim appointments. Petitioner argues that an ad interim appointment to the COMELEC is a temporary
Respondents insist that the real issue in this case is the legality of petitioners reassignment appointment that is prohibited by Section 1 (2), Article IX-C of the Constitution, which provides as
from the EID to the Law Department. Consequently, the constitutionality of the ad follows:
interim appointments is not the lis mota of this case.
The Chairman and the Commissioners shall be appointed by the President with the consent of the
We are not persuaded. Commission on Appointments for a term of seven years without reappointment. Of those first
appointed, three Members shall hold office for seven years, two Members for five years, and the
Benipayo reassigned petitioner from the EID, where she was Acting Director, to the Law
last Members for three years, without reappointment. Appointment to any vacancy shall be only for
Department, where she was placed on detail service.[20] Respondents claim that the reassignment
the unexpired term of the predecessor. In no case shall any Member be appointed or designated
was pursuant to x x x Benipayos authority as Chairman of the Commission on Elections, and as the
in a temporary or acting capacity. (Emphasis supplied)
Commissions Chief Executive Officer.[21] Evidently, respondents anchor the legality of petitioners
reassignment on Benipayos authority as Chairman of the COMELEC. The real issue then turns on
whether or not Benipayo is the lawful Chairman of the COMELEC. Even if petitioner is only an Acting Petitioner posits the view that an ad interim appointment can be withdrawn or revoked by the
Director of the EID, her reassignment is without legal basis if Benipayo is not the lawful COMELEC President at her pleasure, and can even be disapproved or simply by-passed by the Commission on
Chairman, an office created by the Constitution. Appointments. For this reason, petitioner claims that an ad interim appointment is temporary in
character and consequently prohibited by the last sentence of Section 1 (2), Article IX-C of the The Constitution imposes no condition on the effectivity of an ad interim appointment, and
Constitution. thus an ad interim appointment takes effect immediately. The appointee can at once assume office
and exercise, as a de jureofficer, all the powers pertaining to the office. In Pacete vs. Secretary of the
Based on petitioners theory, there can be no ad interim appointment to the COMELEC or to Commission on Appointments,[26] this Court elaborated on the nature of an ad interim appointment
the other two constitutional commissions, namely the Civil Service Commission and the Commission as follows:
on Audit. The last sentence of Section 1 (2), Article IX-C of the Constitution is also found in Article IX-
B and Article IX-D providing for the creation of the Civil Service Commission and the Commission on
Audit, respectively. Petitioner interprets the last sentence of Section 1 (2) of Article IX-C to mean A distinction is thus made between the exercise of such presidential prerogative requiring
that the ad interim appointee cannot assume office until his appointment is confirmed by the confirmation by the Commission on Appointments when Congress is in session and when it is in
Commission on Appointments for only then does his appointment become permanent and no longer recess. In the former, the President nominates, and only upon the consent of the Commission on
temporary in character. Appointments may the person thus named assume office. It is not so with reference to ad interim
appointments. It takes effect at once. The individual chosen may thus qualify and perform his
The rationale behind petitioners theory is that only an appointee who is confirmed by the function without loss of time. His title to such office is complete. In the language of the Constitution,
Commission on Appointments can guarantee the independence of the COMELEC. A confirmed the appointment is effective until disapproval by the Commission on Appointments or until the next
appointee is beyond the influence of the President or members of the Commission on Appointments adjournment of the Congress.
since his appointment can no longer be recalled or disapproved. Prior to his confirmation, the
appointee is at the mercy of both the appointing and confirming powers since his appointment can Petitioner cites Blacks Law Dictionary which defines the term ad interim to mean in the
be terminated at any time for any cause. In the words of petitioner, a Sword of Damocles hangs over meantime or for the time being. Hence, petitioner argues that an ad interim appointment is
the head of every appointee whose confirmation is pending with the Commission on Appointments. undoubtedly temporary in character. This argument is not new and was answered by this Court
We find petitioners argument without merit. in Pamantasan ng Lungsod ng Maynila vs. Intermediate Appellate Court,[27] where we explained
that:
An ad interim appointment is a permanent appointment because it takes effect immediately
and can no longer be withdrawn by the President once the appointee has qualified into office. The x x x From the arguments, it is easy to see why the petitioner should experience difficulty in
fact that it is subject to confirmation by the Commission on Appointments does not alter its understanding the situation. Private respondent had been extended several ad
permanent character. The Constitution itself makes an ad interim appointment permanent in interim appointments which petitioner mistakenly understands as appointments temporary in
character by making it effective until disapproved by the Commission on Appointments or until the nature. Perhaps, it is the literal translation of the word ad interim which creates such belief. The
next adjournment of Congress. The second paragraph of Section 16, Article VII of the Constitution term is defined by Black to mean in the meantime or for the time being. Thus, an officer ad interim is
provides as follows: one appointed to fill a vacancy, or to discharge the duties of the office during the absence or
temporary incapacity of its regular incumbent (Blacks Law Dictionary, Revised Fourth Edition,
The President shall have the power to make appointments during the recess of the Congress, 1978). But such is not the meaning nor the use intended in the context of Philippine law. In referring
whether voluntary or compulsory, but such appointments shall be effective only until disapproval to Dr. Estebans appointments, the term is not descriptive of the nature of the appointments given
by the Commission on Appointments or until the next adjournment of the Congress. (Emphasis to him. Rather, it is used to denote the manner in which said appointments were made, that is,
supplied) done by the President of the Pamantasan in the meantime, while the Board of Regents, which is
originally vested by the University Charter with the power of appointment, is unable to act. x x
x. (Emphasis supplied)
Thus, the ad interim appointment remains effective until such disapproval or next adjournment,
signifying that it can no longer be withdrawn or revoked by the President. The fear that the President
can withdraw or revoke at any time and for any reason an ad interim appointment is utterly without Thus, the term ad interim appointment, as used in letters of appointment signed by the
basis. President, means a permanent appointment made by the President in the meantime that Congress
is in recess. It does not mean a temporary appointment that can be withdrawn or revoked at any
More than half a century ago, this Court had already ruled that an ad interim appointment is time. The term, although not found in the text of the Constitution, has acquired a definite legal
permanent in character. In Summers vs. Ozaeta,[25] decided on October 25, 1948, we held that: meaning under Philippine jurisprudence. The Court had again occasion to explain the nature of an ad
interim appointment in the more recent case of Marohombsar vs. Court of Appeals,[28] where the
x x x an ad interim appointment is one made in pursuance of paragraph (4), Section 10, Article VII of Court stated:
the Constitution, which provides that the President shall have the power to make appointments
during the recess of the Congress, but such appointments shall be effective only until disapproval by We have already mentioned that an ad interim appointment is not descriptive of the nature of the
the Commission on Appointments or until the next adjournment of the Congress. It is an appointment, that is, it is not indicative of whether the appointment is temporary or in an acting
appointment permanent in nature, and the circumstance that it is subject to confirmation by the capacity, rather it denotes the manner in which the appointment was made. In the instant case, the
Commission on Appointments does not alter its permanent character. An ad interim appointment appointment extended to private respondent by then MSU President Alonto, Jr. was issued without
is disapproved certainly for a reason other than that its provisional period has expired. Said condition nor limitation as to tenure. The permanent status of private respondents appointment as
appointment is of course distinguishable from an acting appointment which is merely temporary, Executive Assistant II was recognized and attested to by the Civil Service Commission Regional Office
good until another permanent appointment is issued. (Emphasis supplied) No. 12. Petitioners submission that private respondents ad interim appointment is synonymous
with a temporary appointment which could be validly terminated at any time is clearly untenable. member of the COMELEC. This Court ruled that the designation of an acting Commissioner would
Ad interim appointments are permanent but their terms are only until the Board disapproves undermine the independence of the COMELEC and hence violate the Constitution. We declared
them. (Emphasis supplied) then: It would be more in keeping with the intent, purpose and aim of the framers of the Constitution
to appoint a permanent Commissioner than to designate one to act temporarily. (Emphasis
An ad interim appointee who has qualified and assumed office becomes at that moment a supplied)
government employee and therefore part of the civil service. He enjoys the constitutional protection In the instant case, the President did in fact appoint permanent Commissioners to fill the
that [n]o officer or employee in the civil service shall be removed or suspended except for cause vacancies in the COMELEC, subject only to confirmation by the Commission on
provided by law.[29] Thus, an ad interim appointment becomes complete and irrevocable once the Appointments. Benipayo, Borra and Tuason were extended permanent appointments during the
appointee has qualified into office. The withdrawal or revocation of an ad interim appointment is recess of Congress. They were not appointed or designated in a temporary or acting capacity, unlike
possible only if it is communicated to the appointee before the moment he qualifies, and any Commissioner Haydee Yorac in Brillantes vs. Yorac[34] and Solicitor General Felix Bautista
withdrawal or revocation thereafter is tantamount to removal from office.[30] Once an appointee has in Nacionalista Party vs. Bautista.[35] The ad interim appointments of Benipayo, Borra and Tuason
qualified, he acquires a legal right to the office which is protected not only by statute but also by the are expressly allowed by the Constitution which authorizes the President, during the recess of
Constitution. He can only be removed for cause, after notice and hearing, consistent with the Congress, to make appointments that take effect immediately.
requirements of due process.
While the Constitution mandates that the COMELEC shall be independent [36], this provision
An ad interim appointment can be terminated for two causes specified in the Constitution. should be harmonized with the Presidents power to extend ad interim appointments. To hold that
The first cause is the disapproval of his ad interim appointment by the Commission on the independence of the COMELEC requires the Commission on Appointments to first confirm ad
Appointments. The second cause is the adjournment of Congress without the Commission on interim appointees before the appointees can assume office will negate the Presidents power to
Appointments acting on his appointment. These two causes are resolutory conditions expressly make ad interim appointments. This is contrary to the rule on statutory construction to give
imposed by the Constitution on all ad interim appointments. These resolutory conditions constitute, meaning and effect to every provision of the law. It will also run counter to the clear intent of the
in effect, a Sword of Damocles over the heads of ad interim appointees. No one, however, can framers of the Constitution.
complain because it is the Constitution itself that places the Sword of Damocles over the heads of
the ad interim appointees. The original draft of Section 16, Article VII of the Constitution - on the nomination of officers
subject to confirmation by the Commission on Appointments - did not provide for ad
While an ad interim appointment is permanent and irrevocable except as provided by law, an interim appointments. The original intention of the framers of the Constitution was to do away
appointment or designation in a temporary or acting capacity can be withdrawn or revoked at the with ad interim appointments because the plan was for Congress to remain in session throughout
pleasure of the appointing power.[31] A temporary or acting appointee does not enjoy any security the year except for a brief 30-day compulsory recess. However, because of the need to avoid
of tenure, no matter how briefly. This is the kind of appointment that the Constitution prohibits the disruptions in essential government services, the framers of the Constitution thought it wise to
President from making to the three independent constitutional commissions, including the reinstate the provisions of the 1935 Constitution on ad interim appointments. The following
COMELEC. Thus, in Brillantes vs. Yorac,[32] this Court struck down discussion during the deliberations of the Constitutional Commission elucidates this:
as unconstitutional the designation by then President Corazon Aquino of Associate Commissioner
Haydee Yorac as Acting Chairperson of the COMELEC. This Court ruled that:
FR. BERNAS: X x x our compulsory recess now is only 30 days. So under such circumstances, is it
necessary to provide for ad interim appointments? Perhaps there should be a little discussion on
A designation as Acting Chairman is by its very terms essentially temporary and that.
therefore revocable at will. No cause need be established to justify its
revocation. Assuming its validity, the designation of the respondent as Acting Chairman
of the Commission on Elections may be withdrawn by the President of the Philippines xxx
at any time and for whatever reason she sees fit. It is doubtful if the respondent, having
accepted such designation, will not be estopped from challenging its withdrawal. MS. AQUINO: My concern is that unless this problem is addressed, this might present problems in
terms of anticipating interruption of government business, considering that we are not certain of
xxx the length of involuntary recess or adjournment of the Congress. We are certain, however, of the
involuntary adjournment of the Congress which is 30 days, but we cannot leave to conjecture the
matter of involuntary recess.
The Constitution provides for many safeguards to the independence of the Commission
on Elections, foremost among which is the security of tenure of its members. That
guarantee is not available to the respondent as Acting Chairman of the Commission on FR. BERNAS: That is correct, but we are trying to look for a formula. I wonder if the Commissioner
Elections by designation of the President of the Philippines. has a formula x x x.

Earlier, in Nacionalista Party vs. Bautista,[33] a case decided under the 1935 Constitution, xxx
which did not have a provision prohibiting temporary or acting appointments to the COMELEC, this
Court nevertheless declared unconstitutional the designation of the Solicitor General as acting
MR. BENGZON: Madam President, apropos of the matter raised by Commissioner Aquino and after In a Manifestation[43] dated December 28, 2000 filed with this Court in the Gaminde case,
conferring with the Committee, Commissioner Aquino and I propose the following amendment as Chairperson Demetriou stated that she was vacating her office on February 2, 2001, as she believed
the last paragraph of Section 16, the wordings of which are in the 1935 Constitution: THE PRESIDENT any delay in choosing her successor might create a constitutional crisis in view of the proximity of
SHALL HAVE THE POWER TO MAKE APPOINTMENTS DURING THE RECESS OF CONGRESS WHETHER the May 2001 national elections. Commissioner Desamito chose to file a petition for
IT BE VOLUNTARY OR COMPULSORY BUT SUCH APPOINTMENTS SHALL BE EFFECTIVE ONLY UNTIL intervention[44] in the Gaminde case but this Court denied the intervention. Thus, Commissioner
DISAPPROVAL BY THE COMMISSION ON APPOINTMENTS OR UNTIL THE NEXT ADJOURNMENT OF Desamito also vacated his office on February 2, 2001.
THE CONGRESS.
During an election year, Congress normally goes on voluntary recess between February and
June considering that many of the members of the House of Representatives and the Senate run for
This is otherwise called the ad interim appointments. re-election. In 2001, the Eleventh Congress adjourned from January 9, 2001 to June 3,
2001.[45] Concededly, there was no more time for Benipayo, Borra and Tuason, who were originally
xxx extended ad interim appointments only on March 22, 2001, to be confirmed by the Commission on
Appointments before the May 14, 2001 elections.
THE PRESIDENT: Is there any objection to the proposed amendment of Commissioners Aquino and If Benipayo, Borra and Tuason were not extended ad interim appointments to fill up the three
Bengzon, adding a paragraph to the last paragraph of Section 16? (Silence) The Chair hears none; vacancies in the COMELEC, there would only have been one division functioning in the COMELEC
the amendment is approved.[37] (Emphasis supplied) instead of two during the May 2001 elections. Considering that the Constitution requires that all x x
x election cases shall be heard and decided in division,[46] the remaining one division would have
Clearly, the reinstatement in the present Constitution of the ad interim appointing power of been swamped with election cases.Moreover, since under the Constitution motions for
the President was for the purpose of avoiding interruptions in vital government services that reconsideration shall be decided by the Commission en banc, the mere absence of one of the four
otherwise would result from prolonged vacancies in government offices, including the three remaining members would have prevented a quorum, a less than ideal situation considering that
constitutional commissions. In his concurring opinion in Guevara vs. Inocentes,[38] decided under the the Commissioners are expected to travel around the country before, during and after the
1935 Constitution, Justice Roberto Concepcion, Jr. explained the rationale behind ad elections. There was a great probability that disruptions in the conduct of the May 2001 elections
interim appointments in this manner: could occur because of the three vacancies in the COMELEC. The successful conduct of the May 2001
national elections, right after the tumultuous EDSA II and EDSA III events, was certainly essential in
Now, why is the lifetime of ad interim appointments so limited? Because, if they expired before the safeguarding and strengthening our democracy.
session of Congress, the evil sought to be avoided interruption in the discharge of essential Evidently, the exercise by the President in the instant case of her constitutional power to
functions may take place.Because the same evil would result if the appointments ceased to be make ad interim appointments prevented the occurrence of the very evil sought to be avoided by
effective during the session of Congress and before its adjournment. Upon the other hand, once the second paragraph of Section 16, Article VII of the Constitution. This power to make ad
Congress has adjourned, the evil aforementioned may easily be conjured by the issuance of other ad interim appointments is lodged in the President to be exercised by her in her sound judgment. Under
interim appointments or reappointments. (Emphasis supplied) the second paragraph of Section 16, Article VII of the Constitution, the President can choose either
of two modes in appointing officials who are subject to confirmation by the Commission on
Indeed, the timely application of the last sentence of Section 16, Article VII of the Constitution Appointments. First, while Congress is in session, the President may nominate the prospective
barely avoided the interruption of essential government services in the May 2001 national appointee, and pending consent of the Commission on Appointments, the nominee cannot qualify
elections. Following the decision of this Court in Gaminde vs. Commission on and assume office. Second, during the recess of Congress, the President may extend an ad
Appointments,[39] promulgated on December 13, 2000, the terms of office of constitutional officers interim appointment which allows the appointee to immediately qualify and assume office.
first appointed under the Constitution would have to be counted starting February 2, 1987, the date
of ratification of the Constitution, regardless of the date of their actual appointment. By this Whether the President chooses to nominate the prospective appointee or extend an ad
reckoning, the terms of office of three Commissioners of the COMELEC, including the Chairman, interim appointment is a matter within the prerogative of the President because the Constitution
would end on February 2, 2001.[40] grants her that power. This Court cannot inquire into the propriety of the choice made by the
President in the exercise of her constitutional power, absent grave abuse of discretion amounting
Then COMELEC Chairperson Harriet O. Demetriou was appointed only on January 11, 2000 to to lack or excess of jurisdiction on her part, which has not been shown in the instant case.
serve, pursuant to her appointment papers, until February 15, 2002,[41] the original expiry date of
the term of her predecessor, Justice Bernardo P. Pardo, who was elevated to this Court. The original The issuance by Presidents of ad interim appointments to the COMELEC is a long-standing
expiry date of the term of Commissioner Teresita Dy-Liacco Flores was also February 15, 2002, while practice. Former President Corazon Aquino issued an ad interim appointment to Commissioner
that of Commissioner Julio F. Desamito was November 3, 2001. [42] The original expiry dates of the Alfredo E. Abueg.[47]Former President Fidel V. Ramos extended ad interim appointments to
terms of office of Chairperson Demetriou and Commissioners Flores and Desamito were therefore Commissioners Julio F. Desamito, Japal M. Guiani, Graduacion A. Reyes-Claravall and Manolo F.
supposed to fall after the May 2001 elections.Suddenly and unexpectedly, because of Gorospe.[48] Former President Joseph Estrada also extended ad interim appointments to
the Gaminde ruling, there were three vacancies in the seven-person COMELEC, with national Commissioners Abdul Gani M. Marohombsar, Luzviminda Tancangco, Mehol K. Sadain and Ralph C.
elections looming less than three and one-half months away. To their credit, Chairperson Demetriou Lantion.[49]
and Commissioner Flores vacated their offices on February 2, 2001 and did not question any more
before this Court the applicability of the Gaminde ruling to their own situation.
The Presidents power to extend ad interim appointments may indeed briefly put the because of a final decision by the Commission on Appointments to withhold its consent to the
appointee at the mercy of both the appointing and confirming powers. This situation, however, is appointment.
only for a short period - from the time of issuance of the ad interim appointment until the
Commission on Appointments gives or withholds its consent. The Constitution itself sanctions this An ad interim appointment that is by-passed because of lack of time or failure of the
situation, as a trade-off against the evil of disruptions in vital government services. This is also part Commission on Appointments to organize is another matter. A by-passed appointment is one that
of the check-and-balance under the separation of powers, as a trade-off against the evil of granting has not been finally acted upon on the merits by the Commission on Appointments at the close of
the President absolute and sole power to appoint. The Constitution has wisely subjected the the session of Congress. There is no final decision by the Commission on Appointments to give or
Presidents appointing power to the checking power of the legislature. withhold its consent to the appointment as required by the Constitution. Absent such decision, the
President is free to renew the ad interim appointment of a by-passed appointee. This is recognized
This situation, however, does not compromise the independence of the COMELEC as a in Section 17 of the Rules of the Commission on Appointments, which provides as follows:
constitutional body. The vacancies in the COMELEC are precisely staggered to insure that the
majority of its members hold confirmed appointments, and not one President will appoint all the Section 17. Unacted Nominations or Appointments Returned to the President. Nominations or
COMELEC members.[50] In the instant case, the Commission on Appointments had long confirmed appointments submitted by the President of the Philippines which are not finally acted upon at the
four[51] of the incumbent COMELEC members, comprising a majority, who could now be removed close of the session of Congress shall be returned to the President and, unless new nominations or
from office only by impeachment. The special constitutional safeguards that insure the appointments are made, shall not again be considered by the Commission. (Emphasis supplied)
independence of the COMELEC remain in place.[52] The COMELEC enjoys fiscal autonomy, appoints
its own officials and employees, and promulgates its own rules on pleadings and practice. Moreover,
the salaries of COMELEC members cannot be decreased during their tenure. Hence, under the Rules of the Commission on Appointments, a by-passed appointment can be
considered again if the President renews the appointment.
In fine, we rule that the ad interim appointments extended by the President to Benipayo,
Borra and Tuason, as COMELEC Chairman and Commissioners, respectively, do not constitute It is well settled in this jurisdiction that the President can renew the ad interim appointments
temporary or acting appointments prohibited by Section 1 (2), Article IX-C of the Constitution. of by-passed appointees. Justice Roberto Concepcion, Jr. lucidly explained in his concurring opinion
in Guevara vs. Inocentes[53] why by-passed ad interim appointees could be extended new
appointments, thus:
Third Issue: The Constitutionality of Renewals of Appointments

Petitioner also agues that assuming the first ad interim appointments and the first assumption In short, an ad interim appointment ceases to be effective upon disapproval by the Commission,
of office by Benipayo, Borra and Tuason are constitutional, the renewal of the their ad because the incumbent can not continue holding office over the positive objection of the
interim appointments and their subsequent assumption of office to the same positions violate the Commission. It ceases, also, upon the next adjournment of the Congress, simply because the
prohibition on reappointment under Section 1 (2), Article IX-C of the Constitution, which provides President may then issue new appointments - not because of implied disapproval of the Commission
as follows: deduced from its inaction during the session of Congress, for, under the Constitution, the
Commission may affect adversely the interim appointments only by action, never by omission. If the
adjournment of Congress were an implied disapproval of ad interim appointments made prior
The Chairman and the Commissioners shall be appointed by the President with the consent of the thereto, then the President could no longer appoint those so by-passed by the Commission. But, the
Commission on Appointments for a term of seven years without reappointment. Of those first fact is that the President may reappoint them, thus clearly indicating that the reason for said
appointed, three Members shall hold office for seven years, two Members for five years, and the termination of the ad interimappointments is not the disapproval thereof allegedly inferred from
last members for three years, without reappointment. X x x. (Emphasis supplied) said omission of the Commission, but the circumstance that upon said adjournment of the
Congress, the President is free to make ad interim appointments or reappointments. (Emphasis
Petitioner theorizes that once an ad interim appointee is by-passed by the Commission on supplied)
Appointments, his ad interim appointment can no longer be renewed because this will violate
Section 1 (2), Article IX-C of the Constitution which prohibits reappointments. Petitioner asserts that Guevara was decided under the 1935 Constitution from where the second paragraph of Section 16,
this is particularly true to permanent appointees who have assumed office, which is the situation of Article VII of the present Constitution on ad interim appointments was lifted verbatim.[54] The
Benipayo, Borra and Tuason if their ad interimappointments are deemed permanent in character. jurisprudence under the 1935 Constitution governing ad interim appointments by the President is
There is no dispute that an ad interim appointee disapproved by the Commission on doubtless applicable to the present Constitution. The established practice under the present
Appointments can no longer be extended a new appointment. The disapproval is a final decision of Constitution is that the President can renew the appointments of by-passed ad
the Commission on Appointments in the exercise of its checking power on the appointing authority interim appointees. This is a continuation of the well-recognized practice under the 1935
of the President. The disapproval is a decision on the merits, being a refusal by the Commission on Constitution, interrupted only by the 1973 Constitution which did not provide for a Commission on
Appointments to give its consent after deliberating on the qualifications of the appointee. Since the Appointments but vested sole appointing power in the President.
Constitution does not provide for any appeal from such decision, the disapproval is final and binding The prohibition on reappointment in Section 1 (2), Article IX-C of the Constitution applies
on the appointee as well as on the appointing power. In this instance, the President can no longer neither to disapproved nor by-passed ad interim appointments. A disapproved ad
renew the appointment not because of the constitutional prohibition on reappointment, but interim appointment cannot be revived by another ad interim appointment because the disapproval
is final under Section 16, Article VII of the Constitution, and not because a reappointment is
prohibited under Section 1 (2), Article IX-C of the Constitution. A by-passed ad interim appointment To foreclose this interpretation, the phrase without reappointment appears twice in Section
can be revived by a new ad interim appointment because there is no final disapproval under Section 1 (2), Article IX-C of the present Constitution. The first phrase prohibits reappointment of any person
16, Article VII of the Constitution, and such new appointment will not result in the appointee serving previously appointed for a term of seven years. The second phrase prohibits reappointment of any
beyond the fixed term of seven years. person previously appointed for a term of five or three years pursuant to the first set of appointees
under the Constitution. In either case, it does not matter if the person previously appointed
Section 1 (2), Article IX-C of the Constitution provides that [t]he Chairman and the completes his term of office for the intention is to prohibit any reappointment of any kind.
Commissioners shall be appointed x x x for a term of seven years without reappointment. (Emphasis
supplied) There are four situations where this provision will apply. The first situation is where an ad However, an ad interim appointment that has lapsed by inaction of the Commission on
interim appointee to the COMELEC, after confirmation by the Commission on Appointments, serves Appointments does not constitute a term of office. The period from the time the ad
his full seven-year term. Such person cannot be reappointed to the COMELEC, whether as a member interim appointment is made to the time it lapses is neither a fixed term nor an unexpired term. To
or as a chairman, because he will then be actually serving more than seven years. The second hold otherwise would mean that the President by his unilateral action could start and complete the
situation is where the appointee, after confirmation, serves a part of his term and then resigns running of a term of office in the COMELEC without the consent of the Commission on
before his seven-year term of office ends. Such person cannot be reappointed, whether as a Appointments. This interpretation renders inutile the confirming power of the Commission on
member or as a chair, to a vacancy arising from retirement because a reappointment will result in Appointments.
the appointee also serving more than seven years. The third situation is where the appointee is
confirmed to serve the unexpired term of someone who died or resigned, and the appointee The phrase without reappointment applies only to one who has been appointed by the
completes the unexpired term. Such person cannot be reappointed, whether as a member or chair, President and confirmed by the Commission on Appointments, whether or not such person
to a vacancy arising from retirement because a reappointment will result in the appointee also completes his term of office. There must be a confirmation by the Commission on Appointments of
serving more than seven years. the previous appointment before the prohibition on reappointment can apply. To hold otherwise
will lead to absurdities and negate the Presidents power to make ad interim appointments.
The fourth situation is where the appointee has previously served a term of less than seven
years, and a vacancy arises from death or resignation. Even if it will not result in his serving more In the great majority of cases, the Commission on Appointments usually fails to act, for lack
than seven years, a reappointment of such person to serve an unexpired term is also prohibited of time, on the ad interim appointments first issued to appointees. If such ad interim appointments
because his situation will be similar to those appointed under the second sentence of Section 1 (2), can no longer be renewed, the President will certainly hesitate to make ad interim appointments
Article IX-C of the Constitution. This provision refers to the first appointees under the Constitution because most of her appointees will effectively be disapproved by mere inaction of the Commission
whose terms of office are less than seven years, but are barred from ever being reappointed under on Appointments. This will nullify the constitutional power of the President to make ad
any situation. Not one of these four situations applies to the case of Benipayo, Borra or Tuason. interim appointments, a power intended to avoid disruptions in vital government services. This
Court cannot subscribe to a proposition that will wreak havoc on vital government services.
The framers of the Constitution made it quite clear that any person who has served any term
of office as COMELEC member whether for a full term of seven years, a truncated term of five or The prohibition on reappointment is common to the three constitutional commissions. The
three years, or even for an unexpired term of any length of time can no longer be reappointed to framers of the present Constitution prohibited reappointments for two reasons. The first is to
the COMELEC. Commissioner Foz succinctly explained this intent in this manner: prevent a second appointment for those who have been previously appointed and confirmed even
if they served for less than seven years. The second is to insure that the members of the three
constitutional commissions do not serve beyond the fixed term of seven years. As reported in
MR. FOZ. But there is the argument made in the concurring opinion of Justice Angelo the Journal of the Constitutional Commission, Commissioner Vicente B. Foz, who sponsored[58]the
Bautista in the case of Visarra vs. Miraflor, to the effect that the prohibition on proposed articles on the three constitutional commissions, outlined the four important features of
reappointment applies only when the term or tenure is for seven years. But in cases the proposed articles, to wit:
where the appointee serves only for less than seven years, he would be entitled to
reappointment. Unless we put the qualifying words without reappointment in the case
of those appointed, then it is possible that an interpretation could be made later on Mr. Foz stated that the Committee had introduced basic changes in the common
their case, they can still be reappointed to serve for a total of seven years. provision affecting the three Constitutional Commissions, and which are: 1) fiscal
autonomy which provides (that) appropriations shall be automatically and regularly
released to the Commission in the same manner (as) provided for the Judiciary; 2) fixed
Precisely, we are foreclosing that possibility by making it clear that even in the case term of office without reappointment on a staggered basis to ensure continuity of
of those first appointed under the Constitution, no reappointment can be functions and to minimize the opportunity of the President to appoint all the members
made.[55] (Emphasis supplied) during his incumbency; 3) prohibition to decrease salaries of the members of the
Commissions during their term of office; and 4) appointments of members would not
In Visarra vs. Miraflor,[56] Justice Angelo Bautista, in his concurring opinion, quoted Nacionalista vs. require confirmation.[59] (Emphasis supplied)
De Vera[57] that a [r]eappointment is not prohibited when a Commissioner has held office only for,
say, three or six years, provided his term will not exceed nine years in all. This was the interpretation There were two important amendments subsequently made by the Constitutional
despite the express provision in the 1935 Constitution that a COMELEC member shall hold office for Commission to these four features. First, as discussed earlier, the framers of the Constitution
a term of nine years and may not be reappointed. decided to require confirmation by the Commission on Appointments of all appointments to the
constitutional commissions. Second, the framers decided to strengthen further the prohibition on
serving beyond the fixed seven-year term, in the light of a former chair of the Commission on Audit MR. DE LOS REYES: Mr. Presiding Officer, the reason for this amendment is that some lawyers make
remaining in office for 12 years despite his fixed term of seven years. The following exchange in the a distinction between an appointment and a designation. The Gentleman will recall that in the case
deliberations of the Constitutional Commission is instructive: of Commissioner on Audit Tantuico, I think his term exceeded the constitutional limit but the
Minister of Justice opined that it did not because he was only designated during the time that he
MR. SUAREZ: These are only clarificatory questions, Madam President. May I call the acted as Commissioner on Audit. So, in order to erase that distinction between appointment and
sponsors attention, first of all, to Section 2 (2) on the Civil Service Commission wherein designation, we should specifically place the word so that there will be no more ambiguity. In no
it is stated: In no case shall any Member be appointed in a temporary or acting capacity. I case shall any Member be appointed OR DESIGNATED in a temporary or acting capacity.
detect in the Committees proposed resolutions a constitutional hangover, if I may use
the term, from the past administration. Am I correct in concluding that the reason the MR. FOZ: The amendment is accepted, Mr. Presiding Officer.
Committee introduced this particular provision is to avoid an incident similar to the case
of the Honorable Francisco Tantuico who was appointed in an acting capacity as MR. DE LOS REYES: Thank you.
Chairman of the Commission on Audit for about 5 years from 1975 until 1980, and then
in 1980, was appointed as Chairman with a tenure of another 7 years. So, if we follow
that appointment to (its) logical conclusion, he occupied that position for about 12 years THE PRESIDING OFFICER (Mr. Trenas): Is there any objection? (Silence) The Chair hears none; the
in violation of the Constitution? amendment is approved.[62]

MR. FOZ: It is only one of the considerations. Another is really to make sure that any The ad interim appointments and subsequent renewals of appointments of Benipayo, Borra
member who is appointed to any of the commissions does not serve beyond 7 and Tuason do not violate the prohibition on reappointments because there were no previous
years.[60] (Emphasis supplied) appointments that were confirmed by the Commission on Appointments. A reappointment
presupposes a previous confirmed appointment. The same ad interim appointments and renewals
of appointments will also not breach the seven-year term limit because all the appointments and
Commissioner Christian Monsod further clarified the prohibition on reappointment in this renewals of appointments of Benipayo, Borra and Tuason are for a fixed term expiring on February
manner: 2, 2008.[63] Any delay in their confirmation will not extend the expiry date of their terms of office.
Consequently, there is no danger whatsoever that the renewal of the ad interim appointments of
"MR. MONSOD. If the (Commissioner) will read the whole Article, she will notice these three respondents will result in any of the evils intended to be exorcised by the twin
that there is no reappointment of any kind and, therefore as a whole there is no way prohibitions in the Constitution. The continuing renewal of the ad interim appointment of these
that somebody can serve for more than seven years. The purpose of the last sentence three respondents, for so long as their terms of office expire on February 2, 2008, does not violate
is to make sure that this does not happen by including in the appointment both the prohibition on reappointments in Section 1 (2), Article IX-C of the Constitution.
temporary and acting capacities."[61] (Emphasis supplied)
Fourth Issue: Respondent Benipayos Authority to Reassign Petitioner
Plainly, the prohibition on reappointment is intended to insure that there will be no reappointment
of any kind. On the other hand, the prohibition on temporary or acting appointments is intended to Petitioner claims that Benipayo has no authority to remove her as Director IV of the EID and
prevent any circumvention of the prohibition on reappointment that may result in an appointees reassign her to the Law Department. Petitioner further argues that only the COMELEC, acting as a
total term of office exceeding seven years. The evils sought to be avoided by the twin prohibitions collegial body, can authorize such reassignment. Moreover, petitioner maintains that a
are very specific - reappointment of any kind and exceeding ones term in office beyond the reassignment without her consent amounts to removal from office without due process and
maximum period of seven years. therefore illegal.

Not contented with these ironclad twin prohibitions, the framers of the Constitution tightened Petitioners posturing will hold water if Benipayo does not possess any color of title to the
even further the screws on those who might wish to extend their terms of office. Thus, the word office of Chairman of the COMELEC. We have ruled, however, that Benipayo is the de jure COMELEC
designated was inserted to plug any loophole that might be exploited by violators of the Chairman, and consequently he has full authority to exercise all the powers of that office for so long
Constitution, as shown in the following discussion in the Constitutional Commission: as his ad interim appointment remains effective. Under Section 7 (4), Chapter 2, Subtitle C, Book V
of the Revised Administrative Code, the Chairman of the COMELEC is vested with the following
MR. DE LOS REYES: On line 32, between the words appointed and in, I propose to insert the words power:
OR DESIGNATED so that the whole sentence will read: In no case shall any Member be appointed
OR DESIGNATED in a temporary or acting capacity. Section 7. Chairman as Executive Officer; Powers and Duties. The Chairman, who shall be the Chief
Executive Officer of the Commission, shall:
THE PRESIDING OFFICER (Mr. Trenas): What does the Committee say?
xxx
MR. FOZ: But it changes the meaning of this sentence. The sentence reads: In no case shall any
Member be appointed in a temporary or acting capacity.
(4) Make temporary assignments, rotate and transfer personnel in accordance with the provisions including public school teachers, within the election period except upon prior approval
of the Civil Service Law. (Emphasis supplied) of the Commission.

The Chairman, as the Chief Executive of the COMELEC, is expressly empowered on his own authority Petitioner claims that Benipayo failed to secure the approval of the COMELEC en banc to effect
to transfer or reassign COMELEC personnel in accordance with the Civil Service Law. In the exercise transfers or reassignments of COMELEC personnel during the election period. [67] Moreover,
of this power, the Chairman is not required by law to secure the approval of the COMELEC en banc. petitioner insists that the COMELEC en banc must concur to every transfer or reassignment of
COMELEC personnel during the election period.
Petitioners appointment papers dated February 2, 1999, February 15, 2000 and February 15,
2001, attached as Annexes X, Y and Z to her Petition, indisputably show that she held her Director Contrary to petitioners allegation, the COMELEC did in fact issue COMELEC Resolution No.
IV position in the EID only in an acting or temporary capacity.[64] Petitioner is not a Career Executive 3300 dated November 6, 2000,[68] exempting the COMELEC from Section 261 (h) of the Omnibus
Service (CES) officer, and neither does she hold Career Executive Service Eligibility, which are Election Code. The resolution states in part:
necessary qualifications for holding the position of Director IV as prescribed in the Qualifications
Standards (Revised 1987) issued by the Civil Service Commission.[65] Obviously, petitioner does not WHEREAS, Sec. 56 and Sec. 261, paragraphs (g) and (h), of the Omnibus Election Code provides as
enjoy security of tenure as Director IV. In Secretary of Justice Serafin Cuevas vs. Atty. Josefina G. follows:
Bacal,[66] this Court held that:

xxx
As respondent does not have the rank appropriate for the position of Chief Public Attorney, her
appointment to that position cannot be considered permanent, and she can claim no security of
tenure in respect of that position. As held in Achacoso v. Macaraig: Sec. 261. Prohibited Acts. The following shall be guilty of an election offense:

It is settled that a permanent appointment can be issued only to a person who meets all the xxx
requirements for the position to which he is being appointed, including the appropriate eligibility
prescribed. Achacoso did not. At best, therefore, his appointment could be regarded only as (h) Transfer of officers and employees in the civil service Any public official who
temporary. And being so, it could be withdrawn at will by the appointing authority and at a moments makes or causes any transfer or detail whatever of any officer or employee in
notice, conformably to established jurisprudence x x x. the civil service including public school teachers, within the election period
except upon approval of the Commission.
The mere fact that a position belongs to the Career Service does not automatically confer security
of tenure on its occupant even if he does not possess the required qualifications. Such right will have WHEREAS, the aforequoted provisions are applicable to the national and local elections on May 14,
to depend on the nature of his appointment, which in turn depends on his eligibility or lack of it. A 2001;
person who does not have the requisite qualifications for the position cannot be appointed to it in
the first place, or as an exception to the rule, may be appointed to it merely in an acting capacity in WHEREAS, there is an urgent need to appoint, transfer or reassign personnel of the Commission on
the absence of appropriate eligibles. The appointment extended to him cannot be regarded as Elections during the prohibited period in order that it can carry out its constitutional duty to conduct
permanent even if it may be so designated x x x. free, orderly, honest, peaceful and credible elections;

Having been appointed merely in a temporary or acting capacity, and not possessed of the NOW, THEREFORE, the Commission on Elections by virtue of the powers conferred upon it by the
necessary qualifications to hold the position of Director IV, petitioner has no legal basis in claiming Constitution, the Omnibus Election Code and other election laws, as an exception to the foregoing
that her reassignment was contrary to the Civil Service Law. This time, the vigorous argument of prohibitions, has RESOLVED, as it is hereby RESOLVED, to appoint, hire new employees or fill new
petitioner that a temporary or acting appointment can be withdrawn or revoked at the pleasure of positions and transfer or reassign its personnel, when necessary in the effective performance of its
the appointing power happens to apply squarely to her situation. mandated functions during the prohibited period, provided that the changes in the assignment of
Still, petitioner assails her reassignment, carried out during the election period, as a prohibited its field personnel within the thirty-day period before election day shall be effected after due notice
act under Section 261 (h) of the Omnibus Election Code, which provides as follows: and hearing. (Emphasis supplied)

Section 261. Prohibited Acts. The following shall be guilty of an election offense: The proviso in COMELEC Resolution No. 3300, requiring due notice and hearing before any transfer
or reassignment can be made within thirty days prior to election day, refers only to
COMELEC field personnel and not to head office personnel like the petitioner. Under the Revised
xxx Administrative Code,[69] the COMELEC Chairman is the sole officer specifically vested with the power
to transfer or reassign COMELEC personnel. The COMELEC Chairman will logically exercise the
(h) Transfer of officers and employees in the civil service - Any public official who makes authority to transfer or reassign COMELEC personnel pursuant to COMELEC Resolution No.
or causes any transfer or detail whatever of any officer or employee in the civil service
3300. The COMELEC en banc cannot arrogate unto itself this power because that will mean
amending the Revised Administrative Code, an act the COMELEC en banc cannot legally do.

COMELEC Resolution No. 3300 does not require that every transfer or reassignment of
COMELEC personnel should carry the concurrence of the COMELEC as a collegial body. Interpreting
Resolution No. 3300 to require such concurrence will render the resolution meaningless since the
COMELEC en banc will have to approve every personnel transfer or reassignment, making the
resolution utterly useless. Resolution No. 3300 should be interpreted for what it is, an approval to
effect transfers and reassignments of personnel, without need of securing a second approval from
the COMELEC en banc to actually implement such transfer or reassignment.

The COMELEC Chairman is the official expressly authorized by law to transfer or reassign
COMELEC personnel. The person holding that office, in a de jure capacity, is Benipayo. The
COMELEC en banc, in COMELEC Resolution No. 3300, approved the transfer or reassignment of
COMELEC personnel during the election period. Thus, Benipayos order reassigning petitioner from
the EID to the Law Department does not violate Section 261 (h) of the Omnibus Election Code. For
the same reason, Benipayos order designating Cinco Officer-in-Charge of the EID is legally
unassailable.

Fifth Issue: Legality of Disbursements to Respondents

Based on the foregoing discussion, respondent Gideon C. De Guzman, Officer-in-Charge of the


Finance Services Department of the Commission on Elections, did not act in excess of jurisdiction in
paying the salaries and other emoluments of Benipayo, Borra, Tuason and Cinco.

WHEREFORE, the petition is dismissed for lack of merit. Costs against petitioner.

SO ORDERED.

Davide, Jr., C.J., Bellosillo, Melo, Kapunan, Mendoza, Panganiban, Quisumbing, Ynares-
Santiago, De Leon, Jr., and Sandoval-Gutierrez, JJ., concur.
Puno, and Vitug, JJ., on official leave.
JOSE D. VILLENA, petitioner, vs. THE SECRETARY OF THE INTERIOR, respondent. contained in section 2188 of the Administrative Code as amended. In other
words, the Secretary of the Interior must exercise his supervision over local
LAUREL, J.: G.R. No. L-46570 April 21, 1939 governments, if he has that power under existing law, in accordance with
section 2188 of the Administrative Code, as amended, as the latter provision
govern the procedure to be followed in suspending and punishing elective local
This is an original action of prohibition with prayer for preliminary injunction against the Secretary officials while section 79 (C) of the Administrative Code is the general law which
of the Interior to restrain him and his agents from proceeding with the investigation of the herein must yield to the special law;
petitioner, Jose D. Villena, mayor of Makati, Rizal, which was scheduled to take place on March 28,
1939, until this case is finally determined by this court. The respondent was required to answer, but
the petition for preliminary injunction was denied. (c) Because the respondent Secretary of the Interior is exercising an arbitrary
power by converting himself into a complainant and at the same time judge of
the charges he has preferred against the petitioner;
It appears that the Division of Investigation of the Department of Justice, upon the request of the
Secretary of the Interior, conducted an inquiry into the conduct of the petitioner, as a result of which
the latter was found to have committed bribery, extortion, malicious abuse of authority and (d) Because the action of the respondent Secretary of the Interior is not based
unauthorized practice of the law profession. The respondent, therefore, on February 8, 1939, on any sworn statement of any private person or citizen of this government
recommended to the President of the Philippines the suspension of the petitioner to prevent when section 2188 of the Administrative Code requires the complaint against
possible coercion of witnesses, which recommendation was granted, according to the answer of the elective municipal officials to be under oath in order to merit consideration by
Solicitor-General of March 20, 1939, verbally by the President on the same day. The Secretary of the the authorities.
Interior suspended the petitioner from office on February 9, 1939, and then and thereafter wired
the Provincial Governor of Rizal with instruction that the petitioner be advised accordingly. On Petitioner prays this Honorable Court:
February 13, 1939, the respondent wrote the petitioner a letter, specifying the many charges against
him and notifying him of the designation of Emiliano Anonas as special investigator to investigate (a) To issue a writ of preliminary injunction against the respondent restraining him, his
the charges. The special investigator forthwith notified the petitioner that the formal investigation agents, attorneys and all persons acting by virtue of his authority from further proceeding
would be commenced on February 17, 1939, at 9 a. m., but due to several incidents and against the petitioner until this case is finally determined by this court;
postponements, the same had to be set definitely for March 28, 1939. Hence, the petition for
preliminary injunction referred to in the beginning of this opinion.
(b) To declare, after the hearing of this petition, that the respondent is without authority
or jurisdiction to suspend the petitioner from the office of mayor of Makati and to order
The petitioner contends in his petition: his immediate reinstatement in office;

(1) That the Secretary of the Interior has no jurisdiction or authority to suspend and much (c) To declare that the respondent has no authority to prefer charges against the
less to prefer by himself administrative charges against the petitioner and decide also by petitioner and to investigate those charges for the grant him that power the respondent
himself the merits of the charges as the power to suspend municipal elective officials and would be acting as prosecutor and judge of the case of his own creation.
to try and punish them for misconduct in office or dereliction of duty is lodged in some
other agencies of the government;
Upon the other hand, the Solicitor-General contends in his answer:

(2) That the acts of the respondent in suspending the petitioner from office and in
preferring by himself charges against him and in designating a special investigator to hear 1. That section 79 (C) in relation with section 86 of the Revised Administrative Code
the charges specified in Exhibit A are null and void for the following reasons: expressly empowers the respondent as Secretary of the Interior to "order the
investigation of any act or conduct of any person in the service of any bureau or office
under his department" and in connection therewith to "designate an official or person
(a) Because the Secretary of the Interior, by suspending the petitioner, has who shall conduct such investigation"; (Par. 4.)
exercised control over local governments when that power has been taken
away from the President of the Philippines by the Constitution for the to
abrogate and the power to abrogate means the power to power to control has 2. That although section 2188 of the Revised Administrative Code, invoked by the
been interpreted to include the power usurp and the power to usurp petitioner, empowers the provincial governor to `receive and investigate complaints
necessarily includes the power to destroy; made under oath against municipal officers for neglect of duty, oppression, corruption or
other form of maladministration of office', said section does not preclude the respondent
as Secretary of the Interior from exercising the power vested in him by section 79 (C) in
(b) Because even if the respondent Secretary of the Interior has power of relation with section 86 of the Revised Administrative Code; and that, moreover, said
supervision over local governments, that power, according to the constitution, section 2188 must be read in relation with section 37 of Act No. 4007, known as the
must be exercised in accordance with the provisions of law and the provisions Reorganization Law of 1932; (Par. 4 [b].)
of law governing trials of charges against elective municipal officials are those
3. That at the commencement of the investigation the petitioner did not question the but it at least implies authority to inquire into facts and conditions in order to render the power real
power or jurisdiction of the Department of the Interior to investigate the administrative and effective. If supervision is to be conscientious and rational, and not automatic and brutal, it must
charges against him but merely contended that the filing of said charges was not in be founded upon a knowledge of actual facts and conditions disclosed after careful study and
accordance with law for the reason that they did not bear the oaths of the complainants; investigation." The principle there enunciated is applicable with equal force to the present case.
(Par. 5.)
We hold, therefore, that the Secretary of the Interior is invested with authority to order the
4. That the authority of a department head order the investigation of any act or conduct investigation of the charges against the petitioner and to appoint a special investigator for that
of any person under his department necessarily carries with it by implication the authority purpose.
to take such measures as he may deem necessary to accomplish the purpose of the
investigation, such as by suspending the officer under investigation to prevent coercion As regards the challenged power of the Secretary of the Interior to decree the suspension of the
of witnesses; and that, furthermore, the suspension from office of the herein petitioner herein petitioner pending an administrative investigation of the charges against him, the question,
by the respondent was authorized by the Chief Executive, who is empowered by section it may be admitted, is not free from difficulties. There is no clear and express grant of power to the
64 (B) of the Administrative Code to remove officials from office; (Par. 7.) secretary to suspend a mayor of a municipality who is under investigation. On the contrary, the
power appears lodged in the provincial governor by section 2188 of the Administrative Code which
5. That the petition does not allege facts and circumstances that would warrant the provides that "The provincial governor shall receive and investigate complaints made under oath
granting of the writ of preliminary injunction under section 164 of the Code of Civil against municipal officers for neglect of duty, oppression, corruption or other form of
Procedure; (Par. 8.) maladministration of office, and conviction by final judgment of any crime involving moral turpitude.
For minor delinquency he may reprimand the offender; and if a more severe punishment seems to
6. That it is a well-settled rule "that courts of equity have no power to restrain public be desirable he shall submit written charges touching the matter to the provincial board, furnishing
officers by injunction from performing any official act which they are by law required to a copy of such charges to the accused either personally or by registered mail, and he may in such
perform, or acts which are not in excess of the authority and discretion reposed in them." case suspend the officer (not being the municipal treasurer) pending action by the board, if in his
(Par. 9) opinion the charge be one affecting the official integrity of the officer in question. Where suspension
is thus effected, the written charges against the officer shall be filed with the board within five days."
The fact, however, that the power of suspension is expressly granted by section 2188 of the
The issues presented in this case may be reduced to an inquiry into the legal authority of the Administrative Code to the provincial governor does not mean that the grant is necessarily exclusive
Secretary of the Interior (a) to order an investigation, by a special investigation appointed by him, of and precludes the Secretary of the Interior from exercising a similar power. For instance, counsel for
the charges of corruption and irregularity brought to his attention against the mayor of the the petitioner admitted in the oral argument that the President of the Philippines may himself
municipality of Makati, Province of Rizal, who is the petitioner herein, and (b) to decree the suspend the petitioner from office in virtue of his greater power of removal (sec. 2191, as amended,
suspension of the said mayor pending the investigation of the charges. Administrative Code) to be exercised conformably to law. Indeed, if the President could, in the
manner prescribed by law, remove a municipal official, it would be a legal incongruity if he were to
Section 79 (C) of the Administrative Code provides as follows: be devoid of the lesser power of suspension. And the incongruity would be more patent if, possessed
of the power both to suspend and to remove a provincial official (sec. 2078, Administrative Code),
The Department Head shall have direct control, direction, and supervision over all the President were to be without the power to suspend a municipal official. Here is, parenthetically,
bureaus and offices under his jurisdiction and may, any provision of existing law to the an instance where, as counsel for petitioner admitted, the power to suspend a municipal official is
contrary notwithstanding, repeal or modify the decisions of the chiefs of said bureaus of not exclusive. Upon the other hand, it may be argued with some degree of plausibility that, if the
offices when advisable in the public interest. Secretary of the Interior is, as we have hereinabove concluded, empowered to investigate the
charges against the petitioner and to appoint a special investigator for that purpose, preventive
suspension may be a means by which to carry into effect a fair and impartial investigation. This is a
The Department Head may order the investigation of any act conduct of any person in the point, however, which, for the reason hereinafter indicated, we do not have to decide.
service of any bureau of office under his department and in connection therewith may
appoint a committee or designate an official or person who shall conduct such
investigations, and such committee, official, or person may summon, witness The Solicitor-General argues that section 37 of Act No. 4007, known as the Reorganization Law of
by subpoena and subpoena duces tecum, administer oath and take testimony relevant to 1932, by providing, "the provisions of the existing law to the contrary notwithstanding," that
the investigation. "whenever a specific power, authority, duty, function, or activity is entrusted to a chief of bureau,
office, division or service, the same shall be understood as also conferred upon the proper
Department Head who shall have authority to act directly in pursuance thereof, or to review, modify
The above section speaks, it is true, of direct control, direction, and supervision over bureaus and or revoke any decision or action of said chief of bureau, office, division or service", should be
offices under the jurisdiction of the Secretary of the Interior, but this section should be interpreted interpreted to concede to the Secretary of the Interior the power to suspend a mayor of a
in relation to section 86 of the same Code which grants to the Department of the Interior "executive municipality. The argument is so generally sweeping that, unless distinctions are made, the effect
supervision over the administration of provinces, municipalities, chartered cities and other local would be the complete abrogation at will of the powers of provincial and municipal officials even in
political subdivisions." In the case of Planas vs. Gil (37 Off. Gaz., 1228), we observed that corporate affairs of local governments. Under the theory suggested by the Solicitor-General, the
"Supervision is not a meaningless thing. It is an active power. It is certainly not without limitation,
Secretary of the Interior could, as observed by able counsel for the petitioner, enter into a contract Executive is required by the Constitution or the law to act in person or the exigencies of the situation
and sign a deed of conveyance of real property in behalf of a municipality against the opposition of demand that he act personally, the multifarious executive and administrative functions of the Chief
the mayor thereof who is the local official authorized by law to do so (sec. 2196, Revised Executive are performed by and through the executive departments, and the acts of the secretaries
Administrative Code), or in behalf of a province in lieu of the provincial governor thereof (sec of such departments, performed and promulgated in the regular course of business, are, unless
2068, Ibid.), and otherwise exercise powers of corporate character mentioned in sections 2067 and disapproved or reprobated by the Chief Executive, presumptively the acts of the Chief Executive.
2175 of the Revised Administrative Code and which are lodged in the corresponding provincial and (Runkle vs. United States [1887], 122 U. S., 543; 30 Law. ed., 1167: 7 Sup. Ct. Rep., 1141; see also U.
municipal officials. And if the power of suspension of the Secretary of the Interior is to be justified S. vs. Eliason [1839], 16 Pet., 291; 10 Law. ed., 968; Jones vs. U. S. [1890], 137 U. S., 202; 34 Law. ed.,
on the plea that the pretended power is governmental and not corporate, the result would be more 691; 11 Sup. Ct., Rep., 80; Wolsey vs. Chapman [1880], 101 U. S., 755; 25 Law. ed., 915:
disastrous. Then and thereunder, the Secretary of the Interior, in lieu of the mayor of the Wilcox vs. Jackson [1836], 13 Pet., 498; 10 Law. ed., 264.)
municipality, could directly veto municipal ordinances and resolutions under section 2229 of the
Revised Administrative Code; he could, without any formality, elbow aside the municipal mayor and Fear is expressed by more than one member of this court that the acceptance of the principle of
himself make appointments to all non-elective positions in the municipal service, under section 2199 qualified political agency in this and similar cases would result in the assumption of responsibility by
of the Revised Administrative Code; he could, instead of the provincial governor, fill a temporary the President of the Philippines for acts of any member of his cabinet, however illegal, irregular or
vacancy in any municipal office under subsection (a), section 2188, as amended, of the said Code; improper may be these acts. The implications, it is said, are serious. Fear, however, is no valid
he-could even directly appoint lieutenants of barrios and wrest the authority given by section 2218 argument against the system once adopted, established and operated. Familiarity with the essential
of the Revised Administrative Code to a municipal councilor. Instances may be multiplied but it is background of the type of government established under our Constitution, in the light of certain
unnecessary to go any further. Prudence, then, dictates that we should hesitate to accept the well-known principles and practices that go with the system, should offer the necessary explanation.
suggestion urged upon us by the Solicitor-General, especially where we find the path indicated by With reference to the Executive Department of the government, there is one purpose which is
him neither illuminated by the light of our own experience nor cemented by the virtuality of legal crystal-clear and is readily visible without the projection of judicial searchlight, and that is, the
principles but is, on the contrary, dimmed by the recognition however limited in our own establishment of a single, not plural, Executive. The first section of Article VII of the Constitution,
Constitution of the right of local self-government and by the actual operation and enforcement of dealing with the Executive Department, begins with the enunciation of the principles that "The
the laws governing provinces, chartered cities, municipalities and other political subdivisions. It is executive power shall be vested in a President of the Philippines." This means that the President of
not any question of wisdom of legislation but the existence of any such destructive authority in the the Philippines is the Executive of the Government of the Philippines, and no other. The heads of
law invoked by the Government that we are called upon to pass and determine here. the executive departments occupy political positions and hold office in an advisory capacity, and, in
the language of Thomas Jefferson, "should be of the President's bosom confidence" (7 Writings,
In the deliberation of this case it has also been suggested that, admitting that the President of the Ford ed., 498), and, in the language of Attorney-General Cushing (7 Op., Attorney-General, 453),
Philippines is invested with the authority to suspend the petitioner, and it appearing that he had "are subject to the direction of the President." Without minimizing the importance of the heads of
verbally approved or at least acquiesced in the action taken by the Secretary of the Interior, the the various departments, their personality is in reality but the projection of that of the President.
suspension of the petitioner should be sustained on the principle of approval or ratification of the Stated otherwise, and as forcibly characterized by Chief Justice Taft of the Supreme Court of the
act of the Secretary of the Interior by the President of the Philippines. There is, to be sure, more United States, "each head of a department is, and must be, the President's alter ego in the matters
weight in this argument than in the suggested generalization of section 37 of Act No. 4007. Withal, of that department where the President is required by law to exercise authority" (Myers vs. United
at first blush, the argument of ratification may seem plausible under the circumstances, it should be States, 47 Sup. Ct. Rep., 21 at 30; 272 U. S., 52 at 133; 71 Law. ed., 160). Secretaries of departments,
observed that there are certain prerogative acts which, by their very nature, cannot be validated by of course, exercise certain powers under the law but the law cannot impair or in any way affect the
subsequent approval or ratification by the President. There are certain constitutional power and constitutional power of control and direction of the President. As a matter of executive policy, they
prerogatives of the Chief Executive of the Nation which must be exercised by him in person and no may be granted departmental autonomy as to certain matters but this is by mere concession of the
amount of approval or ratification will validate the exercise of any of those powers by any other executive, in the absence of valid legislation in the particular field. If the President, then, is the
person. Such, for instance, is his power to suspend the writ of habeas corpus and proclaim martial authority in the Executive Department, he assumes the corresponding responsibility. The head of a
law (par. 3, sec. 11, Art. VII) and the exercise by him of the benign prerogative of mercy (par. 6, sec. department is a man of his confidence; he controls and directs his acts; he appoints him and can
11, idem). Upon the other hand, doubt is entertained by some members of the court whether the remove him at pleasure; he is the executive, not any of his secretaries. It is therefore logical that he,
statement made by the Secretary to the President in the latter's behalf and by his authority that the the President, should be answerable for the acts of administration of the entire Executive
President had no objection to the suspension of the petitioner could be accepted as an affirmative Department before his own conscience no less than before that undefined power of public opinion
exercise of the power of suspension in this case, or that the verbal approval by the President of the which, in the language of Daniel Webster, is the last repository of popular government. These are
suspension alleged in a pleading presented in this case by the Solicitor-General could be considered the necessary corollaries of the American presidential type of government, and if there is any defect,
as a sufficient ratification in law. it is attributable to the system itself. We cannot modify the system unless we modify the
Constitution, and we cannot modify the Constitution by any subtle process of judicial interpretation
After serious reflection, we have decided to sustain the contention of the government in this case or constitution.
on the board proposition, albeit not suggested, that under the presidential type of government
which we have adopted and considering the departmental organization established and continued The petition is hereby dismissed, with costs against the petitioner. So ordered.
in force by paragraph 1, section 12, Article VII, of our Constitution, all executive and administrative
organizations are adjuncts of the Executive Department, the heads of the various executive Avancea, C. J., Diaz, and Concepcion, JJ., concur.
departments are assistants and agents of the Chief Executive, and except in cases where the Chief
G.R. No. 196231 September 4, 2012 the President in OP-DC-Case No. 11-B-003, the administrative case initiated against petitioner as a
Special Prosecutor of the Office of the Ombudsman. The petition likewise seeks to declare as
EMILIO A. GONZALES III, Petitioner, unconstitutional Section 8(2) of R.A. No. 6770 giving the President the power to dismiss a Special
vs. Prosecutor of the Office of the Ombudsman.
OFFICE OF THE PRESIDENT OF THE PHILIPPINES, acting through and represented by EXECUTIVE
SECRETARY PAQUITO N. OCHOA, JR., SENIOR DEPUTY EXECUTIVE SECRETARY JOSE AMOR M. The facts from which these two cases separately took root are neither complicated nor unfamiliar.
AMORANDO, Officer in Charge, Office of the Deputy Executive Secretary for Legal Affairs, ATTY.
RONALDO A. GERON, DIR. ROWENA TURINGAN-SANCHEZ, and ATTY. CARLITOD. In the morning of August 23, 2010, news media scampered for a minute-by-minute coverage of a
CATAYONG, Respondents. hostage drama that had slowly unfolded right at the very heart of the City of Manila. While initial
news accounts were fragmented it was not difficult to piece together the story on the hostage-
x-----------------------x taker, Police Senior Inspector Rolando Mendoza. He was a disgruntled former police officer
attempting to secure his reinstatement in the police force and to restore the benefits of a life-long,
G.R. No. 196232 and erstwhile bemedaled, service. The following day, broadsheets and tabloids were replete with
stories not just of the deceased hostage-taker but also of the hostage victims, eight of whom died
during the bungled police operation to rescue the hapless innocents. Their tragic deaths triggered
WENDELL BARRERAS-SULIT, Petitioner, word wars of foreign relation proportions. One newspaper headline ran the story in detail, as
vs. follows:
ATTY. PAQUITO N. OCHOA, JR., in his capacity as EXECUTIVE SECRETARY, OFFICE OF THE
PRESIDENT, ATTY. DENNIS F. ORTIZ, ATTY. CARLO D.SULAY and ATTY. FROILAN MONTALBAN,
.JR., in their capacities as CHAIRMAN and MEMBERS of the OFFICE OF MALACAANG LEGAL MANILA, Philippines - A dismissed policeman armed with an assault rifle hijacked a bus packed
AFFAIRS, Respondents. with tourists, and killed most of its passengers in a 10 hour-hostage drama shown live on national
television until last night.

DECISION
Former police senior inspector Rolando Mendoza was shot dead by a sniper at past 9 p.m.
Mendoza hijacked the bus and took 21 Chinese tourists hostage, demanding his reinstatement to
PERLAS-BERNABE, J.: the police force.

The Case The hostage drama dragged on even after the driver of the bus managed to escape and told police
that all the remaining passengers had been killed.
These two petitions have been consolidated not because they stem from the same factual milieu
but because they raise a common thread of issues relating to the President's exercise of the power Late into the night assault forces surrounded the bus and tried to gain entry, but a pair of dead
to remove from office herein petitioners who claim the protective cloak of independence of the hostages hand-cuffed to the door made it difficult for them. Police said they fired at the wheels of
constitutionally-created office to which they belong - the Office of the Ombudsman. the bus to immobilize it.

The first case, docketed as G.R. No. 196231, is a Petition for Certiorari (with application for Police used hammers to smash windows, door and wind-shield but were met with intermittent fire
issuance of temporary restraining order or status quo order) which assails on jurisdictional grounds from the hos-tage taker.
the Decision1 dated March 31, 2011 rendered by the Office of the President in OP Case No. 10-J-
460 dismissing petitioner Emilio A. Gonzales III, Deputy Ombudsman for the Military and Other
Law Enforcement Offices (MOLEO), upon a finding of guilt on the administrative charges of Gross Police also used tear gas in an effort to confirm if the remaining hostages were all dead or alive.
Neglect of Duty and Grave Misconduct constituting a Betrayal of Public Trust. The petition When the standoff ended at nearly 9 p.m., some four hostages were rescued alive while Mendoza
primarily seeks to declare as unconstitutional Section 8(2) of Republic Act (R.A.) No. 6770, was killed by a sniper.
otherwise known as the Ombudsman Act of 1989, which gives the President the power to dismiss
a Deputy Ombudsman of the Office of the Ombudsman. Initial reports said some 30 policemen stormed the bus. Shots also rang out, sending bystanders
scampering for safety.
The second case, docketed as G.R. No. 196232, is a Petition for Certiorari and Prohibition (with
application for issuance of a temporary restraining order or status quo order) seeking to annul, It took the policemen almost two hours to assault the bus because gunfire reportedly rang out
reverse and set aside (1) the undated Order2 requiring petitioner Wendell Barreras-Sulit to submit from inside the bus.
a written explanation with respect to alleged acts or omissions constituting serious/grave offenses
in relation to the Plea Bargaining Agreement (PLEBARA) entered into with Major General Carlos F. Mendoza hijacked the tourist bus in the morning and took the tourists hostage.
Garcia; and (2) the April 7, 2011 Notice of Preliminary Investigation,3 both issued by the Office of
Mendoza, who claimed he was illegally dismissed from the police service, initially released nine of Internal Affairs Service and the Manila Regional Trial Court had already dismissed crim-inal cases
the hostages during the drama that began at 10 a.m. and played out live on national television. against him.

Live television footage showed Mendoza asking for food for those remaining in the bus, which was The hostage drama began when Mendoza flagged down the Hong Thai Travel Tourist bus (TVU-
delivered, and fuel to keep the air-conditioning going. The disgruntled former police officer was 799), pretend-ing to hitch a ride. Margarejo said the bus had just left Fort Santiago in Intramuros
reportedly armed with an M-16 rifle, a 9 mm pistol and two hand grenades. when Mendoza asked the driver to let him get on and ride to Quirino Grandstand. Upon reaching
the Quirino Grandstand, Mendoza an-nounced to the passengers that they would be taken
Mendoza posted a handwritten note on the windows of the bus, saying "big deal will start after 3 hostage. "Having worn his (police) uniform, of course there is no doubt that he already planned
p.m. today." Another sign stuck to another window said "3 p.m. today deadlock." the hostage taking," Margarejo said. - Sandy Araneta, Nestor Etolle, Delon Porcalla, Amanda
Fisher, Cecille Suerte Felipe, Christi-na Mendez, AP Grandstand Carnage, The Philippine Star,
Updated August 24, 2010 12:00 AM, Val Rodri-guez.4
Stressing his demand, Mendoza stuck a piece of paper with a handwritten message: "Big mistake
to correct a big wrong decision." A larger piece of paper on the front windshield was headed,
"Release final decision," apparently referring to the case that led to his dismissal from the police In a completely separate incident much earlier in time, more particularly in December of 2003, 28-
force. year-old Juan Paolo Garcia and 23-year-old Ian Carl Garcia were caught in the United States
smuggling $100,000 from Manila by concealing the cash in their luggage and making false
statements to US Customs Officers. The Garcia brothers pleaded guilty to bulk cash smuggling and
Negotiations dragged on even after Mendoza's self-imposed deadline. agreed to forfeit the amount in favor of the US Government in exchange for the dismissal of the
rest of the charges against them and for being sentenced to time served. Inevitably, however, an
Senior Police Officer 2 Gregorio Mendoza said his brother was upset over his dismissal from the investigation into the source of the smuggled currency conducted by US Federal Agents and the
police force. "His problem was he was unjustly removed from service. There was no due process, Philippine Government unraveled a scandal of military corruption and amassed wealth -- the boys'
no hearing, no com-plaint," Gregorio said. father, Retired Major General Carlos F. Garcia, former Chief Procurement Officer of the Armed
Forces, had accumulated more than 300 Million during his active military service. Plunder and
Last night, Gregorio was arrested by his colleagues on suspicions of being an accessory to his Anti-Money Laundering cases were eventually filed against Major General Garcia, his wife and
brother's action. Tensions rose as relatives tried to prevent lawmen from arresting Gregorio in their two sons before the Sandiganbayan.
front of national television. This triggered the crisis that eventually forced Mendoza to carry out his
threat and kill the remaining hostages. G.R. No. 196231

Negotiators led by Superintendent Orlando Yebra and Chief Inspector Romeo Salvador tried to talk Sometime in 2008, a formal charge5 for Grave Misconduct (robbery, grave threats, robbery
Mendoza into surrendering and releasing the 21 hostages, mostly children and three Filipinos, extortion and physical injuries) was filed before the Philippine National Police-National Capital
including the driver, the tourist guide and a photographer. Yebra reportedly lent a cellphone to Region (PNP-NCR) against Manila Police District Senior Inspector (P/S Insp.) Rolando Mendoza, and
allow communications with Mendoza in-side the bus, which was parked in front ofthe Quirino four others, namely, Police Inspector Nelson Lagasca, Senior Police Inspector I Nestor David, Police
Grandstand. Officer III Wilson Gavino, and Police Officer II Roderick Lopena. A similar charge was filed by the
private complainant, Christian M. Kalaw, before the Office of the City Prosecutor, Manila, docketed
Children could be seen peeking from the drawn curtains of the bus while police negotiators as I.S. No. 08E-09512.
hovered near the scene.
On July 24, 2008, while said cases were still pending, the Office of the Regional Director of the
Manila Police District (MPD) director Chief Superinten-dent Rodolfo Magtibay ordered the National Police Commission (NPC) turned over, upon the request of petitioner Emilio A. Gonzales
deployment of crack police teams and snipers near the scene. A crisis man-agement committee III, all relevant documents and evidence in relation to said case to the Office of the Deputy
had been activated with Manila Vice Mayor Isko Moreno coordinating the actions with the MPD. Ombudsman for appropriate administrative adjudication.6 Subsequently, Case No. OMB-P-A-08-
0670-H for Grave Misconduct was lodged against P/S Insp. Rolando Mendoza and his fellow police
officers, who filed their respective verified position papers as directed.
Earlier last night, Ombudsman Merceditas Gutierrez had a meeting with Moreno to discuss
Mendoza's case that led to his dismissal from the service. Ombudsman spokesman Jose de Jesus
said Gutierrez gave a "sealed letter" to Moreno to be delivered to Mendoza. De Jesus did not Meanwhile, on August 26, 2008, I.S. No. 08E-09512 was dismissed7 upon a finding that the material
elaborate on the contents of the letter but said Moreno was tasked to personally deliver the letter allegations made by the complainant had not been substantiated "by any evidence at all to
to Mendoza. warrant the indictment of respondents of the offenses charged." Similarly, the Internal Affairs
Service of the PNP issued a Resolution8 dated October 17, 2008 recommending the dismissal
without prejudice of the administrative case against the same police officers, for failure of the
MPD spokesman Chief Inspector Edwin Margarejo said Mendoza was apparently distraught by the complainant to appear in three (3) consecutive hearings despite due notice.
slow process of the Ombudsman in deciding his motion for reconside-ration. He said the PNP-
However, on February 16, 2009, upon the recommendation of petitioner Emilio Gonzales III, a More so, Mendoza's demand for immediate resolution of his motion for reconsideration is not
Decision9 in Case No. OMB-P-A-08-0670-H finding P/S Insp. Rolando Mendoza and his fellow police without legal and compelling bases considering the following:
officers guilty of Grave Misconduct was approved by the Ombudsman. The dispositive portion of
said Decision reads: (a) PSI Mendoza and four policemen were investigated by the Ombudsman
involving a case for alleged robbery (extortion), grave threats and physical
WHEREFORE, it is respectfully recommended that respondents P/S Insp. ROLANDO DEL ROSARIO injuries amounting to grave misconduct allegedly committed against a certain
MENDOZA and PO3 WILSON MATIC GAVINO of PRO-ARMM, Camp Brig. Gen. Salipada K. Christian Kalaw. The same case, however, was previously dismissed by the
Pendatun, Parang, Shariff Kabunsuan; P/INSP. NELSON URBANO LAGASCA, SPO1 NESTOR REYES Manila City Prosecutors Office for lack of probable cause and by the PNP-NCR
DAVID and PO2 RODERICK SALVA LOPEA of Manila Police District, Headquarters, United Nations Internal Affairs Service for failure of the complainant (Christian Kalaw) to
Avenue, Manila, be meted the penalty of DISMISSAL from the Service, pursuant to Section 52 (A), submit evidence and prosecute the case. On the other hand, the case which
Rule IV, Uniform Rules on Administrative Cases in the Civil Service, with the accessory penalties of was filed much ahead by Mendoza et al. against Christian Kalaw involving the
forfeiture of retirement benefits and perpetual disqualification from reemployment in the same incident, was given due course by the City Prosecutors Office.
government service pursuant to Section 58, Rule IV of the same Uniform Rules of Administrative
Cases in the Civil Service, for having committed GRAVE MISCONDUCT. (b) The Ombudsman exercised jurisdiction over the case based on a letter
issued motu proprio for Deputy Ombudsman Emilio A. Gonzalez III, directing
On November 5, 2009, they filed a Motion for Reconsideration10 of the foregoing Decision, the PNP-NCR - without citing any reason - to endorse the case against
followed by a Supplement to the Motion for Reconsideration11 on November 19, 2009. On Mendoza and the arresting policemen to his office for administrative
December 14, 2009, the pleadings mentioned and the records of the case were assigned for review adjudication, thereby showing undue interest on the case. He also caused the
and recommendation to Graft Investigation and Prosecutor Officer Dennis L. Garcia, who released docketing of the case and named Atty. Clarence V. Guinto of the PNP-CIDG-
a draft Order12 on April 5, 2010 for appropriate action by his immediate superior, Director Eulogio NCR, who indorsed the case records, as the nominal complainant, in lieu of
S. Cecilio, who, in turn, signed and forwarded said Order to petitioner Gonzalez's office on April 27, Christian Kalaw. During the proceedings, Christian Kalaw did not also affirm
2010. Not more than ten (10) days after, more particularly on May 6, 2010, petitioner endorsed his complaint-affidavit with the Ombudsman or submit any position paper as
the Order, together with the case records, for final approval by Ombudsman Merceditas N. required.
Gutierrez, in whose office it remained pending for final review and action when P/S Insp. Mendoza
hijacked a bus-load of foreign tourists on that fateful day of August 23, 2010 in a desperate (c) Subsequently, Mendoza, after serving preventive suspension, was
attempt to have himself reinstated in the police service. adjudged liable for grave misconduct by Deputy Ombudsman Gonzales (duly
approved on May 21, 2009) based on the sole and uncorroborated complaint-
In the aftermath of the hostage-taking incident, which ended in the tragic murder of eight affidavit of Christian Kalaw, which was not previously sustained by the City
HongKong Chinese nationals, the injury of seven others and the death of P/S Insp. Rolando Prosecutor's Office and the PNP Internal Affairs Service. From the said
Mendoza, a public outcry against the blundering of government officials prompted the creation of Resolution, Mendoza interposed a timely motion for reconsideration (dated
the Incident Investigation and Review Committee (IIRC),13 chaired by Justice Secretary Leila de Lima and filed November 5, 2009) as well as a supplement thereto. No opposition
and vice-chaired by Interior and Local Government Secretary Jesus Robredo. It was tasked to or comment was filed thereto.
determine accountability for the incident through the conduct of public hearings and executive
sessions. However, petitioner, as well as the Ombudsman herself, refused to participate in the IIRC (d) Despite the pending and unresolved motion for reconsideration, the
proceedings on the assertion that the Office of the Ombudsman is an independent constitutional judgment of dismissal was enforced, thereby abruptly ending Mendoza's 30
body. years of service in the PNP with forfeiture of all his benefits. As a result,
Mendoza sought urgent relief by sending several hand-written letter-requests
Sifting through testimonial and documentary evidence, the IIRC eventually identified petitioner to the Ombudsman for immediate resolution of his motion for
Gonzales to be among those in whom culpability must lie. In its Report,14 the IIRC made the reconsideration. But his requests fell on deaf ears.
following findings:
xxxx
Deputy Ombudsman Gonzales committed serious and inexcusable negligence and gross violation
of their own rules of procedure by allowing Mendoza's motion for reconsideration to languish for By allowing Mendoza's motion for reconsideration to languish for nine long (9) months without
more than nine (9) months without any justification, in violation of the Ombudsman prescribed any justification, Ombudsman Gutierrez and Deputy Ombudsman Gonzales committed complete
rules to resolve motions for reconsideration in administrative disciplinary cases within five (5) days and wanton violation of the Ombudsman prescribed rule to resolve motions for reconsideration in
from submission. The inaction is gross, considering there is no opposition thereto. The prolonged administrative disciplinary cases within five (5) days from submission (Sec. 8, Ombudsman Rules of
inaction precipitated the desperate resort to hostage-taking. Procedure). The inaction is gross, there being no opposition to the motion for reconsideration.
Besides, the Ombudsman, without first resolving the motion for reconsideration, arbitrarily The IIRC recommended that its findings with respect to petitioner Gonzales be referred to the
enforced the judgment of dismissal and ignored the intervening requests for immediate resolution, Office of the President (OP) for further determination of possible administrative offenses and for
thereby rendering the inaction even more inexcusable and unjust as to amount to gross negligence the initiation of the proper administrative proceedings.
and grave misconduct.
On October 15, 2010, the OP instituted a Formal Charge15 against petitioner Gonzales for Gross
SECOND, Ombudsman Gutierrez and Deputy Ombudsman Gonzales committed serious disregard Neglect of Duty and/or Inefficiency in the Performance of Official Duty under Rule XIV, Section 22
of due process, manifest injustice and oppression in failing to provisionally suspend the further of the Omnibus Rules Implementing Book V of E.O. No. 292 and other pertinent Civil
implementation of the judgment of dismissal against Mendoza pending disposition of his
unresolved motion for reconsideration. Service Laws, rules and regulations, and for Misconduct in Office under Section 3 of the Anti-Graft
and Corrupt Practices Act.16 Petitioner filed his Answer17 thereto in due time.
By enforcing the judgment of dismissal without resolving the motion for reconsideration for over
nine months, the two Ombudsman officials acted with arbitrariness and without regard to due Shortly after the filing by the OP of the administrative case against petitioner, a complaint dated
process and the constitutional right of an accused to the speedy disposition of his case. As long as October 29, 2010 was filed by Acting Assistant Ombudsman Joselito P. Fangon before the Internal
his motion for reconsideration remained pending and unresolved, Mendoza was also effectively Affairs Board of the Office of the Ombudsman charging petitioner with "directly or indirectly
deprived of the right to avail of the ordinary course of appeal or review to challenge the judgment requesting or receiving any gift, present, share, percentage, or benefit, for himself or for any other
of dismissal before the higher courts and seek a temporary restraining order to prevent the further person, in connection with any contract or transaction between the Government and any other
execution thereof. party, wherein the public officer in his official capacity has to intervene under the law" under
Section 3(b) of the Anti-Graft and Corrupt Practices Act, and also, with solicitation or acceptance of
As such, if the Ombudsman cannot resolve with dispatch the motion for reconsideration, it should gifts under Section 7(d) of the Code of Conduct and Ethical Standards.18 In a Joint
have provisionally suspended the further enforcement of the judgment of dismissal without Resolution19 dated February 17, 2011, which was approved by Ombudsman Ma. Merceditas N.
prejudice to its re-implementation if the reconsideration is eventually denied. Otherwise, the Gutierrez, the complaint was dismissed, as follows:
Ombudsman will benefit from its own inaction. Besides, the litigant is entitled to a stay of the
execution pending resolution of his motion for reconsideration. Until the motion for WHEREFORE, premises considered, finding no probable cause to indict respondent Emilio A.
reconsideration is denied, the adjudication process before the Ombudsman cannot be considered Gonzales III for violations of Section 3(b) of R.A. No. 3019 and Section 7(d) of R.A. No. 6713, the
as completely finished and, hence, the judgment is not yet ripe for execution. complaint is hereby be [sic] DISMISSED.

xxxx Further, finding no sufficient evidence to hold respondent administratively liable for Misconduct,
the same is likewise DISMISSED.
When the two Ombudsman officials received Mendoza's demand for the release of the final order
resolving his motion for reconsideration, they should have performed their duty by resolving the Meanwhile, the OP notified20 petitioner that a Preliminary Clarificatory Conference relative to the
reconsideration that same day since it was already pending for nine months and the prescribed administrative charge against him was to be conducted at the Office of the Deputy Executive
period for its resolution is only five days. Or if they cannot resolve it that same day, then they Secretary for Legal Affairs (ODESLA) on February 8, 2011. Petitioner Gonzales alleged,21 however,
should have acted decisively by issuing an order provisionally suspending the further enforcement that on February 4, 2011, he heard the news that the OP had announced his suspension for one
of the judgment of dismissal subject to revocation once the reconsideration is denied and without year due to his delay in the disposition of P/S Insp. Mendoza's motion for reconsideration. Hence,
prejudice to the arrest and prosecution of Mendoza for the hostage-taking. Had they done so, the believing that the OP had already prejudged his case and that any proceeding before it would
crisis may have ended peacefully, without necessarily compromising the integrity of the institution. simply be a charade, petitioner no longer attended the scheduled clarificatory conference. Instead,
After all, as relayed to the negotiators, Mendoza did express willingness to take full responsibility he filed an Objection to Proceedings22 on February 7, 2011. Despite petitioner's absence, however,
for the hostage-taking if his demand for release of the final decision or reinstatement was met. the OP pushed through with the proceedings and, on March 31, 2011, rendered the assailed
Decision,23 the dispositive portion of which reads:
But instead of acting decisively, the two Ombudsman officials merely offered to review a pending
motion for review of the case, thereby prolonging their inaction and aggravating the situation. As WHEREFORE, in view of the foregoing, this Office finds Deputy Ombudsman Emilio A. Gonzales III
expected, Mendoza - who previously berated Deputy Gonzales for allegedly demanding guilty of Gross Neglect of Duty and Grave Misconduct constituting betrayal of public trust, and
Php150,000 in exchange for favorably resolving the motion for reconsideration - rejected and hereby meted out the penalty of DISMISSAL from service.
branded as trash ("basura") the Ombudsman [sic] letter promising review, triggering the collapse
of the negotiations. To prevent the situation from getting out of hand, the negotiators sought the
alternative option of securing before the PNP-NCRPO an order for Mendoza's provisional SO ORDERED.
reinstatement pending resolution of the motion for reconsideration. Unfortunately, it was already
too late. But had the Ombudsman officials performed their duty under the law and acted Hence, the petition.
decisively, the entire crisis may have ended differently.
G.R. No. 196232 JURISDICTION WHEN IT CONDUCTED ITS INVESTIGATION AND RENDERED ITS DECISION IN
VIOLATION OF PETITIONER'S RIGHT TO DUE PROCESS.
In April of 2005, the Acting Deputy Special Prosecutor of the Office of the Ombudsman charged
Major General Carlos F. Garcia, his wife Clarita D. Garcia, their sons Ian Carl Garcia, Juan Paulo (C)
Garcia and Timothy Mark Garcia and several unknown persons with Plunder (Criminal Case No.
28107) and Money Laundering (Criminal Case No. SB09CRM0194) before the Sandiganbayan. RESPONDENT OFFICE OF THE PRESIDENT, ACTING THROUGH THE INDIVIDUAL RESPONDENTS,
GRAVELY ABUSED ITS DISCRETION AMOUNTING TO LACK OR EXCESS OF JURISDICTION IN FINDING
On January 7, 2010, the Sandiganbayan denied Major General Garcia's urgent petition for bail THAT PETITIONER COMMITTED DELAY IN THE DISPOSITION OF MENDOZA'S MOTION FOR
holding that strong prosecution evidence militated against the grant of bail. On March 16, 2010, RECONSIDERATION.
however, the government, represented by petitioner, Special Prosecutor Wendell Barreras-Sulit
("Barreras-Sulit") and her prosecutorial staff sought the Sandiganbayan's approval of a Plea (D)
Bargaining Agreement (hereinafter referred to as "PLEBARA") entered into with the accused. On
May 4, 2010, the Sandiganbayan issued a Resolution finding the change of plea warranted and the
PLEBARA compliant with jurisprudential guidelines. RESPONDENT OFFICE OF THE PRESIDENT, ACTING THROUGH THE INDIVIDUAL RESPONDENTS,
GRAVELY ABUSED ITS DISCRETION AMOUNTING TO LACK OR EXCESS OF JURISDICTION IN FINDING
THAT PETITIONER TOOK UNDUE INTEREST IN MENDOZA'S CASE.
Outraged by the backroom deal that could allow Major General Garcia to get off the hook with
nothing but a slap on the hand notwithstanding the prosecution's apparently strong evidence of
his culpability for serious public offenses, the House of Representatives' Committee on Justice (E)
conducted public hearings on the PLEBARA. At the conclusion of these public hearings, the
Committee on Justice passed and adopted Committee Resolution No. 3, 24recommending to the RESPONDENT OFFICE OF THE PRESIDENT, ACTING THROUGH THE INDIVIDUAL RESPONDENTS,
President the dismissal of petitioner Barreras-Sulit from the service and the filing of appropriate GRAVELY ABUSED ITS DISCRETION AMOUNTING TO LACK OR EXCESS OF JURISDICTION IN
charges against her Deputies and Assistants before the appropriate government office for having FAULTING PETITIONER FOR NOT RELEASING THE RESOLUTION ON MENDOZA'S MOTION FOR
committed acts and/or omissions tantamount to culpable violations of the Constitution and RECONSIDERATION OR FOR NOT SUSPENDING MENDOZA'S DISMISSAL FROM SERVICE DURING THE
betrayal of public trust, which are violations under the Anti-Graft and Corrupt Practices Act and HOSTAGE CRISIS.
grounds for removal from office under the Ombudsman Act.
(F)
The Office of the President initiated OP-DC-Case No. 11-B-003 against petitioner Barreras-Sulit. In
her written explanation, petitioner raised the defenses of prematurity and the lack of jurisdiction RESPONDENT OFFICE OF THE PRESIDENT, ACTING THROUGH THE INDIVIDUAL RESPONDENTS,
of the OP with respect to the administrative disciplinary proceeding against her. The OP, however, GRAVELY ABUSED ITS DISCRETION AMOUNTING TO LACK OR EXCESS OF JURISDICTION IN FINDING
still proceeded with the case, setting it for preliminary investigation on April 15, 2011. THAT THERE WAS SUBSTANTIAL EVIDENCE TO SHOW THAT PETITIONER DEMANDED A BRIBE FROM
MENDOZA.25
Hence, the petition.
On the other hand, in G.R. No. 196232, petitioner Barreras-Sulit poses for the Court the question -
The Issues
AS OF THIS POINT IN TIME, WOULD TAKING AND CONTINUING TO TAKE ADMINISTRATIVE
In G.R. No. 196231, petitioner Gonzales raises the following grounds, to wit: DISCIPLINARY PROCEEDING AGAINST PETITIONER BE LAWFUL AND JUSTIFIABLE?26

(A) Re-stated, the primordial question in these two petitions is whether the Office of the President has
jurisdiction to exercise administrative disciplinary power over a Deputy Ombudsman and a Special
RESPONDENT OFFICE OF THE PRESIDENT, ACTING THROUGH THE OTHER INDIVIDUAL Prosecutor who belong to the constitutionally-created Office of the Ombudsman.
RESPONDENTS, HAS NO CONSTITUTIONAL OR VALID STATUTORY AUTHORITY TO SUBJECT
PETITIONER TO AN ADMINISTRATIVE INVESTIGATION AND TO THEREAFTER ORDER HIS REMOVAL The Court's Ruling
AS DEPUTY OMBUDSMAN.
Short of claiming themselves immune from the ordinary means of removal, petitioners asseverate
(B) that the President has no disciplinary jurisdiction over them considering that the Office of the
Ombudsman to which they belong is clothed with constitutional independence and that they, as
RESPONDENT OFFICE OF THE PRESIDENT, ACTING THROUGH THE OTHER INDIVIDUAL Deputy Ombudsman and Special Prosecutor therein, necessarily bear the constitutional attributes
RESPONDENTS, GRAVELY ABUSED ITS DISCRETION AMOUNTING TO LACK OR EXCESS OF of said office.
The Court is not convinced. entered into by his office involving the disbursement or use of public funds or properties, and
report any irregularity to the Commission on Audit for appropriate action.
The Ombudsman's administrative disciplinary power over a Deputy Ombudsman and Special
Prosecutor is not exclusive. (5) Request any government agency for assistance and information necessary in the discharge of
its responsibilities, and to examine, if necessary, pertinent records and documents.
It is true that the authority of the Office of the Ombudsman to conduct administrative
investigations proceeds from its constitutional mandate to be an effective protector of the people (6) Publicize matters covered by its investigation when circumstances so warrant and with due
against inept and corrupt government officers and employees,27 and is subsumed under the broad prudence.
powers "explicitly conferred" upon it by the 1987 Constitution and R.A. No. 6770. 28
(7) Determine the causes of inefficiency, red tape, mismanagement, fraud, and corruption in the
The ombudsman traces its origins to the primitive legal order of Germanic tribes. The Swedish Government and make recommendations for their elimination and the observance of high
term, which literally means "agent" or "representative," communicates the concept that has been standards of ethics and efficiency.
carried on into the creation of the modern-day ombudsman, that is, someone who acts as a
neutral representative of ordinary citizens against government abuses. 29 This idea of a people's (8) Promulgate its rules of procedure and exercise such other powers or perform such functions
protector was first institutionalized in the Philippines under the 1973 Constitution with the or duties as may be provided by law.31
creation of the Tanodbayan, which wielded the twin powers of investigation and prosecution.
Section 6, Article XIII of the 1973 Constitution provided thus:
Congress thereafter passed, on November 17, 1989, Republic Act No. 6770, the Ombudsman Act of
1989, to shore up the Ombudsman's institutional strength by granting it "full administrative
Sec. 6. The Batasang Pambansa shall create an office of the Ombudsman, to be known as disciplinary power over public officials and employees,"32 as follows:
Tanodbayan, which shall receive and investigate complaints relative to public office, including
those in government-owned or controlled corporations, make appropriate recommendations, and
in case of failure of justice as defined by law, file and prosecute the corresponding criminal, civil, or Sec. 21. Officials Subject to Disciplinary Authority; Exceptions. - The Office of the Ombudsman shall
administrative case before the proper court or body. have disciplinary authority over all elective and appointive officials of the Government and its
subdivisions, instrumentalities and agencies, including Members of the Cabinet, local government,
government-owned or controlled corporations and their subsidiaries, except over officials who
The framers of the 1987 Constitution later envisioned a more effective ombudsman vested with may be removed only by impeachment or over Members of Congress, and the Judiciary.(Emphasis
authority to "act in a quick, inexpensive and effective manner on complaints against administrative supplied)
officials", and to function purely with the "prestige and persuasive powers of his office" in
correcting improprieties, inefficiencies and corruption in government freed from the hampering
effects of prosecutorial duties.30 Accordingly, Section 13, Article XI of the 1987 Constitution In the exercise of such full administrative disciplinary authority, the Office of the Ombudsman was
enumerates the following powers, functions, and duties of the Office of the Ombudsman, viz: explicitly conferred the statutory power to conduct administrative investigations under Section 19
of the same law, thus:

(1) Investigate on its own, or on complaint by any person, any act or omission of any public
official, employee, office or agency, when such act or omission appears to be illegal, unjust, Sec. 19. Administrative complaints. - The Ombudsman shall act on all complaints relating, but not
improper, or inefficient. limited, to acts or omissions which:

(2) Direct, upon complaint or at its own instance, any public official or employee of the 1. Are contrary to law or regulation;
Government, or any subdivision, agency or instrumentality thereof, as well as of any government-
owned or controlled corporation with original charter, to perform and expedite any act or duty 2. Are unreasonable, unfair, oppressive or discriminatory;
required by law, or to stop, prevent, and correct any abuse or impropriety in the performance of
duties. 3. Are inconsistent with the general course of an agency's functions, though in accordance with
law;
(3) Direct the officer concerned to take appropriate action against a public official or employee at
fault, and recommend his removal, suspension, demotion, fine, censure, or prosecution, and 4. Proceed from a mistake of law or an arbitrary ascertainment of facts;
ensure compliance therewith.

5. Are in the exercise of discretionary powers but for an improper purpose; or


(4) Direct the officer concerned, in any appropriate case, and subject to such limitations as may
be provided by law, to furnish it with copies of documents relating to contracts or transactions
6. Are otherwise irregular, immoral or devoid of justification.
While the Ombudsman's authority to discipline administratively is extensive and covers all Replying thereto, Senator Angara stated that originally, he was not averse to the proposal,
government officials, whether appointive or elective, with the exception only of those officials however, considering the Chair's observation that vesting such authority upon the Tanodbayan
removable by impeachment, the members of congress and the judiciary, such authority is by no itself could result in mutual protection, it is necessary that an outside official should be vested with
means exclusive. Petitioners cannot insist that they should be solely and directly subject to the such authority to effect a check and balance.35
disciplinary authority of the Ombudsman. For, while Section 21 declares the Ombudsman's
disciplinary authority over all government officials, Section 8(2), on the other hand, grants the Indubitably, the manifest intent of Congress in enacting both provisions - Section 8(2) and Section
President express power of removal over a Deputy Ombudsman and a Special Prosecutor. Thus: 21 - in the same Organic Act was to provide for an external authority, through the person of the
President, that would exercise the power of administrative discipline over the Deputy Ombudsman
Section 8. Removal; Filling of Vacancy.- and Special Prosecutor without in the least diminishing the constitutional and plenary authority of
the Ombudsman over all government officials and employees. Such legislative design is simply a
xxxx measure of "check and balance" intended to address the lawmakers' real and valid concern that
the Ombudsman and his Deputy may try to protect one another from administrative liabilities.

(2) A Deputy or the Special Prosecutor, may be removed from office by the President for any of the
grounds provided for the removal of the Ombudsman, and after due process. This would not be the first instance that the Office of the President has locked horns with the
Ombudsman on the matter of disciplinary jurisdiction. An earlier conflict had been settled in favor
of shared authority in Hagad v. Gozo Dadole.36 In said case, the Mayor and Vice-Mayor of Mandaue
It is a basic canon of statutory construction that in interpreting a statute, care should be taken that City, and a member of the Sangguniang Panlungsod, were charged before the Office of the Deputy
every part thereof be given effect, on the theory that it was enacted as an integrated measure and Ombudsman for the Visayas with violations of R.A. No. 3019, R.A. No. 6713, and the Revised Penal
not as a hodge-podge of conflicting provisions. A construction that would render a provision Code. The pivotal issue raised therein was whether the Ombudsman had been divested of his
inoperative should be avoided; instead, apparently inconsistent provisions should be reconciled authority to conduct administrative investigations over said local elective officials by virtue of the
whenever possible as parts of a coordinated and harmonious whole.33Otherwise stated, the law subsequent enactment of the Local Government Code of 1991 (R.A. No. 7160), the pertinent
must not be read in truncated parts. Every part thereof must be considered together with the provision of which states:
other parts, and kept subservient to the general intent of the whole enactment.34

Sec. 61. Form and Filing of Administrative Complaints.- A verified complaint against any erring local
A harmonious construction of these two apparently conflicting provisions in R.A. No. 6770 leads to elective official shall be prepared as follows:
the inevitable conclusion that Congress had intended the Ombudsman and the President to
exercise concurrent disciplinary jurisdiction over petitioners as Deputy Ombudsman and Special
(a) A complaint against any elective official of a province, a highly urbanized city, an independent
component city or component city shall be filed before the Office of the President.
Prosecutor, respectively. This sharing of authority goes into the wisdom of the legislature, which
prerogative falls beyond the pale of judicial inquiry. The Congressional deliberations on this matter
are quite insightful, viz: The Court resolved said issue in the negative, upholding the ratiocination of the Solicitor General
that R.A. No. 7160 should be viewed as having conferred on the Office of the President, but not on
an exclusive basis, disciplinary authority over local elective officials. Despite the fact that R.A. No.
x x x Senator Angara explained that the phrase was added to highlight the fact that the Deputy 7160 was the more recent expression of legislative will, no repeal of pertinent provisions in the
Tanodbayan may only be removed for cause and after due process. He added that the President Ombudsman Act was inferred therefrom. Thus said the Court:
alone has the power to remove the Deputy Tanodbayan.

Indeed, there is nothing in the Local Government Code to indicate that it has repealed, whether
Reacting thereto, Senator Guingona observed that this might impair the independence of the expressly or impliedly, the pertinent provisions of the Ombudsman Act. The two statutes on the
Tanodbayan and suggested that the procedural removal of the Deputy Tanodbayan...; and that he specific matter in question are not so inconsistent, let alone irreconcilable, as to compel us to only
can be removed not by the President but by the Ombudsman. uphold one and strike down the other. Well settled is the rule that repeals of laws by implication
are not favored, and that courts must generally assume their congruent application. The two laws
However, the Chair expressed apprehension that the Ombudsman and the Deputy Ombudsman must be absolutely incompatible, and a clear finding thereof must surface, before the inference of
may try to protect one another. The Chair suggested the substitution of the phrase "after due implied repeal may be drawn. The rule is expressed in the maxim, interpretare et concordare
process" with the words after due notice and hearing with the President as the ultimate authority. legibus est optimus interpretendi, i.e., every statute must be so interpreted and brought into
accord with other laws as to form a uniform system of jurisprudence. The fundament is that the
Senator Guingona contended, however, that the Constitution provides for an independent Office legislature should be presumed to have known the existing laws on the subject and not to have
of the Tanodbayan, and to allow the Executive to have disciplinary powers over the Tanodbayan enacted conflicting statutes. Hence, all doubts must be resolved against any implied repeal, and all
Deputies would be an encroachment on the independence of the Tanodbayan. efforts should be exerted in order to harmonize and give effect to all laws on the subject.37
While Hagad v. Gozo Dadole38 upheld the plenary power of the Office of the Ombudsman to While the removal of the Ombudsman himself is also expressly provided for in the Constitution,
discipline elective officials over the same disciplinary authority of the President under R.A. No. which is by impeachment under Section 244 of the same Article, there is, however, no
7160, the more recent case of the Office of the Ombudsman v. Delijero39 tempered the exercise by constitutional provision similarly dealing with the removal from office of a Deputy Ombudsman, or
the Ombudsman of such plenary power invoking Section 23(2)40 of R.A. No. 6770, which gives the a Special Prosecutor, for that matter. By enacting Section 8(2) of R.A. 6770, Congress simply filled a
Ombudsman the option to "refer certain complaints to the proper disciplinary authority for the gap in the law without running afoul of any provision in the Constitution or existing statutes. In
institution of appropriate administrative proceedings against erring public officers or employees." fact, the Constitution itself, under Section 2, authorizes Congress to provide for the removal of all
The Court underscored therein the clear legislative intent of imposing "a standard and a separate other public officers, including the Deputy Ombudsman and Special Prosecutor, who are not
set of procedural requirements in connection with administrative proceedings involving public subject to impeachment.
school teachers"41 with the enactment of R.A. No. 4670, otherwise known as "The Magna Carta for
Public School Teachers." It thus declared that, while the Ombudsman's administrative disciplinary That the Deputies of the Ombudsman were intentionally excluded from the enumeration of
authority over a public school teacher is concurrent with the proper investigating committee of the impeachable officials is clear from the following deliberations45 of the Constitutional Commission,
Department of Education, it would have been more prudent under the circumstances for the thus:
Ombudsman to have referred to the DECS the complaint against the public school teacher.

MR. REGALADO. Yes, thank you. On Section 10, regarding the Ombudsman, there has been
Unquestionably, the Ombudsman is possessed of jurisdiction to discipline his own people and concern aired by Commissioner Rodrigo about who will see to it that the Ombudsman will perform
mete out administrative sanctions upon them, including the extreme penalty of dismissal from the his duties because he is something like a guardian of the government. This recalls the statement of
service. However, it is equally without question that the President has concurrent authority with Juvenal that while the Ombudsman is the guardian of the people, "Quis custodiet ipsos custodies",
respect to removal from office of the Deputy Ombudsman and Special Prosecutor, albeit under who will guard the guardians? I understand here that the Ombudsman who has the rank of a
specified conditions. Considering the principles attending concurrence of jurisdiction where the chairman of a constitutional commission is also removable only by impeachment.
Office of the President was the first to initiate a case against petitioner Gonzales, prudence should
have prompted the Ombudsman to desist from proceeding separately against petitioner through
its Internal Affairs Board, and to defer instead to the President's assumption of authority, MR. ROMULO. That is the intention, Madam President.
especially when the administrative charge involved "demanding and soliciting a sum of money"
which constitutes either graft and corruption or bribery, both of which are grounds reserved for MR. REGALADO. Only the Ombudsman?
the President's exercise of his authority to remove a Deputy Ombudsman.
MR. MONSOD. Only the Ombudsman.
In any case, assuming that the Ombudsman's Internal Affairs Board properly conducted a
subsequent and parallel administrative action against petitioner, its earlier dismissal of the charge MR. REGALADO. So not his deputies, because I am concerned with the phrase "have the rank of".
of graft and corruption against petitioner could not have the effect of preventing the Office of the We know, for instance, that the City Fiscal of Manila has the rank of a justice of the Intermediate
President from proceeding against petitioner upon the same ground of graft and corruption. After Appellate Court, and yet he is not a part of the judiciary. So I think we should clarify that also and
all, the doctrine of res judicata applies only to judicial or quasi-judicial proceedings, not to the read our discussions into the Record for purposes of the Commission and the Committee.46
exercise of administrative powers.42 In Montemayor v. Bundalian,43 the Court sustained the
President's dismissal from service of a Regional Director of the Department of Public Works and
Highways (DPWH) who was found liable for unexplained wealth upon investigation by the now xxx
defunct Philippine Commission Against Graft and Corruption (PCAGC). The Court categorically
ruled therein that the prior dismissal by the Ombudsman of similar charges against said official did THE PRESIDENT. The purpose of the amendment of Commissioner Davide is not just to include the
not operate as res judicata in the PCAGC case. Ombudsman among those officials who have to be removed from office only onimpeachment. Is
that right?
By granting express statutory power to the President to remove a Deputy Ombudsman and a
Special Prosecutor, Congress merely filled an obvious gap in the law. MR. DAVIDE. Yes, Madam President.

Section 9, Article XI of the 1987 Constitution confers upon the President the power to appoint the MR. RODRIGO. Before we vote on the amendment, may I ask a question?
Ombudsman and his Deputies, viz:
THE PRESIDENT. Commissioner Rodrigo is recognized.
Section 9. The Ombudsman and his Deputies shall be appointed by the President from a list of at
least six nominees prepared by the Judicial and Bar Council, and from a list of three nominees for
MR. RODRIGO. The Ombudsman, is this only one man?
every vacancy thereafter. Such appointments shall require no confirmation. All vacancies shall be
filled within three months after they occur.
MR. DAVIDE. Only one man.
MR. RODRIGO. Not including his deputies. This proposed amendment merely seeks to extend the office of the Ombudsman to the military
establishment, just as it champions the common people against bureaucratic indifference. The
MR. MONSOD. No.47 (Emphasis supplied) Ombudsman can designate a deputy to help the ordinary foot soldier get through with his
grievance to higher authorities. This deputy will, of course work in close cooperation with the
Minister of National Defense because of the necessity to maintain the integrity of the chain of
The Power of the President to Remove a Deputy Ombudsman and a Special Prosecutor is Implied command. Ordinary soldiers, when they know they can turn to a military Ombudsman for their
from his Power to Appoint. complaints, may not have to fall back on their own informal devices to obtain redress for their
grievances. The Ombudsman will help raise troop morale in accordance with a major professed
Under the doctrine of implication, the power to appoint carries with it the power to remove.48 As a goal of the President and the military authorities themselves. x x x
general rule, therefore, all officers appointed by the President are also removable by him. 49 The
exception to this is when the law expressly provides otherwise - that is, when the power to remove The add-on now forms part of Section 5, Article XI which reads as follows:
is expressly vested in an office or authority other than the appointing power. In some cases, the
Constitution expressly separates the power to remove from the President's power to appoint.
Under Section 9, Article VIII of the 1987 Constitution, the Members of the Supreme Court and Section 5. There is hereby created the independent Office of the Ombudsman, composed of the
judges of lower courts shall be appointed by the President. However, Members of the Supreme Ombudsman to be known as Tanodbayan, one over-all Deputy and at least one Deputy each for
Court may be removed after impeachment proceedings initiated by Congress (Section 2, Article XI), Luzon, Visayas and Mindanao. A separate deputy for the military establishment shall likewise be
while judges of lower courts may be removed only by the Supreme Court by virtue of its appointed.(Emphasis supplied)
administrative supervision over all its personnel (Sections 6 and 11, Article VIII). The Chairpersons
and Commissioners of the Civil Service Commission Section 1(2), Article IX(B), the Commission on The integrity and effectiveness of the Deputy Ombudsman for the MOLEO as a military watchdog
Elections Section 1(2), Article IX(C), and the Commission on Audit Section 1(2), Article IX(D) shall looking into abuses and irregularities that affect the general morale and professionalism in the
likewise be appointed by the President, but they may be removed only by impeachment (Section military is certainly of primordial importance in relation to the President's own role asCommander-
2, Article XI). As priorly stated, the Ombudsman himself shall be appointed by the President in-Chief of the Armed Forces. It would not be incongruous for Congress, therefore, to grant the
(Section 9, Article XI) but may also be removed only by impeachment (Section 2, Article XI). President concurrent disciplinary authority over the Deputy Ombudsman for the military and other
law enforcement offices.
In giving the President the power to remove a Deputy Ombudsman and Special Prosecutor,
Congress simply laid down in express terms an authority that is already implied from the Granting the President the Power to Remove a Deputy Ombudsman does not Diminish the
President's constitutional authority to appoint the aforesaid officials in the Office of the Independence of the Office of the Ombudsman.
Ombudsman.
The claim that Section 8(2) of R.A. No. 6770 granting the President the power to remove a Deputy
The Office of the Ombudsman is charged with monumental tasks that have been generally Ombudsman from office totally frustrates, if not resultantly negates the independence of the
categorized into investigatory power, prosecutorial power, public assistance, authority to inquire Office of the Ombudsman is tenuous. The independence which the Office of the Ombudsman is
and obtain information and the function to adopt, institute and implement preventive vested with was intended to free it from political considerations in pursuing its constitutional
measures.50 In order to ensure the effectiveness of his constitutional role, the Ombudsman was mandate to be a protector of the people. What the Constitution secures for the Office of the
provided with an over-all deputy as well as a deputy each for Luzon, Visayas and Mindanao. Ombudsman is, essentially, political independence. This means nothing more than that "the terms
However, well into the deliberations of the Constitutional Commission, a provision for the of office, the salary, the appointments and discipline of all persons under the office" are
appointment of a separate deputy for the military establishment was necessitated by "reasonably insulated from the whims of politicians."52 And so it was that Section 5, Article XI of
Commissioner Ople's lament against the rise within the armed forces of "fraternal associations the 1987 Constitution had declared the creation of the independent Office of the Ombudsman,
outside the chain of command" which have become the common soldiers' "informal grievance composed of the Ombudsman and his Deputies, who are described as "protectors of the people"
machinery" against injustice, corruption and neglect in the uniformed service, 51 thus: and constitutionally mandated to act promptly on complaints filed in any form or manner against
public officials or employees of the Government Section 12, Article XI. Pertinent provisions under
In our own Philippine Armed Forces, there has arisen in recent years a type of fraternal association Article XI prescribes a term of office of seven years without reappointment Section 11, prohibits a
outside the chain of command proposing reformist objectives. They constitute, in fact, an informal decrease in salaries during the term of office Section 10, provides strict qualifications for the office
grievance machinery against injustices to the rank and file soldiery and perceive graft in higher Section 8, grants fiscal autonomy Section 14 and ensures the exercise of constitutional functions
rank and neglect of the needs of troops in combat zones. The Reform the Armed Forces Movement Section 12 and 13. The cloak of independence is meant to build up the Office of the Ombudsman's
of RAM has kept precincts for pushing logistics to the field, the implied accusation being that most institutional strength to effectively function as official critic, mobilizer of government,
of the resources are used up in Manila instead of sent to soldiers in the field. The Guardians, the El constitutional watchdog53 and protector of the people. It certainly cannot be made to extend to
Diablo and other organizations dominated by enlisted men function, more or less, as grievance wrongdoings and permit the unbridled acts of its officials to escape administrative discipline.
collectors and as mutual aid societies.
Being aware of the constitutional imperative of shielding the Office of the Ombudsman from
political influences and the discretionary acts of the executive, Congress laid down two restrictions
on the President's exercise of such power of removal over a Deputy Ombudsman, namely: (1) that to elucidate upon his pleaded defenses by adamantly refusing to attend the scheduled Clarificatory
the removal of the Deputy Ombudsman must be for any of the grounds provided for the removal Conference despite notice. The OP recounted as follows -
of the Ombudsman and (2) that there must be observance of due process. Reiterating the grounds
for impeachment laid down in Section 2, Article XI of the 1987 Constitution, paragraph 1 of Section It bears noting that respondent Deputy Ombudsman Gonzalez was given two separate
8 of R.A. No. 6770 states that the Deputy Ombudsman may be removed from office for the same opportunities to explain his side and answer the Formal Charge against him.
grounds that the Ombudsman may be removed through impeachment, namely, "culpable violation
of the Constitution, treason, bribery, graft and corruption, other high crimes, or betrayal of public
trust." Thus, it cannot be rightly said that giving the President the power to remove a Deputy In the first instance, respondent was given the opportunity to submit his answer together with his
Ombudsman, or a Special Prosecutor for that matter, would diminish or compromise the documentary evidence, which opportunity respondent actually availed of. In the second instance,
constitutional independence of the Office of the Ombudsman. It is, precisely, a measure of this Office called a Clarificatory Conference on 8 February 2011 pursuant to respondent's express
protection of the independence of the Ombudsman's Deputies and Special Prosecutor in the election of a formal investigation. Despite due notice, however, respondent Deputy Ombudsman
discharge of their duties that their removal can only be had on grounds provided by law. refused to appear for said conference, interposing an objection based on the unfounded notion
that this Office has prejudged the instant case. Respondent having been given actual and
reasonable opportunity to explain or defend himself in due course, the requirement of due
In Espinosa v. Office of the Ombudsman,54 the Court elucidated on the nature of the Ombudsman's process has been satisfied.59
independence in this wise -

In administrative proceedings, the quantum of proof necessary for a finding of guilt is substantial
The prosecution of offenses committed by public officers is vested in the Office of the evidence,60 which is more than a mere scintilla and means such relevant evidence as a reasonable
Ombudsman. To insulate the Office from outside pressure and improper influence, the mind might accept as adequate to support a conclusion.61 The fact, therefore, that petitioner later
Constitution as well as RA 6770 has endowed it with a wide latitude of investigatory and refused to participate in the hearings before the OP is not a hindrance to a finding of his culpability
prosecutory powers virtually free from legislative, executive or judicial intervention. This Court based on substantial evidence, which only requires that a decision must "have something upon
consistently refrains from interfering with the exercise of its powers, and respects the initiative which it is based."62
and independence inherent in the Ombudsman who, 'beholden to no one, acts as the champion of
the people and the preserver of the integrity of public service.
Factual findings of administrative bodies are controlling when supported by substantial
evidence.63 The OP's pronouncement of administrative accountability against petitioner and the
Petitioner Gonzales may not be removed from office where the questioned acts, falling short of imposition upon him of the corresponding penalty of removal from office was based on the finding
constitutional standards, do not constitute betrayal of public trust. of gross neglect of duty and grave misconduct in office amounting to a betrayal of public trust,
which is a constitutional ground for the removal by impeachment of the Ombudsman (Section 2,
Having now settled the question concerning the validity of the President's power to remove the Article XI, 1987 Constitution), and a statutory ground for the President to remove from office a
Deputy Ombudsman and Special Prosecutor, we now go to the substance of the administrative Deputy Ombudsman and a Special Prosecutor Section 8(2) of the Ombudsman Act.
findings in OP Case No. 10-J-460 which led to the dismissal of herein petitioner, Deputy
Ombudsman Emilio A. Gonzales, III. The OP held that petitioner's want of care and wrongful conduct consisted of his unexplained
action in directing the PNP-NCR to elevate P/S Insp. Mendoza's case records to his office; his
At the outset, the Court finds no cause for petitioner Gonzales to complain simply because the OP failure to verify the basis for requesting the Ombudsman to take over the case; his pronouncement
proceeded with the administrative case against him despite his non-attendance thereat. Petitioner of administrative liability and imposition of the extreme penalty of dismissal on P/S Insp. Mendoza
was admittedly able to file an Answer in which he had interposed his defenses to the formal based upon an unverified complaint-affidavit; his inordinate haste in implementing P/S Insp.
charge against him. Due process is satisfied when a person is notified of the charge against him Mendoza's dismissal notwithstanding the latter's non-receipt of his copy of the Decision and the
and given an opportunity to explain or defend himself. In administrative proceedings, the filing of subsequent filing of a motion for reconsideration; and his apparent unconcern that the pendency
charges and giving reasonable opportunity for the person so charged to answer the accusations of the motion for reconsideration for more than five months had deprived P/S Insp. Mendoza of
against him constitute the minimum requirements of due process. 55 Due process is simply having available remedies against the immediate implementation of the Decision dismissing him from the
the opportunity to explain one's side, or an opportunity to seek a reconsideration of the action or service.
ruling complained of.56
Thus, taking into consideration the factual determinations of the IIRC, the allegations and evidence
The essence of due process is that a party is afforded reasonable opportunity to be heard and to of petitioner in his Answer as well as other documentary evidence, the OP concluded that: (1)
submit any evidence he may have in support of his defense.57 Mere opportunity to be heard is petitioner failed to supervise his subordinates to act with dispatch on the draft resolution of P/S
sufficient. As long as petitioner was given the opportunity to explain his side and present evidence, Insp. Mendoza's motion for reconsideration and thereby caused undue prejudice to P/S Insp.
the requirements of due process are satisfactorily complied with because what the law abhors is Mendoza by effectively depriving the latter of the right to challenge the dismissal before the courts
an absolute lack of opportunity to be heard.58 Besides, petitioner only has himself to blame for and prevent its immediate execution, and (2) petitioner showed undue interest by having P/S Insp.
limiting his defense through the filing of an Answer. He had squandered a subsequent opportunity Mendoza's case endorsed to the Office of the Ombudsman and resolving the same against P/S
Insp. Mendoza on the basis of the unverified complaint-affidavit of the alleged victim Christian MR. NOLLEDO: x x x I think we will miss a golden opportunity if we fail to adopt the words
Kalaw. "betrayal of public trust" in the 1986 Constitution. But I would like him to know that we are
amenable to any possible amendment. Besides, I think plain error of judgment, where
The invariable rule is that administrative decisions in matters within the executive jurisdiction can circumstances may indicate that there is good faith, to my mind, will not constitute betrayal of
only be set aside on proof of gross abuse of discretion, fraud, or error of law.64 In the instant case, public trust if that statement will allay the fears of difficulty in interpreting the term."68 (Emphasis
while the evidence may show some amount of wrongdoing on the part of petitioner, the Court supplied)
seriously doubts the correctness of the OP's conclusion that the imputed acts amount to gross
neglect of duty and grave misconduct constitutive of betrayal of public trust. To say that The Constitutional Commission eventually found it reasonably acceptable for the phrase betrayal
petitioner's offenses, as they factually appear, weigh heavily enough to constitute betrayal of of public trust to refer to "acts which are just short of being criminal but constitute gross
public trust would be to ignore the significance of the legislature's intent in prescribing the faithlessness against public trust, tyrannical abuse of power, inexcusable negligence of duty,
removal of the Deputy Ombudsman or the Special Prosecutor for causes that, theretofore, had favoritism, and gross exercise of discretionary powers."69 In other words, acts that should
been reserved only for the most serious violations that justify the removal by impeachment of the constitute betrayal of public trust as to warrant removal from office may be less than criminal but
highest officials of the land. must be attended by bad faith and of such gravity and seriousness as the other grounds for
impeachment.
Would every negligent act or misconduct in the performance of a Deputy Ombudsman's duties
constitute betrayal of public trust warranting immediate removal from office? The question calls A Deputy Ombudsman and a Special Prosecutor are not impeachable officers. However, by
for a deeper, circumspective look at the nature of the grounds for the removal of a Deputy providing for their removal from office on the same grounds as removal by impeachment, the
Ombudsman and a Special Prosecutor vis-a-vis common administrative offenses. legislature could not have intended to redefine constitutional standards of culpable violation of
the Constitution, treason, bribery, graft and corruption, other high crimes, as well as betrayal of
Betrayal of public trust is a new ground for impeachment under the 1987 Constitution added to public trust, and apply them less stringently. Hence, where betrayal of public trust, for purposes of
the existing grounds of culpable violation of the Constitution, treason, bribery, graft and impeachment, was not intended to cover all kinds of official wrongdoing and plain errors of
corruption and other high crimes. While it was deemed broad enough to cover any violation of the judgment, this should remain true even for purposes of removing a Deputy Ombudsman and
oath of office,65 the impreciseness of its definition also created apprehension that "such an Special Prosecutor from office. Hence, the fact that the grounds for impeachment have been made
overarching standard may be too broad and may be subject to abuse and arbitrary exercise by the statutory grounds for the removal by the President of a Deputy Ombudsman and Special
legislature."66 Indeed, the catch-all phrase betrayal of public trust that referred to "all acts not Prosecutor cannot diminish the seriousness of their nature nor the acuity of their scope. Betrayal
punishable by statutes as penal offenses but, nonetheless, render the officer unfit to continue in of public trust could not suddenly "overreach" to cover acts that are not vicious or malevolent on
office"67 could be easily utilized for every conceivable misconduct or negligence in office. However, the same level as the other grounds for impeachment.
deliberating on some workable standard by which the ground could be reasonably interpreted, the
Constitutional Commission recognized that human error and good faith precluded an adverse The tragic hostage-taking incident was the result of a confluence of several unfortunate events
conclusion. including system failure of government response. It cannot be solely attributed then to what
petitioner Gonzales may have negligently failed to do for the quick, fair and complete resolution of
MR. VILLACORTA: x x x One last matter with respect to the use of the words "betrayal of public the case, or to his error of judgment in the disposition thereof. Neither should petitioner's official
trust" as embodying a ground for impeachment that has been raised by the Honorable Regalado. I acts in the resolution of P/S Insp. Mendoza's case be judged based upon the resulting deaths at the
am not a lawyer so I can anticipate the difficulties that a layman may encounter in understanding Quirino Grandstand. The failure to immediately act upon a party's requests for an early resolution
this provision and also the possible abuses that the legislature can commit in interpreting this of his case is not, by itself, gross neglect of duty amounting to betrayal of public trust. Records
phrase. It is to be noted that this ground was also suggested in the 1971 Constitutional show that petitioner took considerably less time to act upon the draft resolution after the same
Convention. A review of the Journals of that Convention will show that it was not included; it was was submitted for his appropriate action compared to the length of time that said draft remained
construed as encompassing acts which are just short of being criminal but constitute gross pending and unacted upon in the Office of Ombudsman Merceditas N. Gutierrez. He reviewed and
faithlessness against public trust, tyrannical abuse of power, inexcusable negligence of duty, denied P/S Insp. Mendoza's motion for reconsideration within nine (9) calendar days reckoned
favoritism, and gross exercise of discretionary powers. I understand from the earlier discussions from the time the draft resolution was submitted to him on April 27, 2010 until he forwarded his
that these constitute violations of the oath of office, and also I heard the Honorable Davide say recommendation to the Office of Ombudsman Gutierrez on May 6, 2010 for the latter's final
that even the criminal acts that were enumerated in the earlier 1973 provision on this matter action. Clearly, the release of any final order on the case was no longer in his hands.
constitute betrayal of public trust as well. In order to avoid confusion, would it not be clearer to
stick to the wording of Section 2 which reads: "may be removed from office on impeachment for Even if there was inordinate delay in the resolution of P/S Insp. Mendoza's motion and an
and conviction of, culpable violation of the Constitution, treason, bribery, and other high crimes, unexplained failure on petitioner's part to supervise his subordinates in its prompt disposition, the
graft and corruption or VIOLATION OF HIS OATH OF OFFICE", because if betrayal of public trust same cannot be considered a vicious and malevolent act warranting his removal for betrayal of
encompasses the earlier acts that were enumerated, then it would behoove us to be equally clear public trust. More so because the neglect imputed upon petitioner appears to be an isolated case.
about this last provision or phrase.
Similarly, petitioner's act of directing the PNP-IAS to endorse P/S Insp. Mendoza's case to the
Ombudsman without citing any reason therefor cannot, by itself, be considered a manifestation of
his undue interest in the case that would amount to wrongful or unlawful conduct. After all, taking The argument will not hold water. The incidents that have taken place subsequent to the
cognizance of cases upon the request of concerned agencies or private parties is part and parcel of submission in court of the PLEBARA shows that the PLEBARA has been practically approved, and
the constitutional mandate of the Office of the Ombudsman to be the "champion of the people." that the only thing which remains to be done by the Sandiganbayan is to promulgate a judgment
The factual circumstances that the case was turned over to the Office of the Ombudsman upon imposing the proper sentence on the accused Major General Garcia based on his new pleas to
petitioner's request; that administrative liability was pronounced against P/S Insp. Mendoza even lesser offenses. On May 4, 2010, the Sandiganbayan issued a resolution declaring that the change
without the private complainant verifying the truth of his statements; that the decision was of plea under the PLEBARA was warranted and that it complied with jurisprudential guidelines. The
immediately implemented; or that the motion for reconsideration thereof remained pending for Sandiganbayan, thereafter, directed the accused Major General Garcia to immediately convey in
more than nine months cannot be simply taken as evidence of petitioner's undue interest in the favor of the State all the properties, both real and personal, enumerated therein. On August 11,
case considering the lack of evidence of any personal grudge, social ties or business affiliation with 2010, the Sandiganbayan issued a resolution, which, in order to put into effect the reversion of
any of the parties to the case that could have impelled him to act as he did. There was likewise no Major General Garcia's ill-gotten properties, ordered the corresponding government agencies to
evidence at all of any bribery that took place, or of any corrupt intention or questionable cause the transfer of ownership of said properties to the Republic of the Philippines. In the
motivation. meantime, the Office of the Special Prosecutor (OSP) informed the Sandiganbayan that an
Order70 had been issued by the Regional Trial Court of Manila, Branch 21 on November 5, 2010
Accordingly, the OP's pronouncement of administrative accountability against petitioner and the allowing the transfer of the accused's frozen accounts to the Republic of the Philippines pursuant
imposition upon him of the corresponding penalty of dismissal must be reversed and set aside, as to the terms of the PLEBARA as approved by the Sandiganbayan. Immediately after the OSP
the findings of neglect of duty or misconduct in office do not amount to a betrayal of public trust. informed the Sandiganbayan that its May 4, 2010 Resolution had been substantially complied with,
Hence, the President, while he may be vested with authority, cannot order the removal of Major General Garcia manifested71 to the Sandiganbayan on November 19, 2010 his readiness for
petitioner as Deputy Ombudsman, there being no intentional wrongdoing of the grave and serious sentencing and for the withdrawal of the criminal information against his wife and two sons. Major
kind amounting to a betrayal of public trust. General Garcia's Motion to Dismiss,72 dated December 16, 2010 and filed with the Sandiganbayan,
reads:

This is not to say, however, that petitioner is relieved of all liability for his acts showing less than
diligent performance of official duties. Although the administrative acts imputed to petitioner fall 1.0 The Co-Accused were impleaded under the theory of conspiracy with the Principal Accused
short of the constitutional standard of betrayal of public trust, considering the OP's factual findings MGen. Carlos F. Garcia (AFP Ret.), (Principal Accused) with the allegation that the act of one is the
of negligence and misconduct against petitioner, the Court deems it appropriate to refer the case act of the others. Therefore, with the approval by the Honorable Court of the Plea Bargaining
to the Office of the Ombudsman for further investigation of the charges in OP Case No. 10-J-460 Agreement executed by the Principal Accused, the charges against the Co-Accused should likewise
and the imposition of the corresponding administrative sanctions, if any. be dismissed since the charges against them are anchored on the same charges against the
Principal Accused.

Inasmuch as there is as yet no existing ground justifying his removal from office, petitioner is
entitled to reinstatement to his former position as Deputy Ombudsman and to the payment of On December 16, 2010, the Sandiganbayan allowed accused Major General Garcia to plead guilty
backwages and benefits corresponding to the period of his suspension. to the lesser offenses of direct bribery and violation of Section 4(b), R.A. No. 9160, as amended.
Upon Major General Garcia's motion, and with the express conformity of the OSP, the
Sandiganbayan allowed him to post bail in both cases, each at a measly amount of 30,000.00.
The Office of the President is vested with statutory authority to proceed administratively against
petitioner Barreras-Sulit to determine the existence of any of the grounds for her removal from
office as provided for under the Constitution and the Ombudsman Act. The approval or disapproval of the PLEBARA by the Sandiganbayan is of no consequence to an
administrative finding of liability against petitioner Barreras-Sulit. While the court's determination
of the propriety of a plea bargain is on the basis of the existing prosecution evidence on record,
Petitioner Barreras-Sulit, on the other hand, has been resisting the President's authority to remove the disciplinary authority's determination of the prosecutor's administrative liability is based on
her from office upon the averment that without the Sandiganbayan's final approval and judgment whether the plea bargain is consistent with the conscientious consideration of the government's
on the basis of the PLEBARA, it would be premature to charge her with acts and/or omissions best interest and the diligent and efficient performance by the prosecution of its public duty to
"tantamount to culpable violations of the Constitution and betrayal of public trust," which are prosecute crimes against the State. Consequently, the disciplining authority's finding of ineptitude,
grounds for removal from office under Section 8, paragraph (2) of the Ombudsman Act of 1989; neglect or willfulness on the part of the prosecution, more particularly petitioner Special
and which also constitute a violation of Section 3, paragraph (e) of Republic Act No. 3019 (Anti- Prosecutor Barreras-Sulit, in failing to pursue or build a strong case for the government or, in this
Graft and Corrupt Practices Act) - causing undue injury to the Government or giving any private case, entering into an agreement which the government finds "grossly disadvantageous," could
party any unwarranted benefits, advantage or preference through manifest partiality, evident bad result in administrative liability, notwithstanding court approval of the plea bargaining agreement
faith or gross inexcusable negligence. With reference to the doctrine of prejudicial procedural entered into.
antecedent, petitioner Barreras-Sulit asserts that the propriety of taking and continuing to take
administrative disciplinary proceeding against her must depend on the final disposition by the
Sandiganbayan of the PLEBARA, explaining that if the Sandiganbayan would uphold the PLEBARA, Plea bargaining is a process in criminal cases whereby the accused and the prosecution work out a
there would no longer be any cause of complaint against her; if not, then the situation becomes mutually satisfactory disposition of the case subject to court approval.73 The essence of a plea
ripe for the determination of her failings. bargaining agreement is the allowance of an accused to plead guilty to a lesser offense than that
charged against him. Section 2, Rule 116 of the Revised Rules of Criminal Procedure provides the 11-B-003 against Special Prosecutor Wendell Barreras-Sulit for alleged acts and omissions
procedure therefor, to wit: tantamount to culpable violation of the Constitution and a betrayal of public trust, in accordance
with Section 8(2) of the Ombudsman Act of 1989.
SEC. 2. Plea of guilty to a lesser offense. -- At arraignment, the accused, with the consent of the
offended party and the prosecutor, may be allowed by the trial court to plead guilty to a lesser The challenge to the constitutionality of Section 8(2) of the Ombudsman Act is hereby DENIED.
offense which is necessarily included in the offense charged. After arraignment but before trial,
the accused may still be allowed to plead guilty to said lesser offense after withdrawing his plea of SO ORDERED.
not guilty. No amendment of the complaint or information is necessary. (Sec. 4, Cir. 38-98)

Plea bargaining is allowable when the prosecution does not have sufficient evidence to establish
the guilt of the accused of the crime charged.74 However, if the basis for the allowance of a plea
bargain in this case is the evidence on record, then it is significant to state that in its earlier
Resolution75 promulgated on January 7, 2010, the Sandiganbayan had evaluated the testimonies of
twenty (20) prosecution witnesses and declared that "the conglomeration of evidence presented
by the prosecution is viewed by the Court to be of strong character that militates against the grant
of bail."

Notwithstanding this earlier ruling by the Sandiganbayan, the OSP, unexplainably, chose to plea
bargain with the accused Major General Garcia as if its evidence were suddenly insufficient to
secure a conviction. At this juncture, it is not amiss to emphasize that the "standard of strong
evidence of guilt which is sufficient to deny bail to an accused is markedly higher than the standard
of judicial probable cause which is sufficient to initiate a criminal case."76Hence, in light of the
apparently strong case against accused Major General Garcia, the disciplining authority would be
hard-pressed not to look into the whys and wherefores of the prosecution's turnabout in the case.

The Court need not touch further upon the substantial matters that are the subject of the pending
administrative proceeding against petitioner Barreras-Sulit and are, thus, better left to the
complete and effective resolution of the administrative case before the Office of the President.

The challenge to the constitutionality of Section 8(2) of the Ombudsman Act has, nonetheless,
failed to obtain the necessary votes to invalidate the law, thus, keeping said provision part of the
law of the land. To recall, these cases involve two distinct issues: (a) the constitutionality of Section
8(2) of the Ombudsman Act; and (b) the validity of the administrative action of removal taken
against petitioner Gonzales. While the Court voted unanimously to reverse the decision of the OP
removing petitioner Gonzales from office, it was equally divided in its opinion on the
constitutionality of the assailed statutory provision in its two deliberations held on April 17, 2012
and September 4, 2012. There being no majority vote to invalidate the law, the Court, therefore,
dismisses the challenge to the constitutionality of Section 8(2) of the Ombudsman Act in
accordance with Section 2(d), Rule 12 of the Internal Rules of the Court.

Indeed, Section 4(2), Article VIII of the 1987 Constitution requires the vote of the majority of the
Members of the Court actually taking part in the deliberation to sustain any challenge to the
constitutionality or validity of a statute or any of its provisions.

WHEREFORE, in G.R. No. 196231, the decision of the Office of the President in OP Case No. 10-J-
460 isREVERSED and SET ASIDE. Petitioner Emilio A. Gonzales III is ordered REINSTATED with
payment of backwages corresponding to the period of suspension effective immediately, even as
the Office of the Ombudsman is directed to proceed with the investigation in connection with the
above case against petitioner. In G.R. No. 196232, We AFFIRM the continuation of OP-DC Case No.
G.R. No. 196231 January 28, 2014 ll-B-003 against Special Prosecutor Wendell Barreras-Sulit for alleged acts and omissions
tantamount to culpable violation of the Constitution and a betrayal of public trust, in accordance
EMILIO A. GONZALES III, Petitioner, with Section 8(2) of the Ombudsman Act of 1989.3
vs.
OFFICE OF THE PRESIDENT OF THE PHILIPPINES, ACTING THROUGH AND REPRESENTED BY In view of the Courts ruling, the OP filed the present motion for reconsideration through the
EXECUTIVE SECRETARY PAQUITO N. OCHOA, JR., SENIOR DEPUTY EXECUTIVE SECRETARY JOSE Office of the Solicitor General (OSG).
AMOR M. AMORANDO, OFFICER-IN-CHARGE - OFFICE OF THE DEPUTY EXECUTIVE SECRETARY
FOR LEGAL AFFAIRS, ATTY. RONALDO A. GERON, DIR. ROWENA TURINGAN-SANCHEZ, AND ATTY. We briefly narrate the facts that preceded the filing of the petitions and the present motion for
CARLITO D. CATAYONG, Respondents. reconsideration.

x-----------------------x I. ANTECEDENTS

G.R. No. 196232 A. Gonzales petition (G.R. No. 196231)

WENDELL BARRERAS-SULIT Petitioner, a. Factual antecedents


vs.
ATTY. PAQUITO N. OCHOA, JR., IN HIS CAP A CITY AS EXECUTIVE SECRETARY, OFFICE OF THE
PRESIDENT, ATTY. DENNIS F. ORTIZ, ATTY. CARLO D. SULAY AND ATTY. FROILAN D. MONTALBAN, On May 26, 2008, Christian Kalaw filed separate charges with the Philippine National Police
JR., IN THEIR CAPACITIES AS CHAIRMAN AND MEMBERS OF OFFICE OF MALACANANG LEGAL Internal Affairs Service (PNP-IAS) and with the Manila City Prosecutors Office against Manila Police
AFFAIRS,Respondents. District Senior Inspector Rolando Mendoza and four others (Mendoza, et al.) for robbery, grave
threat, robbery extortion and physical injury.4

DECISION
On May 29, 2008, Police Senior Superintendent Atty. Clarence Guinto filed an administrative
charge for grave misconduct with the National Police Commission (NAPOLCOM) PNP-NCRPO
BRION, J.: against Mendoza, et al. based on the same allegations made by Kalaw before the PNP-IAS.5

We resolve the Office of the President's (OP 's) motion for reconsideration of our September 4, On July 2, 2008, Gonzales, Deputy Ombudsman for Military and Other Law Enforcement Officers
2012 Decision1which ruled on the petitions filed by Deputy Ombudsman Emilio Gonzales III and (MOLEO), directed the NAPOLCOM to turn over the records of Mendozas case to his office. The
Special Prosecutor Wendell Barreras-Sulit. Their petitions challenged the constitutionality of Office of the Regional Director of the NAPOLCOM duly complied on July 24, 2008.6 Mendoza, et al.
Section 8(2) of Republic Act (RA) No. 6770.2 filed their position papers with Gonzales, in compliance with his Order.7

In the challenged Decision, the Court upheld the constitutionality of Section 8(2) of RA No. 6770 Pending Gonzales action on Mendoza, et al.s case (on August 26, 2008), the Office of the City
and ruled that the President has disciplinary jurisdiction over a Deputy Ombudsman and a Special Prosecutor of Manila City dismissed Kalaws complaint against Mendoza, et al. for his failure to
Prosecutor. The Court, however, reversed the OP ruling that: (i) found Gonzales guilty of Gross substantiate his allegations.8 Similarly, on October 17, 2008, the PNP-IAS recommended the
Neglect of Duty and Grave Misconduct constituting betrayal of public trust; and (ii) imposed on dismissal without prejudice of the administrative case against Mendoza, et al. for Kalaws failure to
him the penalty of dismissal. prosecute.9

Sulit, who had not then been dismissed and who simply sought to restrain the disciplinary On February 16, 2009, after preparing a draft decision on Mendoza, et al.s case, Gonzales
proceedings against her, solely questioned the jurisdiction of the OP to subject her to disciplinary forwarded the entire records to the Office of then Ombudsman Merceditas Gutierrez for her
proceedings. The Court affirmed the continuation of the proceedings against her after upholding review.10 In his draft decision, Gonzales found Mendoza, et al. guilty of grave misconduct and
the constitutionality of Section 8(2) of RA No. 6770. imposed on them the penalty of dismissal from the service.11

The fallo of our assailed Decision reads: Mendoza, et al. received a copy of the Ombudsmans decision that approved Gonzales
recommendation on October 30, 2009. Mendoza, et al. filed a motion for reconsideration12 on
WHEREFORE, in G.R. No. 196231, the decision of the Office of the President in OP Case No. 1 O-J- November 5, 2009, followed by a Supplement to the Motion for Reconsideration.13
460 is REVERSED and SET ASIDE. Petitioner Emilio A. Gonzales III is ordered REINSTATED with
payment of backwages corresponding to the period of suspension effective immediately, even as On December 10, 2009, the MOLEO-Records Section forwarded Mendoza, et al.s case records to
the Office of the Ombudsman is directed to proceed with the investigation in connection with the the Criminal Investigation, Prosecution and Administrative Bureau-MOLEO. On December 14,
above case against petitioner. In G.R. No. 196232, We AFFIRM the continuation of OP-DC Case No.
2009, the case was assigned to Graft Investigation and Prosecution Officer (GIPO) Dennis Garcia for took to resolve the motion could not be unjustified, since he himself acted on the draft order only
review and recommendation.14 within nine (9) calendars days from his receipt of the order.23

GIPO Garcia released a draft order15 to his immediate superior, Director Eulogio S. Cecilio, for B. Sulits petition (G.R. No. 196232)
appropriate action on April 5, 2010. Dir. Cecilio signed and forwarded the draft order to Gonzales
office on April 27, 2010. Gonzales reviewed the draft and endorsed the order, together with the In April 2005, the Office of the Ombudsman charged Major General Carlos F. Garcia and several
case records, on May 6, 2010 for the final approval by the Ombudsman.16 others, before the Sandiganbayan, with plunder and money laundering. On May 7, 2007, Garcia
filed an Urgent Petition for Bail which the prosecution opposed. The Sandiganbayan denied
On August 23, 2010, pending final action by the Ombudsman on Mendoza, et al.s case, Mendoza Garcia's urgent petition for bail on January 7, 2010, in view of the strength of the prosecutions
hijacked a tourist bus and held the 21 foreign tourists and the four Filipino tour assistants on board evidence against Garcia.
as hostages. While the government exerted earnest attempts to peacefully resolve the hostage-
taking, it ended tragically, resulting in the deaths of Mendoza and several others on board the On February 25, 2010, the Office of the Ombudsman, through Sulit and her prosecutorial staff,
hijacked bus. entered into a plea bargaining agreement (Agreement) with Garcia.24 Garcia thereby agreed to: (i)
withdraw his plea of not guilty to the charge of plunder and enter a plea of guilty to the lesser
In the aftermath, President Benigno C. Aquino III directed the Department of Justice and the offense of indirect bribery; and (ii) withdraw his plea of not guilty to the charge of money
Department of Interior and Local Government to conduct a joint thorough investigation of the laundering and enter a guilty plea to the lesser offense of facilitating money laundering. In
incident. The two departments issued Joint Department Order No. 01-2010, creating an Incident exchange, he would convey to the government his ownership, rights and other interests over the
Investigation and Review Committee (IIRC). real and personal properties enumerated in the Agreement and the bank deposits alleged in the
information.25
In its September 16, 2010 First Report, the IIRC found the Ombudsman and Gonzales accountable
for their "gross negligence and grave misconduct in handling the case against Mendoza."17 The IIRC The Sandiganbayan approved the Agreement on May 4, 201026 based on the parties submitted
stated that the Ombudsman and Gonzales failure to promptly resolve Mendozas motion for Joint Motion for Approval.27
reconsideration, "without justification and despite repeated pleas" xxx "precipitated the desperate
resort to hostage-taking."18 The IIRC recommended the referral of its findings to the OP for further The apparent one-sidedness of the Agreement drew public outrage and prompted the Committee
determination of possible administrative offenses and for the initiation of the proper on Justice of the House of Representatives to conduct an investigation. After public hearings, the
administrative proceedings.19 Committee found that Sulit, her deputies and assistants committed culpable violations of the
Constitution and betrayal of public trust grounds for removal under Section 8(2) of RA No.
Accordingly, on October 15, 2010, Gonzales was formally charged before the OP for Gross Neglect 6770.28The Committee recommended to the President the dismissal from the service of Sulit and
of Duty and/or Inefficiency in the Performance of Official Duty and for Misconduct in Office. 20 the filing of appropriate charges against her deputies and assistants before the appropriate
government office.
b. The OP ruling
Accordingly, the OP initiated an administrative disciplinary proceeding against Sulit. 29 On March
On March 31, 2011, the OP found Gonzales guilty as charged and dismissed him from the 24, 2011, Sulit filed her Written Explanation, questioning the OPs jurisdiction.30 The question of
service.21According to the OP, "the inordinate and unjustified delay in the resolution of jurisdiction notwithstanding, the OP set the case for preliminary investigation on April 15, 2011,
[Mendozas] Motion for Reconsideration [that spanned for nine (9) long months] xxx amounted prompting Sulit to seek relief from this Court.
to gross neglect of duty" and "constituted a flagrant disregard of the Office of the Ombudsmans
own Rules of Procedure."22 II. COURTS RULING

c. The Petition On motion for reconsideration and further reflection, the Court votes to grant Gonzales petition
and to declare Section 8(2) of RA No. 6770 unconstitutional with respect to the Office of the
Gonzales posited in his petition that the OP has no administrative disciplinary jurisdiction over a Ombudsman. (As the full explanation of the Courts vote describes below, this conclusion does not
Deputy Ombudsman. Under Section 21 of RA No. 6770, it is the Ombudsman who exercises apply to Sulit as the grant of independence is solely with respect to the Office of the Ombudsman
administrative disciplinary jurisdiction over the Deputy Ombudsman. which does not include the Office of the Special Prosecutor under the Constitution. The prevailing
ruling on this latter point is embodied in the Concurring and Dissenting Opinion of J. Marvic Mario
Victor Leonen).
On the merits, Gonzales argued that his office received the draft order from GIPO Garcia on April
27, 2010. On May 6, 2010, he completed his review of the draft, approved it, and transmitted it to
the Office of the Ombudsman for final approval. Since the draft order on Mendozas motion for A. Preliminary considerations:
reconsideration had to undergo different levels of preparation, review and approval, the period it
a. Absence of motion for reconsideration on the part of the petitioners It was under the 1973 Constitution that the Office of the Ombudsman became a constitutionally-
mandated office to give it political independence and adequate powers to enforce its mandate.
At the outset, the Court notes that Gonzales and Sulit did not file a motion for reconsideration of Pursuant to the 1973 Constitution, President Ferdinand Marcos enacted Presidential Decree (PD)
the Courts September 4, 2012 Decision; only the OP, through the OSG, moved for the No. 1487, as amended by PD No. 1607 and PD No. 1630, creating the Office of the Ombudsman to
reconsideration of our ruling reinstating Gonzales. be known as Tanodbayan. It was tasked principally to investigate, on complaint or motu proprio,
any administrative act of any administrative agency, including any government-owned or
controlled corporation. When the Office of the Tanodbayan was reorganized in 1979, the powers
This omission, however, poses no obstacle for the Courts review of its ruling on the whole case previously vested in the Special Prosecutor were transferred to the Tanodbayan himself. He was
since a serious constitutional question has been raised and is one of the underlying bases for the given the exclusive authority to conduct preliminary investigation of all cases cognizable by the
validity or invalidity of the presidential action. If the President does not have any constitutional Sandiganbayan, file the corresponding information, and control the prosecution of these cases. 34
authority to discipline a Deputy Ombudsman and/or a Special Prosecutor in the first place, then
any ruling on the legal correctness of the OPs decision on the merits will be an empty one.
With the advent of the 1987 Constitution, a new Office of the Ombudsman was created by
constitutional fiat. Unlike in the 1973 Constitution, its independence was expressly and
In other words, since the validity of the OPs decision on the merits of the dismissal is inextricably constitutionally guaranteed. Its objectives are to enforce the state policy in Section 27, Article
anchored on the final and correct ruling on the constitutional issue, the whole case including the II35 and the standard of accountability in public service under Section 1, Article XI of the 1987
constitutional issue remains alive for the Courts consideration on motion for reconsideration. Constitution. These provisions read:

b. The justiciability of the constitutional issue raised in the petitions Section 27. The State shall maintain honesty and integrity in the public service and take positive
and effective measures against graft and corruption.
We clarify, too, that the issue of whether a Deputy Ombudsman may be subjected to the
administrative disciplinary jurisdiction of the President (concurrently with that of the Ombudsman) Section 1. Public office is a public trust. Public officers and employees must, at all times, be
is a justiciable not a political question. A justiciable question is one which is inherently accountable to the people, serve them with utmost responsibility, integrity, loyalty, and efficiency;
susceptible of being decided on grounds recognized by law,31 as where the court finds that there act with patriotism and justice, and lead modest lives.
are constitutionally-imposed limits on the exercise of the powers conferred on a political branch of
the government.32
Under Section 12, Article XI of the 1987 Constitution, the Office of the Ombudsman is envisioned
to be the "protector of the people" against the inept, abusive, and corrupt in the Government, to
In resolving the petitions, we do not inquire into the wisdom of the Congress choice to grant function essentially as a complaints and action bureau. 36 This constitutional vision of a Philippine
concurrent disciplinary authority to the President. Our inquiry is limited to whether such statutory Ombudsman practically intends to make the Ombudsman an authority to directly check and guard
grant violates the Constitution, particularly whether Section 8(2) of RA No. 6770 violates the core against the ills, abuses and excesses of the bureaucracy. Pursuant to Section 13(8), Article XI of the
constitutional principle of the independence of the Office of the Ombudsman as expressed in 1987 Constitution, Congress enacted RA No. 6770 to enable it to further realize the vision of the
Section 5, Art. XI of the Constitution. Constitution. Section 21 of RA No. 6770 provides:

To be sure, neither the Executive nor the Legislative can create the power that Section 8(2) of RA Section 21. Official Subject to Disciplinary Authority; Exceptions. The Office of the Ombudsman
No. 6770 grants where the Constitution confers none. When exercised authority is drawn from a shall have disciplinary authority over all elective and appointive officials of the Government and its
vacuum, more so when the authority runs counter to a core constitutional principle and subdivisions, instrumentalities and agencies, including Members of the Cabinet, local government,
constitutional intents, the Court is duty-bound to intervene under the powers and duties granted government-owned or controlled corporations and their subsidiaries, except over officials who
and imposed on it by Article VIII of the Constitution. may be removed only by impeachment or over Members of Congress, and the Judiciary. [emphasis
ours, italics supplied]
B. The Deputy Ombudsman: Constitutional Issue
As the Ombudsman is expected to be an "activist watchman,"37 the Court has upheld its actions,
a. The Philippine Ombudsman although not squarely falling under the broad powers granted it by the Constitution and by RA No.
6770, if these actions are reasonably in line with its official function and consistent with the law
Prior to the 1973 Constitution, past presidents established several Ombudsman-like agencies to and the Constitution.38
serve as the people's medium for airing grievances and for direct redress against abuses and
misconduct in the government. Ultimately, however, these agencies failed to fully realize their The Ombudsmans broad investigative and disciplinary powers include all acts of malfeasance,
objective for lack of the political independence necessary for the effective performance of their misfeasance, and nonfeasance of all public officials, including Members of the Cabinet and key
function as government critic.33 Executive officers, during their tenure. To support these broad powers, the Constitution saw it fit
to insulate the Office of the Ombudsman from the pressures and influence of officialdom and
partisan politics and from fear of external reprisal by making it an "independent" office. Section 5,
Article XI of the Constitution expressed this intent, as follows: Notably, the Constitution also created an "independent" Commission on Human Rights, although it
enjoys a lesser degree of independence since it is not granted fiscal autonomy in the manner fiscal
Section 5. There is hereby created the independent Office of the Ombudsman, composed of the autonomy is granted to the constitutional commissions. The lack of fiscal autonomy
Ombudsman to be known as Tanodbayan, one overall Deputy and at least one Deputy each for notwithstanding, the framers of the 1987 Constitution clearly expressed their desire to keep the
Luzon, Visayas, and Mindanao. A separate Deputy for the military establishment may likewise be Commission independent from the executive branch and other political leaders:
appointed. [emphasis ours]
MR. MONSOD. We see the merits of the arguments of Commissioner Rodrigo. If we explain to him
Given the scope of its disciplinary authority, the Office of the Ombudsman is a very powerful our concept, he can advise us on how to reconcile his position with ours. The position of the
government constitutional agency that is considered "a notch above other grievance-handling committee is that we need a body that would be able to work and cooperate with the executive
investigative bodies."39 It has powers, both constitutional and statutory, that are commensurate because the Commissioner is right. Many of the services needed by this commission would need
with its daunting task of enforcing accountability of public officers. 40 not only the cooperation of the executive branch of the government but also of the judicial branch
of government. This is going to be a permanent constitutional commission over time. We also
want a commission to function even under the worst circumstance when the executive may not be
b. "Independence" of constitutional bodies vis-a-vis the Ombudsmans independence very cooperative. However, the question in our mind is: Can it still function during that time?
Hence, we are willing to accept suggestions from Commissioner Rodrigo on how to reconcile this.
Under the Constitution, several constitutional bodies have been expressly labeled as We realize the need for coordination and cooperation. We also would like to build in some
"independent."41The extent of the independence enjoyed by these constitutional bodies however safeguards that it will not be rendered useless by an uncooperative executive.
varies and is to be interpreted with two significant considerations in mind: first, the functions
performed or the powers involved in a given case; and second, consistency of any allowable xxxx
interference to these powers and functions, with the principle of checks and balances.

MR. GARCIA. xxx Very often, when international commissions or organizations on human rights go
Notably, the independence enjoyed by the Office of the Ombudsman and by the Constitutional to a country, the most credible organizations are independent human rights bodies. Very often
Commissions shares certain characteristics they do not owe their existence to any act of these are private organizations, many of which are prosecuted, such as those we find in many
Congress, but are created by the Constitution itself; additionally, they all enjoy fiscal autonomy. In countries in Latin America. In fact, what we are proposing is an independent body on human
general terms, the framers of the Constitution intended that these "independent" bodies be rights, which would provide governments with credibility precisely because it is independent of the
insulated from political pressure to the extent that the absence of "independence" would result in present administration. Whatever it says on the human rights situation will be credible because it
the impairment of their core functions. is not subject to pressure or control from the present political leadership.

In Bengzon v. Drilon,42 involving the fiscal autonomy of the Judiciary, we ruled against the Secondly, we all know how political fortunes come and go. Those who are in power yesterday are
interference that the President may bring and maintained that the independence and the flexibility in opposition today and those who are in power today may be in the opposition tomorrow.
of the Judiciary, the Constitutional Commissions and the Office of the Ombudsman are crucial to Therefore, if we have a Commission on Human Rights that would investigate and make sure that
our legal system. the rights of each one is protected, then we shall have a body that could stand up to any power, to
defend the rights of individuals against arrest, unfair trial, and so on.45
The Judiciary, the Constitutional Commissions, and the Ombudsman must have the independence
and flexibility needed in the discharge of their constitutional duties. The imposition of restrictions These deliberative considerations abundantly show that the independent constitutional
and constraints on the manner the independent constitutional offices allocate and utilize the funds commissions have been consistently intended by the framers to be independent from executive
appropriated for their operations is anathema to fiscal autonomy and violative not only the control or supervision or any form of political influence. At least insofar as these bodies are
express mandate of the Constitution but especially as regards the Supreme Court, of the concerned, jurisprudence is not scarce on how the "independence" granted to these bodies
independence and separation of powers upon which the entire fabric of our constitutional system prevents presidential interference.
is based.

In Brillantes, Jr. v. Yorac,46 we emphasized that the Constitutional Commissions, which have been
The constitutional deliberations explain the Constitutional Commissions need for independence. characterized under the Constitution as "independent," are not under the control of the President,
In the deliberations of the 1973 Constitution, the delegates amended the 1935 Constitution by even if they discharge functions that are executive in nature. The Court declared as
providing for a constitutionally-created Civil Service Commission, instead of one created by law, on unconstitutional the Presidents act of temporarily appointing the respondent in that case as
the premise that the effectivity of this body is dependent on its freedom from the tentacles of Acting Chairman of the Comelec "however well-meaning"47 it might have been.
politics.43 In a similar manner, the deliberations of the 1987 Constitution on the Commission on
Audit highlighted the developments in the past Constitutions geared towards insulating the
Commission on Audit from political pressure.44 In Bautista v. Senator Salonga,48 the Court categorically stated that the tenure of the
commissioners of the independent Commission on Human Rights could not be placed under the
discretionary power of the President:
Indeed, the Court finds it extremely difficult to conceptualize how an office conceived and created The deliberations of the Constitutional Commission on the independence of the Ombudsman fully
by the Constitution to be independent as the Commission on Human Rights and vested with support this position. Commissioner Florenz Regalado of the Constitutional Commission expressed
the delicate and vital functions of investigating violations of human rights, pinpointing his apprehension that any form of presidential control over the Office of the Ombudsman would
responsibility and recommending sanctions as well as remedial measures therefor, can truly diminish its independence.51 The following exchanges between Commissioners Blas Ople and
function with independence and effectiveness, when the tenure in office of its Chairman and Christian Monsod further reveal the constitutional intent to keep the Office of the Ombudsman
Members is made dependent on the pleasure of the President. Executive Order No. 163-A, being independent from the President:
antithetical to the constitutional mandate of independence for the Commission on Human Rights
has to be declared unconstitutional. MR. OPLE. xxx

Again, in Atty. Macalintal v. Comelec,49 the Court considered even the mere review of the rules of May I direct a question to the Committee? xxx [W]ill the Committee consider later an amendment
the Commission on Elections by Congress a "trampling" of the constitutional mandate of xxx, by way of designating the office of the Ombudsman as a constitutional arm for good
independence of this body. Obviously, the mere review of rules places considerably less pressure government, efficiency of the public service and the integrity of the President of the Philippines,
on a constitutional body than the Executives power to discipline and remove key officials of the instead of creating another agency in a kind of administrative limbo which would be accountable
Office of the Ombudsman, yet the Court struck down the law as unconstitutional. to no one on the pretext that it is a constitutional body?

The kind of independence enjoyed by the Office of the Ombudsman certainly cannot be inferior MR. MONSOD. The Committee discussed that during our committee deliberations and when we
but is similar in degree and kind to the independence similarly guaranteed by the Constitution to prepared the report, it was the opinion of the Committee and I believe it still is that it may
the Constitutional Commissions since all these offices fill the political interstices of a republican not contribute to the effectiveness of this office of the Ombudsman precisely because many of the
democracy that are crucial to its existence and proper functioning.50 culprits in inefficiency, injustice and impropriety are in the executive department. Therefore, as we
saw the wrong implementation of the Tanodbayan which was under the tremendous influence of
c. Section 8(2) of RA No. 6770 vesting disciplinary authority in the President over the the President, it was an ineffectual body and was reduced to the function of a special fiscal. The
Deputy Ombudsman violates the independence of the Office of the Ombudsman and is thus whole purpose of our proposal is precisely to separate those functions and to produce a vehicle
unconstitutional that will give true meaning to the concept of Ombudsman. Therefore, we regret that we cannot
accept the proposition.52
Our discussions, particularly the Courts expressed caution against presidential interference with
the constitutional commissions, on one hand, and those expressed by the framers of the 1987 The statements made by Commissioner Monsod emphasized a very logical principle: the Executive
Constitution, on the other, in protecting the independence of the Constitutional Commissions, power to remove and discipline key officials of the Office of the Ombudsman, or to exercise any
speak for themselves as overwhelming reasons to invalidate Section 8(2) of RA No. 6770 for power over them, would result in an absurd situation wherein the Office of the Ombudsman is
violating the independence of the Office of the Ombudsman. given the duty to adjudicate on the integrity and competence of the very persons who can remove
or suspend its members. Equally relevant is the impression that would be given to the public if the
In more concrete terms, we rule that subjecting the Deputy Ombudsman to discipline and removal rule were otherwise. A complainant with a grievance against a high-ranking official of the
by the President, whose own alter egos and officials in the Executive Department are subject to Executive, who appears to enjoy the Presidents favor, would be discouraged from approaching
the Ombudsmans disciplinary authority, cannot but seriously place at risk the independence of the Ombudsman with his complaint; the complainants impression (even if misplaced), that the
the Office of the Ombudsman itself. The Office of the Ombudsman, by express constitutional Ombudsman would be susceptible to political pressure, cannot be avoided. To be sure, such an
mandate, includes its key officials, all of them tasked to support the Ombudsman in carrying out impression would erode the constitutional intent of creating an Office of the Ombudsman as
her mandate. Unfortunately, intrusion upon the constitutionally-granted independence is what champion of the people against corruption and bureaucracy.
Section 8(2) of RA No. 6770 exactly did. By so doing, the law directly collided not only with the
independence that the Constitution guarantees to the Office of the Ombudsman, but inevitably d. The mutual-protection argument for crafting Section 8(2)of RA No. 6770
with the principle of checks and balances that the creation of an Ombudsman office seeks to
revitalize. In crafting Section 8(2) of RA No. 6770, Congress apparently addressed the concern that a lack of
an external check against the Deputy Ombudsman would result in mutual protection between the
What is true for the Ombudsman must be equally and necessarily true for her Deputies who act as Ombudsman and her Deputies.
agents of the Ombudsman in the performance of their duties. The Ombudsman can hardly be
expected to place her complete trust in her subordinate officials who are not as independent as While the preceding discussion already suffices to address this concern, it should be added that
she is, if only because they are subject to pressures and controls external to her Office. This need this concern stands on shaky grounds since it ignores the existing checks and balances already in
for complete trust is true in an ideal setting and truer still in a young democracy like the Philippines place. On the one hand, the Ombudsmans Deputies cannot protect the Ombudsman because she
where graft and corruption is still a major problem for the government. For these reasons, Section is subject to the impeachment power of Congress. On the other hand, the Ombudsmans attempt
8(2) of RA No. 6770 (providing that the President may remove a Deputy Ombudsman) should be to cover up the misdeeds of her Deputies can be questioned before the Court on appeal or
declared void. certiorari. The same attempt can likewise subject her to impeachment.
The judicial recourse available is only consistent with the nature of the Supreme Court as a non- In short, the authority granted by the Constitution to Congress to provide for the manner and
political independent body mandated by the Constitution to settle judicial and quasi-judicial cause of removal of all other public officers and employees does not mean that Congress can
disputes, whose judges and employees are not subject to the disciplinary authority of the ignore the basic principles and precepts established by the Constitution.
Ombudsman and whose neutrality would be less questionable. The Members of the Court
themselves may be subjected to the impeachment power of Congress. In the same manner, the congressional determination of the identity of the disciplinary authority is
not a blanket authority for Congress to repose it on whomsoever Congress chooses without
In these lights, the appeal, if any, of the mutual protection argument becomes distinctly running afoul of the independence enjoyed by the Office of the Ombudsman and without
implausible. At the same time, the Court remains consistent with its established rulings - that the disrupting the delicate check and balance mechanism under the Constitution. Properly viewed
independence granted to the Constitutional Commissions bars any undue interference from either from this perspective, the core constitutional principle of independence is observed and any
the Executive or Congress and is in full accord with constitutional intent. possible absurdity resulting from a contrary interpretation is avoided. In other words, while the
Constitution itself vested Congress with the power to determine the manner and cause of removal
e. Congress power determines the manner and causes for the removal of non-impeachable officers of all non-impeachable officials, this power must be interpreted consistent with the core
is not a carte blanch authority constitutional principle of independence of the Office of the Ombudsman. Our observation in
Macalintal v. Comelec63 is apt:

Under Section 2, Article XI of the 1987 Constitution,53 Congress is empowered to determine the
modes of removal from office of all public officers and employees except the President, the Vice- The ambit of legislative power under Article VI of the Constitution is circumscribed by other
President, the Members of the Supreme Court, the Members of the Constitutional Commissions, constitutional provisions. One such provision is Section 1 of Article IX-A of the 1987 Constitution
and the Ombudsman, who are all impeachable officials. ordaining that constitutional commissions such as the COMELEC shall be "independent."

The intent of the framers of the Constitution in providing that "[a]ll other public officers and While one may argue that the grounds for impeachment under Section 8(2) of RA No. 6770 is
employees may be removed from office as provided by law, but not by impeachment" in the intended as a measure of protection for the Deputy Ombudsman and Special Prosecutor since
second sentence of Section 2, Article XI is to prevent Congress from extending the more stringent these grounds are not intended to cover all kinds of official wrongdoing and plain errors of
rule of "removal only by impeachment" to favored public officers.54 Understandably so, judgment - this argument seriously overlooks the erosion of the independence of the Office of the
impeachment is the most difficult and cumbersome mode of removing a public officer from office. Ombudsman that it creates. The mere fact that a statutorily-created sword of Damocles hangs
It is, by its nature, a sui generis politico-legal process55 that signals the need for a judicious and over the Deputy Ombudsmans head, by itself, opens up all the channels for external pressures
careful handling as shown by the process required to initiate the proceeding;56 the one-year and influence of officialdom and partisan politics. The fear of external reprisal from the very office
limitation or bar for its initiation;57 the limited grounds for impeachment;58 the defined he is to check for excesses and abuses defeats the very purpose of granting independence to the
instrumentality given the power to try impeachment cases;59 and the number of votes required for Office of the Ombudsman.
a finding of guilt.60 All these argue against the extension of this removal mechanism beyond those
mentioned in the Constitution. That a judicial remedy is available (to set aside dismissals that do not conform to the high standard
required in determining whether a Deputy Ombudsman committed an impeachable offense) and
On the practical side, our nation has witnessed the complications and problems an impeachment that the Presidents power of removal is limited to specified grounds are dismally inadequate
proceeding entails, thus justifying its limited application only to the officials occupying the highest when balanced with the constitutional principle of independence. The mere filing of an
echelons of responsibility in our government. To name a few, some of the negative practical administrative case against the Deputy Ombudsman and the Special Prosecutor before the OP can
effects of impeachment are: it stalls legislative work; it is an expensive process in terms of the cost already result in their suspension and can interrupt the performance of their functions, in violation
of prosecution alone; and, more importantly, it is inherently divisive of the nation. 61 Thus, in a of Section 12, Article XI of the Constitution. With only one term allowed under Section 11, a
cost-benefit analysis of adopting impeachment as a mechanism, limiting Congress power to Deputy Ombudsman or Special Prosecutor, if removable by the President, can be reduced to the
otherwise legislate on the matter is far more advantageous to the country. very same ineffective Office of the Ombudsman that the framers had foreseen and carefully tried
to avoid by making these offices independent constitutional bodies.

It is in these lights that the second sentence in Section 2, Article XI of the 1987 Constitution should
be read. Contrary to the implied view of the minority, in no way can this provision be regarded as At any rate, even assuming that the OP has disciplinary authority over the Deputy Ombudsman, its
blanket authority for Congress to provide for any ground of removal it deems fit. While the decision finding Gonzales guilty of Gross Neglect of Duty and Grave Misconduct constituting
manner and cause of removal are left to congressional determination, this must still be consistent betrayal of public trust is patently erroneous. The OPs decision perfectly illustrates why the
with constitutional guarantees and principles, namely: the right to procedural and substantive due requirement of impeachment-grounds in Section 8(2) of RA No. 6770 cannot be considered, even
process; the constitutional guarantee of security of tenure; the principle of separation of powers; at a minimum, a measure of protection of the independence of the Office of the Ombudsman.
and the principle of checks and balances.62
C. The Deputy Ombudsman: The Dismissal Issue

a. The Office of the Presidents finding of gross negligence has no legal and factual leg to stand on
The OPs decision found Gonzales guilty of Gross Neglect of Duty and of Grave Misconduct. The a) New evidence had been discovered which materially affects the order, directive or
assailed Decision of the OP reads: decision;

Upon consideration of the First Report, the evidence and allegations of respondent Deputy b) Grave errors of facts or laws or serious irregularities have been committed prejudicial
Ombudsman himself, and other documentary evidence gathered, this Office finds that the to the interest of the movant.
inordinate and unjustified delay in the resolution of Captain Mendozas Motion for
Reconsideration timely filed on 5 November 2009 xxx amounted to gross neglect of duty and/or Only one motion for reconsideration or reinvestigation shall be allowed, and the Hearing Officer
inefficiency in the performance of official duty.64 shall resolve the same within five (5) days from the date of submission for resolution. [emphasis
and underscore ours]
b. No gross neglect of duty or inefficiency
Even if we consider this provision to be mandatory, the period it requires cannot apply to Gonzales
Let us again briefly recall the facts. since he is a Deputy Ombudsman whose obligation is to review the case; he is not simply a Hearing
Officer tasked with the initial resolution of the motion. In Section 6 of Administrative Order No. 7
1. November 5, 2009 - Mendoza filed a Motion for Reconsideration of the decision of on the resolution of the case and submission of the proposed decision, the period for resolving the
the Ombudsman,65 which was followed by a Supplement to the Motion for case does not cover the period within which it should be reviewed:
Reconsideration;66
Section 6. Rendition of decision. Not later than thirty (30) days after the case is declared
2. December 14, 200967 - GIPO Garcia, who was assigned to review these motions and submitted for resolution, the Hearing Officer shall submit a proposed decision containing his
make his recommendation for the appropriate action, received the records of the case; findings and recommendation for the approval of the Ombudsman. Said proposed decision shall
be reviewed by the Directors, Assistant Ombudsmen and Deputy Ombudsmen concerned. With
respect to low ranking public officials, the Deputy Ombudsman concerned shall be the approving
3. April 5, 2010 GIPO Garcia released a draft order to be reviewed by his immediate authority. Upon approval, copies thereof shall be served upon the parties and the head of the
superior, Dir. Cecilio;68 office or agency of which the respondent is an official or employee for his information and
compliance with the appropriate directive contained therein. [italics and emphases supplied]
4. April 27, 2010 Dir. Cecilio signed and forwarded to Gonzales this draft order;69
Thus, the OPs ruling that Gonzales had been grossly negligent for taking nine days, instead of five
5. May 6, 2010 (or nine days after the records were forwarded to Gonzales) Gonzales days, to review a case was totally baseless.
endorsed the draft order for the final approval of the Ombudsman.70
c. No actionable failure to supervise subordinates
Clearly, when Mendoza hijacked the tourist bus on August 23, 2010, the records of the case were
already pending before Ombudsman Gutierrez. The OPs claims that Gonzales could have supervised his subordinates to promptly act on
Mendozas motion and apprised the Tanodbayan of the urgency of resolving the same are similarly
Gross negligence refers to negligence characterized by the want of even the slightest care, acting groundless.
or omitting to act in a situation where there is a duty to act, not inadvertently but willfully and
intentionally, with a conscious indifference to consequences insofar as other persons may be The Office of the Ombudsman is not a corner office in our bureaucracy. It handles numerous cases
affected. In the case of public officials, there is gross negligence when a breach of duty is flagrant that involve the potential loss of employment of many other public employees. We cannot
and palpable.71 conclusively state, as the OP appears to suggest, that Mendozas case should have been prioritized
over other similar cases.
Gonzales cannot be guilty of gross neglect of duty and/or inefficiency since he acted on the case
forwarded to him within nine days. In finding Gonzales guilty, the OP72 relied on Section 8, Rule III The Court has already taken judicial notice of the steady stream of cases reaching the Office of the
of Administrative Order No. 7 (or the Rules of Procedure of the Office of the Ombudsman, series of Ombudsman.73 This consideration certainly militates against the OSGs observation that there was
1990, as amended) in ruling that Gonzales should have acted on Mendozas Motion for "a grossly inordinate and inexcusable delay"74 on the part of Gonzales.
Reconsideration within five days:

Equally important, the constitutional guarantee of "speedy disposition of cases" before, among
Section 8. Motion for reconsideration or reinvestigation: Grounds Whenever allowable, a motion others, quasi-judicial bodies,75 like the Office of the Ombudsman, is itself a relative
for reconsideration or reinvestigation may only be entertained if filed within ten (10) days from concept.76 Thus, the delay, if any, must be measured in this objective constitutional sense.
receipt of the decision or order by the party on the basis of any of the following grounds: Unfortunately, because of the very statutory grounds relied upon by the OP in dismissing
Gonzales, the political and, perhaps, "practical" considerations got the better of what is legal and e. Penalty of dismissal totally incommensurate with established facts
constitutional.
Given the lack of factual basis for the charges against Gonzales, the penalty of removal imposed by
The facts do not show that Gonzales subordinates had in any way been grossly negligent in their the OP necessarily suffers grave infirmity. Basic strictures of fair play dictate that we can only be
work. While GIPO Garcia reviewed the case and drafted the order for more than three months, it is held liable for our own misdeeds; we can be made to account only for lapses in our
noteworthy that he had not drafted the initial decision and, therefore, had to review the case for responsibilities. It is notable that of all the officers, it was Gonzales who took the least time nine
the first time.77 Even the Ombudsman herself could not be faulted for acting on a case within four days followed by Cecilio, who took 21 days; Garcia the writer of the draft took less than
months, given the amount of cases that her office handles. four months, and the Ombudsman, less than four months until the kidnapping incident rendered
Mendozas motion moot.
The point is that these are not inordinately long periods for the work involved: examination of the
records, research on the pertinent laws and jurisprudence, and exercise of legal judgment and In these lights, the decision of the OP is clearly and patently wrong. This conclusion, however, does
discretion. If this Court rules that these periods per se constitute gross neglect of duty, the not preclude the Ombudsman from looking into any other possible administrative liability of
Ombudsmans constitutional mandate to prosecute all the erring officials of this country would be Gonzales under existing Civil Service laws, rules and regulations.
subjected to an unreasonable and overwhelming constraint. Similarly, if the Court rules that these
periods per se constitute gross neglect of duty, then we must be prepared to reconcile this with D. The Special Prosecutor: The Constitutional Issue
the established concept of the right of speedy disposition of cases something the Court may be
hard put to justify.
The 1987 Constitution created a new, independent Office of the Ombudsman. The existing
Tanodbayan at the time83 became the Office of the Special Prosecutor under the 1987
d. No undue interest Constitution. While the composition of the independent Office of the Ombudsman under the 1987
Constitution does not textually include the Special Prosecutor, the weight of the foregoing
The OP also found Gonzales guilty of showing undue interest in Mendozas case by having the case discussions on the unconstitutionality of Section 8(2) of RA No. 6770 should equally apply to the
endorsed to the Office of the Ombudsman and by resolving it against Mendoza on the basis of the
unverified complaint-affidavit of the alleged victim, Kalaw. Special Prosecutor on the basis of the legislative history of the Office of the Ombudsman as
expounded in jurisprudence.
The fact that Gonzales had Mendozas case endorsed to his office lies within his mandate, even if it
were based merely on the request of the alleged victims father. The Constitution empowers the Under the 1973 Constitution,84 the legislature was mandated to create the Office of the
Ombudsman and her Deputies to act promptly on complaints filed in any form or manner against Ombudsman, known as the Tanodbayan, with investigative and prosecutorial powers. Accordingly,
any public official or employee of the government.78 This provision is echoed by Section 13 of RA on June 11, 1978, President Ferdinand Marcos enacted PD No. 1487.85
No. 6770,79 and by Section 3, Rule III of Administrative Order No. 7, series of 1990, as amended.80

Under PD No. 1486,86 however, the "Chief Special Prosecutor" (CSP) was given the "exclusive
Moreover, Gonzales and his subordinates did not resolve the complaint only on the basis of the authority" to conduct preliminary investigation and to prosecute cases that are within the
unverified affidavit of Kalaw. Based on the prosecution officers recommendations, the finding of jurisdiction of the Sandiganbayan.87 PD No. 1486 expressly gave the Secretary of Justice the power
guilt on the part of Mendoza, et al. was based on their admissions as well. Mendoza, et al. of control and supervision over the Special Prosecutor.88 Consistent with this grant of power, the
admitted that they had arrested Kalaw based on two traffic violations and allowed him to stay the law also authorized the Secretary of Justice to appoint or detail to the Office of the CSP "any
whole night until the following morning in the police precinct. The next morning, Kalaw was officer or employee of Department of Justice or any Bureau or Office under the executive
allowed to leave the precinct despite his failure to show a valid license and based merely on his supervision thereof" to assist the Office of the CSP.
promise to return with the proper documents.81 These admissions led Gonzales and his staff to
conclude that Mendoza, et al. irregularly acted in apprehending Kalaw, since the proper procedure
for the apprehension of traffic violators would be to give them a ticket and to file a case, when In December 1978, PD No. 160789 practically gave back to the Tanodbayan the powers taken away
appropriate.82 from it by the Office of the CSP. The law "created in the Office of the Tanodbayan an Office of the
Chief Special Prosecutor" under the Tanodbayans control,90 with the exclusive authority to
conduct preliminary investigation and prosecute all cases cognizable by the Sandiganbayan. Unlike
Lastly, we cannot deduce undue interest simply because Gonzales decision differs from the the earlier decree, the law also empowered the Tanodbayan to appoint Special Investigators and
decision of the PNP-IAS (which dismissed the complaint against Mendoza). To be sure, we cannot subordinate personnel and/or to detail to the Office of the CSP any public officer or employees
tie the hands of any judicial or quasi-judicial body by ruling that it should always concur with the who "shall be under the supervision and control of the Chief Special Prosecutor."91 In 1979, PD No.
decisions of other judicial or quasi-judicial bodies which may have also taken cognizance of the 1630 further amended the earlier decrees by transferring the powers previously vested in the
case. To do so in the case of a Deputy Ombudsman would be repugnant to the independence that Special Prosecutor directly to the Tanodbayan himself.92
our Constitution has specifically granted to this office and would nullify the very purpose for which
it was created.
This was the state of the law at the time the 1987 Constitution was ratified. Under the 1987 are subject to the prosecutorial authority of the Special Prosecutor, would seriously place the
Constitution, an "independent Office of the Ombudsman" is created.93 The existing Tanodbayan is independence of the Office of the Ombudsman itself at risk.
made the Office of the Special Prosecutor, "who shall continue to function and exercise its powers
as now94 or hereafter may be provided by law."95 Thus, even if the Office of the Special Prosecutor is not expressly made part of the composition of
the Office of the Ombudsman, the role it performs as an organic component of that Office
Other than the Ombudsmans Deputies, the Ombudsman shall appoint all other officials and militates against a differential treatment between the Ombudsmans Deputies, on one hand, and
employees of the Office of the Ombudsman.96 Section 13(8), Article XI of the 1987 Constitution the Special Prosecutor himself, on the other. What is true for the Ombudsman must be equally
provides that the Ombudsman may exercise "such other powers or perform such functions or true, not only for her Deputies but, also for other lesser officials of that Office who act directly as
duties as may be provided by law." Pursuant to this constitutional command, Congress enacted RA agents of the Ombudsman herself in the performance of her duties.
No. 6770 to provide for the functional and structural organization of the Office of the Ombudsman
and the extent of its disciplinary authority. In Acop v. Office of the Ombudsman,106 the Court was confronted with an argument that, at
bottom, the Office of the Special Prosecutor is not a subordinate agency of the Office of the
In terms of composition, Section 3 of RA No. 6770 defines the composition of the Office of the Ombudsman and is, in fact, separate and distinct from the latter. In debunking that argument, the
Ombudsman, including in this Office not only the offices of the several Deputy Ombudsmen but Court said:
the Office of the Special Prosecutor as well. In terms of appointment, the law gave the President
the authority to appoint the Ombudsman, his Deputies and the Special Prosecutor, from a list of Firstly, the petitioners misconstrue Commissioner Romulo's statement as authority to advocate
nominees prepared by the Judicial and Bar Council. In case of vacancy in these positions, the law that the intent of the framers of the 1987 Constitution was to place the Office of the Special
requires that the vacancy be filled within three (3) months from occurrence. 97 Prosecutor under the Office of the President. Xxx

The law also imposes on the Special Prosecutor the same qualifications it imposes on the In the second place, Section 7 of Article XI expressly provides that the then existing Tanodbayan, to
Ombudsman himself/herself and his/her deputies.98 Their terms of office,99 prohibitions and be henceforth known as the Office of the Special Prosecutor, "shall continue to function and
qualifications,100 rank and salary are likewise the same.101 The requirement on disclosure102 is exercise its powers as now or hereafter may be provided by law, except those conferred on the
imposed on the Ombudsman, the Deputies and the Special Prosecutor as well. In case of vacancy Office of the Ombudsman created under this Constitution." The underscored phrase evidently
in the Office of the Ombudsman, the Overall Deputy cannot assume the role of Acting refers to the Tanodbayan's powers under P.D. No. 1630 or subsequent amendatory legislation. It
Ombudsman; the President may designate any of the Deputies or the Special Prosecutor as Acting follows then that Congress may remove any of the Tanodbayan's/Special Prosecutor's powers
Ombudsman.103 The power of the Ombudsman and his or her deputies to require other under P.D. N0. 1630 or grant it other powers, except those powers conferred by the Constitution
government agencies to render assistance to the Office of the Ombudsman is likewise enjoyed by on the Office of the Ombudsman.
the Special Prosecutor.104

Pursuing the present line of reasoning, when one considers that by express mandate of paragraph
Given this legislative history, the present overall legal structure of the Office of the Ombudsman, 8, Section 13, Article XI of the Constitution, the Ombudsman may "exercise such other powers or
both under the 1987 Constitution and RA No. 6770, militates against an interpretation that would perform functions or duties as may be provided by law," it is indubitable then that Congress has
insulate the Deputy Ombudsman from the disciplinary authority of the OP and yet expose the the power to place the Office of the Special Prosecutor under the Office of the Ombudsman. 107
Special Prosecutor to the same ills that a grant of independence to the Office of the Ombudsman
was designed for.
Thus, under the present Constitution, there is every reason to treat the Special Prosecutor to be at
par with the Ombudsman's deputies, at least insofar as an extraneous disciplinary authority is
Congress recognized the importance of the Special Prosecutor as a necessary adjunct of the concerned, and must also enjoy the same grant of independence under the Constitution.
Ombudsman, aside from his or her deputies, by making the Office of the Special Prosecutor an
organic component of the Office of the Ombudsman and by granting the Ombudsman control and
supervision over that office.105 This power of control and supervision includes vesting the Office of III. SUMMARY OF VOTING
the Ombudsman with the power to assign duties to the Special Prosecutor as he/she may deem fit.
Thus, by constitutional design, the Special Prosecutor is by no means an ordinary subordinate but In the voting held on January 28, 2014, by a vote of 8-7,108 the Court resolved to reverse its
one who effectively and directly aids the Ombudsman in the exercise of his/her duties, which September 4, 2012 Decision insofar as petitioner Gonzales is concerned (G.R. No. 196231). We
include investigation and prosecution of officials in the Executive Department. declared Section 8(2) of RA No. 6770 unconstitutional by granting disciplinary jurisdiction to the
President over a Deputy Ombudsman, in violation of the independence of the Office of the
Under Section 11(4) of RA No. 6770, the Special Prosecutor handles the prosecution of criminal Ombudsman.
cases within the jurisdiction of the Sandiganbayan and this prosecutorial authority includes high-
ranking executive officials. For emphasis, subjecting the Special Prosecutor to disciplinary and However, by another vote of 8-7,109 the Court resolved to maintain the validity of Section 8(2) of
removal powers of the President, whose own alter egos and officials in the Executive Department RA No. 6770 insofar as Sulit is concerned. The Court did not consider the Office of the Special
Prosecutor to be constitutionally within the Office of the Ombudsman and is, hence, not entitled
to the independence the latter enjoys under the Constitution.

WHEREFORE, premises considered, the Court resolves to declare Section 8(2)


UNCONSTITUTIONAL. This ruling renders any further ruling on the dismissal of Deputy
Ombudsman Emilio Gonzales III unnecessary, but is without prejudice to the power of the
Ombudsman to conduct an administrative investigation, if warranted, into the possible
administrative liability of Deputy Ombudsman Emilio Gonzales III under pertinent Civil Service
laws, rules and regulations.

SO ORDERED.
[G.R. No. 142801-802. July 10, 2001] responsibility for anti-smuggling operations in all land areas and inland waters and waterways
outside the areas of sole jurisdiction of the Bureau of Customs. [6]
BUKLOD NG KAWANING EIIB, CESAR POSADA, REMEDIOS G. PRINCESA, BENJAMIN KHO, BENIGNO
MANGA, LULU MENDOZA, petitioners, vs. HON. EXECUTIVE SECRETARY RONALDO B. Eleven years after, or on January 7, 2000, President Joseph Estrada issued Executive Order No.
ZAMORA, HON. SECRETARY JOSE PARDO, DEPARTMENT OF FINANCE, HON. SECRETARY 191 entitled Deactivation of the Economic Intelligence and Investigation Bureau. [7] Motivated by the
BENJAMIN DIOKNO, DEPARTMENT OF BUDGET AND MANAGEMENT, HON. SECRETARY fact that the designated functions of the EIIB are also being performed by the other existing agencies
ARTEMIO TUQUERO, DEPARTMENT OF JUSTICE, respondents. of the government and that there is a need to constantly monitor the overlapping of functions
among these agencies, former President Estrada ordered the deactivation of EIIB and the transfer
DECISION of its functions to the Bureau of Customs and the National Bureau of Investigation.

SANDOVAL-GUTIERREZ, J.: Meanwhile, President Estrada issued Executive Order No. 196[8] creating the Presidential Anti-
Smuggling Task Force Aduana.[9]

In this petition for certiorari, prohibition and mandamus, petitioners Buklod Ng Kawaning EIIB, Then the day feared by the EIIB employees came. On March 29, 2000, President Estrada issued
Cesar Posada, Remedios Princesa, Benjamin Kho, Benigno Manga and Lulu Mendoza, for themselves Executive Order No. 223[10] providing that all EIIB personnel occupying positions specified therein
and in behalf of others with whom they share a common or general interest, seek the nullification shall be deemed separated from the service effective April 30, 2000, pursuant to a bona
of Executive Order No. 191[1] and Executive Order No. 223[2] on the ground that they were issued by fide reorganization resulting to abolition, redundancy, merger, division, or consolidation of
the Office of the President with grave abuse of discretion and in violation of their constitutional right positions.[11]
to security of tenure.
Agonizing over the loss of their employment, petitioners now come before this Court invoking
The facts are undisputed: our power of judicial review of Executive Order Nos. 191 and 223. They anchor their petition on the
following arguments:
On June 30, 1987, former President Corazon C. Aquino, issued Executive Order No.
127[3] establishing the Economic Intelligence and Investigation Bureau (EIIB) as part of the structural A
organization of the Ministry of Finance.[4] The EIIB was designated to perform the following
functions: Executive Order Nos. 191 and 223 should be annulled as they are unconstitutional for being
violative of Section 2(3), Article IX-B of the Philippine Constitution and/or for having been issued
(a) Receive, gather and evaluate intelligence reports and information and evidence on the with grave abuse of discretion amounting to lack or excess of jurisdiction.
nature, modes and extent of illegal activities affecting the national economy, such as, but
not limited to, economic sabotage, smuggling, tax evasion, and dollar-salting, investigate the B.
same and aid in the prosecution of cases;
The abolition of the EIIB is a hoax. Similarly, if Executive Order Nos. 191 and 223 are considered
(b) Coordinate with external agencies in monitoring the financial and economic activities of to effect a reorganization of the EIIB, such reorganization was made in bad faith.
persons or entities, whether domestic or foreign, which may adversely affect national
financial interest with the goal of regulating, controlling or preventing said activities;
C.

(c) Provide all intelligence units of operating Bureaus or Offices under the Ministry with the
general framework and guidelines in the conduct of intelligence and investigating works; The President has no authority to abolish the EIIB.

(d) Supervise, monitor and coordinate all the intelligence and investigation operations of the Petitioners contend that the issuance of the afore-mentioned executive orders is: (a) a
operating Bureaus and Offices under the Ministry; violation of their right to security of tenure; (b) tainted with bad faith as they were not actually
intended to make the bureaucracy more efficient but to give way to Task Force Aduana, the
functions of which are essentially and substantially the same as that of EIIB; and (c) a usurpation of
(e) Investigate, hear and file, upon clearance by the Minister, anti-graft and corruption cases the power of Congress to decide whether or not to abolish the EIIB.
against personnel of the Ministry and its constituents units;
Arguing in behalf of respondents, the Solicitor General maintains that: (a) the President
(f) Perform such other appropriate functions as may be assigned by the Minister or his enjoys the totality of the executive power provided under Sections 1 and 7, Article VII of the
deputies.[5] Constitution, thus, he has the authority to issue Executive Order Nos. 191 and 223; (b) the said
executive orders were issued in the interest of national economy, to avoid duplicity of work and to
streamline the functions of the bureaucracy; and (c) the EIIB was not abolished, it was
In a desire to achieve harmony of efforts and to prevent possible conflicts among agencies in only deactivated.
the course of their anti-smuggling operations, President Aquino issued Memorandum Order No. 225
on March 17, 1989, providing, among others, that the EIIB shall be the agency of primary The petition is bereft of merit.
Despite the presence of some procedural flaws in the instant petition, such as, petitioners Sec. 48. Scaling Down and Phase Out of Activities of Agencies Within the Executive Branch. The
disregard of the hierarchy of courts and the non-exhaustion of administrative remedies, we deem it heads of departments, bureaus and offices and agencies are hereby directed to identify their
necessary to address the issues. It is in the interest of the State that questions relating to the status respective activities which are no longer essential in the delivery of public services and which may
and existence of a public office be settled without delay. We are not without precedent. In Dario v. be scaled down, phased out or abolished, subject to civil service rules and regulations. X x x. Actual
Mison,[12] we liberally decreed: scaling down, phasing out or abolition of the activities shall be effected pursuant to Circulars or
Orders issued for the purpose by the Office of the President.
The Court disregards the questions raised as to procedure, failure to exhaust administrative
remedies, the standing of certain parties to sue, for two reasons, `[b]ecause of the demands of Said provision clearly mentions the acts of scaling down, phasing out and abolition of offices only
public interest, including the need for stability in the public service,' and because of the serious and does not cover the creation of offices or transfer of functions. Nevertheless, the act of creating
implications of these cases on the administration of the Philippine civil service and the rights of and decentralizing is included in the subsequent provision of Section 62 which provides that:
public servants.
Sec. 62. Unauthorized organizational charges.- Unless otherwise created by law or directed by the
At first glance, it seems that the resolution of this case hinges on the question - Does the President of the Philippines, no organizational unit or changes in key positions in any department
deactivation of EIIB constitute abolition of an office? However, after coming to terms with the or agency shall be authorized in their respective organization structures and be funded from
prevailing law and jurisprudence, we are certain that the ultimate queries should be a) Does the appropriations by this Act. (italics ours)
President have the authority to reorganize the executive department? and, b) How should the
reorganization be carried out? The foregoing provision evidently shows that the President is authorized to effect organizational
Surely, there exists a distinction between the words deactivate and abolish. changes including the creation of offices in the department or agency concerned.
To deactivate means to render inactive or ineffective or to break up by discharging or reassigning
personnel,[13] while to abolish means to do away with, to annul, abrogate or destroy xxxxxx
completely.[14] In essence, abolition denotes an intention to do away with the
office wholly and permanently.[15] Thus, while in abolition, the office ceases to exist, the same is not Another legal basis of E.O. No. 132 is Section 20, Book III of E.O. No. 292 which states:
true in deactivation where the office continues to exist, albeit remaining dormant or inoperative. Be
that as it may, deactivation and abolition are both reorganization measures.
Sec. 20. Residual Powers. Unless Congress provides otherwise, the President shall exercise such
The Solicitor General only invokes the above distinctions on the mistaken assumption that the other powers and functions vested in the President which are provided for under the laws and
President has no power to abolish an office. which are not specifically enumerated above or which are not delegated by the President in
accordance with law. (italic ours)
The general rule has always been that the power to abolish a public office is lodged with the
legislature.[16] This proceeds from the legal precept that the power to create includes the power to
destroy. A public office is either created by the Constitution, by statute, or by authority of This provision speaks of such other powers vested in the President under the law. What law then
law.[17] Thus, except where the office was created by the Constitution itself, it may be abolished by gives him the power to reorganize? It is Presidential Decree No. 1772 which amended
the same legislature that brought it into existence.[18] Presidential Decree No. 1416. These decrees expressly grant the President of the Philippines the
continuing authority to reorganize the national government, which includes the power to group,
The exception, however, is that as far as bureaus, agencies or offices in the executive consolidate bureaus and agencies, to abolish offices, to transfer functions, to create and classify
department are concerned, the Presidents power of control may justify him to inactivate the functions, services and activities and to standardize salaries and materials. The validity of these
functions of a particular office,[19] or certain laws may grant him the broad authority to carry out two decrees are unquestionable. The 1987 Constitution clearly provides that all laws, decrees,
reorganization measures.[20] The case in point is Larin v. Executive Secretary.[21] In this case, it was executive orders, proclamations, letters of instructions and other executive issuances not
argued that there is no law which empowers the President to reorganize the BIR. In decreeing inconsistent with this Constitution shall remain operative until amended, repealed or revoked.So
otherwise, this Court sustained the following legal basis, thus: far, there is yet no law amending or repealing said decrees. (Emphasis supplied)

Initially, it is argued that there is no law yet which empowers the President to issue E.O. No. 132 or Now, let us take a look at the assailed executive order.
to reorganize the BIR.
In the whereas clause of E.O. No. 191, former President Estrada anchored his authority to
deactivate EIIB on Section 77 of Republic Act 8745 (FY 1999 General Appropriations Act), a provision
We do not agree. similar to Section 62 of R.A. 7645 quoted in Larin, thus;

xxxxxx Sec. 77. Organized Changes. Unless otherwise provided by law or directed by the President of the
Philippines, no changes in key positions or organizational units in any department or agency shall
Section 48 of R.A. 7645 provides that: be authorized in their respective organizational structures and funded from appropriations
provided by this Act.
We adhere to the precedent or ruling in Larin that this provision recognizes the authority of Firstly, there is no employment of new personnel to man the Task Force. E.O. No. 196
the President to effect organizational changes in the department or agency under the executive provides that the technical, administrative and special staffs of EIIB are to be composed of people
structure. Such a ruling further finds support in Section 78 of Republic Act No. 8760. [22] Under this who are already in the public service, they being employees of other existing agencies. Their
law, the heads of departments, bureaus, offices and agencies and other entities in the Executive tenure with the Task Force would only be temporary, i.e., only when the agency where they
Branch are directed (a) to conduct a comprehensive review of their respective mandates, missions, belong is called upon to assist the Task Force. Since their employment with the Task force is only
objectives, functions, programs, projects, activities and systems and procedures; (b) identify by way of detail or assignment, they retain their employment with the existing agencies. And
activities which are no longer essential in the delivery of public services and which may be scaled should the need for them cease, they would be sent back to the agency concerned.
down, phased-out or abolished; and (c) adopt measures that will result in the streamlined
organization and improved overall performance of their respective agencies. [23] Section 78 ends up Secondly, the thrust of E.O. No. 196 is to have a small group of military men under the direct
with the mandate that the actual streamlining and productivity improvement in agency organization control and supervision of the President as base of the governments anti-smuggling campaign. Such
and operation shall be effected pursuant to Circulars or Orders issued for the purpose by the Office a smaller base has the necessary powers 1) to enlist the assistance of any department, bureau, or
of the President.[24] The law has spoken clearly. We are left only with the duty to sustain. office and to use their respective personnel, facilities and resources; and 2) to select and recruit
personnel from within the PSG and ISAFP for assignment to the Task Force. Obviously, the idea is
But of course, the list of legal basis authorizing the President to reorganize any department or to encourage the utilization of personnel, facilities and resources of the already existing
agency in the executive branch does not have to end here. We must not lose sight of the very source departments, agencies, bureaus, etc., instead of maintaining an independent office with a whole
of the power that which constitutes an express grant of power. Under Section 31, Book III of set of personnel and facilities. The EIIB had proven itself burdensome for the government because
Executive Order No. 292 (otherwise known as the Administrative Code of 1987), the President, it maintained separate offices in every region in the Philippines.
subject to the policy in the Executive Office and in order to
achieve simplicity, economy and efficiency, shall have the continuing authority to reorganize the And thirdly, it is evident from the yearly budget appropriation of the government that the
administrative structure of the Office of the President. For this purpose, he may transfer the creation of the Task Force Aduana was especially intended to lessen EIIBs expenses. Tracing from
functions of other Departments or Agencies to the Office of the President. In Canonizado v. the yearly General Appropriations Act, it appears that the allotted amount for the EIIBs general
Aguirre,[25] we ruled that reorganization involves the reduction of personnel, consolidation of administration, support, and operations for the year 1995, was P128,031,000;[31] for
offices, or abolition thereof by reason of economy or redundancy of functions. It takes place when 1996, P182,156,000;[32] for 1998, P219,889,000;[33] and, for 1999, P238,743,000.[34] These amounts
there is an alteration of the existing structure of government offices or units therein, including the were far above the P50,000,000[35] allocation to the Task Force Aduana for the year 2000.
lines of control, authority and responsibility between them. The EIIB is a bureau attached to the While basically, the functions of the EIIB have devolved upon the Task Force Aduana, we find
Department of Finance.[26] It falls under the Office of the President. Hence, it is subject to the the latter to have additional new powers. The Task Force Aduana, being composed of elements from
Presidents continuing authority to reorganize. the Presidential Security Group (PSG) and Intelligence Service Armed Forces of the Philippines
It having been duly established that the President has the authority to carry out reorganization (ISAFP),[36] has the essential power to effect searches, seizures and arrests. The EIIB did not have this
in any branch or agency of the executive department, what is then left for us to resolve is whether power. The Task Force Aduana has the power to enlist the assistance of any department, bureau,
or not the reorganization is valid. In this jurisdiction, reorganizations have been regarded as valid office, or instrumentality of the government, including government-owned or controlled
provided they are pursued in good faith. Reorganization is carried out in good faith if it is for the corporations; and to use their personnel, facilities and resources. Again, the EIIB did not have this
purpose of economy or to make bureaucracy more efficient.[27] Pertinently, Republic Act No. power. And, the Task Force Aduana has the additional authority to conduct investigation of cases
6656[28] provides for the circumstances which may be considered as evidence of bad faith in the involving ill-gotten wealth. This was not expressly granted to the EIIB.
removal of civil service employees made as a result of reorganization, to wit: (a) where there is a Consequently, it cannot be said that there is a feigned reorganization. In Blaquera v. Civil
significant increase in the number of positions in the new staffing pattern of the department or Sevice Commission, [37] we ruled that a reorganization in good faith is one designed to trim the fat
agency concerned; (b) where an office is abolished and another performing substantially the same off the bureaucracy and institute economy and greater efficiency in its operation.
functions is created; (c) where incumbents are replaced by those less qualified in terms of status of
appointment, performance and merit; (d) where there is a classification of offices in the department Lastly, we hold that petitioners right to security of tenure is not violated. Nothing is better
or agency concerned and the reclassified offices perform substantially the same functions as the settled in our law than that the abolition of an office within the competence of a legitimate body if
original offices, and (e) where the removal violates the order of separation.[29] done in good faith suffers from no infirmity. Valid abolition of offices is neither removal nor
separation of the incumbents.[38] In the instructive words laid down by this Court in Dario v.
Petitioners claim that the deactivation of EIIB was done in bad faith because four days after Mison,[39] through Justice Abraham F. Sarmiento:
its deactivation, President Estrada created the Task Force Aduana.

We are not convinced. Reorganizations in this jurisdiction have been regarded as valid provided they are pursued in good
faith. As a general rule, a reorganization is carried out in good faith if it is for the purpose of
An examination of the pertinent Executive Orders[30] shows that the deactivation of EIIB and economy or to make bureaucracy more efficient. In that event, no dismissal (in case of dismissal)
the creation of Task Force Aduana were done in good faith. It was not for the purpose of removing or separation actually occurs because the position itself ceases to exist. And in that case, security
the EIIB employees, but to achieve the ultimate purpose of E.O. No. 191, which is economy. While of tenure would not be a Chinese wall. Be that as it may, if the abolition, which is nothing else but
Task Force Aduana was created to take the place of EIIB, its creation does not entail expense to the a separation or removal, is done for political reasons or purposely to defeat security of tenure,
government. otherwise not in good faith, no valid abolition takes and whatever abolition is done, is void ab
initio. There is an invalid abolition as where there is merely a change of nomenclature of positions,
or where claims of economy are belied by the existence of ample funds.

Indeed, there is no such thing as an absolute right to hold office. Except constitutional offices
which provide for special immunity as regards salary and tenure, no one can be said to have any
vested right in an office or its salary.[40]

While we cast a commiserating look upon the plight of all the EIIB employees whose lives
perhaps are now torn with uncertainties, we cannot ignore the unfortunate reality that our
government is also battling the impact of a plummeting economy. Unless the government is given
the chance to recuperate by instituting economy and efficiency in its system, the EIIB will not be the
last agency to suffer the impact. We cannot frustrate valid measures which are designed to rebuild
the executive department.

WHEREFORE, the petition is hereby DENIED. No costs.

SO ORDERED.
PROF. RANDOLF S. DAVID, LORENZO TAADA III, RONALD G.R. No. 171396 USTAREZ, ANTONIO C. PASCUAL, SALVADOR T.
LLAMAS, H. HARRY L. ROQUE, JR., JOEL RUIZ BUTUYAN, CARRANZA, EMILIA P. DAPULANG, MARTIN CUSTODIO, JR.,
ROGER R. RAYEL, GARY S. MALLARI, ROMEL REGALADO AND ROQUE M. TAN,
BAGARES, CHRISTOPHER F.C. BOLASTIG, Petitioners,
Petitioners, - versus -
- versus -
HER EXCELLENCY, PRESIDENT GLORIA MACAPAGAL-
GLORIA MACAPAGAL-ARROYO, AS PRESIDENT ARROYO, THE HONORABLE EXECUTIVE SECRETARY,
AND COMMANDER-IN-CHIEF, EXECUTIVE SECRETARY EDUARDO ERMITA, THE CHIEF OF STAFF, ARMED FORCES
EDUARDO ERMITA, HON. AVELINO CRUZ II, SECRETARY OF OF THE PHILIPPINES, GENEROSO SENGA, AND THE PNP
NATIONAL DEFENSE, GENERAL GENEROSO SENGA, CHIEF DIRECTOR GENERAL, ARTURO LOMIBAO, Respondents.
OF STAFF, ARMED FORCES OF THE PHILIPPINES, DIRECTOR x-------------------------------------------------x
GENERAL ARTURO LOMIBAO, CHIEF, PHILIPPINE
NATIONAL POLICE, ALTERNATIVE LAW GROUPS, INC. (ALG), G.R. No. 171400
Respondents. Petitioner
x-------------------------------------------------x - versus -

NIEZ CACHO-OLIVARES AND TRIBUNE PUBLISHING CO., G.R. No. 171409 EXECUTIVE SECRETARY EDUARDO R. ERMITA, LT. GEN.
INC., GENEROSO SENGA, AND DIRECTOR GENERAL ARTURO
Petitioners, LOMIBAO,
- versus - Respondents.
x-------------------------------------------------x
HONORABLE SECRETARY EDUARDO ERMITA AND
HONORABLE DIRECTOR GENERAL ARTURO C. LOMIBAO, JOSE ANSELMO I. CADIZ, FELICIANO M. BAUTISTA, G.R. No. 171489
Respondents. ROMULO R. RIVERA, JOSE AMOR M. AMORADO, ALICIA A.
x-------------------------------------------------x RISOS-VIDAL, FELIMON C. ABELITA III, MANUEL P.
LEGASPI, J.B. JOVY C. BERNABE, BERNARD L. DAGCUTA,
FRANCIS JOSEPH G. ESCUDERO, JOSEPH A. SANTIAGO, G.R. No. 171485 ROGELIO V. GARCIA AND INTEGRATED BAR OF THE
TEODORO A. CASINO, AGAPITO A. AQUINO, MARIO J. PHILIPPINES (IBP),
AGUJA, SATUR C. OCAMPO, MUJIV S. HATAMAN, JUAN Petitioners,
EDGARDO ANGARA, TEOFISTO DL. GUINGONA III, - versus -
EMMANUEL JOSEL J. VILLANUEVA, LIZA L. MAZA, IMEE R.
MARCOS, RENATO B. MAGTUBO, JUSTIN MARC SB. HON. EXECUTIVE SECRETARY EDUARDO ERMITA,
CHIPECO, ROILO GOLEZ, DARLENE ANTONINO-CUSTODIO, GENERAL GENEROSO SENGA, IN HIS CAPACITY AS AFP
LORETTA ANN P. ROSALES, JOSEL G. VIRADOR, RAFAEL V. CHIEF OF STAFF, AND DIRECTOR GENERAL ARTURO
MARIANO, GILBERT C. REMULLA, FLORENCIO G. NOEL, LOMIBAO, IN HIS CAPACITY AS PNP CHIEF,
ANA THERESIA HONTIVEROS-BARAQUEL, IMELDA C. Respondents.
NICOLAS, MARVIC M.V.F. LEONEN, NERI JAVIER x-------------------------------------------------x
COLMENARES, MOVEMENT OF CONCERNED CITIZENS FOR
CIVIL LIBERTIES REPRESENTED BY AMADO GAT INCIONG, LOREN B. LEGARDA, G.R. No. 171424
Petitioners, Petitioner
- versus - - versus -

EDUARDO R. ERMITA, EXECUTIVE SECRETARY, AVELINO J. GLORIA MACAPAGAL-ARROYO, IN HER CAPACITY AS


CRUZ, JR., SECRETARY, DND RONALDO V. PUNO, PRESIDENT AND COMMANDER-IN-CHIEF; ARTURO
SECRETARY, DILG, GENEROSO SENGA, AFP CHIEF OF LOMIBAO, IN HIS CAPACITY AS DIRECTOR-GENERAL OF
STAFF, ARTURO LOMIBAO, CHIEF PNP, THE PHILIPPINE NATIONAL POLICE (PNP); GENEROSO
Respondents. SENGA, IN HIS CAPACITY AS CHIEF OF STAFF OF THE
x-------------------------------------------------x ARMED FORCES OF THE PHILIPPINES (AFP); AND EDUARDO
ERMITA, IN HIS CAPACITY AS EXECUTIVE SECRETARY,
KILUSANG MAYO UNO, REPRESENTED BY ITS G.R. No. 171483 Respondents.
CHAIRPERSON ELMER C. LABOG AND SECRETARY
GENERAL JOEL MAGLUNSOD, NATIONAL FEDERATION OF x---------------------------------------------------------------------------------------------x
LABOR UNIONS KILUSANG MAYO UNO (NAFLU-KMU),
REPRESENTED BY ITS NATIONAL PRESIDENT, JOSELITO V. DECISION
violence as well as any act of insurrection or rebellion and to enforce
SANDOVAL-GUTIERREZ, J.: obedience to all the laws and to all decrees, orders and regulations
promulgated by me personally or upon my direction; and as provided in
Section 17, Article 12 of the Constitution do hereby declare a State of
National Emergency.
All powers need some restraint; practical adjustments rather than rigid formula are

necessary.[1] Superior strength the use of force cannot make wrongs into rights. In this regard,
She cited the following facts as bases:
the courts should be vigilant in safeguarding the constitutional rights of the citizens, specifically their
WHEREAS, over these past months, elements in the political
liberty. opposition have conspired with authoritarians of the extreme Left
represented by the NDF-CPP-NPA and the extreme Right, represented by
military adventurists the historical enemies of the democratic Philippine
Chief Justice Artemio V. Panganibans philosophy of liberty is thus most relevant. He
State who are now in a tactical alliance and engaged in a concerted and
said: In cases involving liberty, the scales of justice should weigh heavily against government systematic conspiracy, over a broad front, to bring down the duly constituted
Government elected in May 2004;
and in favor of the poor, the oppressed, the marginalized, the dispossessed and the weak. Laws
WHEREAS, these conspirators have repeatedly tried to bring down
and actions that restrict fundamental rights come to the courts with a heavy presumption against the President;

their constitutional validity.[2] WHEREAS, the claims of these elements have been recklessly
magnified by certain segments of the national media;

WHEREAS, this series of actions is hurting the Philippine State


These seven (7) consolidated petitions for certiorari and prohibition allege that in issuing by obstructing governance including hindering the growth of the economy
Presidential Proclamation No. 1017 (PP 1017) and General Order No. 5 (G.O. No. 5), President and sabotaging the peoples confidence in government and their faith in
the future of this country;
Gloria Macapagal-Arroyo committed grave abuse of discretion. Petitioners contend that respondent
WHEREAS, these actions are adversely affecting the economy;
officials of the Government, in their professed efforts to defend and preserve democratic institutions,
WHEREAS, these activities give totalitarian forces of both the
are actually trampling upon the very freedom guaranteed and protected by the Constitution. Hence, extreme Left and extreme Right the opening to intensify their avowed aims
to bring down the democratic Philippine State;
such issuances are void for being unconstitutional.
WHEREAS, Article 2, Section 4 of the our Constitution makes the
Once again, the Court is faced with an age-old but persistently modern problem. How does the defense and preservation of the democratic institutions and the State the primary
duty of Government;
Constitution of a free people combine the degree of liberty, without which, law becomes tyranny, with

the degree of law, without which, liberty becomes license?[3] WHEREAS, the activities above-described, their consequences,
ramifications and collateral effects constitute a clear and present danger to the
safety and the integrity of the Philippine State and of the Filipino people;

On February 24, 2006, as the nation celebrated the 20th Anniversary of the Edsa People Power

I, President Arroyo issued PP 1017 declaring a state of national emergency, thus: On the same day, the President issued G. O. No. 5 implementing PP 1017, thus:

NOW, THEREFORE, I, Gloria Macapagal-Arroyo, President of the WHEREAS, over these past months, elements in the political opposition
Republic of the Philippines and Commander-in-Chief of the Armed Forces of have conspired with authoritarians of the extreme Left, represented by the NDF-
the Philippines, by virtue of the powers vested upon me by Section 18, Article 7 CPP-NPA and the extreme Right, represented by military adventurists - the
of the Philippine Constitution which states that: The President. . . whenever it historical enemies of the democratic Philippine State and who are now in a
becomes necessary, . . . may call out (the) armed forces to prevent or suppress. . tactical alliance and engaged in a concerted and systematic conspiracy, over a
.rebellion. . ., and in my capacity as their Commander-in-Chief, do hereby broad front, to bring down the duly-constituted Government elected in May
command the Armed Forces of the Philippines, to maintain law and order 2004;
throughout the Philippines, prevent or suppress all forms of lawless
WHEREAS, these conspirators have repeatedly tried to bring down our WHEREAS, by virtue of General Order No.5 and No.6 dated
republican government; February 24, 2006, which were issued on the basis of Proclamation No. 1017,
the Armed Forces of the Philippines (AFP) and the Philippine National Police
WHEREAS, the claims of these elements have been recklessly (PNP), were directed to maintain law and order throughout the Philippines,
magnified by certain segments of the national media; prevent and suppress all form of lawless violence as well as any act of
rebellion and to undertake such action as may be necessary;
WHEREAS, these series of actions is hurting the Philippine State by
obstructing governance, including hindering the growth of the economy and WHEREAS, the AFP and PNP have effectively prevented,
sabotaging the peoples confidence in the government and their faith in the suppressed and quelled the acts lawless violence and rebellion;
future of this country;
NOW, THEREFORE, I, GLORIA MACAPAGAL-
WHEREAS, these actions are adversely affecting the economy; ARROYO, President of the Republic of the Philippines, by virtue of the
powers vested in me by law, hereby declare that the state of national
WHEREAS, these activities give totalitarian forces; of both the emergency has ceased to exist.
extreme Left and extreme Right the opening to intensify their avowed aims to
bring down the democratic Philippine State;

WHEREAS, Article 2, Section 4 of our Constitution makes the In their presentation of the factual bases of PP 1017 and G.O. No. 5, respondents stated
defense and preservation of the democratic institutions and the State the primary
duty of Government; that the proximate cause behind the executive issuances was the conspiracy among some military

officers, leftist insurgents of the New Peoples Army (NPA), and some members of the political
WHEREAS, the activities above-described, their consequences,
ramifications and collateral effects constitute a clear and present danger to the opposition in a plot to unseat or assassinate President Arroyo. [4] They considered the aim to oust or
safety and the integrity of the Philippine State and of the Filipino people;
assassinate the President and take-over the reigns of government as a clear and present danger.
WHEREAS, Proclamation 1017 date February 24, 2006 has been
issued declaring a State of National Emergency;

NOW, THEREFORE, I GLORIA MACAPAGAL-ARROYO, by During the oral arguments held on March 7, 2006, the Solicitor General specified the facts
virtue of the powers vested in me under the Constitution as President of the
Republic of the Philippines, and Commander-in-Chief of the Republic of the leading to the issuance of PP 1017 and G.O. No. 5. Significantly, there was no refutation from
Philippines, and pursuant to Proclamation No. 1017 dated February 24, 2006,
do hereby call upon the Armed Forces of the Philippines (AFP) and the petitioners counsels.
Philippine National Police (PNP), to prevent and suppress acts of terrorism and
lawless violence in the country;
The Solicitor General argued that the intent of the Constitution is to give full discretionary
I hereby direct the Chief of Staff of the AFP and the Chief of the PNP,
as well as the officers and men of the AFP and PNP, to immediately carry out powers to the President in determining the necessity of calling out the armed forces. He emphasized
the necessary and appropriate actions and measures to suppress and
prevent acts of terrorism and lawless violence. that none of the petitioners has shown that PP 1017 was without factual bases. While he explained

that it is not respondents task to state the facts behind the questioned Proclamation, however, they

On March 3, 2006, exactly one week after the declaration of a state of national emergency are presenting the same, narrated hereunder, for the elucidation of the issues.

and after all these petitions had been filed, the President lifted PP 1017. She issued Proclamation On January 17, 2006, Captain Nathaniel Rabonza and First Lieutenants Sonny
No. 1021 which reads: Sarmiento, Lawrence San Juan and Patricio Bumidang, members of the Magdalo Group indicted in

the Oakwood mutiny, escaped their detention cell in Fort Bonifacio, Taguig City. In a public
WHEREAS, pursuant to Section 18, Article VII and Section 17,
Article XII of the Constitution, Proclamation No. 1017 dated February 24, statement, they vowed to remain defiant and to elude arrest at all costs. They called upon the people
2006, was issued declaring a state of national emergency;
to show and proclaim our displeasure at the sham regime. Let us demonstrate our disgust, not only Commander of the Armys elite Scout Ranger. Lim said it was all systems go for the planned

by going to the streets in protest, but also by wearing red bands on our left arms. [5] movement against Arroyo.[8]

On February 17, 2006, the authorities got hold of a document entitled Oplan Hackle I B/Gen. Danilo Lim and Brigade Commander Col. Ariel Querubin confided to Gen.

which detailed plans for bombings and attacks during the Philippine Military Academy Alumni Generoso Senga, Chief of Staff of the Armed Forces of the Philippines (AFP), that a huge number of

Homecoming in Baguio City. The plot was to assassinate selected targets including some cabinet soldiers would join the rallies to provide a critical mass and armed component to the Anti-Arroyo

members and President Arroyo herself.[6] Upon the advice of her security, President Arroyo decided protests to be held on February 24, 2005. According to these two (2) officers, there was no way they

not to attend the Alumni Homecoming. The next day, at the height of the celebration, a bomb was could possibly stop the soldiers because they too, were breaking the chain of command to join the

found and detonated at the PMA parade ground. forces foist to unseat the President. However, Gen. Senga has remained faithful to his Commander-

in-Chief and to the chain of command. He immediately took custody of B/Gen. Lim and directed

On February 21, 2006, Lt. San Juan was recaptured in a communist safehouse in Batangas Col. Querubin to return to the Philippine Marines Headquarters in Fort Bonifacio.

province. Found in his possession were two (2) flash disks containing minutes of the meetings

between members of the Magdalo Group and the National Peoples Army (NPA), a tape recorder, Earlier, the CPP-NPA called for intensification of political and revolutionary work within

audio cassette cartridges, diskettes, and copies of subversive documents. [7] Prior to his arrest, Lt. San the military and the police establishments in order to forge alliances with its members and key

Juan announced through DZRH that the Magdalos D-Day would be on February 24, 2006, the officials. NPA spokesman Gregorio Ka Roger Rosal declared: The Communist Party and

20th Anniversary of Edsa I. revolutionary movement and the entire people look forward to the possibility in the coming year of

accomplishing its immediate task of bringing down the Arroyo regime; of rendering it to weaken and

On February 23, 2006, PNP Chief Arturo Lomibao intercepted information that members unable to rule that it will not take much longer to end it.[9]

of the PNP- Special Action Force were planning to defect. Thus, he immediately ordered SAF

Commanding General Marcelino Franco, Jr. to disavow any defection. The latter promptly obeyed On the other hand, Cesar Renerio, spokesman for the National Democratic Front (NDF) at

and issued a public statement: All SAF units are under the effective control of responsible and North Central Mindanao, publicly announced: Anti-Arroyo groups within the military and police

trustworthy officers with proven integrity and unquestionable loyalty. are growing rapidly, hastened by the economic difficulties suffered by the families of AFP officers

and enlisted personnel who undertake counter-insurgency operations in the field. He claimed that

On the same day, at the house of former Congressman Peping Cojuangco, President Cory with the forces of the national democratic movement, the anti-Arroyo conservative political parties,

Aquinos brother, businessmen and mid-level government officials plotted moves to bring down the coalitions, plus the groups that have been reinforcing since June 2005, it is probable that the

Arroyo administration. Nelly Sindayen of TIME Magazine reported that Pastor Saycon, longtime Presidents ouster is nearing its concluding stage in the first half of 2006.

Arroyo critic, called a U.S. government official about his groups plans if President Arroyo is

ousted. Saycon also phoned a man code-named Delta. Saycon identified him as B/Gen. Danilo Lim, Respondents further claimed that the bombing of telecommunication towers and cell sites

in Bulacan and Bataan was also considered as additional factual basis for the issuance of PP 1017 and

G.O. No. 5. So is the raid of an army outpost in Benguet resulting in the death of three (3)
soldiers. And also the directive of the Communist Party of the Philippines ordering its front According to petitioner Kilusang Mayo Uno, the police cited PP 1017 as the ground for the

organizations to join 5,000 Metro Manila radicals and 25,000 more from the provinces in mass dispersal of their assemblies.

protests.[10]

During the dispersal of the rallyists along EDSA, police arrested (without warrant)

By midnight of February 23, 2006, the President convened her security advisers and several petitioner Randolf S. David, a professor at the University of the Philippines and newspaper columnist.

cabinet members to assess the gravity of the fermenting peace and order situation. She directed both Also arrested was his companion, Ronald Llamas, president of party-list Akbayan.

the AFP and the PNP to account for all their men and ensure that the chain of command remains solid

and undivided. To protect the young students from any possible trouble that might break loose on At around 12:20 in the early morning of February 25, 2006, operatives of the Criminal

the streets, the President suspended classes in all levels in the entire National Capital Region. Investigation and Detection Group (CIDG) of the PNP, on the basis of PP 1017 and G.O. No. 5, raided

the Daily Tribune offices in Manila. The raiding team confiscated news stories by reporters,

For their part, petitioners cited the events that followed after the issuance of PP 1017 documents, pictures, and mock-ups of the Saturday issue. Policemen from Camp Crame in Quezon

and G.O. No. 5. City were stationed inside the editorial and business offices of the newspaper; while policemen from

the Manila Police District were stationed outside the building.[13]

Immediately, the Office of the President announced the cancellation of all programs and

activities related to the 20th anniversary celebration of Edsa People Power I; and revoked the permits A few minutes after the search and seizure at the Daily Tribune offices, the police

to hold rallies issued earlier by the local governments. Justice Secretary Raul Gonzales stated that surrounded the premises of another pro-opposition paper, Malaya, and its sister publication, the

political rallies, which to the Presidents mind were organized for purposes of destabilization, are tabloid Abante.

cancelled. Presidential Chief of Staff Michael Defensor announced that warrantless arrests and

take-over of facilities, including media, can already be implemented.[11] The raid, according to Presidential Chief of Staff Michael Defensor, is meant to show a

strong presence, to tell media outlets not to connive or do anything that would help the rebels in

Undeterred by the announcements that rallies and public assemblies would not be allowed, bringing down this government. The PNP warned that it would take over any media organization

groups of protesters (members of Kilusang Mayo Uno [KMU] and National Federation of Labor that would not follow standards set by the government during the state of national

Unions-Kilusang Mayo Uno [NAFLU-KMU]), marched from various parts of Metro Manila with the emergency. Director General Lomibao stated that if they do not follow the standards and the

intention of converging at the EDSA shrine. Those who were already near the EDSA site were standards are - if they would contribute to instability in the government, or if they do not subscribe

violently dispersed by huge clusters of anti-riot police. The well-trained policemen used truncheons, to what is in General Order No. 5 and Proc. No. 1017 we will recommend a

big fiber glass shields, water cannons, and tear gas to stop and break up the marching groups, and takeover. National Telecommunications Commissioner Ronald Solis urged television and

scatter the massed participants. The same police action was used against the protesters marching radio networks to cooperate with the government for the duration of the state of national

forward to Cubao, Quezon City and to the corner of Santolan Street and EDSA. That same evening, emergency. He asked for balanced reporting from broadcasters when covering the events

hundreds of riot policemen broke up an EDSA celebration rally held along Ayala Avenue and Paseo surrounding the coup attempt foiled by the government. He warned that his agency will not hesitate
[12]
de Roxas Street in Makati City.
to recommend the closure of any broadcast outfit that violates rules set out for media coverage when On March 3, 2006, President Arroyo issued PP 1021 declaring that the state of national

the national security is threatened.[14] emergency has ceased to exist.

Also, on February 25, 2006, the police arrested Congressman Crispin Beltran, representing In the interim, these seven (7) petitions challenging the constitutionality of PP 1017 and

the Anakpawis Party and Chairman of Kilusang Mayo Uno (KMU), while leaving his farmhouse in G.O. No. 5 were filed with this Court against the above-named respondents. Three (3) of these

Bulacan. The police showed a warrant for his arrest dated 1985. Beltrans lawyer explained that petitions impleaded President Arroyo as respondent.

the warrant, which stemmed from a case of inciting to rebellion filed during the Marcos regime, had

long been quashed. Beltran, however, is not a party in any of these petitions. In G.R. No. 171396, petitioners Randolf S. David, et al. assailed PP 1017 on the grounds

that (1) it encroaches on the emergency powers of Congress; (2) it is a subterfuge to avoid the

When members of petitioner KMU went to Camp Crame to visit Beltran, they were told constitutional requirements for the imposition of martial law; and (3) it violates the constitutional

they could not be admitted because of PP 1017 and G.O. No. 5. Two members were arrested and guarantees of freedom of the press, of speech and of assembly.

detained, while the rest were dispersed by the police.

In G.R. No. 171409, petitioners Ninez Cacho-Olivares and Tribune Publishing Co.,

Bayan Muna Representative Satur Ocampo eluded arrest when the police went after him Inc. challenged the CIDGs act of raiding the Daily Tribune offices as a clear case of censorship

during a public forum at the Sulo Hotel in Quezon City. But his two drivers, identified as Roel and or prior restraint. They also claimed that the term emergency refers only to tsunami, typhoon,

Art, were taken into custody. hurricane and similar occurrences, hence, there is absolutely no emergency that warrants the

issuance of PP 1017.

Retired Major General Ramon Montao, former head of the Philippine Constabulary, was

arrested while with his wife and golfmates at the Orchard Golf and Country Club in Dasmarias, In G.R. No. 171485, petitioners herein are Representative Francis Joseph G. Escudero, and

Cavite. twenty one (21) other members of the House of Representatives, including Representatives Satur

Ocampo, Rafael Mariano, Teodoro Casio, Liza Maza, and Josel Virador. They asserted that PP

Attempts were made to arrest Anakpawis Representative Satur Ocampo, Representative 1017 and G.O. No. 5 constitute usurpation of legislative powers; violation of freedom of

Rafael Mariano, Bayan Muna Representative Teodoro Casio and Gabriela Representative Liza expression and a declaration of martial law. They alleged that President Arroyo gravely

Maza. Bayan Muna Representative Josel Virador was arrested at the PAL Ticket Office in Davao abused her discretion in calling out the armed forces without clear and verifiable factual basis of the

City. Later, he was turned over to the custody of the House of Representatives where the Batasan possibility of lawless violence and a showing that there is necessity to do so.

5 decided to stay indefinitely.

In G.R. No. 171483, petitioners KMU, NAFLU-KMU, and their members averred that PP

Let it be stressed at this point that the alleged violations of the rights of Representatives 1017 and G.O. No. 5 are unconstitutional because (1) they arrogate unto President Arroyo the power

Beltran, Satur Ocampo, et al., are not being raised in these petitions. to enact laws and decrees; (2) their issuance was without factual basis; and (3) they violate freedom

of expression and the right of the people to peaceably assemble to redress their grievances.
In G.R. No. 171400, petitioner Alternative Law Groups, Inc. (ALGI) alleged that PP 1017 1) Whether the issuance of PP 1021 renders the petitions moot and

and G.O. No. 5 are unconstitutional because they violate (a) Section 4[15] of Article II, (b)Sections academic.

1,[16] 2,[17] and 4[18] of Article III, (c) Section 23[19] of Article VI, and (d) Section 17[20] of Article 2) Whether petitioners in 171485 (Escudero et al.), G.R. Nos.

XII of the Constitution. 171400 (ALGI), 171483 (KMU et al.), 171489 (Cadiz et al.),

and 171424 (Legarda) have legal standing.

In G.R. No. 171489, petitioners Jose Anselmo I. Cadiz et al., alleged that PP 1017 is an B. SUBSTANTIVE:

arbitrary and unlawful exercise by the President of her Martial Law powers. And assuming that 1) Whether the Supreme Court can review the factual bases of PP

PP 1017 is not really a declaration of Martial Law, petitioners argued that it amounts to an exercise 1017.

by the President of emergency powers without congressional approval. In addition, petitioners 2) Whether PP 1017 and G.O. No. 5 are unconstitutional.
a. Facial Challenge
asserted that PP 1017 goes beyond the nature and function of a proclamation as defined under the b. Constitutional Basis
c. As Applied Challenge
Revised Administrative Code.
A. PROCEDURAL

And lastly, in G.R. No. 171424, petitioner Loren B. Legarda maintained that PP 1017 and
First, we must resolve the procedural roadblocks.
G.O. No. 5 are unconstitutional for being violative of the freedom of expression, including its

cognate rights such as freedom of the press and the right to access to information on matters of public
I- Moot and Academic Principle
concern, all guaranteed under Article III, Section 4 of the 1987 Constitution. In this regard, she

stated that these issuances prevented her from fully prosecuting her election protest pending before
One of the greatest contributions of the American system to this country is the concept of judicial
the Presidential Electoral Tribunal.
review enunciated in Marbury v. Madison.[21] This concept rests on the extraordinary simple

foundation --
In respondents Consolidated Comment, the Solicitor General countered that: first, the

petitions should be dismissed for being moot; second, petitioners in G.R. Nos. 171400 The Constitution is the supreme law. It was ordained by the people, the
ultimate source of all political authority. It confers limited powers on the
(ALGI), 171424 (Legarda), 171483 (KMU et al.), 171485 (Escudero et al.) and 171489 (Cadiz et al.) national government. x x x If the government consciously or unconsciously
oversteps these limitations there must be some authority competent to hold
have no legal standing; third, it is not necessary for petitioners to implead President Arroyo as it in control, to thwart its unconstitutional attempt, and thus to vindicate
and preserve inviolate the will of the people as expressed in the
respondent; fourth, PP 1017 has constitutional and legal basis; and fifth, PP 1017 does not violate Constitution. This power the courts exercise. This is the beginning and the
end of the theory of judicial review.[22]
the peoples right to free expression and redress of grievances.

But the power of judicial review does not repose upon the courts a self-starting
On March 7, 2006, the Court conducted oral arguments and heard the parties on the above
capacity.[23] Courts may exercise such power only when the following requisites are
interlocking issues which may be summarized as follows:
present: first, there must be an actual case or controversy; second, petitioners have to raise a question

of constitutionality; third, the constitutional question must be raised at the earliest opportunity;
A. PROCEDURAL:
and fourth, the decision of the constitutional question must be necessary to the determination of the if: first, there is a grave violation of the Constitution;[31] second, the exceptional character of the

case itself.[24] situation and the paramount public interest is involved;[32] third, when constitutional issue raised

requires formulation of controlling principles to guide the bench, the bar, and the

Respondents maintain that the first and second requisites are absent, hence, we shall limit public;[33] and fourth, the case is capable of repetition yet evading review.[34]

our discussion thereon.

All the foregoing exceptions are present here and justify this Courts assumption of

An actual case or controversy involves a conflict of legal right, an opposite legal claims jurisdiction over the instant petitions. Petitioners alleged that the issuance of PP 1017 and G.O. No.

susceptible of judicial resolution. It is definite and concrete, touching the legal relations of parties 5 violates the Constitution. There is no question that the issues being raised affect the publics

having adverse legal interest; a real and substantial controversy admitting of specific interest, involving as they do the peoples basic rights to freedom of expression, of assembly and of
[25]
relief. The Solicitor General refutes the existence of such actual case or controversy, contending the press. Moreover, the Court has the duty to formulate guiding and controlling constitutional

that the present petitions were rendered moot and academic by President Arroyos issuance of precepts, doctrines or rules. It has the symbolic function of educating the bench and the bar, and in

PP 1021. the present petitions, the military and the police, on the extent of the protection given by

constitutional guarantees.[35] And lastly, respondents contested actions are capable of

Such contention lacks merit. repetition. Certainly, the petitions are subject to judicial review.

A moot and academic case is one that ceases to present a justiciable controversy by virtue In their attempt to prove the alleged mootness of this case, respondents cited Chief Justice

of supervening events,[26] so that a declaration thereon would be of no practical use or Artemio V. Panganibans Separate Opinion in Sanlakas v. Executive Secretary.[36] However, they

value.[27] Generally, courts decline jurisdiction over such case[28] or dismiss it on ground of failed to take into account the Chief Justices very statement that an otherwise moot case may

mootness.[29] still be decided provided the party raising it in a proper case has been and/or continues to be

prejudiced or damaged as a direct result of its issuance. The present case falls right within this

The Court holds that President Arroyos issuance of PP 1021 did not render the present exception to the mootness rule pointed out by the Chief Justice.

petitions moot and academic. During the eight (8) days that PP 1017 was operative, the police

officers, according to petitioners, committed illegal acts in implementing it. Are PP 1017 and G.O. II- Legal Standing

No. 5 constitutional or valid? Do they justify these alleged illegal acts? These are the vital issues

that must be resolved in the present petitions. It must be stressed that an unconstitutional act is In view of the number of petitioners suing in various personalities, the Court deems it

not a law, it confers no rights, it imposes no duties, it affords no protection; it is in legal imperative to have a more than passing discussion on legal standing or locus standi.

contemplation, inoperative.[30]

Locus standi is defined as a right of appearance in a court of justice on a given

The moot and academic principle is not a magical formula that can automatically question.[37] In private suits, standing is governed by the real-parties-in interest rule as

dissuade the courts in resolving a case. Courts will decide cases, otherwise moot and academic, contained in Section 2, Rule 3 of the 1997 Rules of Civil Procedure, as amended. It provides that
every action must be prosecuted or defended in the name of the real party in that for a private individual to invoke the judicial power to determine the validity of an executive or

interest. Accordingly, the real-party-in interest is the party who stands to be benefited or legislative action, he must show that he has sustained a direct injury as a result of that action,

injured by the judgment in the suit or the party entitled to the avails of the suit.[38]Succinctly and it is not sufficient that he has a general interest common to all members of the public.

put, the plaintiffs standing is based on his own right to the relief sought.

This Court adopted the direct injury test in our jurisdiction. In People v. Vera,[44] it

The difficulty of determining locus standi arises in public suits. Here, the plaintiff who held that the person who impugns the validity of a statute must have a personal and substantial

asserts a public right in assailing an allegedly illegal official action, does so as a representative of interest in the case such that he has sustained, or will sustain direct injury as a

the general public. He may be a person who is affected no differently from any other person. He result. The Vera doctrine was upheld in a litany of cases, such as, Custodio v. President of the

could be suing as a stranger, or in the category of a citizen, or taxpayer. In either case, he Senate,[45] Manila Race Horse Trainers Association v. De la Fuente,[46] Pascual v. Secretary of

has to adequately show that he is entitled to seek judicial protection. In other words, he has to make Public Works[47] and Anti-Chinese League of the Philippines v. Felix.[48]

out a sufficient interest in the vindication of the public order and the securing of relief as a citizen

or taxpayer. However, being a mere procedural technicality, the requirement of locus standi may be

waived by the Court in the exercise of its discretion. This was done in the 1949 Emergency Powers

Case law in most jurisdictions now allows both citizen and taxpayer standing in public Cases, Araneta v. Dinglasan,[49] where the transcendental importance of the cases prompted the

actions. The distinction was first laid down in Beauchamp v. Silk,[39] where it was held that the Court to act liberally. Such liberality was neither a rarity nor accidental. In Aquino v.

plaintiff in a taxpayers suit is in a different category from the plaintiff in a citizens suit. In the Comelec,[50] this Court resolved to pass upon the issues raised due to the far-reaching

former, the plaintiff is affected by the expenditure of public funds, while in the latter, he is but implications of the petition notwithstanding its categorical statement that petitioner therein had no

the mere instrument of the public concern. As held by the New York Supreme Court in People personality to file the suit. Indeed, there is a chain of cases where this liberal policy has been

ex rel Case v. Collins:[40] In matter of mere public right, howeverthe people are the real observed, allowing ordinary citizens, members of Congress, and civic organizations to prosecute

partiesIt is at least the right, if not the duty, of every citizen to interfere and see that a public actions involving the constitutionality or validity of laws, regulations and rulings.[51]

offence be properly pursued and punished, and that a public grievance be remedied. With

respect to taxpayers suits, Terr v. Jordan[41] held that the right of a citizen and a taxpayer to Thus, the Court has adopted a rule that even where the petitioners have failed to show direct

maintain an action in courts to restrain the unlawful use of public funds to his injury cannot be injury, they have been allowed to sue under the principle of transcendental importance. Pertinent

denied. are the following cases:


(1) Chavez v. Public Estates Authority,[52] where the Court ruled
that the enforcement of the constitutional right to information and the
However, to prevent just about any person from seeking judicial interference in any official equitable diffusion of natural resources are matters of transcendental
importance which clothe the petitioner with locus standi;
policy or act with which he disagreed with, and thus hinders the activities of governmental agencies

engaged in public service, the United State Supreme Court laid down the more stringent direct (2) Bagong Alyansang Makabayan v. Zamora,[53] wherein the Court
held that given the transcendental importance of the issues involved, the
injury test in Ex Parte Levitt,[42] later reaffirmed in Tileston v. Ullman.[43] The same Court ruled Court may relax the standing requirements and allow the suit to prosper
despite the lack of direct injury to the parties seeking judicial review of
the Visiting Forces Agreement; taxpayer absent any allegation that public funds are being misused. Nor can it sue as a concerned

citizen as it does not allege any specific injury it has suffered.


(3) Lim v. Executive Secretary,[54] while the Court noted that the
petitioners may not file suit in their capacity as taxpayers absent a showing that
Balikatan 02-01 involves the exercise of Congress taxing or spending
powers, it reiterated its ruling in Bagong Alyansang Makabayan v. In Telecommunications and Broadcast Attorneys of the Philippines, Inc. v. Comelec,[57] the
Zamora,[55] that in cases of transcendental importance, the cases must be
settled promptly and definitely and standing requirements may be relaxed. Court reiterated the direct injury test with respect to concerned citizens cases involving

constitutional issues. It held that there must be a showing that the citizen personally suffered some

actual or threatened injury arising from the alleged illegal official act.
By way of summary, the following rules may be culled from the cases decided by this

Court. Taxpayers, voters, concerned citizens, and legislators may be accorded standing to sue, In Lacson v. Perez,[58] the Court ruled that one of the petitioners, Laban ng Demokratikong
provided that the following requirements are met: Pilipino (LDP), is not a real party-in-interest as it had not demonstrated any injury to itself or to its

leaders, members or supporters.


(1) the cases involve constitutional issues;

In Sanlakas v. Executive Secretary,[59] the Court ruled that only the petitioners who are
(2) for taxpayers, there must be a claim of illegal disbursement of public funds
members of Congress have standing to sue, as they claim that the Presidents declaration of a state
or that the tax measure is unconstitutional;
of rebellion is a usurpation of the emergency powers of Congress, thus impairing their legislative

(3) for voters, there must be a showing of obvious interest in the validity of the powers. As to petitioners Sanlakas, Partido Manggagawa, and Social Justice Society, the Court

election law in question; declared them to be devoid of standing, equating them with the LDP in Lacson.

(4) for concerned citizens, there must be a showing that the issues raised are of Now, the application of the above principles to the present petitions.
transcendental importance which must be settled early; and

The locus standi of petitioners in G.R. No. 171396, particularly David and Llamas, is
(5) for legislators, there must be a claim that the official action complained of
beyond doubt. The same holds true with petitioners in G.R. No. 171409, Cacho-Olivares
infringes upon their prerogatives as legislators.
and Tribune Publishing Co. Inc. They alleged direct injury resulting from illegal arrest and

unlawful search committed by police operatives pursuant to PP 1017. Rightly so, the Solicitor
Significantly, recent decisions show a certain toughening in the Courts attitude toward
General does not question their legal standing.
legal standing.

In G.R. No. 171485, the opposition Congressmen alleged there was usurpation of legislative
In Kilosbayan, Inc. v. Morato,[56] the Court ruled that the status of Kilosbayan as a peoples
powers. They also raised the issue of whether or not the concurrence of Congress is necessary
organization does not give it the requisite personality to question the validity of the on-line lottery
whenever the alarming powers incident to Martial Law are used. Moreover, it is in the interest of
contract, more so where it does not raise any issue of constitutionality. Moreover, it cannot sue as a
justice that those affected by PP 1017 can be represented by their Congressmen in bringing to the these issuances prevented her from pursuing her occupation. Her submission that she has pending

attention of the Court the alleged violations of their basic rights. electoral protest before the Presidential Electoral Tribunal is likewise of no relevance. She has not

sufficiently shown that PP 1017 will affect the proceedings or result of her case. But considering

In G.R. No. 171400, (ALGI), this Court applied the liberality rule in Philconsa v. once more the transcendental importance of the issue involved, this Court may relax the standing

Enriquez,[60] Kapatiran Ng Mga Naglilingkod sa Pamahalaan ng Pilipinas, Inc. v. Tan,[61]Association rules.

of Small Landowners in the Philippines, Inc. v. Secretary of Agrarian Reform,[62] Basco v. Philippine

Amusement and Gaming Corporation,[63] and Taada v. Tuvera,[64] that when the issue concerns a It must always be borne in mind that the question of locus standi is but corollary to the

public right, it is sufficient that the petitioner is a citizen and has an interest in the execution of the bigger question of proper exercise of judicial power. This is the underlying legal tenet of the

laws. liberality doctrine on legal standing. It cannot be doubted that the validity of PP No. 1017 and

G.O. No. 5 is a judicial question which is of paramount importance to the Filipino people. To

In G.R. No. 171483, KMUs assertion that PP 1017 and G.O. No. 5 violated its right to paraphrase Justice Laurel, the whole of Philippine society now waits with bated breath the ruling of

peaceful assembly may be deemed sufficient to give it legal standing. Organizations may be this Court on this very critical matter. The petitions thus call for the application of the

granted standing to assert the rights of their members.[65] We take judicial notice of the transcendental importance doctrine, a relaxation of the standing requirements for the petitioners

announcement by the Office of the President banning all rallies and canceling all permits for public in the PP 1017 cases.

assemblies following the issuance of PP 1017 and G.O. No. 5.

This Court holds that all the petitioners herein have locus standi.

In G.R. No. 171489, petitioners, Cadiz et al., who are national officers of the Integrated

Bar of the Philippines (IBP) have no legal standing, having failed to allege any direct or potential Incidentally, it is not proper to implead President Arroyo as respondent. Settled is the

injury which the IBP as an institution or its members may suffer as a consequence of the issuance of doctrine that the President, during his tenure of office or actual incumbency, [67] may not be sued

PP No. 1017 and G.O. No. 5. In Integrated Bar of the Philippines v. Zamora,[66] the Court held that in any civil or criminal case, and there is no need to provide for it in the Constitution or law. It will

the mere invocation by the IBP of its duty to preserve the rule of law and nothing more, while degrade the dignity of the high office of the President, the Head of State, if he can be dragged into

undoubtedly true, is not sufficient to clothe it with standing in this case. This is too general an interest court litigations while serving as such. Furthermore, it is important that he be freed from any form

which is shared by other groups and the whole citizenry. However, in view of the transcendental of harassment, hindrance or distraction to enable him to fully attend to the performance of his official

importance of the issue, this Court declares that petitioner have locus standi. duties and functions. Unlike the legislative and judicial branch, only one constitutes the executive

branch and anything which impairs his usefulness in the discharge of the many great and important

In G.R. No. 171424, Loren Legarda has no personality as a taxpayer to file the instant duties imposed upon him by the Constitution necessarily impairs the operation of the

petition as there are no allegations of illegal disbursement of public funds. The fact that she is a Government. However, this does not mean that the President is not accountable to anyone. Like any

former Senator is of no consequence. She can no longer sue as a legislator on the allegation that her other official, he remains accountable to the people[68] but he may be removed from office only in the

prerogatives as a lawmaker have been impaired by PP 1017 and G.O. No. 5. Her claim that she is a mode provided by law and that is by impeachment.[69]

media personality will not likewise aid her because there was no showing that the enforcement of
B. SUBSTANTIVE The Integrated Bar of the Philippines v. Zamora[80] -- a recent case most pertinent to these

cases at bar -- echoed a principle similar to Lansang. While the Court considered the Presidents
I. Review of Factual Bases
calling-out power as a discretionary power solely vested in his wisdom, it stressed that this does
Petitioners maintain that PP 1017 has no factual basis. Hence, it was not necessary for
not prevent an examination of whether such power was exercised within permissible
President Arroyo to issue such Proclamation.
constitutional limits or whether it was exercised in a manner constituting grave abuse of

discretion. This ruling is mainly a result of the Courts reliance on Section 1, Article VIII of
The issue of whether the Court may review the factual bases of the Presidents exercise of
1987 Constitution which fortifies the authority of the courts to determine in an appropriate action the
his Commander-in-Chief power has reached its distilled point - from the indulgent days of Barcelon
validity of the acts of the political departments. Under the new definition of judicial power, the courts
v. Baker[70] nd Montenegro v. Castaneda[71] to the volatile era of Lansang v. Garcia,[72] Aquino, Jr.
are authorized not only to settle actual controversies involving rights which are legally demandable
v. Enrile,[73] and Garcia-Padilla v. Enrile.[74] The tug-of-war always cuts across the line defining
and enforceable, but also to determine whether or not there has been a grave abuse of
political questions, particularly those questions in regard to which full discretionary authority
discretion amounting to lack or excess of jurisdiction on the part of any branch or
has been delegated to the legislative or executive branch of the government.[75] Barcelon and
instrumentality of the government. The latter part of the authority represents a broadening of
Montenegro were in unison in declaring that the authority to decide whether an exigency has arisen
judicial power to enable the courts of justice to review what was before a forbidden territory, to wit,
belongs to the President and his decision is final and conclusive on the courts. Lansang took the [81]
the discretion of the political departments of the government. It speaks of judicial prerogative not
opposite view. There, the members of the Court were unanimous in the conviction that the Court has
only in terms of power but also of duty.[82]
the authority to inquire into the existence of factual bases in order to determine their constitutional
As to how the Court may inquire into the Presidents exercise of power, Lansang adopted
sufficiency. From the principle of separation of powers, it shifted the focus to the system of
the test that judicial inquiry can go no further than to satisfy the Court not that the Presidents
checks and balances, under which the President is supreme, x x x only if and when he acts

within the sphere allotted to him by the Basic Law, and the authority to determine decision is correct, but that the President did not act arbitrarily. Thus, the standard laid down is

whether or not he has so acted is vested in the Judicial Department, which in this respect, not correctness, but arbitrariness.[83] In Integrated Bar of the Philippines, this Court further ruled that

is, in turn, constitutionally supreme.[76] In 1973, the unanimous Court it is incumbent upon the petitioner to show that the Presidents decision is totally bereft of
[77]
of Lansang was divided in Aquino v. Enrile. There, the Court was almost evenly
factual basis and that if he fails, by way of proof, to support his assertion, then this Court cannot
divided on the issue of whether the validity of the imposition of Martial Law is a
undertake an independent investigation beyond the pleadings.
political or justiciable question.[78] Then came Garcia-Padilla v. Enrile which greatly

diluted Lansang. It declared that there is a need to re-examine the latter case, ratiocinating that in Petitioners failed to show that President Arroyos exercise of the calling-out power, by

times of war or national emergency, the President must be given absolute control for the very issuing PP 1017, is totally bereft of factual basis. A reading of the Solicitor Generals Consolidated

life of the nation and the government is in great peril. The President, it intoned, is answerable Comment and Memorandum shows a detailed narration of the events leading to the issuance of PP

only to his conscience, the People, and God.[79] 1017, with supporting reports forming part of the records. Mentioned are the escape of the Magdalo

Group, their audacious threat of the Magdalo D-Day, the defections in the military, particularly in the

Philippine Marines, and the reproving statements from the communist leaders. There was also the
Minutes of the Intelligence Report and Security Group of the Philippine Army showing the growing Jean-Jacques Rousseau also assumed the need for temporary suspension of democratic
alliance between the NPA and the military. Petitioners presented nothing to refute such
processes of government in time of emergency. According to him:
events. Thus, absent any contrary allegations, the Court is convinced that the President was justified

in issuing PP 1017 calling for military aid. The inflexibility of the laws, which prevents them from adopting
themselves to circumstances, may, in certain cases, render them disastrous and
make them bring about, at a time of crisis, the ruin of the State
Indeed, judging the seriousness of the incidents, President Arroyo was not expected to
It is wrong therefore to wish to make political institutions as strong as
simply fold her arms and do nothing to prevent or suppress what she believed was lawless violence, to render it impossible to suspend their operation. Even Sparta allowed its law
to lapse...
invasion or rebellion. However, the exercise of such power or duty must not stifle liberty.
If the peril is of such a kind that the paraphernalia of the laws are an
II. Constitutionality of PP 1017 and G.O. No. 5 obstacle to their preservation, the method is to nominate a supreme lawyer, who
shall silence all the laws and suspend for a moment the sovereign authority. In
Doctrines of Several Political Theorists such a case, there is no doubt about the general will, and it clear that the
on the Power of the President peoples first intention is that the State shall not perish. [86]
in Times of Emergency

Rosseau did not fear the abuse of the emergency dictatorship or supreme magistracy
This case brings to fore a contentious subject -- the power of the President in times of as he termed it. For him, it would more likely be cheapened by indiscreet use. He was unwilling

emergency. A glimpse at the various political theories relating to this subject provides an adequate to rely upon an appeal to heaven. Instead, he relied upon a tenure of office of prescribed duration

backdrop for our ensuing discussion. to avoid perpetuation of the dictatorship.[87]

John Stuart Mill concluded his ardent defense of representative government: I am far
John Locke, describing the architecture of civil government, called upon the English
from condemning, in cases of extreme necessity, the assumption of absolute power in the form
doctrine of prerogative to cope with the problem of emergency. In times of danger to the nation,
of a temporary dictatorship.[88]
positive law enacted by the legislature might be inadequate or even a fatal obstacle to the promptness

of action necessary to avert catastrophe. In these situations, the Crown retained a prerogative power
Nicollo Machiavellis view of emergency powers, as one element in the whole scheme of
to act according to discretion for the public good, without the proscription of the law and
limited government, furnished an ironic contrast to the Lockean theory of prerogative. He recognized
sometimes even against it.[84] But Locke recognized that this moral restraint might not suffice to
and attempted to bridge this chasm in democratic political theory, thus:
avoid abuse of prerogative powers. Who shall judge the need for resorting to the prerogative
Now, in a well-ordered society, it should never be necessary to resort to
and how may its abuse be avoided? Here, Locke readily admitted defeat, suggesting that the extra constitutional measures; for although they may for a time be beneficial,
yet the precedent is pernicious, for if the practice is once established for good
people have no other remedy in this, as in all other cases where they have no judge on earth, objects, they will in a little while be disregarded under that pretext but for evil
purposes. Thus, no republic will ever be perfect if she has not by law provided
but to appeal to Heaven.[85] for everything, having a remedy for every emergency and fixed rules for
applying it.[89]
must be legitimate; he should not enjoy power to determine the existence of an emergency;

Machiavelli in contrast to Locke, Rosseau and Mill sought to incorporate into the emergency powers should be exercised under a strict time limitation; and last, the objective of

constitution a regularized system of standby emergency powers to be invoked with suitable checks emergency action must be the defense of the constitutional order.[97]

and controls in time of national danger. He attempted forthrightly to meet the problem of combining

a capacious reserve of power and speed and vigor in its application in time of emergency, with
Clinton L. Rossiter, after surveying the history of the employment of emergency powers in
effective constitutional restraints.[90]
Great Britain, France, Weimar, Germany and the United States, reverted to a description of a scheme

of constitutional dictatorship as solution to the vexing problems presented by emergency. [98] Like

Contemporary political theorists, addressing themselves to the problem of response to Watkins and Friedrich, he stated a priori the conditions of success of the constitutional

emergency by constitutional democracies, have employed the doctrine of constitutional dictatorship, thus:

dictatorship.[91] Frederick M. Watkins saw no reason why absolutism should not be used as a
1) No general regime or particular institution of
means for the defense of liberal institutions, provided it serves to protect established constitutional dictatorship should be initiated unless it is necessary or
even indispensable to the preservation of the State and its
institutions from the danger of permanent injury in a period of temporary emergency and is constitutional order
followed by a prompt return to the previous forms of political life.[92] He recognized the two
2) the decision to institute a constitutional dictatorship
(2) key elements of the problem of emergency governance, as well as all constitutional should never be in the hands of the man or men who will constitute
the dictator
governance: increasing administrative powers of the executive, while at the same
3) No government should initiate a constitutional
time imposing limitation upon that power.[93] Watkins placed his real faith in a scheme of dictatorship without making specific provisions for its termination

constitutional dictatorship. These are the conditions of success of such a dictatorship: The period 4) all uses of emergency powers and all readjustments
in the organization of the government should be effected in pursuit of
of dictatorship must be relatively shortDictatorship should always be strictly legitimate in constitutional or legal requirements
characterFinal authority to determine the need for dictatorship in any given case must never 5) no dictatorial institution should be adopted, no right
rest with the dictator himself[94] and the objective of such an emergency dictatorship should be invaded, no regular procedure altered any more than is absolutely
necessary for the conquest of the particular crisis . . .
strict political conservatism.
6) The measures adopted in the prosecution of the a
constitutional dictatorship should never be permanent in character or
effect

Carl J. Friedrich cast his analysis in terms similar to those of Watkins. [95] It is a problem 7) The dictatorship should be carried on by persons
representative of every part of the citizenry interested in the defense
of concentrating power in a government where power has consciously been divided to cope with of the existing constitutional order. . .
situations of unprecedented magnitude and gravity. There must be a broad grant of powers, subject 8) Ultimate responsibility should be maintained for every
action taken under a constitutional dictatorship. . .
to equally strong limitations as to who shall exercise such powers, when, for how long, and to what

end.[96] Friedrich, too, offered criteria for judging the adequacy of any of scheme of emergency 9) The decision to terminate a constitutional dictatorship,
like the decision to institute one should never be in the hands of the
powers, to wit: The emergency executive must be appointed by constitutional means i.e., he man or men who constitute the dictator. . .
10) No constitutional dictatorship should extend beyond
the termination of the crisis for which it was instituted In the final analysis, the various approaches to emergency of the above political theorists

11) the termination of the crisis must be followed by a - from Locks theory of prerogative, to Watkins doctrine of constitutional dictatorship
complete return as possible to the political and governmental
and, eventually, to McIlwains principle of constitutionalism --- ultimately aim to solve one real
conditions existing prior to the initiation of the constitutional
dictatorship[99] problem in emergency governance, i.e., that of allotting increasing areas of discretionary power

to the Chief Executive, while insuring that such powers will be exercised with a sense of political
Rossiter accorded to legislature a far greater role in the oversight exercise of emergency powers than responsibility and under effective limitations and checks.
did Watkins. He would secure to Congress final responsibility for declaring the existence or

termination of an emergency, and he places great faith in the effectiveness of congressional Our Constitution has fairly coped with this problem. Fresh from the fetters of a repressive
[100]
investigating committees. regime, the 1986 Constitutional Commission, in drafting the 1987 Constitution, endeavored to create
Scott and Cotter, in analyzing the above contemporary theories in light of recent experience, a government in the concept of Justice Jacksons balanced power structure.[102] Executive,
were one in saying that, the suggestion that democracies surrender the control of government legislative, and judicial powers are dispersed to the President, the Congress, and the Supreme Court,
to an authoritarian ruler in time of grave danger to the nation is not based upon sound respectively. Each is supreme within its own sphere. But none has the monopoly of power in times
constitutional theory. To appraise emergency power in terms of constitutional dictatorship serves of emergency. Each branch is given a role to serve as limitation or check upon the
merely to distort the problem and hinder realistic analysis. It matters not whether the term dictator other. This system does not weaken the President, it just limits his power, using the language
is used in its normal sense (as applied to authoritarian rulers) or is employed to embrace all chief of McIlwain. In other words, in times of emergency, our Constitution reasonably demands that we
executives administering emergency powers. However used, constitutional dictatorship cannot be repose a certain amount of faith in the basic integrity and wisdom of the Chief Executive but, at the
divorced from the implication of suspension of the processes of constitutionalism. Thus, they same time, it obliges him to operate within carefully prescribed procedural limitations.
favored instead the concept of constitutionalism articulated by Charles H. McIlwain:

A concept of constitutionalism which is less misleading in the a. Facial Challenge


analysis of problems of emergency powers, and which is consistent with the
findings of this study, is that formulated by Charles H. McIlwain. While it does
not by any means necessarily exclude some indeterminate limitations upon the Petitioners contend that PP 1017 is void on its face because of its overbreadth. They
substantive powers of government, full emphasis is placed upon procedural
limitations, and political responsibility. McIlwain clearly recognized the need claim that its enforcement encroached on both unprotected and protected rights under Section 4,
to repose adequate power in government. And in discussing the meaning of
constitutionalism, he insisted that the historical and proper test of Article III of the Constitution and sent a chilling effect to the citizens.
constitutionalism was the existence of adequate processes for keeping
government responsible. He refused to equate constitutionalism with the
enfeebling of government by an exaggerated emphasis upon separation of A facial review of PP 1017, using the overbreadth doctrine, is uncalled for.
powers and substantive limitations on governmental power. He found that the
really effective checks on despotism have consisted not in the weakening of
government but, but rather in the limiting of it; between which there is a great First and foremost, the overbreadth doctrine is an analytical tool developed for testing
and very significant difference. In associating constitutionalism with
limited as distinguished from weak government, McIlwain meant on their faces statutes in free speech cases, also known under the American Law as First
government limited to the orderly procedure of law as opposed to the
processes of force. The two fundamental correlative elements of Amendment cases.[103]
constitutionalism for which all lovers of liberty must yet fight are the legal
limits to arbitrary power and a complete political responsibility of
government to the governed.[101]
A plain reading of PP 1017 shows that it is not primarily directed to speech or even speech-
The most distinctive feature of the overbreadth technique is that
related conduct. It is actually a call upon the AFP to prevent or suppress all forms it marks an exception to some of the usual rules of constitutional
litigation. Ordinarily, a particular litigant claims that a statute is
of lawless violence. In United States v. Salerno,[104] the US Supreme Court held that we have not unconstitutional as applied to him or her; if the litigant prevails, the courts
carve away the unconstitutional aspects of the law by invalidating its
recognized an overbreadth doctrine outside the limited context of the First Amendment
improper applications on a case to case basis. Moreover, challengers to a
(freedom of speech). law are not permitted to raise the rights of third parties and can only assert
their own interests. In overbreadth analysis, those rules give way;
challenges are permitted to raise the rights of third parties; and the court
invalidates the entire statute on its face, not merely as applied for so that
Moreover, the overbreadth doctrine is not intended for testing the validity of a law that the overbroad law becomes unenforceable until a properly authorized court
construes it more narrowly. The factor that motivates courts to depart from the
reflects legitimate state interest in maintaining comprehensive control over harmful, constitutionally normal adjudicatory rules is the concern with the chilling; deterrent effect of
the overbroad statute on third parties not courageous enough to bring suit. The
unprotected conduct. Undoubtedly, lawless violence, insurrection and rebellion are considered Court assumes that an overbroad laws very existence may cause others not
before the court to refrain from constitutionally protected speech or
harmful and constitutionally unprotected conduct. In Broadrick v. Oklahoma,[105] it was held: expression. An overbreadth ruling is designed to remove that deterrent effect
on the speech of those third parties.
It remains a matter of no little difficulty to determine when a law
may properly be held void on its face and when such summary action is
inappropriate. But the plain import of our cases is, at the very least, that
facial overbreadth adjudication is an exception to our traditional rules of
practice and that its function, a limited one at the outset, attenuates as the In other words, a facial challenge using the overbreadth doctrine will require the Court to
otherwise unprotected behavior that it forbids the State to sanction moves
from pure speech toward conduct and that conduct even if expressive examine PP 1017 and pinpoint its flaws and defects, not on the basis of its actual operation to
falls within the scope of otherwise valid criminal laws that reflect
legitimate state interests in maintaining comprehensive controls over petitioners, but on the assumption or prediction that its very existence may cause others not before
harmful, constitutionally unprotected conduct.
the Court to refrain from constitutionally protected speech or expression. In Younger v. Harris,[109] it

was held that:


Thus, claims of facial overbreadth are entertained in cases involving statutes which, by their
[T]he task of analyzing a proposed statute, pinpointing its
terms, seek to regulate only spoken words and again, that overbreadth claims, if entertained deficiencies, and requiring correction of these deficiencies before the statute is
put into effect, is rarely if ever an appropriate task for the judiciary. The
at all, have been curtailed when invoked against ordinary criminal laws that are sought to be combination of the relative remoteness of the controversy, the impact on the
legislative process of the relief sought, and above all the speculative and
applied to protected conduct.[106] Here, the incontrovertible fact remains that PP 1017 pertains to amorphous nature of the required line-by-line analysis of detailed
statutes,...ordinarily results in a kind of case that is wholly unsatisfactory for
a spectrum of conduct, not free speech, which is manifestly subject to state regulation. deciding constitutional questions, whichever way they might be decided.

And third, a facial challenge on the ground of overbreadth is the most difficult challenge
Second, facial invalidation of laws is considered as manifestly strong medicine, to be used
to mount successfully, since the challenger must establish that there can be no instance when the
sparingly and only as a last resort, and is generally disfavored;[107] The reason for this is
assailed law may be valid. Here, petitioners did not even attempt to show whether this situation
obvious. Embedded in the traditional rules governing constitutional adjudication is the principle that
exists.
a person to whom a law may be applied will not be heard to challenge a law on the ground that it may
Petitioners likewise seek a facial review of PP 1017 on the ground of vagueness. This,
conceivably be applied unconstitutionally to others, i.e., in other situations not before the
too, is unwarranted.
Court.[108] A writer and scholar in Constitutional Law explains further:
Related to the overbreadth doctrine is the void for vagueness doctrine which holds The first provision pertains to the Presidents calling-out power. In

that a law is facially invalid if men of common intelligence must necessarily guess at its Sanlakas v. Executive Secretary,[111] this Court, through Mr. Justice Dante O. Tinga, held that Section

meaning and differ as to its application.[110] It is subject to the same principles governing 18, Article VII of the Constitution reproduced as follows:

overbreadth doctrine. For one, it is also an analytical tool for testing on their faces statutes in Sec. 18. The President shall be the Commander-in-Chief of all armed
forces of the Philippines and whenever it becomes necessary, he may call out
free speech cases. And like overbreadth, it is said that a litigant may challenge a statute on its face such armed forces to prevent or suppress lawless violence, invasion or
rebellion. In case of invasion or rebellion, when the public safety requires it, he
only if it is vague in all its possible applications. Again, petitioners did not even attempt to show
may, for a period not exceeding sixty days, suspend the privilege of the writ
that PP 1017 is vague in all its application. They also failed to establish that men of common of habeas corpus or place the Philippines or any part thereof under martial law.
Within forty-eight hours from the proclamation of martial law or the suspension
intelligence cannot understand the meaning and application of PP 1017. of the privilege of the writ of habeas corpus, the President shall submit a report
in person or in writing to the Congress. The Congress, voting jointly, by a vote
of at least a majority of all its Members in regular or special session, may revoke
such proclamation or suspension, which revocation shall not be set aside by the
b. Constitutional Basis of PP 1017 President. Upon the initiative of the President, the Congress may, in the same
manner, extend such proclamation or suspension for a period to be determined
by the Congress, if the invasion or rebellion shall persist and public safety
requires it.
Now on the constitutional foundation of PP 1017.
The Congress, if not in session, shall within twenty-four hours following such
proclamation or suspension, convene in accordance with its rules without need
The operative portion of PP 1017 may be divided into three important provisions, thus: of a call.

The Supreme Court may review, in an appropriate proceeding filed by


any citizen, the sufficiency of the factual bases of the proclamation of martial
First provision: law or the suspension of the privilege of the writ or the extension thereof, and
must promulgate its decision thereon within thirty days from its filing.

by virtue of the power vested upon me by Section 18, Artilce VII A state of martial law does not suspend the operation of the Constitution,
do hereby command the Armed Forces of the Philippines, to maintain law and nor supplant the functioning of the civil courts or legislative assemblies, nor
order throughout the Philippines, prevent or suppress all forms of lawless authorize the conferment of jurisdiction on military courts and agencies over
violence as well any act of insurrection or rebellion civilians where civil courts are able to function, nor automatically suspend the
privilege of the writ.

Second provision: The suspension of the privilege of the writ shall apply only to persons judicially
charged for rebellion or offenses inherent in or directly connected with invasion.
and to enforce obedience to all the laws and to all decrees, orders
and regulations promulgated by me personally or upon my direction; During the suspension of the privilege of the writ, any person thus arrested or
detained shall be judicially charged within three days, otherwise he shall be
released.
Third provision:

as provided in Section 17, Article XII of the Constitution do hereby


grants the President, as Commander-in-Chief, a sequence of graduated powers. From the most to
declare a State of National Emergency.
the least benign, these are: the calling-out power, the power to suspend the privilege of the writ

of habeas corpus, and the power to declare Martial Law. Citing Integrated Bar of the Philippines v.
First Provision: Calling-out Power
Zamora,[112] the Court ruled that the only criterion for the exercise of the calling-out power is that

whenever it becomes necessary, the President may call the armed forces to prevent or
suppress lawless violence, invasion or rebellion. Are these conditions present in the instant an awesome power. Obviously, such Proclamation cannot be deemed harmless, without legal

cases? As stated earlier, considering the circumstances then prevailing, President Arroyo found it significance, or not written, as in the case of Sanlakas.

necessary to issue PP 1017. Owing to her Offices vast intelligence network, she is in the best

position to determine the actual condition of the country. Some of the petitioners vehemently maintain that PP 1017 is actually a declaration of Martial

Law. It is no so. What defines the character of PP 1017 are its wordings. It is plain therein that what

Under the calling-out power, the President may summon the armed forces to aid him in the President invoked was her calling-out power.

suppressing lawless violence, invasion and rebellion. This involves ordinary police action. But

every act that goes beyond the Presidents calling-out power is considered illegal or ultra vires. For The declaration of Martial Law is a warn[ing] to citizens that the military power has been

this reason, a President must be careful in the exercise of his powers. He cannot invoke a greater called upon by the executive to assist in the maintenance of law and order, and that, while the

power when he wishes to act under a lesser power. There lies the wisdom of our Constitution, the emergency lasts, they must, upon pain of arrest and punishment, not commit any acts which will in

greater the power, the greater are the limitations. any way render more difficult the restoration of order and the enforcement of law.[113]

It is pertinent to state, however, that there is a distinction between the Presidents authority to In his Statement before the Senate Committee on Justice on March 13, 2006, Mr.

declare a state of rebellion (in Sanlakas) and the authority to proclaim a state of national Justice Vicente V. Mendoza,[114] an authority in constitutional law, said that of the three powers of the

emergency. While President Arroyos authority to declare a state of rebellion emanates from her President as Commander-in-Chief, the power to declare Martial Law poses the most severe threat to

powers as Chief Executive, the statutory authority cited in Sanlakas was Section 4, Chapter 2, Book civil liberties. It is a strong medicine which should not be resorted to lightly. It cannot be used to

II of the Revised Administrative Code of 1987, which provides: stifle or persecute critics of the government. It is placed in the keeping of the President for the purpose

of enabling him to secure the people from harm and to restore order so that they can enjoy their
SEC. 4. Proclamations. Acts of the President fixing a date
or declaring a status or condition of public moment or interest, upon individual freedoms. In fact, Section 18, Art. VII, provides:
the existence of which the operation of a specific law or regulation is
made to depend, shall be promulgated in proclamations which shall A state of martial law does not suspend the operation of the
have the force of an executive order. Constitution, nor supplant the functioning of the civil courts or legislative
assemblies, nor authorize the conferment of jurisdiction on military courts and
agencies over civilians where civil courts are able to function, nor automatically
suspend the privilege of the writ.
President Arroyos declaration of a state of rebellion was merely an act declaring a status

or condition of public moment or interest, a declaration allowed under Section 4 cited above. Such Justice Mendoza also stated that PP 1017 is not a declaration of Martial Law. It is no more than

declaration, in the words of Sanlakas, is harmless, without legal significance, and deemed not a call by the President to the armed forces to prevent or suppress lawless violence. As such, it cannot

written. In these cases, PP 1017 is more than that. In declaring a state of national emergency, be used to justify acts that only under a valid declaration of Martial Law can be done. Its use for any

President Arroyo did not only rely on Section 18, Article VII of the Constitution, a provision calling other purpose is a perversion of its nature and scope, and any act done contrary to its command is ultra

on the AFP to prevent or suppress lawless violence, invasion or rebellion. She also relied on Section vires.

17, Article XII, a provision on the States extraordinary power to take over privately-owned public Justice Mendoza further stated that specifically, (a) arrests and seizures without judicial

utility and business affected with public interest. Indeed, PP 1017 calls for the exercise of warrants; (b) ban on public assemblies; (c) take-over of news media and agencies and press
censorship; and (d) issuance of Presidential Decrees, are powers which can be exercised by the Petitioners contention is understandable. A reading of PP 1017 operative clause shows

President as Commander-in-Chief only where there is a valid declaration of Martial Law or that it was lifted[120] from Former President Marcos Proclamation No. 1081, which partly reads:

suspension of the writ of habeas corpus.


NOW, THEREFORE, I, FERDINAND E. MARCOS, President of
Based on the above disquisition, it is clear that PP 1017 is not a declaration of Martial the Philippines by virtue of the powers vested upon me by Article VII, Section
10, Paragraph (2) of the Constitution, do hereby place the entire Philippines as
Law. It is merely an exercise of President Arroyos calling-out power for the armed forces to
defined in Article 1, Section 1 of the Constitution under martial law and, in my
assist her in preventing or suppressing lawless violence. capacity as their Commander-in-Chief, do hereby command the Armed
Forces of the Philippines, to maintain law and order throughout the
Philippines, prevent or suppress all forms of lawless violence as well as any
Second Provision: Take Care Power act of insurrection or rebellion and to enforce obedience to all the laws and
decrees, orders and regulations promulgated by me personally or upon my
direction.

The second provision pertains to the power of the President to ensure that the laws be
We all know that it was PP 1081 which granted President Marcos legislative power. Its
faithfully executed. This is based on Section 17, Article VII which reads:
enabling clause states: to enforce obedience to all the laws and decrees, orders and regulations

SEC. 17. The President shall have control of all the executive promulgated by me personally or upon my direction. Upon the other hand, the enabling clause
departments, bureaus, and offices. He shall ensure that the laws be faithfully
executed. of PP 1017 issued by President Arroyo is: to enforce obedience to all the laws and to all decrees,

orders and regulations promulgated by me personally or upon my direction.

As the Executive in whom the executive power is vested,[115] the primary function of the
Is it within the domain of President Arroyo to promulgate decrees?
President is to enforce the laws as well as to formulate policies to be embodied in existing laws. He

sees to it that all laws are enforced by the officials and employees of his department. Before assuming
PP 1017 states in part: to enforce obedience to all the laws and decrees x x
office, he is required to take an oath or affirmation to the effect that as President of the Philippines,
x promulgated by me personally or upon my direction.
he will, among others, execute its laws.[116] In the exercise of such function, the President, if

needed, may employ the powers attached to his office as the Commander-in-Chief of all the armed

forces of the country,[117] including the Philippine National Police[118] under the Department of The President is granted an Ordinance Power under Chapter 2, Book III of Executive Order

Interior and Local Government.[119] No. 292 (Administrative Code of 1987). She may issue any of the following:

Petitioners, especially Representatives Francis Joseph G. Escudero, Satur Ocampo, Rafael Sec. 2. Executive Orders. Acts of the President providing for rules
of a general or permanent character in implementation or execution of
Mariano, Teodoro Casio, Liza Maza, and Josel Virador argue that PP 1017 is unconstitutional as it constitutional or statutory powers shall be promulgated in executive orders.
arrogated upon President Arroyo the power to enact laws and decrees in violation of Section 1, Article Sec. 3. Administrative Orders. Acts of the President which relate
to particular aspect of governmental operations in pursuance of his duties as
VI of the Constitution, which vests the power to enact laws in Congress. They assail the clause to administrative head shall be promulgated in administrative orders.
enforce obedience to all the laws and to all decrees, orders and regulations promulgated by me Sec. 4. Proclamations. Acts of the President fixing a date or
declaring a status or condition of public moment or interest, upon the existence
personally or upon my direction. of which the operation of a specific law or regulation is made to depend, shall
be promulgated in proclamations which shall have the force of an executive
order.
Sec. 5. Memorandum Orders. Acts of the President on matters of
administrative detail or of subordinate or temporary interest which only concern
a particular officer or office of the Government shall be embodied in
memorandum orders.
Sec. 6. Memorandum Circulars. Acts of the President on matters
relating to internal administration, which the President desires to bring to the Third Provision: Power to Take Over
attention of all or some of the departments, agencies, bureaus or offices of the
Government, for information or compliance, shall be embodied in memorandum The pertinent provision of PP 1017 states:
circulars.
Sec. 7. General or Special Orders. Acts and commands of the x x x and to enforce obedience to all the laws
President in his capacity as Commander-in-Chief of the Armed Forces of the and to all decrees, orders, and regulations promulgated by
Philippines shall be issued as general or special orders. me personally or upon my direction; and as provided in
Section 17, Article XII of the Constitution do hereby
declare a state of national emergency.
President Arroyos ordinance power is limited to the foregoing issuances. She cannot

issue decrees similar to those issued by Former President Marcos under PP 1081. Presidential The import of this provision is that President Arroyo, during the state of national emergency
Decrees are laws which are of the same category and binding force as statutes because they were under PP 1017, can call the military not only to enforce obedience to all the laws and to all decrees
issued by the President in the exercise of his legislative power during the period of Martial Law under x x x but also to act pursuant to the provision of Section 17, Article XII which reads:
[121]
the 1973 Constitution. Sec. 17. In times of national emergency, when the public interest so
requires, the State may, during the emergency and under reasonable terms
prescribed by it, temporarily take over or direct the operation of any privately-
This Court rules that the assailed PP 1017 is unconstitutional insofar as it grants owned public utility or business affected with public interest.

President Arroyo the authority to promulgate decrees. Legislative power is peculiarly within What could be the reason of President Arroyo in invoking the above provision when she

the province of the Legislature. Section 1, Article VI categorically states that [t]he legislative issued PP 1017?

power shall be vested in the Congress of the Philippines which shall consist of a Senate and a

House of Representatives. To be sure, neither Martial Law nor a state of rebellion nor a state of The answer is simple. During the existence of the state of national emergency, PP 1017

emergency can justify President Arroyos exercise of legislative power by issuing decrees. purports to grant the President, without any authority or delegation from Congress, to take over or

direct the operation of any privately-owned public utility or business affected with public interest.

Can President Arroyo enforce obedience to all decrees and laws through the military?
This provision was first introduced in the 1973 Constitution, as a product of the martial

law thinking of the 1971 Constitutional Convention.[122] In effect at the time of its approval was
As this Court stated earlier, President Arroyo has no authority to enact decrees. It follows that
President Marcos Letter of Instruction No. 2 dated September 22, 1972 instructing the Secretary of
these decrees are void and, therefore, cannot be enforced. With respect to laws, she cannot call
National Defense to take over the management, control and operation of the Manila Electric
the military to enforce or implement certain laws, such as customs laws, laws governing family and
Company, the Philippine Long Distance Telephone Company, the National Waterworks
property relations, laws on obligations and contracts and the like. She can only order the military,
and Sewerage Authority, the Philippine National Railways, the Philippine Air Lines, Air Manila
under PP 1017, to enforce laws pertinent to its duty to suppress lawless violence.
(and) Filipinas Orient Airways . . . for the successful prosecution by the Government of its effort to emergency. The logical conclusion then is that President Arroyo could validly declare the existence

contain, solve and end the present national emergency. of a state of national emergency even in the absence of a Congressional enactment.

Petitioners, particularly the members of the House of Representatives, claim that President But the exercise of emergency powers, such as the taking over of privately owned public

Arroyos inclusion of Section 17, Article XII in PP 1017 is an encroachment on the legislatures utility or business affected with public interest, is a

emergency powers. different matter. This requires a delegation from Congress.

This is an area that needs delineation. Courts have often said that constitutional provisions in pari materia are to be construed

together. Otherwise stated, different clauses, sections, and provisions of a constitution which relate

A distinction must be drawn between the Presidents authority to declare a state of national to the same subject matter will be construed together and considered in the light of each

emergency and to exercise emergency powers. To the first, as elucidated by the Court, other.[123] Considering that Section 17 of Article XII and Section 23 of Article VI, previously quoted,

Section 18, Article VII grants the President such power, hence, no legitimate constitutional objection relate to national emergencies, they must be read together to determine the limitation of the exercise

can be raised. But to the second, manifold constitutional issues arise. of emergency powers.

Generally, Congress is the repository of emergency powers. This is evident in the tenor
Section 23, Article VI of the Constitution reads:
of Section 23 (2), Article VI authorizing it to delegate such powers to the President. Certainly, a
SEC. 23. (1) The Congress, by a vote of two-thirds of both Houses body cannot delegate a power not reposed upon it. However, knowing that during grave
in joint session assembled, voting separately, shall have the sole power to
declare the existence of a state of war. emergencies, it may not be possible or practicable for Congress to meet and exercise its powers, the

(2) In times of war or other national emergency, the Congress may, Framers of our Constitution deemed it wise to allow Congress to grant emergency powers to the
by law, authorize the President, for a limited period and subject to such
restrictions as it may prescribe, to exercise powers necessary and proper to carry President, subject to certain conditions, thus:
out a declared national policy. Unless sooner withdrawn by resolution of the
Congress, such powers shall cease upon the next adjournment thereof.
(1) There must be a war or other emergency.

(2) The delegation must be for a limited period only.


It may be pointed out that the second paragraph of the above provision refers not only to war
(3) The delegation must be subject to such restrictions as the
but also to other national emergency. If the intention of the Framers of our Constitution was to Congress may prescribe.
(4) The emergency powers must be exercised to carry out a
withhold from the President the authority to declare a state of national emergency pursuant to national policy declared by Congress.[124]

Section 18, Article VII (calling-out power) and grant it to Congress (like the declaration of the
Section 17, Article XII must be understood as an aspect of the emergency powers clause. The
existence of a state of war), then the Framers could have provided so. Clearly, they did not intend
taking over of private business affected with public interest is just another facet of the emergency
that Congress should first authorize the President before he can declare a state of national
powers generally reposed upon Congress. Thus, when Section 17 states that the the State may,

during the emergency and under reasonable terms prescribed by it, temporarily take over or
direct the operation of any privately owned public utility or business affected with public in this definitions are the elements of intensity, variety, and perception. [127] Emergencies, as

interest, it refers to Congress, not the President. Now, whether or not the President may exercise perceived by legislature or executive in the United Sates since 1933, have been occasioned by a wide

such power is dependent on whether Congress may delegate it to him pursuant to a law prescribing range of situations, classifiable under three (3) principal heads: a) economic,[128] b) natural

the reasonable terms thereof. Youngstown Sheet & Tube Co. et al. v. Sawyer,[125] held: disaster,[129] and c) national security.[130]

It is clear that if the President had authority to issue the order he did, Emergency, as contemplated in our Constitution, is of the same breadth. It may include
it must be found in some provision of the Constitution. And it is not claimed
that express constitutional language grants this power to the President. The rebellion, economic crisis, pestilence or epidemic, typhoon, flood, or other similar catastrophe of
contention is that presidential power should be implied from the aggregate of his
powers under the Constitution. Particular reliance is placed on provisions in nationwide proportions or effect.[131] This is evident in the Records of the Constitutional Commission,
Article II which say that The executive Power shall be vested in a President . .
thus:
. .; that he shall take Care that the Laws be faithfully executed; and that he
shall be Commander-in-Chief of the Army and Navy of the United States.
MR. GASCON. Yes. What is the Committees definition of national
emergency which appears in Section 13, page 5? It reads:
The order cannot properly be sustained as an exercise of the
Presidents military power as Commander-in-Chief of the Armed Forces. The
When the common good so requires, the State may temporarily take over
Government attempts to do so by citing a number of cases upholding broad
or direct the operation of any privately owned public utility or business affected
powers in military commanders engaged in day-to-day fighting in a theater of
with public interest.
war. Such cases need not concern us here. Even though theater of war be
an expanding concept, we cannot with faithfulness to our constitutional
system hold that the Commander-in-Chief of the Armed Forces has the MR. VILLEGAS. What I mean is threat from external aggression, for
ultimate power as such to take possession of private property in order to example, calamities or natural disasters.
keep labor disputes from stopping production. This is a job for the
nations lawmakers, not for its military authorities. MR. GASCON. There is a question by Commissioner de los
Reyes. What about strikes and riots?
Nor can the seizure order be sustained because of the several
constitutional provisions that grant executive power to the President. In MR. VILLEGAS. Strikes, no; those would not be covered by the term
the framework of our Constitution, the Presidents power to see that the national emergency.
laws are faithfully executed refutes the idea that he is to be a
lawmaker. The Constitution limits his functions in the lawmaking process MR. BENGZON. Unless they are of such proportions such that they
to the recommending of laws he thinks wise and the vetoing of laws he would paralyze government service.[132]
thinks bad. And the Constitution is neither silent nor equivocal about who
shall make laws which the President is to execute. The first section of the
first article says that All legislative Powers herein granted shall be vested x x x x x x
in a Congress of the United States. . .[126]
MR. TINGSON. May I ask the committee if national emergency refers
to military national emergency or could this be economic emergency?
Petitioner Cacho-Olivares, et al. contends that the term emergency under Section 17,
MR. VILLEGAS. Yes, it could refer to both military or economic
Article XII refers to tsunami, typhoon, hurricane and similar occurrences. This
dislocations.
is a limited view of emergency.
MR. TINGSON. Thank you very much.[133]

It may be argued that when there is national emergency, Congress may not be able to convene
Emergency, as a generic term, connotes the existence of conditions suddenly intensifying and, therefore, unable to delegate to the President the power to take over privately-owned public utility
the degree of existing danger to life or well-being beyond that which is accepted as normal. Implicit or business affected with public interest.
have ceased. Likewise, without legislation, the President has no power to point out the types of

In Araneta v. Dinglasan,[134] this Court emphasized that legislative power, through which businesses affected with public interest that should be taken over. In short, the President has no

extraordinary measures are exercised, remains in Congress even in times of crisis. absolute authority to exercise all the powers of the State under Section 17, Article VII in the absence

of an emergency powers act passed by Congress.

x x x

After all the criticisms that have been made against the c. AS APPLIED CHALLENGE
efficiency of the system of the separation of powers, the fact remains
that the Constitution has set up this form of government, with all its
defects and shortcomings, in preference to the commingling of
powers in one man or group of men. The Filipino people by adopting One of the misfortunes of an emergency, particularly, that which pertains to security, is
parliamentary government have given notice that they share the faith
that military necessity and the guaranteed rights of the individual are often not compatible. Our
of other democracy-loving peoples in this system, with all its faults,
as the ideal. The point is, under this framework of government, history reveals that in the crucible of conflict, many rights are curtailed and trampled upon. Here,
legislation is preserved for Congress all the time, not excepting
periods of crisis no matter how serious. Never in the history of the the right against unreasonable search and seizure; the right against warrantless arrest;and the
United States, the basic features of whose Constitution have been
copied in ours, have specific functions of the legislative branch of freedom of speech, of expression, of the press, and of assembly under the Bill of Rights suffered
enacting laws been surrendered to another department unless we
regard as legislating the carrying out of a legislative policy according the greatest blow.
to prescribed standards; no, not even when that Republic was fighting
a total war, or when it was engaged in a life-and-death struggle to
preserve the Union. The truth is that under our concept of
constitutional government, in times of extreme perils more than in Of the seven (7) petitions, three (3) indicate direct injury.
normal circumstances the various branches, executive, legislative,
and judicial, given the ability to act, are called upon to perform
the duties and discharge the responsibilities committed to them In G.R. No. 171396, petitioners David and Llamas alleged that, on February 24, 2006, they
respectively.
were arrested without warrants on their way to EDSA to celebrate the 20 th Anniversary ofPeople

Power I. The arresting officers cited PP 1017 as basis of the arrest.

Following our interpretation of Section 17, Article XII, invoked by President Arroyo in

issuing PP 1017, this Court rules that such Proclamation does not authorize her during the emergency In G.R. No. 171409, petitioners Cacho-Olivares and Tribune Publishing Co., Inc. claimed

to temporarily take over or direct the operation of any privately owned public utility or business that on February 25, 2006, the CIDG operatives raided and ransacked without warrant their

affected with public interest without authority from Congress. office. Three policemen were assigned to guard their office as a possible source of

destabilization. Again, the basis was PP 1017.

Let it be emphasized that while the President alone can declare a state of national

emergency, however, without legislation, he has no power to take over privately-owned public And in G.R. No. 171483, petitioners KMU and NAFLU-KMU et al. alleged that their
utility or business affected with public interest. The President cannot decide whether members were turned away and dispersed when they went to EDSA and later, to Ayala Avenue,

exceptional circumstances exist warranting the take over of privately-owned public utility to celebrate the 20th Anniversary of People Power I.

or business affected with public interest. Nor can he determine when such exceptional circumstances
A perusal of the direct injuries allegedly suffered by the said petitioners shows that they object.[140] For these reasons, one requirement for these rules to be valid is that they must

resulted from the implementation, pursuant to G.O. No. 5, of PP 1017. be reasonable, not arbitrary or capricious.

G.O. No. 5 mandates the AFP and the PNP to immediately carry out the necessary and

Can this Court adjudge as unconstitutional PP 1017 and G.O. No 5 on the basis of these appropriate actions and measures to suppress and prevent acts of terrorism and

illegal acts? In general, does the illegal implementation of a law render it unconstitutional? lawless violence.

Unlike the term lawless violence which is unarguably extant in our statutes and the

Settled is the rule that courts are not at liberty to declare statutes invalid although they Constitution, and which is invariably associated with invasion, insurrection or rebellion, the phrase

may be abused and misabused[135] and may afford an opportunity for abuse in the manner of acts of terrorism is still an amorphous and vague concept. Congress has yet to enact a law defining

application.[136] The validity of a statute or ordinance is to be determined from its general purpose and punishing acts of terrorism.
[137]
and its efficiency to accomplish the end desired, not from its effects in a particular case. PP In fact, this definitional predicament or the absence of an agreed definition of

1017 is merely an invocation of the Presidents calling-out power. Its general purpose is to command terrorism confronts not only our country, but the international

the AFP to suppress all forms of lawless violence, invasion or rebellion. It had accomplished the end community as well. The following observations are quite apropos:

desired which prompted President Arroyo to issue PP 1021. But there is nothing in PP 1017 allowing
In the actual unipolar context of international relations, the fight
the police, expressly or impliedly, to conduct illegal arrest, search or violate the citizens against terrorism has become one of the basic slogans when it comes to the
justification of the use of force against certain states and against groups
constitutional rights. operating internationally. Lists of states sponsoring terrorism and of terrorist
organizations are set up and constantly being updated according to criteria that
Now, may this Court adjudge a law or ordinance unconstitutional on the ground that its
are not always known to the public, but are clearly determined by strategic
implementor committed illegal acts? The answer is no. The criterion by which the validity of the interests.

statute or ordinance is to be measured is the essential basis for the exercise of power, and not a mere The basic problem underlying all these military actions or threats
of the use of force as the most recent by the United States against Iraq consists
incidental result arising from its exertion.[138] This is logical. Just imagine the absurdity of in the absence of an agreed definition of terrorism.

situations when laws maybe declared unconstitutional just because the officers implementing them Remarkable confusion persists in regard to the legal categorization of
acts of violence either by states, by armed groups such as liberation movements,
have acted arbitrarily. If this were so, judging from the blunders committed by policemen in the cases or by individuals.
passed upon by the Court, majority of the provisions of the Revised Penal Code would have been The dilemma can by summarized in the saying One countrys
declared unconstitutional a long time ago. terrorist is another countrys freedom fighter. The apparent contradiction or
lack of consistency in the use of the term terrorism may further be
President Arroyo issued G.O. No. 5 to carry into effect the provisions of PP 1017. General demonstrated by the historical fact that leaders of national liberation movements
such as Nelson Mandela in South Africa, Habib Bourgouiba in Tunisia, or
orders are acts and commands of the President in his capacity as Commander-in-Chief of the Armed Ahmed Ben Bella in Algeria, to mention only a few, were originally labeled as
terrorists by those who controlled the territory at the time, but later became
Forces of the Philippines. They are internal rules issued by the executive officer to his subordinates internationally respected statesmen.

precisely for the proper and efficient administration of law. Such rules and regulations create no What, then, is the defining criterion for terrorist acts the differentia
[139]
specifica distinguishing those acts from eventually legitimate acts of national
relation except between the official who issues them and the official who receives them. They are resistance or self-defense?
based on and are the product of, a relationship in which power is their source, and obedience, their
Since the times of the Cold War the United Nations Organization has
been trying in vain to reach a consensus on the basic issue of definition. The
organization has intensified its efforts recently, but has been unable to bridge the
gap between those who associate terrorism with any violent act by non-state immediately arrest them pursuant to G.O. No. 5. Obviously, this is abuse and oppression on their
groups against civilians, state functionaries or infrastructure or military
installations, and those who believe in the concept of the legitimate use of force part. It must be remembered that an act can only be considered a crime if there is a law defining the
when resistance against foreign occupation or against systematic oppression of
same as such and imposing the corresponding penalty thereon.
ethnic and/or religious groups within a state is concerned.
So far, the word terrorism appears only once in our criminal laws, i.e., in P.D. No. 1835
The dilemma facing the international community can best be
illustrated by reference to the contradicting categorization of organizations and dated January 16, 1981 enacted by President Marcos during the Martial Law regime. This decree is
movements such as Palestine Liberation Organization (PLO) which is a
terrorist group for Israel and a liberation movement for Arabs and Muslims entitled Codifying The Various Laws on Anti-Subversion and Increasing The Penalties for
the Kashmiri resistance groups who are terrorists in the perception of India,
liberation fighters in that of Pakistan the earlier Contras in Nicaragua Membership in Subversive Organizations. The word terrorism is mentioned in the following
freedom fighters for the United States, terrorists for the Socialist camp or,
most drastically, the Afghani Mujahedeen (later to become the Taliban provision: That one who conspires with any other person for the purpose of overthrowing the
movement): during the Cold War period they were a group of freedom fighters
for the West, nurtured by the United States, and a terrorist gang for the Soviet Government of the Philippines x x x by force, violence, terrorism, x x x shall be punished
Union. One could go on and on in enumerating examples of conflicting by reclusion temporal x x x.
categorizations that cannot be reconciled in any way because of opposing
political interests that are at the roots of those perceptions. P.D. No. 1835 was repealed by E.O. No. 167 (which outlaws the Communist Party of the
How, then, can those contradicting definitions and conflicting Philippines) enacted by President Corazon Aquino on May 5, 1985. These two (2) laws, however, do
perceptions and evaluations of one and the same group and its actions be
explained? In our analysis, the basic reason for these striking inconsistencies not define acts of terrorism. Since there is no law defining acts of terrorism, it is President
lies in the divergent interest of states. Depending on whether a state is in the
position of an occupying power or in that of a rival, or adversary, of an Arroyo alone, under G.O. No. 5, who has the discretion to determine what acts constitute
occupying power in a given territory, the definition of terrorism will
fluctuate accordingly. A state may eventually see itself as protector of the terrorism. Her judgment on this aspect is absolute, without restrictions. Consequently, there can be
rights of a certain ethnic group outside its territory and will therefore speak of a
indiscriminate arrest without warrants, breaking into offices and residences, taking over the media
liberation struggle, not of terrorism when acts of violence by this group
are concerned, and vice-versa. enterprises, prohibition and dispersal of all assemblies and gatherings unfriendly to the
The United Nations Organization has been unable to reach a decision administration. All these can be effected in the name of G.O. No. 5. These acts go far beyond the
on the definition of terrorism exactly because of these conflicting interests of
sovereign states that determine in each and every instance how a particular calling-out power of the President. Certainly, they violate the due process clause of the
armed movement (i.e. a non-state actor) is labeled in regard to the terrorists-
freedom fighter dichotomy. A policy of double standards on this vital issue Constitution. Thus, this Court declares that the acts of terrorism portion of G.O. No. 5 is
of international affairs has been the unavoidable consequence.
unconstitutional.
This definitional predicament of an organization consisting of
Significantly, there is nothing in G.O. No. 5 authorizing the military or police to commit
sovereign states and not of peoples, in spite of the emphasis in the Preamble
to the United Nations Charter! has become even more serious in the present acts beyond what are necessary and appropriate to suppress and prevent lawless violence, the
global power constellation: one superpower exercises the decisive role in the
Security Council, former great powers of the Cold War era as well as medium limitation of their authority in pursuing the Order. Otherwise, such acts are considered illegal.
powers are increasingly being marginalized; and the problem has become even
more acute since the terrorist attacks of 11 September 2001 I the United
States.[141] We first examine G.R. No. 171396 (David et al.)

The absence of a law defining acts of terrorism may result in abuse and oppression on The Constitution provides that the right of the people to be secured in their persons,

the part of the police or military. An illustration is when a group of persons are merely engaged in a houses, papers and effects against unreasonable search and seizure of whatever nature and for any

drinking spree. Yet the military or the police may consider the act as an act of terrorism and purpose shall be inviolable, and no search warrant or warrant of arrest shall issue except upon
probable cause to be determined personally by the judge after examination under oath or affirmation insufficiency of evidence. He noted that petitioner David was not wearing the subject t-shirt and even

of the complainant and the witnesses he may produce, and particularly describing the place to be if he was wearing it, such fact is insufficient to charge him with inciting to sedition. Further, he also

searched and the persons or things to be seized.[142] The plain import of the language of the stated that there is insufficient evidence for the charge of violation of BP 880 as it was not even

Constitution is that searches, seizures and arrests are normally unreasonable unless authorized by a known whether petitioner David was the leader of the rally.[147]

validly issued search warrant or warrant of arrest. Thus, the fundamental protection given by this But what made it doubly worse for petitioners David et al. is that not only was their right

provision is that between person and police must stand the protective authority of a magistrate clothed against warrantless arrest violated, but also their right to peaceably assemble.

with power to issue or refuse to issue search warrants or warrants of arrest. [143] Section 4 of Article III guarantees:

In the Brief Account[144] submitted by petitioner David, certain facts are established: first, he No law shall be passed abridging the freedom of speech, of
expression, or of the press, or the right of the people peaceably to assemble and
was arrested without warrant; second, the PNP operatives arrested him on the basis of PP petition the government for redress of grievances.
1017; third, he was brought at Camp Karingal, Quezon City where he was fingerprinted,
Assembly means a right on the part of the citizens to meet peaceably for consultation
photographed and booked like a criminal suspect; fourth, he was treated brusquely by policemen
in respect to public affairs. It is a necessary consequence of our republican institution and
who held his head and tried to push him inside an unmarked car; fifth, he was charged with
complements the right of speech. As in the case of freedom of expression, this right is not to be
Violation of Batas Pambansa Bilang No. 880[145] and Inciting to Sedition; sixth, he was detained
limited, much less denied, except on a showing of a clear and present danger of a substantive evil
for seven (7) hours; and seventh, he was eventually released for insufficiency of evidence.
that Congress has a right to prevent. In other words, like other rights embraced in the freedom of
Section 5, Rule 113 of the Revised Rules on Criminal Procedure provides: expression, the right to assemble is not subject to previous restraint or censorship. It may not be
Sec. 5. Arrest without warrant; when lawful. - A peace conditioned upon the prior issuance of a permit or authorization from the government authorities
officer or a private person may, without a warrant, arrest a person:
except, of course, if the assembly is intended to be held in a public place, a permit for the use of such
(a) When, in his presence, the person to be arrested has
committed, is actually committing, or is attempting to commit an place, and not for the assembly itself, may be validly required.
offense.
The ringing truth here is that petitioner David, et al. were arrested while they were
(b) When an offense has just been committed and he has
exercising their right to peaceful assembly. They were not committing any crime, neither was there
probable cause to believe based on personal knowledge of facts or
circumstances that the person to be arrested has committed it; and a showing of a clear and present danger that warranted the limitation of that right. As can be gleaned
x x x. from circumstances, the charges of inciting to sedition and violation of BP 880 were mere

afterthought. Even the Solicitor General, during the oral argument, failed to justify the arresting
Neither of the two (2) exceptions mentioned above justifies petitioner
officers conduct. In De Jonge v. Oregon,[148] it was held that peaceable assembly cannot be made a
Davids warrantless arrest. During the inquest for the charges of inciting to
crime, thus:
sedition and violation of BP 880, all that the arresting officers could invoke was their
Peaceable assembly for lawful discussion cannot be made a crime.
observation that some rallyists were wearing t-shirts with the invective Oust Gloria The holding of meetings for peaceable political action cannot be proscribed.
Those who assist in the conduct of such meetings cannot be branded as criminals
Now and their erroneous assumption that petitioner David was the leader of the
on that score. The question, if the rights of free speech and peaceful assembly
rally. [146]
Consequently, the Inquest Prosecutor ordered his immediate release on the ground of are not to be preserved, is not as to the auspices under which the meeting was
held but as to its purpose; not as to the relations of the speakers, but whether
their utterances transcend the bounds of the freedom of speech which the
Constitution protects. If the persons assembling have committed crimes conducted in the absence of any official of the Daily Tribune except the security guard of the building;
elsewhere, if they have formed or are engaged in a conspiracy against the public
peace and order, they may be prosecuted for their conspiracy or other violations and fifth, policemen stationed themselves at the vicinity of the Daily Tribune offices.
of valid laws. But it is a different matter when the State, instead of
prosecuting them for such offenses, seizes upon mere participation in a
peaceable assembly and a lawful public discussion as the basis for a Thereafter, a wave of warning came from government officials. Presidential Chief of Staff
criminal charge.
Michael Defensor was quoted as saying that such raid was meant to show a strong presence,

to tell media outlets not to connive or do anything that would help the rebels in bringing down
On the basis of the above principles, the Court likewise considers the dispersal and arrest
this government. Director General Lomibao further stated that if they do not follow the
of the members of KMU et al. (G.R. No. 171483) unwarranted. Apparently, their dispersal was done
standards and the standards are if they would contribute to instability in the government, or
merely on the basis of Malacaangs directive canceling all permits previously issued by local
if they do not subscribe to what is in General Order No. 5 and Proc. No. 1017 we will
government units. This is arbitrary. The wholesale cancellation of all permits to rally is a blatant
recommend a takeover. National Telecommunications Commissioner Ronald Solis urged
disregard of the principle that freedom of assembly is not to be limited, much less denied, except
television and radio networks to cooperate with the government for the duration of the state of
on a showing of a clear and present danger of a substantive evil that the State has a right to
national emergency. He warned that his agency will not hesitate to recommend the closure of
prevent.[149] Tolerance is the rule and limitation is the exception. Only upon a showing that an
any broadcast outfit that violates rules set out for media coverage during times when the
assembly presents a clear and present danger that the State may deny the citizens right to exercise
national security is threatened.[151]
it. Indeed, respondents failed to show or convince the Court that the rallyists committed acts

amounting to lawless violence, invasion or rebellion. With the blanket revocation of permits, the
The search is illegal. Rule 126 of The Revised Rules on Criminal Procedure lays down the
distinction between protected and unprotected assemblies was eliminated.
steps in the conduct of search and seizure. Section 4 requires that a search warrant be issued upon
Moreover, under BP 880, the authority to regulate assemblies and rallies is lodged with
probable cause in connection with one specific offence to be determined personally by the judge after
the local government units. They have the power to issue permits and to revoke such permits after
examination under oath or affirmation of the complainant and the witnesses he may produce. Section
due notice and hearing on the determination of the presence of clear and present danger. Here,
8 mandates that the search of a house, room, or any other premise be made in the presence of
petitioners were not even notified and heard on the revocation of their permits. [150] The first time they
the lawful occupant thereof or any member of his family or in the absence of the latter, in the
learned of it was at the time of the dispersal. Such absence of notice is a fatal defect. When a
presence of two (2) witnesses of sufficient age and discretion residing in the same
persons right is restricted by government action, it behooves a democratic government to see to it
locality. And Section 9 states that the warrant must direct that it be served in the daytime, unless
that the restriction is fair, reasonable, and according to procedure.
the property is on the person or in the place ordered to be searched, in which case a direction may be
G.R. No. 171409, (Cacho-Olivares, et al.) presents another facet of freedom of speech i.e.,
inserted that it be served at any time of the day or night. All these rules were violated by the CIDG
the freedom of the press. Petitioners narration of facts, which the Solicitor General failed to refute,
operatives.
established the following: first, the Daily Tribunes offices were searched without

warrant; second, the police operatives seized several materials for publication; third, the search was
Not only that, the search violated petitioners freedom of the press. The best gauge of a
conducted at about 1:00 o clock in the morning of February 25, 2006; fourth, the search was
free and democratic society rests in the degree of freedom enjoyed by its media. In theBurgos v. Chief

of Staff[152] this Court held that --


As heretofore stated, the premises searched were the business and SOLICITOR GENERAL BENIPAYO:
printing offices of the "Metropolitan Mail" and the "We Forum newspapers.
As a consequence of the search and seizure, these premises were padlocked Under the law they would seem to be, if they were
and sealed, with the further result that the printing and publication of said illegally seized, I think and I know, Your Honor,
newspapers were discontinued. and these are inadmissible for any purpose.[155]

Such closure is in the nature of previous restraint or censorship xxx xxx xxx
abhorrent to the freedom of the press guaranteed under the fundamental
law, and constitutes a virtual denial of petitioners' freedom to express SR. ASSO. JUSTICE PUNO:
themselves in print. This state of being is patently anathematic to a
democratic framework where a free, alert and even militant press is These have been published in the past issues of the
essential for the political enlightenment and growth of the citizenry. Daily Tribune; all you have to do is to get those
past issues. So why do you have to go there at
1 oclock in the morning and without any
While admittedly, the Daily Tribune was not padlocked and sealed like the Metropolitan search warrant? Did they become suddenly part
of the evidence of rebellion or inciting to
Mail and We Forum newspapers in the above case, yet it cannot be denied that the CIDG sedition or what?
operatives exceeded their enforcement duties. The search and seizure of materials for publication,
SOLGEN BENIPAYO:
the stationing of policemen in the vicinity of the The Daily Tribune offices, and the arrogant warning
Well, it was the police that did that, Your
of government officials to media, are plain censorship. It is that officious functionary of the Honor. Not upon my instructions.

repressive government who tells the citizen that he may speak only if allowed to do so, and no more SR. ASSO. JUSTICE PUNO:

and no less than what he is permitted to say on pain of punishment should he be so rash as to Are you saying that the act of the policeman is
[153]
illegal, it is not based on any law, and it is not
disobey. Undoubtedly, the The Daily Tribune was subjected to these arbitrary intrusions because based on Proclamation 1017.
of its anti-government sentiments. This Court cannot tolerate the blatant disregard of a constitutional SOLGEN BENIPAYO:
right even if it involves the most defiant of our citizens. Freedom to comment on public affairs is
It is not based on Proclamation 1017, Your Honor,
essential to the vitality of a representative democracy. It is the duty of the courts to be watchful for because there is nothing in 1017 which says that
the police could go and inspect and gather
the constitutional rights of the citizen, and against any stealthy encroachments thereon. The motto clippings from Daily Tribune or any other
newspaper.
should always be obsta principiis.[154]
SR. ASSO. JUSTICE PUNO:

Incidentally, during the oral arguments, the Solicitor General admitted that the search of Is it based on any law?
the Tribunes offices and the seizure of its materials for publication and other papers are illegal; and SOLGEN BENIPAYO:
that the same are inadmissible for any purpose, thus:
As far as I know, no, Your Honor, from the
facts, no.
JUSTICE CALLEJO:
SR. ASSO. JUSTICE PUNO:
You made quite a mouthful of admission when you
said that the policemen, when inspected the
So, it has no basis, no legal basis whatsoever?
Tribune for the purpose of gathering evidence
and you admitted that the policemen were able
to get the clippings. Is that not in admission of
the admissibility of these clippings that were
taken from the Tribune?
SOLGEN BENIPAYO:
Maybe so, Your Honor. Maybe so, that is why I
said, I dont know if it is premature to say that PP 1017, or one similar to it, may not again be issued. Already, there have been media reports
this, we do not condone this. If the people
who have been injured by this would want to on April 30, 2006 that allegedly PP 1017 would be reimposed if the May 1 rallies become unruly
sue them, they can sue and there are remedies
and violent. Consequently, the transcendental issues raised by the parties should not be evaded;
for this.[156]
they must now be resolved to prevent future constitutional aberration.
Likewise, the warrantless arrests and seizures executed by the police were, according to
The Court finds and so holds that PP 1017 is constitutional insofar as it constitutes a call
the Solicitor General, illegal and cannot be condoned, thus:
by the President for the AFP to prevent or suppress lawless violence. The proclamation is sustained
CHIEF JUSTICE PANGANIBAN: by Section 18, Article VII of the Constitution and the relevant jurisprudence discussed
There seems to be some confusions if not earlier. However, PP 1017s extraneous provisions giving the President express or implied power
contradiction in your theory.
(1) to issue decrees; (2) to direct the AFP to enforce obedience to all laws even those not related to
SOLICITOR GENERAL BENIPAYO:
lawless violence as well as decrees promulgated by the President; and (3) to impose standards on
I dont know whether this will clarify. The acts,
the supposed illegal or unlawful acts committed on the media or any form of prior restraint on the press, are ultra vires and unconstitutional. The Court
occasion of 1017, as I said, it cannot be condoned. You
cannot blame the President for, as you said, a also rules that under Section 17, Article XII of the Constitution, the President, in the absence of a
misapplication of the law. These are acts of the police legislation, cannot take over privately-owned public utility and private business affected with public
officers, that is their responsibility.[157]
interest.
The Dissenting Opinion states that PP 1017 and G.O. No. 5 are constitutional in every In the same vein, the Court finds G.O. No. 5 valid. It is an Order issued by the President
aspect and should result in no constitutional or statutory breaches if applied according to their acting as Commander-in-Chief addressed to subalterns in the AFP to carry out the provisions of
letter. PP 1017. Significantly, it also provides a valid standard that the military and the police should take
The Court has passed upon the constitutionality of these issuances. Its ratiocination has only the necessary and appropriate actions and measures to suppress and prevent acts of
been exhaustively presented. At this point, suffice it to reiterate that PP 1017 is limited to the calling lawless violence. But the words acts of terrorism found in G.O. No. 5 have not been legally
out by the President of the military to prevent or suppress lawless violence, invasion or defined and made punishable by Congress and should thus be deemed deleted from the said
rebellion. When in implementing its provisions, pursuant to G.O. No. 5, the military and the police G.O. While terrorism has been denounced generally in media, no law has been enacted to guide
committed acts which violate the citizens rights under the Constitution, this Court has to declare the military, and eventually the courts, to determine the limits of the AFPs authority in carrying out
such acts unconstitutional and illegal. this portion of G.O. No. 5.
In this connection, Chief Justice Artemio V. Panganibans concurring opinion, attached On the basis of the relevant and uncontested facts narrated earlier, it is also pristine clear
hereto, is considered an integral part of this ponencia. that (1) the warrantless arrest of petitioners Randolf S. David and Ronald Llamas; (2) the dispersal of

the rallies and warrantless arrest of the KMU and NAFLU-KMU members; (3) the imposition of
SUMMATION standards on media or any prior restraint on the press; and (4) the warrantless search of
In sum, the lifting of PP 1017 through the issuance of PP 1021 a supervening event the Tribune offices and the whimsical seizures of some articles for publication and other materials,
would have normally rendered this case moot and academic. However, while PP 1017 was still are not authorized by the Constitution, the law and jurisprudence. Not even by the valid provisions
operative, illegal acts were committed allegedly in pursuance thereof. Besides, there is no guarantee of PP 1017 and G.O. No. 5.
Other than this declaration of invalidity, this Court cannot impose any civil, criminal or The warrantless arrest of Randolf S. David and Ronald Llamas; the dispersal and

administrative sanctions on the individual police officers concerned. They have not been individually warrantless arrest of the KMU and NAFLU-KMU members during their rallies, in the absence of

identified and given their day in court. The civil complaints or causes of action and/or relevant proof that these petitioners were committing acts constituting lawless violence, invasion or rebellion

criminal Informations have not been presented before this Court. Elementary due process bars this and violating BP 880; the imposition of standards on media or any form of prior restraint on the press,

Court from making any specific pronouncement of civil, criminal or administrative liabilities. as well as the warrantless search of the Tribune offices and whimsical seizure of its articles for

It is well to remember that military power is a means to an end and substantive civil publication and other materials, are declared UNCONSTITUTIONAL.

rights are ends in themselves. How to give the military the power it needs to protect the No costs.
Republic without unnecessarily trampling individual rights is one of the eternal balancing tasks

of a democratic state. During emergency, governmental action may vary in breadth and intensity

from normal times, yet they should not be arbitrary as to unduly restrain our peoples liberty. SO ORDERED.

Perhaps, the vital lesson that we must learn from the theorists who studied the various

competing political philosophies is that, it is possible to grant government the authority to cope with

crises without surrendering the two vital principles of constitutionalism: the maintenance of legal

limits to arbitrary power, and political responsibility of the government to the governed.[158]

WHEREFORE, the Petitions are partly granted. The Court rules that PP 1017

is CONSTITUTIONAL insofar as it constitutes a call by President Gloria Macapagal-Arroyo on the

AFP to prevent or suppress lawless violence. However, the provisions of PP 1017 commanding the

AFP to enforce laws not related to lawless violence, as well as decrees promulgated by the President,

are declared UNCONSTITUTIONAL. In addition, the provision in PP 1017 declaring national

emergency under Section 17, Article VII of the Constitution is CONSTITUTIONAL, but such

declaration does not authorize the President to take over privately-owned public utility or business

affected with public interest without prior legislation.

G.O. No. 5 is CONSTITUTIONAL since it provides a standard by which the AFP and

the PNP should implement PP 1017, i.e. whatever is necessary and appropriate actions and

measures to suppress and prevent acts of lawless violence. Considering that acts of terrorism

have not yet been defined and made punishable by the Legislature, such portion of G.O. No. 5 is

declared UNCONSTITUTIONAL.
[ G.R. No. 231658, July 04, 2017 ] forces, or depriving the Chief Executive or the Legislature, wholly or partially, of any of their
powers or prerogatives';
REPRESENTATIVES EDCEL C. LAGMAN, TOMASITO S. VILLARIN, GARY C. ALEJAO, EMMANUEL A.
WHEREAS, part of the reasons for the issuance of Proclamation No. 55 was the series of violent
BILLONES, AND TEDDY BRAWNER BAGUILAT, JR., PETITIONERS, VS. HON. SALVADOR C.
acts committed by the Maute terrorist group such as the attack on the military outpost in Butig,
MEDIALDEA, EXECUTIVE SECRETARY; HON. DELFIN N. LORENZANA, SECRETARY OF THE
Lanao del Sur in February 2016, killing and wounding several soldiers, and the mass jailbreak in
DEPARTMENT OF NATIONAL DEFENSE AND MARTIAL LAW ADMINISTRATOR; AND GEN. EDUARDO
Marawi City in August 2016, freeing their arrested comrades and other detainees;
AO, CHIEF OF STAFF OF THE ARMED FORCES OF THE PHILIPPINES AND MARTIAL LAW
IMPLEMENTOR, RESPONDENTS.
WHEREAS, today 23 May 2017, the same Maute terrorist group has taken over a hospital in
Marawi City, Lanao del Sur, established several checkpoints within the City, burned down certain
[G.R. No. 231771]
government and private facilities and inflicted casualties on the part of Government forces, and
started flying the flag of the Islamic State of Iraq and Syria (ISIS) in several areas, thereby openly
EUFEMIA CAMPOS CULLAMAT, VIRGILIO T. LINCUNA, ATELIANA U. HIJOS, ROLAND A. COBRADO,
attempting to remove from the allegiance to the Philippine Government this part of Mindanao and
CARL ANTHONY D. OLALO, ROY JIM BALANGHIG, RENATO REYES, JR., CRISTINA E. PALABAY,
deprive the Chief Executive of his powers and prerogatives to enforce the laws of the land and to
AMARYLLIS H. ENRIQUEZ, ACT TEACHERS' REPRESENTATIVE ANTONIO L. TINIO, GABRIELA
maintain public order and safety in Mindanao, constituting the crime of rebellion; and
WOMEN'S PARTY REPRESENTATIVE ARLENE D. BROSAS, KABATAAN PARTY-LIST REPRESENTATIVE
SARAH JANE I. ELAGO, MAE PANER, GABRIELA KRISTA DALENA, ANNA ISABELLE ESTEIN, MARK
WHEREAS, this recent attack shows the capability of the Maute group and other rebel groups to
VINCENT D. LIM, VENCER MARI CRISOSTOMO, JOVITA MONTES, PETITIONERS, VS. PRESIDENT
sow terror, and cause death and damage to property not only in Lanao del Sur but also in other
RODRIGO DUTERTE, EXECUTIVE SECRETARY SALVADOR MEDIALDEA, DEFENSE SECRETARY DELFIN
parts of Mindanao.
LORENZANA, ARMED FORCES OF THE PHILIPPINES CHIEF OF STAFF LT. GENERAL EDUARDO AO,
PHILIPPINE NATIONAL POLICE DIRECTOR-GENERAL RONALD DELA ROSA, RESPONDENTS.
NOW, THEREFORE, I, RODRIGO ROA DUTERTE, President of the Republic of the Philippines, by
virtue of the powers vested in me by the Constitution and by law, do hereby proclaim as follows:
[G.R. No. 231774]
SECTION 1. There is hereby declared a state of martial law in the Mindanao group of islands for a
NORKAYA S. MOHAMAD, SITTIE NUR DYHANNA S. MOHAMAD, NORAISAH S. SANI, ZAHRIA P.
period not exceeding sixty days, effective as of the date hereof.
MUTI-MAPANDI, PETITIONERS, VS. EXECUTIVE SECRETARY SALVADOR C. MEDIALDEA,
DEPARTMENT OF NATIONAL DEFENSE (DND) SECRETARY DELFIN N. LORENZANA, DEPARTMENT OF
SECTION 2. The privilege of the writ of habeas corpus shall likewise be suspended in the aforesaid
THE INTERIOR AND LOCAL GOVERNMENT (DILG) SECRETARY (OFFICER-IN-CHARGE) CATALINO S.
area for the duration of the state of martial law.
CUY, ARMED FORCES OF THE PHILIPPINES (AFP) CHIEF OF STAFF GEN. EDUARDO M. AO,
PHILIPPINE NATIONAL POLICE (PNP) CHIEF DIRECTOR GENERAL RONALD M. DELA ROSA, NATIONAL
DONE in the Russian Federation, this 23rd day of May in the year of our Lord, Two Thousand and
SECURITY ADVISER HERMOGENES C. ESPERON, JR., RESPONDENTS.
Seventeen.
Within the timeline set by Section 18, Article VII of the Constitution, the President submitted to
DEL CASTILLO, J.: Congress on May 25, 2017, a written Report on the factual basis of Proclamation No. 216.
Effective May 23, 2017, and for a period not exceeding 60 days, President Rodrigo Roa Duterte The Report pointed out that for decades, Mindanao has been plagued with rebellion and lawless
issued Proclamation No. 216 declaring a state of martial law and suspending the privilege of the violence which only escalated and worsened with the passing of time.
writ of habeas corpus in the whole of Mindanao.
Mindanao has been the hotbed of violent extremism and a brewing rebellion for decades. In more
The full text of Proclamation No. 216 reads as follows: recent years, we have witnessed the perpetration of numerous acts of violence challenging the
authority of the duly constituted authorities, i.e., the Zamboanga siege, the Davao bombing, the
WHEREAS, Proclamation No. 55, series of 2016, was issued on 04 September 2016 declaring a state Mamasapano carnage, and the bombings in Cotabato, Sultan Kudarat, Sulu, and Basilan, among
of national emergency on account of lawless violence in Mindanao; others. Two armed groups have figured prominently in all these, namely, the Abu Sayaff Group
(ASG) and the ISIS-backed Maute Group.[1]
WHEREAS, Section 18, Article VII of the Constitution provides that 'x x x In case of invasion or The President went on to explain that on May 23, 2017, a government operation to capture the
rebellion, when the public safety requires it, he (the President) may, for a period not exceeding high-ranking officers of the Abu Sayyaf Group (ASG) and the Maute Group was conducted. These
sixty days, suspend the privilege of the writ of habeas corpus or place the Philippines or any part groups, which have been unleashing havoc in Mindanao, however, confronted the government
thereof under martial law x x x'; operation by intensifying their efforts at sowing violence aimed not only against the government
authorities and its facilities but likewise against civilians and their properties. As narrated in the
WHEREAS, Article 134 of the Revised Penal Code, as amended by R.A. No. 6968, provides that 'the President's Report:
crime of rebellion or insurrection is committed by rising and taking arms against the Government
for the purpose of removing from the allegiance to said Government or its laws, the territory of On 23 May 2017, a government operation to capture Isnilon Hapilon, a senior leader of the ASG,
the Republic of the Philippines or any part thereof, of any body of land, naval or other armed and Maute Group operational leaders, Abdullah and Omarkhayam Maute, was confronted with
armed resistance which escalated into open hostility against the government. Through these inmates of the City Jail.
groups' armed siege and acts of violence directed towards civilians and government authorities,
institutions and establishments, they were able to take control of major social, economic, and The BJMP directed its personnel at the Marawi City Jail and other affected areas to
political foundations of Marawi City which led to its paralysis. This sudden taking of control was evacuate.
intended to lay the groundwork for the eventual establishment of a DAESH wilayat or province in
Mindanao. By evening of 23 May 2017, at least three (3) bridges in Lanao del Sur, namely, Lilod,
Bangulo, and Sauiaran, fell under the control of these groups. They threatened to bomb
Based on verified intelligence reports, the Maute Group, as of the end of 2016, consisted of the bridges to pre-empt military reinforcement.
around two hundred sixty-three (263) members, fully armed and prepared to wage combat in
furtherance of its aims. The group chiefly operates in the province of Lanao del Sur, but has
extensive networks and linkages with foreign and local armed groups such as the Jemaah
As of 2222H, persons connected with the Maute Group had occupied several areas in
Marawi City, including Naga Street, Bangolo Street, Mapandi, and Camp Keithly, as well
Islamiyah, Mujahidin Indonesia Timur and the ASG. It adheres to the ideals being espoused by the
as the following barangays: Basak Malutlot, Mapandi, Saduc, Lilod Maday, Bangon,
DAESH, as evidenced by, among others, its publication of a video footage declaring its allegiance to
Saber, Bubong, Marantao, Caloocan, Banggolo, Barionaga, and Abubakar.
the DAESH. Reports abound that foreign-based terrorist groups, the ISIS (Islamic State of Iraq and
Syria) in particular, as well as illegal drug money, provide financial and logistical support to the
Maute Group. These lawless armed groups had likewise set up road blockades and checkpoints at the
Iligan City-Marawi City junction.
The events commencing on 23 May 2017 put on public display the groups' clear intention to
establish an Islamic State and their capability to deprive the duly constituted authorities - the Later in the evening, the Maute Group burned Dansalan College Foundation, Cathedral
President, foremost - of their powers and prerogatives.[2] of Maria Auxiliadora, the nun's quarters in the church, and the Shia Masjid Moncado
In particular, the President chronicled in his Report the events which took place on May 23, 2017 Colony. Hostages were taken from the church.
in Marawi City which impelled him to declare a state of martial law and suspend the privilege of
writ of habeas corpus, to wit: About five (5) faculty members of Dansalan College Foundation had been reportedly
killed by the lawless groups.

At 1400H members of the Maute Group and ASG, along with their sympathizers, Other educational institutions were also burned, namely, Senator Ninoy Aquino College
commenced their attack on various facilities government and privately owned - in the Foundation and the Marawi Central Elementary Pilot School.
City of Marawi.
The Maute Group also attacked Amai Pakpak Hospital and hoisted the DAESH flag there,
At 1600H around fifty (50) armed criminals assaulted Marawi City Jail being managed by among other several locations. As of 0600H of 24 May 2017, members of the Maute
the Bureau of Jail Management and Penology (BJMP). Group were seen guarding the entry gates of Amai Pakpak Hospital. They held hostage
the employees of the Hospital and took over the PhilHealth office located thereat.
The Maute Group forcibly entered the jail facilities, destroyed its main gate, and
assaulted on-duty personnel. BJMP personnel were disarmed, tied, and/or locked inside The groups likewise laid siege to another hospital, Filipino-Libyan Friendship Hospital,
the cells. which they later set ablaze.

The group took cellphones, personnel-issued firearms, and vehicles (i.e., two [2] Lawless armed groups likewise ransacked the Landbank of the Philippines and
prisoner vans and private vehicles). commandeered one of its armored vehicles.

By 1630H, the supply of power into Marawi City had been interrupted, and sporadic Latest information indicates that about seventy-five percent (75%) of Marawi City has
gunfights were heard and felt everywhere. By evening, the power outage had spread been infiltrated by lawless armed groups composed of members of the Maute Group
citywide. (As of 24 May 2017, Marawi City's electric supply was still cut off, plunging the and the ASG. As of the time of this Report, eleven (11) members of the Armed Forces
city into total black-out.) and the Philippine National Police have been killed in action, while thirty-five (35) others
have been seriously wounded.
From 1800H to 1900H, the same members of the Maute Group ambushed and burned
the Marawi Police Station. A patrol car of the Police Station was also taken. There are reports that these lawless armed groups are searching for Christian
communities in Marawi City to execute Christians. They are also preventing Maranaos
A member of the Provincial Drug Enforcement Unit was killed during the takeover of the from leaving their homes and forcing young male Muslims to join their groups.
Marawi City Jail. The Maute Group facilitated the escape of at least sixty-eight (68)
Based on various verified intelligence reports from the AFP and the PNP, there exists a absolute control over the entirety of Mindanao. These circumstances demand swift and decisive
strategic mass action of lawless armed groups in Marawi City, seizing public and private action to ensure the safety and security of the Filipino people and preserve our national
facilities, perpetrating killings of government personnel, and committing armed uprising integrity.[6]
against and open defiance of the government.[3] The President ended his Report in this wise:

While the government is presently conducting legitimate operations to address the on-going
The unfolding of these events, as well as the classified reports he received, led the President to
rebellion, if not the seeds of invasion, public safety necessitates the continued implementation of
conclude that -
martial law and the suspension of the privilege of the writ of habeas corpus in the whole of
Mindanao until such time that the rebellion is completely quelled.[7]
These activities constitute not simply a display of force, but a clear attempt to establish the groups'
In addition to the Report, representatives from the Executive Department, the military and police
seat of power in Marawi City for their planned establishment of a DAESH wilayat or province
authorities conducted briefings with the Senate and the House of Representatives relative to the
covering the entire Mindanao.
declaration of martial law.
The cutting of vital lines for transportation and power; the recruitment of young Muslims to
After the submission of the Report and the briefings, the Senate issued P.S. Resolution No.
further expand their ranks and strengthen their force; the armed consolidation of their members
388[8] expressing full support to the martial law proclamation and finding Proclamation No. 216 "to
throughout Marawi City; the decimation of a segment of the city population who resist; and the
be satisfactory, constitutional and in accordance with the law". In the same Resolution, the Senate
brazen display of DAESH flags constitute a clear, pronounced, and unmistakable intent to remove
declared that it found "no compelling reason to revoke the same". The Senate thus resolved as
Marawi City, and eventually the rest of Mindanao, from its allegiance to the Government.
follows:
There exists no doubt that lawless armed groups are attempting to deprive the President of his
NOW, THEREFORE, BE IT RESOLVED, as it is hereby resolved, by way of the sense of the Senate,
power, authority, and prerogatives within Marawi City as a precedent to spreading their control
that the Senate finds the issuance of Proclamation No. 216 to be satisfactory, constitutional and in
over the entire Mindanao, in an attempt to undermine his control over executive departments,
accordance with the law. The Senate hereby supports fully Proclamation No. 216 and finds no
bureaus, and offices in said area; defeat his mandate to ensure that all laws are faithfully executed;
compelling reason to revoke the same.[9]
and remove his supervisory powers over local governments.[4]
The Senate's counterpart in the lower house shared the same sentiments. The House of
According to the Report, the lawless activities of the ASG, Maute Group, and other criminals,
Representatives likewise issued House Resolution No. 1050[10] "EXPRESSING THE FULL SUPPORT OF
brought about undue constraints and difficulties to the military and government personnel,
THE HOUSE OF REPRESENTATIVES TO PRESIDENT RODRIGO DUTERTE AS IT FINDS NO REASON TO
particularly in the performance of their duties and functions, and untold hardships to the
REVOKE PROCLAMATION NO. 216, ENTITLED 'DECLARING A STATE OF MARTIAL LAW AND
civilians, viz.:
SUSPENDING THE PRIVILEGE OF THE WRIT OF HABEAS CORPUS IN THE WHOLE OF MINDANAO'".
Law enforcement and other government agencies now face pronounced difficulty sending their
The Petitions
reports to the Chief Executive due to the city-wide power outages. Personnel from the BJMP have
been prevented from performing their functions. Through the attack and occupation of several
A) G.R. No. 231658 (Lagman Petition)
hospitals, medical services in Marawi City have been adversely affected. The bridge and road
blockades set up by the groups effectively deprive the government of its ability to deliver basic
On June 5, 2017, Representatives Edcel C. Lagman, Tomasito S. Villarin, Gary C. Alejano, Emmanuel
services to its citizens. Troop reinforcements have been hampered, preventing the government
A. Billones, and Teddy Brawner Baguilat, Jr. filed a Petition[11] Under the Third Paragraph of Section
from restoring peace and order in the area. Movement by both civilians and government
18 of Article VII of the 1987 Constitution.
personnel to and from the city is likewise hindered.
First, the Lagman Petition claims that the declaration of martial la has no sufficient factual basis
The taking up of arms by lawless armed groups in the area, with support being provided by
because there is no rebellion or invasion in Marawi City or in any part of Mindanao. It argues that
foreign-based terrorists and illegal drug money, and their blatant acts of defiance which embolden
acts of terrorism in Mindanao do not constitute rebellion[12] since there is no proof that its purpose
other armed groups in Mindanao, have resulted in the deterioration of public order and safety in
is to remove Mindanao or any part thereof from allegiance to the Philippines, its laws, or its
Marawi City; they have likewise compromised the security of the entire Island of Mindanao.[5]
territory.[13] It labels the flying of ISIS flag by the Maute Group in Marawi City and other outlying
The Report highlighted the strategic location of Marawi City and the crucial and significant role it
areas as mere propaganda[14] and not an open attempt to remove such areas from the allegiance
plays in Mindanao, and the Philippines as a whole. In addition, the Report pointed out the possible
to the Philippine Government and deprive the Chief Executive of the assertion and exercise of his
tragic repercussions once Marawi City falls under the control of the lawless groups.
powers and prerogatives therein. It contends that the Maute Group is a mere private army, citing
as basis the alleged interview of Vera Files with Joseph Franco wherein the latter allegedly
The groups' occupation of Marawi City fulfills a strategic objective because of its terrain and the
mentioned that the Maute Group is more of a "clan's private militia latching into the IS brand
easy access it provides to other parts of Mindanao. Lawless armed groups have historically used
theatrically to inflate perceived capability".[15] The Lagman Petition insists that during the briefing,
provinces adjoining Marawi City as escape routes, supply lines, and backdoor passages.
representatives of the military and defense authorities did not categorically admit nor deny the
presence of an ISIS threat in the country but that they merely gave an evasive answer[16] that
Considering the network and alliance-building activities among terrorist groups, local criminals,
"there is ISIS in the Philippines".[17] The Lagman Petition also avers that Lt. Gen. Salvador Mison, Jr.
and lawless armed men, the siege of Marawi City is a vital cog in attaining their long-standing goal:
himself admitted that the current armed conflict in Marawi City was precipitated or initiated by
the government in its bid to capture Hapilon.[18] Based on said statement, it concludes that the Petition and set the case for oral argument on June 13, 14, and 15, 2017.
objective of the Maute Group's armed resistance was merely to shield Hapilon and the Maute
brothers from the government forces, and not to lay siege on Marawi City and remove its On June 9, 2017, two other similar petitions docketed as G.R. Nos. 231771 and 231774 were filed
allegiance to the Philippine Republic.[19] It then posit that if at all, there is only a threat of rebellion and eventually consolidated with G.R. No. 231658.[32]
in Marawi City which is akin to "imminent danger" of rebellion, which is no longer a valid ground
for the declaration of martial law.[20] B) G.R. No. 231771 (Cullamat Petition)

Second, the Lagman Petition claims that the declaration of martial law has no sufficient factual The Cullamat Petition, "anchored on Section 18, Article VII"[33] of the Constitution, likewise seeks
basis because the President's Report contained "false, inaccurate, contrived and hyperbolic the nullification of Proclamation No. 216 for being unconstitutional because it lacks sufficient
accounts".[21] factual basis that there is rebellion in Mindanao and that public safety warrants its declaration.[34]

It labels as false the claim in the President's Report that the Maute Group attacked Amai Pakpak In particular, it avers that the supposed rebellion described in Proclamation No. 216 relates to
Medical Center. Citing online reports on the interview of Dr. Amer Saber (Dr. Saber), the hospital's events happening in Marawi City only an not in the entire region of Mindanao. It concludes that
Chief, the Lagman Petition insists that the Maute Group merely brought an injured member to the Proclamation No. 216 "failed to show any factual basis for the imposition of martial law in
hospital for treatment but did not overrun the hospital or harass the hospital personnel.[22] The the entire Mindanao,"[35] "failed to allege any act of rebellion outside Marawi City, much less x x x
Lagman Petition also refutes the claim in the President's Report that a branch of the Landbank of allege that public safety requires the imposition of martial law in the whole of Mindanao".[36]
the Philippines was ransacked and its armored vehicle commandeered. It alleges that the bank
employees themselves clarified that the bank was not ransacked while the armored vehicle was The Cullamat Petition claims that the alleged "capability of the Maute Group and other rebel
owned by a third party and was empty at the time it was commandeered.[23] It also labels as false groups to sow terror and cause death and damage to property"[37] does not rise to the level of
the report on the burning of the Senator Ninoy Aquino College Foundation and the Marawi Central rebellion sufficient to declare martial law in the whole of Mindanao.[38] It also posits that there is
Elementary Pilot School. It avers that the Senator Ninoy Aquino College Foundation is intact as of no lawless violence in other parts of Mindanao similar to that in Marawi City.[39]
May 24, 2017 and that according to Asst. Superintendent Ana Alonto, the Marawi Central
Elementary Pilot School was not burned by the terrorists.[24] Lastly, it points out as false the report Moreover, the Cullamat Petition assails the inclusion of the phrase "other rebel groups" in the last
on the beheading of the police chief of Malabang, Lanao del Sur, and the occupation of the Marawi Whereas Clause of Proclamation No. 216 for being vague as it failed to identify these rebel groups
City Hall and part of the Mindanao State University.[25] and specify the acts of rebellion that they were supposedly waging.[40]

Third, the Lagman Petition claims that the declaration of martial law has no sufficient factual basis In addition, the Cullamat Petition cites alleged inaccuracies, exaggerations, and falsities in the
since the President's Report mistakenly included the attack on the military outpost in Butig, Lanao Report of the President to Congress, particularly the attack at the Amai Pakpak Hospital, the
del Sur in February 2016, the mass jailbreak in Marawi City in August 2016, the Zamboanga siege, ambush and burning of the Marawi Police Station, the killing of five teachers of Dansalan College
the Davao market bombing, the Mamasapano carnage and other bombing incidents in Cotabato, Foundation, and the attacks on various government facilities.[41]
Sultan Kudarat, and Basilan, as additional factual bases for the proclamation of martial law. It
contends that these events either took place long before the conflict in Marawi City began, had In fine, the Cullamat Petition prays for the Court to declare Proclamation No. 216 as
long been resolved, or with the culprits having already been arrested.[26] unconstitutional or in the alternative, should the Court find justification for the declaration of
martial law and suspension of the privilege of the writ of habeas corpus in Marawi City, to declare
Fourth, the Lagman Petition claims that the declaration of martial law has no sufficient factual the same as unconstitutional insofar as its inclusion of the other parts of Mindanao.[42]
basis considering that the President acted alone and did not consult the military establishment or
any ranking official[27] before making the proclamation. C) G.R. No. 231774 (Mohamad Petition)

Finally, the Lagman Petition claims that the President's proclamation of martial law lacks sufficient The Mohamad Petition, denominated as a "Petition for Review of the Sufficiency of [the] Factual
factual basis owing to the fact that during the presentation before the Committee of the Whole of Basis of [the] Declaration of Martial Law and [the] Suspension of the Privilege of the Writ
the House of Representatives, it was shown that the military was even successful in pre-empting of Habeas Corpus,"[43] labels itself as "a special proceeding"[44] or an "appropriate proceeding filed
the ASG and the Maute Group's plan to take over Marawi City and other parts of Mindanao; there by any citizen"[45] authorized under Section 18, Article VII of the Constitution.
was absence of any hostile plan by the Moro Islamic Liberation Front; and the number of foreign
fighters allied with ISIS was "undetermined"[28]which indicates that there are only a meager The Mohamad Petition posits that martial law is a measure of last resort[46] and should be invoked
number of foreign fighters who can lend support to the Maute Group.[29] by the President only after exhaustion of less severe remedies.[47] It contends that the
extraordinary powers of the President should be dispensed sequentially, i.e., first, the power to
Based on the foregoing argumentation, the Lagman Petition asks the Court to: (1)"exercise its call out the armed forces; second, the power to suspend the privilege of the writ of habeas corpus;
specific and special jurisdiction to review sufficiency of the factual basis of Proclamation No. 216"; and finally, the power to declare martial law.[48] It maintains that the President has no discretion to
and (2) render "a Decision voiding and nullifying Proclamation No. 216" for lack of sufficient factual choose which extraordinary power to use; moreover, his choice must be dictated only by, and
basis.[30] commensurate to, the exigencies of the situation.[49]

In a Resolution[31] dated June 6, 2017, the Court required respondents to comment on the Lagman According to the Mohamad Petition, the factual situation in Marawi is not so grave as to require
the imposition of martial law.[50] It asserts that the Marawi incidents "do not equate to the should be the standard in reviewing the sufficiency of factual basis.
existence of a public necessity brought about by an actual rebellion, which would compel the
imposition of martial law or the suspension of the privilege of the writ of habeas corpus".[51] It The OSG maintains that the burden lies not with the respondents but with the petitioners to prove
proposes that "[m]artial law can only be justified if the rebellion or invasion has reached such that Proclamation No. 216 is bereft of factual basis. It thus takes issue with petitioners' attempt to
gravity that [its] imposition x x x is compelled by the needs of public safety"[52] which, it believes, is shift the burden of proof when they asked the Court "to compel [the] respondents to present
not yet present in Mindanao. proof on the factual basis"[66] of Proclamation No. 216. For the OSG, "he who alleges must
prove"[67] and that governmental actions are presumed to be valid and constitutional.[68]
Moreover, it alleges that the statements contained in the President's Report to the Congress, to
wit: that the Maute Group intended to establish an Islamic State; that they have the capability to Likewise, the OSG posits that the sufficiency of the factual basis must be assessed from the
deprive the duly constituted authorities of their powers and prerogatives; and that the Marawi trajectory or point of view of the President and based on the facts available to him at the time the
armed hostilities is merely a prelude to a grander plan of taking over the whole of Mindanao, are decision was made.[69] It argues that the sufficiency of the factual basis should be examined not
conclusions bereft of substantiation.[53] based on the facts discovered after the President had made his decision to declare martial law
because to do so would subject the exercise of the President's discretion to an impossible
The Mohamad Petition posits that immediately after the declaration of martial law, and without standard.[70] It reiterates that the President's decision should be guided only by the information
waiting for a congressional action, a suit may already be brought before the Court to assail the and data available to him at the time he made the determination.[71] The OSG thus asserts that
sufficiency of the factual basis of Proclamation No. 216. facts that were established after the declaration of martial law should not be considered in the
review of the sufficiency of the factual basis of the proclamation of martial law. The OSG suggests
Finally, in invoking this Court's power to review the sufficiency of the factual basis for the that the assessment of after-proclamation-facts lies with the President and Congress for the
declaration of martial law and the suspension of the privilege of the writ of habeas corpus, the purpose of determining the, propriety of revoking or extending the martial law. The OSG fears that
Mohamad Petition insists that the Court may "look into the wisdom of the [President's] actions, the Court considers after-proclamation-facts in its review of the sufficiency of the factual basis for
[and] not just the presence of arbitrariness".[54] Further, it asserts that since it is making a negative the proclamation, it would in effect usurp the powers of the Congress to determine whether
assertion, then the burden to prove the sufficiency of the factual basis is shifted to and lies on the martial law should be revoked or extended.[72]
respondents.[55] It thus asks the Court "to compel the [r]espondents to divulge relevant
information"[56] in order for it to review the sufficiency of the factual basis. It is also the assertion of the OSG that the President could validly rely on intelligence reports
coming from the Armed Forces of the Philippines;[73] and that he could not be expected to
In closing, the Mohamad Petition prays for the Court to exercise its power to review, "compel personally determine the veracity of the contents of the reports.[74] Also, since the power to
respondents to present proof on the factual basis [of] the declaration of martial law and the impose martial law is vested solely on the President as Commander-in-Chief, the lack of
suspension of the privilege of the writ of habeas corpus in Mindanao"[57] and declare as recommendation from the Defense Secretary, or any official for that matter, will not nullify the
unconstitutional Proclamation No. 216 for lack of sufficient factual basis. said declaration, or affect its validity, or compromise the sufficiency of the factual basis.

The Consolidated Comment Moreover, the OSG opines that the petitioners miserably failed to validly refute the facts cited by
the President in Proclamation No. 216 and in his Report to the Congress by merely citing news
The respondents' Consolidated Comment[58] was filed on June 12, 2017, as required by the Court. reports that supposedly contradict the facts asserted therein or by criticizing in piecemeal the
Noting that the same coincided with the celebration of the 119th anniversary of the independence happenings in Marawi. For the OSG, the said news articles are "hearsay evidence, twice
of this Republic, the Office of the Solicitor General (OSG) felt that "defending the constitutionality removed,"[75] and thus inadmissible and without probative value, and could not overcome the
of Proclamation No. 216" should serve as "a rallying call for every Filipino to unite behind one true "legal presumption bestowed on governmental acts".[76]
flag and defend it against all threats from within and outside our shores".[59]
Finally, the OSG points out that it has no duty or burden to prove that Proclamation No. 216 has
The OSG acknowledges that Section 18, Article VII of the Constitution vests the Court with the sufficient factual basis. It maintains that the burden rests with the petitioners. However, the OSG
authority or power to review the sufficiency of the factual basis of the declaration of martial still endeavors to lay out the factual basis relied upon by the President "if only to remove any
law.[60] The OSG, however, posits that although Section 18, Article VII lays the basis for the exercise doubt as to the constitutionality of Proclamation No. 216".[77]
of such authority or power, the same constitutional provision failed to specify the vehicle, mode or
remedy through which the "appropriate proceeding" mentioned therein may be resorted to. The The facts laid out by the OSG in its Consolidated Comment will be discussed in detail in the Court's
OSG suggests that the "appropriate proceeding" referred to in Section 18, Article VII may be Ruling.
availed of using the vehicle, mode or remedy of a certiorari petition, either under Section 1 or 5, of
Article VIII.[61] Corollarily, the OSG maintains that the review power is not mandatory, but
discretionary only, on the part of the Court.[62] The Court has the discretion not to give due course ISSUES
to the petition.[63]
The issues as contained in the revised Advisory[78] are as follows:
Prescinding from the foregoing, the OSG contends that the sufficiency of the factual basis of
Proclamation No. 216 should be reviewed by the Court "under the lens of grave abuse of
discretion"[64] and not the yardstick of correctness of the facts.[65] Arbitrariness, not correctness,
1. Whether or not the petitions docketed as G.R. Nos. 231658, 231771, and 231774 are the 9. Whether or not nullifying Proclamation No. 216 of 23 May 2017 will:
"appropriate proceeding" covered by Paragraph 3, Section 18, Article VII of the
Constitution sufficient to invoke the mode of review required of this Court when a a. have the effect of recalling Proclamation No. 55 s. 2016; or
declaration of martial law or the suspension of the privilege of the writ of habeas b. also nullify the acts of the President in calling out the armed forces to quell
corpus is promulgated; lawless violence in Marawi and other parts of the Mindanao region.

2. Whether or not the President in declaring martial law and suspending the privilege of After the oral argument, the parties submitted their respective memoranda and supplemental
the writ of habeas corpus: memoranda.

a. is required to be factually correct or only not arbitrary in his appreciation of OUR RULING
facts;
b. is required to obtain the favorable recommendation thereon the Secretary of I. Locus standi of petitioners.
National Defense;
c. is required to take into account only the situation at the time of the One of the requisites for judicial review is locus standi, i.e., "the constitutional question is brought
proclamation, even if subsequent events prove the situation to have not been before [the Court] by a party having the requisite 'standing' to challenge it."[79] As a general rule,
accurately reported; the challenger must have "a personal and substantial interest in the case such that he has
sustained, or will sustain, direct injury as a result of its enforcement."[80] Over the years, there has
3. Whether or not the power of this Court to review the sufficiency of the factual basis [of] been a trend towards relaxation of the rule on legal standing, a prime example of which is found in
the proclamation of martial law or the suspension of the privilege of the writ of habeas Section 18 of Article VII which provides that any citizen may file the appropriate proceeding to
corpus is independent of the actual actions that have been taken by Congress jointly or assail the sufficiency of the factual basis of the declaration of martial law or the suspension of the
separately; privilege of the writ of habeas corpus. "[T]he only requisite for standing to challenge the validity of
the suspension is that the challenger be a citizen. He need not even be a taxpayer."[81]
4. Whether or not there were sufficient factual [basis] for the proclamation of martial law
or the suspension of the privilege of the writ of habeas corpus; Petitioners in the Cullamat Petition claim to be "suing in their capacities as citizens of the
Republic;"[82] similarly, petitioners in the Mohamad Petition all claim to be "Filipino citizens, all
a. What are the parameters for review? women, all of legal [age], and residents of Marawi City".[83] In the Lagman Petition, however,
b. Who has the burden of proof? petitioners therein did not categorically mention that they are suing as citizens but merely referred
c. What is the threshold of evidence? to themselves as duly elected Representatives.[84] That they are suing in their official capacities as
Members of Congress could have elicited a vigorous discussion considering the issuance by the
5. Whether the exercise of the power of judicial review by this Court involves the House of Representatives of House Resolution No. 1050 expressing full support o President
calibration of graduated powers granted the President a Commander-in-Chief, namely Duterte and finding no reason to revoke Proclamation No. 216. By such resolution, the House of
calling out powers, suspension of the privilege of the writ of habeas corpus, and Representatives is declaring that it finds no reason to review the sufficiency of the factual basis of
declaration of martial law; the martial law declaration, which is in direct contrast to the views and arguments being espoused
by the petitioners in the Lagman Petition. Considering, however, the trend towards relaxation of
6. Whether or not Proclamation No. 216 of 23 May 2017 may be considered vague and the rules on legal standing, as well as the transcendental issues involved in the present Petitions,
thus null and void: the Court will exercise judicial self-restraint[85] and will not venture into this matter. After all, "the
Court is not entirely without discretion to accept a suit which does not satisfy the requirements of
a. with its inclusion of "other rebel groups;" or a [bona fide] case or of standing. Considerations paramount to [the requirement of legal standing]
b. since it has no guidelines specifying its actual operational parameters within could compel assumption of jurisdiction."[86] In any case, the Court can take judicial cognizance of
the entire Mindanao region; the fact that petitioners in the Lagman Petition are all citizens of the Philippines since Philippine
citizenship is a requirement for them to be elected as representatives. We will therefore consider
7. Whether or not the armed hostilities mentioned in Proclamation No. 216 and in the them a suing in their own behalf as citizens of this country. Besides, respondents did not question
Report of the President to Congress are sufficient [bases]: petitioners' legal standing.

a. for the existence of actual rebellion; or II. Whether or not the petitions are the "appropriate proceeding" covered by paragraph 3,
b. for a declaration of martial law or the suspension of the privilege of the writ Section 18, Article VII of the Constitution sufficient to invoke the mode of review required by the
of habeas corpus in the entire Mindanao region; Court.
8. Whether or not terrorism or acts attributable to terrorism are equivalent to actual All three petitions beseech the cognizance of this Court based on the third paragraph of Section
rebellion and the requirements of public safety sufficient to declare martial law or 18, Article VII (Executive Department) of the 1987 Constitution which provides:
suspend the privilege of the writ of habeas corpus; and
The Supreme Court may review, in an appropriate proceeding filed by any citizen, the sufficiency The Commander-in-Chief provisions of the 1935 Constitution had enabled President Ferdinand
of the factual basis of the proclamation of martial law or the suspension of the privilege of the writ Marcos to impose authoritarian rule on the Philippines from 1972 to 1986. Supreme Court
or the extension thereof, and must promulgate its decision thereon within thirty days from its decisions during that period upholding the actions taken by Mr. Marcos made authoritarian rule
filing. part of Philippine constitutional jurisprudence. The members of the Constitutional Commission,
During the oral argument, the petitioners theorized that the jurisdiction of this Court under the very much aware of these facts, went about reformulating the Commander-in-Chief powers with a
third paragraph of Section 18, Article VII is sui generis.[87] It is a special and specific jurisdiction of view to dismantling what had been constructed during the authoritarian years. The new formula
the Supreme Court different from those enumerated in Sections 1 and 5 of Article VIII. [88] included revised grounds for the activation of emergency powers, the manner of activating them,
the scope of the powers, and review of presidential action.[94] (Emphasis supplied)
The Court agrees. To recall, the Court held in the 1951 case of Montenegro v. Castaeda[95] that the authority to
decide whether there is a state of rebellion requiring the suspension of the privilege of the writ
a) Jurisdiction must be specifically conferred by the Constitution or by law. of habeas corpus is lodged with the President and his decision thereon is final and conclusive upon
the courts. This ruling was reversed in the 1971 case of Lansang where it was held that the factual
It is settled that jurisdiction over the subject matter is conferred only by the Constitution or by the basis of the declaration of martial law and the suspension of the privilege of the writ of habeas
law.[89] Unless jurisdiction has been specifically conferred by the Constitution or by some legislative corpus is not a political question and is within the ambit of judicial review.[96] However, in 1983, or
act, no body or tribunal has the power to act or pass upon a matter brought before it for after the declaration of martial law by former President Ferdinand E. Marcos, the Court, in Garcia-
resolution. It is likewise settled that in the absence of a clear legislative intent, jurisdiction cannot Padilla v. Enrile,[97] abandoned the ruling in Lansang and reverted to Montenegro. According to the
be implied from the language of the Constitution or a statute.[90] It must appear clearly from the Supreme Court, the constitutional power of the President to suspend the privilege of the writ
law or it will not be held to exist.[91] of habeas corpus is not subject to judicial inquiry.[98]

A plain reading of the afore-quoted Section 18, Article VII reveals that it specifically grants Thus, by inserting Section 18 in Article VII which allows judicial review of the declaration of martial
authority to the Court to determine the sufficiency of the factual basis of the proclamation of law and suspension of the privilege of the writ of habeas corpus, the framers of the 1987
martial law or suspension of the privilege of the writ of habeas corpus. Constitution in effect constitutionalized and reverted to the Lansang doctrine.

b) "In an appropriate proceeding" does not refer to a petition for certiorari filed under Section 1 or d) Purpose of Section 18, Article VII is to provide additional safeguard against possible abuse by the
5 of Article VIII. President on the exercise of the extraordinary powers.

It could not have been the intention of the framers of the Constitution that the phrase "in an Section 18, Article VII is meant to provide additional safeguard against possible abuse by the
appropriate proceeding" would refer to a Petition for Certiorari pursuant to Section 1 or Section 5 President in the exercise of his power to declare martial law or suspend the privilege of the writ
of Article VIII. The standard of review in a petition for certiorari is whether the respondent has of habeas corpus. Reeling from the aftermath of the Marcos martial law, the framers of the
committed any grave abuse of discretion amounting to lack or excess of jurisdiction in the Constitution deemed it wise to insert the now third paragraph of Section 18 of Article VII.[99] This is
performance of his or her functions. Thus, it is not the proper tool to review the sufficiency of the clear from the records of the Constitutional Commission when its members were deliberating on
factual basis of the proclamation or suspension. It must be emphasized that under Section 18, whether the President could proclaim martial law even without the concurrence of Congress.
Article VII, the Court is tasked to review the sufficiency of the factual basis of the President's Thus:
exercise of emergency powers. Put differently, if this Court applies the standard of review used in
a petition for certiorari, the same would emasculate its constitutional task under Section 18, MR. SUAREZ. Thank you, Madam President.
Article VII.
The Commissioner is proposing a very substantial amendment because this means that he is
c) Purpose/significance of Section 18, Article VII is to constitutionalize the pre-Marcos martial law vesting exclusively unto the President the right to determine the factors which may lead to the
ruling in In the Matter of the Petition for Habeas Corpus of Lansang. declaration of martial law and the suspension of the writ of habeas corpus. I suppose he has strong
and compelling reasons in seeking to delete this particular phrase. May we be informed of his good
The third paragraph of Section 18, Article VII was inserted by the framers of the 1987 Constitution and substantial reasons?
to constitutionalize the pre-Marcos martial law ruling of this Court in In the Matter of the Petition
for Habeas Corpus of Lansang,[92] to wit: that the factual basis of the declaration of martial law or MR. MONSOD. This situation arises in cases of invasion or rebellion. And in previous interpellations
the suspension of the privilege of the writ of habeas corpus is not a political question but precisely regarding this phrase, even during the discussions on the Bill of Rights, as I understand it, the
within the ambit of judicial review. interpretation is a situation of actual invasion or rebellion. In these situations, the President has to
act quickly. Secondly, this declaration has a time fuse. It is only good for a maximum of 60 days. At
"In determining the meaning, intent, and purpose of a law or constitutional provision, the history the end of 60 days, it automatically terminates. Thirdly, the right of the judiciary to inquire into the
of the times out of which it grew and to which it may be rationally supposed to bear some direct sufficiency of the factual basis of the proclamation always exists, even during those first 60 days.
relationship, the evils intended to be remedied, and the good to be accomplished are proper
subjects of inquiry."[93] Fr. Joaquin G. Bernas, S.J. (Fr. Bernas), a member of the Constitutional MR. SUAREZ. Given our traumatic experience during the past administration, if we give exclusive
Commission that drafted the 1987 Constitution, explained: right to the President to determine these factors, especially the existence of an invasion or
rebellion and the second factor of determining whether the public safety requires it or not, may I
call the attention of the Gentleman to what happened to us during the past administration. placed in Article VIII or the Judicial Department but remained under Article VII or the Executive
Proclamation No. 1081 was issued by Ferdinand E. Marcos in his capacity as President of the Department.
Philippines by virtue of the powers vested upon him purportedly under Article VII, Section 10 (2) of
the Constitution, wherein he made this predicate under the "Whereas" provision: During the closing session of the Constitutional Commission's deliberations, President Cecilia
Muoz Palma expressed her sentiments on the 1987 Constitution. She said:
Whereas, the rebellion and armed action undertaken by these lawless elements of the
Communists and other armed aggrupations organized to overthrow the Republic of the Philippines The executive power is vested in the President of the Philippines elected by the people for a six-
by armed violence and force have assumed the magnitude of an actual state of war against our year term with no reelection for the duration of his/her life. While traditional powers inherent in
people and the Republic of the Philippines. the office of the President are granted, nonetheless for the first time, there are specific
And may I also call the attention of the Gentleman to General Order No. 3, also promulgated by provisions which curtail the extent of such powers. Most significant is the power of the Chief
Ferdinand E. Marcos, in his capacity as Commander-in-Chief of all the Armed Forces of the Executive to suspend the privilege of the writ of habeas corpus or proclaim martial law.
Philippines and pursuant to Proclamation No. 1081 dated September 21, 1972 wherein he said,
among other things: The flagrant abuse of that power of the Commander-in-Chief by Mr. Marcos caused the imposition
of martial law for more than eight years and the suspension of the privilege of the writ even after
Whereas, martial law having been declared because of wanton destruction of lives and properties, the lifting of martial law in 1981. The new Constitution now provides that those powers can be
widespread lawlessness and anarchy and chaos and disorder now prevailing throughout the exercised only in two cases, invasion or rebellion when public safety demands it, only for a period
country, which condition has been brought about by groups of men who are actively engaged in a not exceeding 60 days, and reserving to Congress the power to revoke such suspension or
criminal conspiracy to seize political and state power in the Philippines in order to take over the proclamation of martial law which congressional action may not be revoked by the President.
government by force and violence, the extent of which has now assumed the proportion of an More importantly, the action of the President is made subject to judicial review, thereby again
actual war against our people and the legitimate government . . . discarding jurisprudence which render[s] the executive action a political question and beyond the
And he gave all reasons in order to suspend the privilege of the writ of habeas corpus and declare jurisdiction of the courts to adjudicate.
martial law in our country without justifiable reason. Would the Gentleman still insist on the
deletion of the phrase 'and, with the concurrence of at least a majority of all the members of the For the first time, there is a provision that the state of martial law does not suspend the operation
Congress'? of the Constitution nor abolish civil courts or legislative assemblies, or vest jurisdiction to military
tribunals over civilians, or suspend the privilege of the writ. Please forgive me if, at this point, I
MR. MONSOD. Yes, Madam President, in the case of Mr. Marcos, he is undoubtedly an state that this constitutional provision vindicates the dissenting opinions I have written during my
aberration in our history and national consciousness. But given the possibility that there would tenure in the Supreme Court in the martial law cases.[101]
be another Marcos, our Constitution now has sufficient safeguards. As I said, it is not really true, f) To interpret "appropriate proceeding" as filed under Section 1 of Article VIII would be contrary to
as the Gentleman has mentioned, that there is an exclusive right to determine the factual basis the intent of the Constitution.
because the paragraph beginning on line 9 precisely tells us that the Supreme Court may review,
in an appropriate proceeding filed by any citizen, the sufficiency of the factual basis of the To conclude that the "appropriate proceeding" refers to a Petition for Certiorari filed under the
proclamation of martial law or the suspension of the privilege of the writ or the extension expanded jurisdiction of this Court would, therefore, contradict the clear intention of the framers
thereof and must promulgate its decision on the same within 30 days from its filing. of the Constitution to place additional safeguards against possible martial law abuse for,
invariably, the third paragraph of Section 18, Article VII would be subsumed under Section I of
I believe that there are enough safeguards. The Constitution is supposed to balance the interests Article VIII. In other words, the framers of the Constitution added the safeguard under the third
of the country. And here we are trying to balance the public interest in case of invasion or rebellion paragraph of Section 18, Article VII on top of the expanded jurisdiction of this Court.
as against the rights of citizens. And I am saying that there are enough safeguards, unlike in 1972
when Mr. Marcos was able to do all those things mentioned.[100] g) Jurisdiction of the Court is not restricted to those enumerated in Sections 1 and 5 of Article VIII.
To give more teeth to this additional safeguard, the framers of the 1987 Constitution not only
placed the President's proclamation of martial law or suspension of the privilege of the writ The jurisdiction of this Court is not restricted to those enumerated in Sections 1 and 5 of Article
of habeas corpus within the ambit of judicial review, it also relaxed the rule on standing by VIII. For instance, its jurisdiction to be the sole judge of all contests relating to the election,
allowing any citizen to question before this Court the sufficiency of the factual basis of such returns, and qualifications of the President or Vice-President can be found in the last paragraph of
proclamation or suspension. Moreover, the third paragraph of Section 18, Article VII veritably Section 4, Article VII.[102] The power of the Court to review on certiorari the decision, order, or
conferred upon any citizen a demandable right to challenge the sufficiency of the factual basis of ruling of the Commission on Elections and Commission on Audit can be found in Section 7, Article
said proclamation or suspension. It further designated this Court as the reviewing tribunal to IX(A).[103]
examine, in an appropriate proceeding, the sufficiency of the factual basis and to render its
decision thereon within a limited period of 30 days from date of filing. h) Unique features of the third paragraph of Section 18, Article VII make it sui generis.

e) Purpose of Section 18, Article VII is to curtail the extent of the powers of the President. The unique features of the third paragraph of Section 18, Article VII clearly indicate that it should
be treated as sui generis separate and different from those enumerated in Article VIII. Under the
The most important objective, however, of Section 18, Article VII is the curtailment of the extent third paragraph of Section 18, Article VII, a petition filed pursuant therewith will follow a different
of the powers of the Commander-in-Chief. This is the primary reason why the provision was not rule on standing as any citizen may file it. Said provision of the Constitution also limits the issue to
the sufficiency of the factual basis of the exercise by the Chief Executive of his emergency powers. b) The framers of the 1987 Constitution intended the judicial power to review to be exercised
The usual period for filing pleadings in Petition for Certiorari is likewise not applicable under the independently from the congressional power to revoke.
third paragraph of Section 18, Article VII considering the limited period within which this Court has
to promulgate its decision. If only to show that the intent of the framers of the 1987 Constitution was to vest the Court and
Congress with veto powers independently from each other, we quote the following exchange:
A proceeding "[i]n its general acceptation, [is] the form in which actions are to be brought and
defended, the manner of intervening in suits, of conducting them, the mode of deciding them, of MS. QUESADA. Yesterday, the understanding of many was that there would be safeguards that
opposing judgments, and of executing."[104] In fine, the phrase "in an appropriate proceeding" Congress will be able to revoke such proclamation.
appearing on the third paragraph of Section 18, Article VII refers to any action initiated by a citizen
for the purpose of questioning the sufficiency of the factual basis of the exercise of the Chief MR. RAMA. Yes.
Executive's emergency powers, as in these cases. It could be denominated as a complaint, a
petition, or a matter to be resolved by the Court. MS. QUESADA. But now, if they cannot meet because they have been arrested or that the
Congress has been padlocked, then who is going to declare that such a proclamation was not
III. The power of the Court to review the sufficiency of the factual basis of the proclamation of warranted?
martial law or the suspension of the privilege, of the writ of habeas corpus under Section 18,
Article VII of the 1987 Constitution is independent of the actions taken by Congress. xxxx

During the oral argument,[105] the OSG urged the Court to give deference to the actions of the two MR. REGALADO. May I also inform Commissioner Quesada that the judiciary is not exactly just
co-equal branches of the Government: on the part of the President as Commander-in-Chief, in standing by. A petition for a writ of habeas corpus, if the Members are detained, can immediately
resorting to his extraordinary powers to declare martial law and suspend the privilege of the writ be applied for, and the Supreme Court shall also review the factual basis. x x x[107]
of habeas corpus; and on the part of Congress, in giving its imprimatur to Proclamation No. 216 c) Re-examination of the Court's pronouncement in Fortun v. President Macapagal-Arroyo.
and not revoking the same.
Considering the above discussion, the Court finds it imperative to re-examine, reconsider, and set
The framers of the 1987 Constitution reformulated the scope of the extraordinary powers of the aside its pronouncement in Fortun v. President Macapagal-Arroyo[108] to the effect that:
President as Commander-in-Chief and the review of the said presidential action. In particular, the
President's extraordinary powers of suspending the privilege of the writ of habeas corpus and Consequently, although the Constitution reserves to the Supreme Court the power to review the
imposing martial law are subject to the veto powers of the Court and Congress. sufficiency of the factual basis of the proclamation or suspension in a proper suit, it is implicit that
the Court must allow Congress to exercise its own review powers, which is automatic rather than
a) The judicial power to review versus the congressional power to revoke. initiated. Only when Congress defaults in its express duty to defend the Constitution through such
review should the Supreme Court step in as its final rampart. The constitutional validity of the
The Court may strike down the presidential proclamation in an appropriate proceeding filed by any President's proclamation of martial law or suspension of the writ of habeas corpus is first a political
citizen on the ground of lack o sufficient factual basis. On the other hand, Congress may revoke th question in the hands of Congress before it becomes a justiciable one in the hands of the Court.[109]
proclamation or suspension, which revocation shall not be set aside by the President.
xxxx
In reviewing the sufficiency of the factual basis of the proclamation o suspension, the Court
considers only the information and data available to the President prior to or at the time of the If the Congress procrastinates or altogether fails to fulfill its duty respecting the proclamation or
declaration; it is not allowed to "undertake an independent investigation beyond the suspension within the short time expected of it, then the Court can step in, hear the petitions
pleadings."[106] On the other hand, Congress may take into consideration not only data available challenging the President's action, and ascertain if it has a factual basis. x x x.[110]
prior to, but likewise events supervening the declaration. Unlike the Court which does not look By the above pronouncement, the Court willingly but unwittingly clipped its own power and
into the absolute correctness of the factual basis as will be discussed below, Congress could probe surrendered the same to Congress as well as, abdicated from its bounden duty to review. Worse,
deeper and further; it can delve into the accuracy of the facts presented before it. the Court considered itself just on stand-by, waiting and willing to act as a substitute in case
Congress "defaults." It is an aberration, a stray declaration, which must be rectified and set aside in
In addition, the Court's review power is passive; it is only initiated by the filing of a petition "in an this proceeding.[111]
appropriate proceeding" by a citizen. On the other hand, Congress' review mechanism is automatic
in the sense that it may be activated by Congress itself at any time after the proclamation or We, therefore, hold that the Court can simultaneously exercise its power of review with, and
suspension was made. independently from, the power to revoke by Congress. Corollary, any perceived inaction or default
on the part of Congress does not deprive or deny the Court of its power to review.
Thus, the power to review by the Court and the power to revoke by Congress are not only totally
different but likewise independent from each other although concededly, they have the same IV. The judicial power to review the sufficiency of factual basis of the declaration of martial law
trajectory, which is, the nullification of the presidential proclamation. Needless to say, the power or the suspension of the privilege of the writ of habeas corpus does not extend to the calibration
of the Court to review can be exercised independently from the power of revocation of Congress. of the President's decision of which among his graduated powers he will avail of in a given
situation.
The provision is put there, precisely, to reverse the doctrine of the Supreme Court. I think it is the
The President as the Commander-in-Chief wields the extraordinary powers of: a) calling out the case of Aquino v. COMELEC where the Supreme Court said that in times of martial law, the
armed forces; b) suspending the privilege of the writ of habeas corpus; and c) declaring martial President automatically has legislative power. So these two clauses denied that. A state of martial
law.[112] These powers may be resorted to only under specified conditions. law does not suspend the operation of the Constitution; therefore, it does not suspend the
principle of separation of powers.
The framers of the 1987 Constitution reformulated the powers of the Commander-in-Chief by
revising the "grounds for the activation of emergency powers, the manner of activating them, the The question now is: During martial law, can the President issue decrees? The answer we gave to
scope of the powers, and review of presidential action."[113] that question in the Committee was: During martial law, the President may have the powers of a
commanding general in a theatre of war. In actual war when there is fighting in an area, the
a) Extraordinary powers of the President distinguished. President as the commanding general has the authority to issue orders which have the effect of
law but strictly in a theater of war, not in the situation we had during the period of martial law. In
Among the three extraordinary powers, the calling out power is the most benign and involves other words, there is an effort here to return to the traditional concept of martial law as it was
ordinary police action.[114] The President may resort to this extraordinary power whenever it developed especially in American jurisprudence, where martial law has reference to the theater of
becomes necessary to prevent or suppress lawless violence, invasion, or rebellion. "[T]he power to war.[124]
call is fully discretionary to the President;"[115] the only limitations being that he acts within
permissible constitutional boundaries or in a manner not constituting grave abuse of xxxx
discretion.[116] In fact, "the actual use to which the President puts the armed forces is x x x not
subject to judicial review."[117] FR. BERNAS. This phrase was precisely put here because we have clarified the meaning of martial
law; meaning, limiting it to martial law as it has existed in the jurisprudence in international law,
The extraordinary powers of suspending the privilege of the writ of habeas corpus and/or that it is a law for the theater of war. In a theater of war, civil courts are unable to function. If in
declaring martial law may be exercised only when there is actual invasion or rebellion, and public the actual theater of war civil courts, in fact, are unable to function, then the military commander
safety requires it. The 1987 Constitution imposed the following limits in the exercise of these is authorized to give jurisdiction even over civilians to military courts precisely because the civil
powers: "(1) a time limit of sixty days; (2) review and possible revocation by Congress; [and] (3) courts are closed in that area. But in the general area where the civil courts are open then in no
review and possible nullification by the Supreme Court."[118] case can the military courts be given jurisdiction over civilians. This is in reference to a theater of
war where the civil courts, in fact, are unable to function.
The framers of the 1987 Constitution eliminated insurrection, and the phrase "imminent danger
thereof" as grounds for the suspension of the privilege of the writ of habeas corpus or declaration MR. FOZ. It is a state of things brought about by the realities of the situation in that specified
of martial law.[119] They perceived the phrase "imminent danger" to be "fraught with possibilities of critical area.
abuse;"[120] besides, the calling out power of the President "is sufficient for handling imminent
danger."[121] FR. BERNAS. That is correct.

The powers to declare martial law and to suspend the privilege of the writ of habeas MR. FOZ. And it is not something that is brought about by a declaration of the Commander-in-
corpus involve curtailment and suppression of civil rights and individual freedom. Thus, the Chief.
declaration of martial law serves as a warning to citizens that the Executive Department has called
upon the military to assist in the maintenance of law and order, and while the emergency remains, FR. BERNAS. It is not brought about by a declaration of the Commander-in-Chief. The
the citizens must, under pain of arrest and punishment, not act in a manner that will render it understanding here is that the phrase 'nor authorize the conferment of jurisdiction on military
more difficult to restore order and enforce the law.[122] As such, their exercise requires more courts and agencies over civilians' has reference to the practice under the Marcos regime where
stringent safeguards by the Congress, and review by the Court.[123] military courts were given jurisdiction over civilians. We say here that we will never allow that
except in areas where civil courts are, in fact, unable to function and it becomes necessary for
b) What really happens during martial law? some kind of court to function.[125]
A state of martial law is peculiar because the President, at such a time, exercises police power,
During the oral argument, the following questions cropped up: What really happens during the which is normally a function of the Legislature. In particular, the President exercises police power,
imposition of martial law? What powers could the President exercise during martial law that he with the military's assistance, to ensure public safety and in place of government agencies which
could not exercise if there is no martial law? Interestingly, these questions were also discussed by for the time being are unable to cope with the condition in a locality, which remains under the
the framers of the 1987 Constitution, viz.: control of the State.[126]

FR. BERNAS. That same question was asked during the meetings of the Committee: What precisely In David v. President Macapagal-Arroyo,[127] the Court, quoting Justice Vicente V. Mendoza's
does martial law add to the power of the President to call on the armed forces? The first and (Justice Mendoza) Statement before the Senate Committee on Justice on March 13, 2006, stated
second lines in this provision state: that under a valid declaration of martial law, the President as Commander-in-Chief may order the
"(a) arrests and seizures without judicial warrants; (b) ban on public assemblies; (c) [takeover] of
A state of martial law does not suspend the operation of the Constitution, nor supplant the news media and agencies and press censorship; and (d) issuance of Presidential Decrees x x x". [128]
functioning of the civil courts or legislative assemblies . . .
Worthy to note, however, that the above-cited acts that the President may perform do not give provided by the Committee, the Congress may revoke, amend, or shorten or even increase the
him unbridled discretion to infringe on the rights of civilians during martial law. This is because period of such suspension.[134]
martial law does not suspend the operation of the Constitution, neither does it supplant the
operation of civil courts or legislative assemblies. Moreover, the guarantees under the Bill of Rights xxxx
remain in place during its pendency. And in such instance where the privilege of the writ of habeas
corpus is also suspended, such suspension applies only to those judicially charged with rebellion or MR. NATIVIDAD. First and foremost, we agree with the Commissioner's thesis that in the first
offensed connected with invasion.[129] imposition of martial law there is no need for concurrence of the Members of Congress because
the provision says 'in case of actual invasion or rebellion.' If there is actual invasion and rebellion,
Clearly, from the foregoing, while martial law poses the most severe threat to civil liberties,[130] the as Commissioner Crispino de Castro said, there is a need for immediate response because there is
Constitution has safeguards against the President's prerogative to declare a state of martial law. an attack. Second, the fact of securing a concurrence may be impractical because the roads might
be blocked or barricaded. x x x So the requirement of an initial concurrence of the majority of all
c) "Graduation" of powers refers to hierarchy based on scope and effect) it does not refer to a Members of the Congress in case of an invasion or rebellion might be impractical as I can see it.
sequence) order, or arrangement by which the Commander-in-Chief must adhere to.
Second, Section 15 states that the Congress may revoke the declaration or lift the suspension.
Indeed, the 1987 Constitution gives the "President, as Commander-in-Chief, a 'sequence' of
'graduated power[s]'. From the most to the least benign, these are: the calling out power, the And third, the matter of declaring martial law is already a justiciable question and no longer a
power to suspend the privilege of the writ of habeas corpus, and the power to declare martial political one in that it is subject to judicial review at any point in time. So on that basis, I agree that
law."[131] It must be stressed, however, that the graduation refers only to hierarchy based on scope there is no need for concurrence as a prerequisite to declare martial law or to suspend the
and effect. It does not in any manner refer to a sequence, arrangement, or order which the privilege of the writ of habeas corpus. x x x[135]
Commander-in-Chief must follow. This socalled "graduation of powers" does not dictate or restrict
the manner by which the President decides which power to choose. xxxx

These extraordinary powers are conferred by the Constitution with the President as Commander- MR. SUAREZ. Thank you.
in-Chief; it therefore necessarily follows that the power and prerogative to determine whether the
situation warrants a mere exercise of the calling out power; or whether the situation demands The Commissioner is suggesting that in connection with Section 15, we delete the phrase 'and,
suspension of the privilege of the writ of habeas corpus; or whether it calls for the declaration of with the concurrence of at least a majority of all the Members of the Congress . . .'
martial law, also lies, at least initially, with the President. The power to choose, initially, which
among these extraordinary powers to wield in a given set of conditions is a judgment call on the MR. PADILLA. That is correct especially for the initial suspension of the privilege of the writ
part of the President. As Commander-in-Chief, his powers are broad enough to include his of habeas corpus or also the declaration of martial law.
prerogative to address exigencies or threats that endanger the government, and the very integrity
of the State.[132] MR. SUAREZ. So in both instances, the Commissioner is suggesting that this would be an exclusive
prerogative of the President?
It is thus beyond doubt that the power of judicial review does not extend to calibrating the
President's decision pertaining to which extraordinary power to avail given a set of facts or MR. PADILLA. At least initially, for a period of 60 days. But even that period of 60 days may be
conditions. To do so would be tantamount to an incursion into the exclusive domain of the shortened by the Congress or the Senate because the next sentence says that the Congress or the
Executive and an infringement on the prerogative that solely, at least initially, lies with the Senate may even revoke the proclamation.[136]
President.
xxxx
d) The framers of the 1987 Constitution intended the Congress not to interfere a priori in the
decision-making process of the President. MR. SUAREZ. x x x

The elimination by the framers of the 1987 Constitution of the requirement of prior concurrence The Commissioner is proposing a very substantial amendment because this means that he is
of the Congress in the initial imposition of martial law or suspension of the privilege of the writ vesting exclusively unto the President the right to determine the factors which may lead to the
of habeas corpus further supports the conclusion that judicial review does not include the declaration of martial law and the suspension of the writ of habeas corpus. I suppose he has strong
calibration of the President's decision of which of his graduated powers will be availed of in a given and compelling reasons in seeking to delete this particular phrase. May we be informed of his good
situation. Voting 28 to 12, the framers of the 1987 Constitution removed the requirement of and substantial reasons?
congressional concurrence in the first imposition of martial law and suspension of the privilege.[133]
MR. MONSOD. This situation arises in cases of invasion or rebellion. And in previous interpellations
MR. PADILLA. x x x regarding this phrase, even during the discussions on the Bill of Rights, as I understand it, the
interpretation is a situation of actual invasion or rebellion. In these situations, the President has to
We all agree with the suspension of the writ or the proclamation of martial law should not require act quickly. Secondly, this declaration has a time fuse. It is only good for a maximum of 60 days. At
beforehand the concurrence of the majority of the Members of the Congress. However, as the end of 60 days, it automatically terminates. Thirdly, the right of the judiciary to inquire into the
sufficiency of the factual basis of the proclamation always exists, even during those first 60 days. element that prompted the Constitutional Commission to eliminate the requirement of
concurrence of the Congress in the initial imposition by the President of martial law or suspension
xxxx of the privilege of the writ of habeas corpus. Considering that the proclamation of martial law or
suspension of the privilege of the writ of habeas corpus is now anchored on actual invasion or
MR. MONSOD. Yes, Madam President, in the case of Mr. Marcos[,] he is undoubtedly an rebellion and when public safety requires it, and is no longer under threat or in imminent danger
aberration in our history and national consciousness. But given the possibility that there would be thereof, there is a necessity and urgency for the President to act quickly to protect the
another Marcos, our Constitution now has sufficient safeguards. As I said, it is not really true, as country.[138] The Court, as Congress does, must thus accord the President the same leeway by not
the Gentleman mentioned, that there is an exclusive right to determine the factual basis because wading into the realm that is reserved exclusively by the Constitution to the Executive
the paragraph being on line 9 precisely tells us that the Supreme court may review, in an Department.
appropriate proceeding filed by any citizen, the sufficiency of the factual basis of the proclamation
of martial law or the suspension of the privilege of the writ or the extension thereof and must f) The recommendation of the Defense Secretary is not a condition for the declaration of martial
promulgate its decision on the same within 30 days from its filing. law or suspension of the privilege of the writ of habeas corpus.

I believe that there are enough safeguards. The Constitution is supposed to balance the interests Even the recommendation of, or consultation with, the Secretary of National Defense, or other
of the country. And here we are trying to balance the public interest in case of invasion or rebellion high-ranking military officials, is not a condition for the President to declare martial law. A plain
as against the rights of citizens. x x x reading of Section 18, Article VII of the Constitution shows that the President's power to declare
martial law is not subject to any condition except for the requirements of actual invasion or
MR. SUAREZ. Will that prevent a future President from doing what Mr. Marcos had done? rebellion and that public safety requires it. Besides, it would be contrary to common sense if the
decision of the President is made dependent on the recommendation of his mere alter ego. Rightly
MR. MONSOD. There is nothing absolute in this world, and there may be another Marcos. What so, it is only on the President and no other that the exercise of the powers of the Commander-in-
we are looking for are safeguards that are reasonable and, I believe, adequate at this point. On the Chief under Section 18, Article VII of the Constitution is bestowed.
other hand, in case of invasion or rebellion, even during the first 60 days when the intention here
is to protect the country in that situation, it would be unreasonable to ask that there should be a g) In any event, the President initially employed the most benign action - the calling out power -
concurrence on the part of the Congress, which situation is automatically terminated at the end of before he declared martial law and suspended the privilege of the writ of habeas corpus.
such 60 days.
At this juncture, it must be stressed that prior to Proclamation No. 216 or the declaration of
xxxx martial law on May 23, 2017, the President had already issued Proclamation No. 55 on September
4, 2016, declaring a state of national emergency on account of lawless violence in Mindanao. This,
MR. SUAREZ. Would the Gentleman not feel more comfortable if we provide for a legislative check in fact, is extant in the first Whereas Clause of Proclamation No. 216. Based on the foregoing
on this awesome power of the Chief Executive acting as Commander-in-Chief? presidential actions, it can be gleaned that although there is no obligation or requirement on his
part to use his extraordinary powers on a graduated or sequential basis, still the President made
MR. MONSOD. I would be less comfortable if we have a presidency that cannot act under those the conscious and deliberate effort to first employ the most benign from among his extraordinary
conditions. powers. As the initial and preliminary step towards suppressing and preventing the armed
hostilities in Mindanao, the President decided to use his calling out power first. Unfortunately, the
MR. SUAREZ. But he can act with the concurrence of the proper or appropriate authority? situation did not improve; on the contrary, it only worsened. Thus, exercising his sole and exclusive
prerogative, the President decided to impose martial law an suspend the privilege of the writ
MR. MONSOD. Yes. But when those situations arise, it is very unlikely that the concurrence of of habeas corpus on the belief that the armed hostilities in Mindanao already amount to actual
Congress would be available; and, secondly, the President will be able to act quickly in order to rebellion and publip safety requires it.
deal with the circumstances.
V. Whether or not Proclamation No. 216 may be considered vague and thus void because of (a)
MR. SUAREZ. So, we would be subordinating actual circumstances to expediency? its inclusion of "other rebel groups"; and (b) the absence of any guideline specifying its actual
operational parameters within the entire Mindanao region.
MR. MONSOD. Ido not believe it is expediency when one is trying to protect the country in the
event of an invasion or a rebellion.[137] Proclamation No. 216 is being facially challenged on the ground of "vagueness" by the insertion of
The foregoing exchange clearly manifests the intent of the Constitution not to allow Congress to the phrase "other rebel groups"[139] in its Whereas Clause and for lack of available guidelines
interfere a priori in the President's choice of extraordinary powers. specifying its actual operational parameters within the entire Mindanao region, making the
proclamation susceptible to broad interpretation, misinterpretation, or confusion.
e) The Court must similarly and necessarily refrain from calibrating the President's decision of
which among his extraordinary powers to avail given a certain situation or condition. This argument lacks legal basis.

It cannot be overemphasized that time is paramount in situations necessitating the proclamation a) Void-for-vagueness doctrine.
of martial law or suspension of the privilege of the writ of habeas corpus. It was precisely this time
The void-for-vagueness doctrine holds that a law is facially invalid if "men of common intelligence Invalidation of statutes "on its face" should be used sparingly because it results in striking down
must necessarily guess at its meaning and differ as to its application."[140] "[A] statute or act may be statutes entirely on the ground that they might be applied to Rarties not before the Court whose
said to be vague when it lacks comprehensible standards that men of common intelligence must activities are constitutionally protected.[146] "Such invalidation would constitute a departure from
necessarily guess at its meaning and differ in its application. [In such instance, the statute] is the usual requirement of 'actual case and controversy' and permit decisions to be made in a sterile
repugnant to the Constitution in two respects: (1) it violates due process for failure to accord abstract context having no factual concreteness."[147]
persons, especially the parties targeted by it, fair notice of the conduct to avoid; and (2) it leaves
law enforcers unbridled discretion in carrying out its provisions and becomes an arbitrary flexing of c) Proclamation No. 216 cannot be facially challenged using the vagueness doctrine.
the Government muscle."[141]
Clearly, facial review of Proclamation No. 216 on the grounds of vagueness is unwarranted.
b) Vagueness doctrine applies only in free speech cases. Proclamation No. 216 does not regulate speech, religious freedom, and other fundamental rights
that may be facially challenged.[148] What it seeks to penalize is conduct, not speech.
The vagueness doctrine is an analytical tool developed for testing "on their faces" statutes in free
speech cases or, as they are called in American law, First Amendment cases.[142] A facial challenge As held by the Court in David v. President Macapagal-Arroyo,[149] the facial review of Proclamation
is allowed to be made to a vague statute and also to one which is overbroad because of possible No. 1017, issued by then President Gloria Macapagal-Arroyo declaring a state of national
"'chilling effect' on protected speech that comes from statutes violating free speech. A person who emergency, on ground of vagueness is uncalled for since a plain reading of Proclamation No. 1017
does not know whether his speech constitutes a crime under an overbroad or vague law may shows that it is not primarily directed at speech or even speech-related conduct. It is actually a call
simply restrain himself from speaking in order to avoid being charged of a crime. The overbroad or upon the Armed Forces of the Philippines (AFP) to prevent or suppress all forms of lawless
vague law thus chills him into silence."[143] violence. Like Proclamation No. 1017, Proclamation No. 216 pertains to a spectrum of conduct, not
free speech, which is manifestly subject to state regulation.
It is best to stress that the vagueness doctrine has a special application only to free-speech cases.
They are not appropriate for testing the validity of penal statutes.[144] Justice Mendoza explained d) Inclusion of "other rebel groups" does not make Proclamation No. 216 vague.
the reason as follows:
The contention that the phrase "other rebel groups" leaves Proclamation No. 216 open to broad
A facial challenge is allowed to be made to a vague statute and to one which is overbroad because interpretation, misinterpretation, and confusion, cannot be sustained.
of possible 'chilling effect' upon protected speech. The theory is that '[w]hen statutes regulate or
proscribe speech and no readily apparent construction suggests itself as a vehicle for rehabilitating In People v. Nazario,[150] the Court enunciated that:
the statutes in a single prosecution, the transcendent value to all society of constitutionally
protected expression is deemed to justify allowing attacks on overly broad statutes with no As a rule, a statute or act may be said to be vague when it lacks comprehensible standards that
requirement that the person making the attack demonstrate that his own conduct could not be men 'of common intelligence must necessarily guess at its meaning and differ as to its application.'
regulated by a statute drawn with narrow specificity.' The possible harm to society in permitting It is repugnant to the Constitution in two respects: (1) it violates due process for failure to accord
some unprotected speech to go unpunished is outweighed by the possibility that the protected persons, especially the parties targetted by it, fair notice of the conduct to avoid; and (2) it leaves
speech of others may be deterred and perceived grievances left to fester because of possible law enforcers unbridled discretion in carrying out its provisions and becomes an arbitrary flexing of
inhibitory effects of overly broad statutes. the Government muscle.

This rationale does not apply to penal statutes. Criminal statutes have general in terrorem effect But the act must be utterly vague on its face, that is to say, it cannot be clarified by either a saving
resulting from their very existence, and, if facial challenge is allowed for this reason alone, the clause or by construction. Thus, in Coates v. City of Cincinnati, the U.S. Supreme Court struck down
State may well be prevented from enacting laws against socially harmful conduct. In the area of an ordinance that had made it illegal for 'three or more persons to assemble on any sidewalk and
criminal law, the law cannot take chances as in the area of free speech. there conduct themselves in a manner annoying to persons passing by.' Clearly, the ordinance
imposed no standard at all 'because one may never know in advance what annoys some people
xxxx but does not annoy others.'

In sum, the doctrines of strict scrutiny, overbreadth, and vagueness are analytical tools developed Coates highlights what has been referred to as a 'perfectly vague' act whose obscurity is evident on
for testing 'on their faces' statutes in free speech cases or, as they are called in American law, First its face. It is to be distinguished, however, from legislation couched in imprecise language - but
Amendment cases. They cannot be made to do service when what is involved is a criminal statute. which nonetheless specifies a standard though defectively phrased - in which case, it may be
With respect to such statute, the established rule is that 'one to whom application of a statute is 'saved' by proper construction.[151]
constitutional will not be heard to attack the statute on the ground that impliedly it might also be The term "other rebel groups" in Proclamation No. 216 is not at all vague when viewed in the
taken as applying to other persons or other situations in which its application might be context of the words that accompany it. Verily, the text of Proclamation No. 216 refers to "other
unconstitutional.' As has been pointed out, 'vagueness challenges in the First Amendment context, rebel groups" found in Proclamation No. 55, which it cited by way of reference in its Whereas
like overbreadth challenges typically produce facial invalidation, while statutes found vague as a clauses.
matter of due process typically are invalidated [only] 'as applied' to a particular defendant.' x x
x[145] e) Lack of guidelines/operational parameters does not make Proclamation No. 216 vague.
Neither could Proclamation No. 216 be described as vague, and thus void, on the ground that it may only be examined by the Court as to whether such power was exercised within permissible
has no guidelines specifying its actual operational parameters within the entire Mindanao region. constitutional limits or in a manner constituting grave abuse of discretion.[155]
Besides, operational guidelines will serve only as mere tools for the implementation of the
proclamation. In Part III, we declared that judicial review covers only the sufficiency of information In Zamora, the Court categorically ruled that the Integrated Bar of the Philippines had failed to
or data available to or known to the President prior to, or at the time of, the declaration or sufficiently comply with the requisites of locus standi, as it was not able to show any specific
suspension. And, as will be discussed exhaustively in Part VII, the review will be confined to the injury which it had suffered or could suffer by virtue of President Joseph Estrada's order deploying
proclamation itself and the Report submitted to Congress. the Philippine Marines to join the PNP in visibility patrols around the metropolis. [156]

Clearly, therefore, there is no need for the Court to determine the constitutionality of the This locus standi requirement, however, need not be complied with in so far as the Court's
implementing and/or operational guidelines, general orders, arrest orders and other orders issued jurisdiction to review the sufficiency of the factual basis of the President's declaration of martial
after the proclamation for being irrelevant to its review. Thus, any act committed under the said law or suspension of the privilege of the writ of habeas corpus is concerned. In fact, by
orders in violation of the Constitution and the laws, such as criminal acts or human rights constitutional design, such review may be instituted by any citizen before the Court,[157] without
violations, should be resolved in a separate proceeding. Finally, there is a risk that if the Court the need to prove that he or she stands to sustain a direct and personal injury as a consequence of
wades into these areas, it would be deemed a trespassing into the sphere that is reserved the questioned Presidential act/s.
exclusively for Congress in exercise of its power to revoke.
But, even assuming arguendo that the Court finds no sufficient basis for the declaration of martial
VI. Whether or not nullifying Proclamation No. 216 will (a) have the effect of recalling law in this case, such ruling could not affect the President's exercise of his calling out power
Proclamation No. 55; or (b) also nullify the acts of the President in calling out the armed forces to through Proclamation No. 55.
quell lawless violence in Marawi and other parts of the Mindanao region.
b) The operative fact doctrine.
a) The calling out power is in a different category from the power to declare martial law and the
power to suspend the privilege of the writ of habeas corpus; nullification of Proclamation No. 216 Neither would the nullification of Proclamation No. 216 result in thcl nullification of the acts of the
will not affict Proclamation No. 55. President done pursuant thereto. Under the "operative fact doctrine," the unconstitutional statute
is recognized as an "operative fact" before it is declared unconstitutional.[158]
The Court's ruling in these cases will not, in any way, affect the President's declaration of a state of
national emergency on account of lawless violence in Mindanao through Proclamation No. 55 Where the assailed legislative or executive act is found by the judiciary to be contrary to the
dated September 4, 2016, where he called upon the Armed Forces and the Philippine National Constitution, it is null and void. As the new Civil Code puts it: 'When the courts declare a law to be
Police (PNP) to undertake such measures to suppress any and all forms of lawless violence in the inconsistent with the Constitution, the former shall be void and the latter shall govern.
Mindanao region, and to prevent such lawless violence from spreading and escalating elsewhere in Administrative or executive acts, orders and regulations shall be valid only when they are not
the Philippines. contrary to the laws or the Constitution.' The above provision of the Civil Code reflects the
orthodox view that an unconstitutional act, whether legislative or executive, is not a law, confers
In Kulayan v. Tan,[152] the Court ruled that the President's calling out power is in a different no rights, imposes no duties, and affords no protection. This doctrine admits of qualifications,
category from the power to suspend the privilege of the writ of habeas corpus and the power to however. As the American Supreme Court stated: 'The actual existence of a statute prior to such a
declare martial law: determination [of constitutionality], is an operative fact and may have consequences which cannot
always be erased by a new judicial declaration. The effect of the subsequent ruling as to the
x x x Congress may revoke such proclamation or suspension and the Court may review the invalidity may have to be considered in various aspects, - with respect to particular regulations,
sufficiency of the factual basis thereof. However, there is no such equivalent provision dealing individual and corporate, and particular conduct, private and official.
with the revocation or review of the President's action to call out the armed forces. The
distinction places the calling out power in a different category from the power to declare martial The orthodox view finds support in the well-settled doctrine that the Constitution is supreme and
law and the power to suspend the privilege of the writ of habeas corpus, otherwise, the framers of provides the measure for the validity of legislative or executive acts. Clearly then, neither the
the Constitution would have simply lumped together the three powers and provided for their legislative nor the executive branch, and for that matter much less, this Court, has power under
revocation and review without any qualification.[153] the Constitution to act contrary to its terms. Any attempted exercise of power in violation of its
In other words, the President may exercise the power to call out the Armed provisions is to that extent unwarranted and null.
Forces independently of the power to suspend the privilege of the writ of habeas corpus and to
declare martial law, although, of course, it may also be a prelude to a possible future exercise of The growing awareness of the role of the judiciary as the governmental organ which has the final
the latter powers, as in this case. say on whether or not a legislative or executive measure is valid leads to a more appreciative
attitude of the emerging concept that a declaration of nullity may have legal consequences which
Even so, the Court's review of the President's declaration of martial law and his calling out the the more orthodox view would deny. That for a period of time such a statute, treaty, executive
Armed Forces necessarily entails separate proceedings instituted for that particular purpose. order, or ordinance was in 'actual existence' appears to be indisputable. What is more
appropriate and logical then than to consider it as 'an operative fact?' (Emphasis supplied)[159]
As explained in Integrated Bar of the Philippines v. Zamora,[154] the President's exercise of his However, it must also be stressed that this "operative fact doctrine" is not a fool-proof shield that
power to call out the armed forces to prevent or suppress lawless violence, invasion or rebellion would repulse any challenge to acts performed during the effectivity of martial law or suspension
of the privilege of the writ of habeas corpus, purportedly in furtherance of quelling rebellion or existing at the time the declaration was made or past events. As to how far the past events should
invasion, and promotion of public safety, when evidence shows otherwise. be from the present depends on the President. Past events may be considered as justifications for
the declaration and/or suspension as long as these are connected or related to the current
VII. The Scope of the Power to Review. situation existing at the time of the declaration.

a) The scope of the power of review under the 1987 Constitution refers only to the determination of As to what facts must be stated in the proclamation and the written Report is up to the
the sufficiency of the factual basis of the declaration of martial law and suspension of the privilege President.[165] As Commander-in-Chief, he has sole discretion to determine what to include and
of habeas corpus. what not to include in the proclamation and the written Report taking into account the urgency of
the situation as well as national security. He cannot be forced to divulge intelligence reports and
To recall, the Court, in the case of In the Matter of the Petition for Habeas Corpus of confidential information that may prejudice the operations and the safety of the military.
Lansang,[160] which was decided under the 1935 Constitution,[161] held that it can inquire
into, within proper bounds, whether there has been adherence to or compliance with the Similarly, events that happened after the issuance of the proclamation, which are included in the
constitutionally-imposed limitations on the Presidential power to suspend the privilege of the writ written report, cannot be considered in determining the sufficiency of the factual basis of the
of habeas corpus.[162] "Lansanglimited the review function of the Court to a very prudentially declaration of martial law and/or the suspension of the privilege of the writ of habeas corpus since
narrow test of arbitrariness."[163] Fr. Bernas described the "proper bounds" in Lansang as follows: these happened after the President had already issued the proclamation. If at all, they may be
used only as tools, guides or reference in the Court's determination of the sufficiency of factual
What, however, are these 'proper bounds' on the power of the courts? The Court first gave the basis, but not as part or component of the portfolio of the factual basis itself.
general answer that its power was 'merely to check not to supplant the Executive, or to ascertain
merely whether he has gone beyond the constitutional limits of his jurisdiction, not to exercise the In determining the sufficiency of the factual basis of the declaration and/or the suspension, the
power vested in him or to determine the wisdom of his act. More specifically, the Court said that Court should look into the full complement or totality of the factual basis, and not piecemeal or
its power was not 'even comparable with its power over civil or criminal cases elevated thereto by individually. Neither should the Court expect absolute correctness of the facts stated in the
appeal . . . in which cases the appellate court has all the powers of the court of origin,' nor to its proclamation and in the written Report as the President could not be expected to verify the
power of quasi-judicial administrative decisions where the Court is limited to asking whether accuracy and veracity of all facts reported to him due to the urgency of the situation. To require
'there is some evidentiary basis' for the administrative finding. Instead, the Court accepted the precision in the President's appreciation of facts would unduly burden him and therefore impede
Solicitor General's suggestion that it 'go no further than to satisfy [itself] not that the President's the process of his decision-making. Such a requirement will practically necessitate the President to
decision is correct and that public safety was endangered by the rebellion and justified the be on the ground to confirm the correctness of the reports submitted to him within a period that
suspension of the writ, but that in suspending the writ, the President did not act arbitrarily.' [164] only the circumstances obtaining would be able to dictate. Such a scenario, of course, would not
Lansang, however, was decided under the 1935 Constitution. The 1987 Constitution, by providing only place the President in peril but would also defeat the very purpose of the grant of emergency
only for judicial review based on th determination of the sufficiency of the factual bases, has in fact powers upon him, that is, to borrow the words of Justice Antonio T. Carpio in Fortun, to
done away with the test of arbitrariness as provided in Lansang. "immediately put an end to the root cause of the emergency".[166] Possibly, by the time the
President is satisfied with the correctness of the facts in his possession, it would be too late in the
b) The "sufficiency of factual basis test". day as the invasion or rebellion could have already escalated to a level that is hard, if not
impossible, to curtail.
Similarly, under the doctrine of contemporaneous construction, the framers of the 1987
Constitution are presumed to know the prevailing jurisprudence at the time they were drafting the Besides, the framers of the 1987 Constitution considered intelligencb reports of military officers as
Constitution. Thus, the phrase "sufficiency of factual basis" in Section 18, Article VII of th credible evidence that the President cah appraise and to which he can anchor his judgment,[167] as
Constitution should be understood as the only test for judicial review of the President's power to appears to be the case here.
declare martial law and suspend the privilege of the writ of habeas corpus under Section 18,
Article VII of the Constitution. The Court does not need to satisfy itself that the President's decision At this point, it is wise to quote the pertinent portions of the Dissenting Opinion of Justice
is correct, rather it only needs to determine whether the President's decision had sufficient factual Presbitero J. Velasco Jr. in Fortun:
bases.
President Arroyo cannot be blamed for relying upon the information given to her by the Armed
We conclude, therefore, that Section 18, Article VII limits the scope of judicial review by the Forces of the Philippines and the Philippine National Police, considering that the matter of the
introduction of the "sufficiency of the factual basis" test. supposed armed uprising was within their realm of competence, and that a state of emergency has
also been declared in Central Mindanao to prevent lawless violence similar to the 'Maguindanao
As Commander-in-Chief, the President has the sole discretion to declare martial law and/or to massacre,' which may be an indication that there is a threat to the public safety warranting a
suspend the privilege of the writ of habeas corpus, subject to the revocation of Congress and the declaration of martial law or suspension of the writ.
review of this Court. Since the exercise of these powers is a judgment call of the President, the
determination of this Court as to whether there is sufficient factual basis for the exercise of such, Certainly, the President cannot be expected to risk being too late before declaring martial law or
must be based only on facts or information known by or available to the President at the time he suspending the writ of habeas corpus. The Constitution, as couched, does not require precision in
made the declaration or suspension, which facts or information are found in the proclamation as establishing the fact of rebellion. The President is called to act as public safety requires. [168]
well as the written Report submitted by him to Congress. These may be based on the situation
Corollary, as the President is expected to decide quickly on whether there is a need to proclaim Revised Penal Code, that presupposes an actual assemblage of men in an armed public uprising for
martial law even only on the basis of intelligencd reports, it is irrelevant, for purposes of the the purposes mentioned in Article 134 and by the means employed under Article 135. x x x [173]
Court's review, if subsequent events prove that the situation had not been accurately reported to Thus, rebellion as mentioned in the Constitution could only refer to rebellion as defined under
him. Article 134 of the RPC. To give it a different definition would not only create confusion but would
also give the President wide latitude of discretion, which may be abused - a situation that the
After all, the Court's review is confined to the sufficiency, not accuracy, of the information at hand Constitution seeks to prevent.[174]
during the declaration or suspension; subsequent events do not have any bearing insofar as the
Court's review is concerned. In any event, safeguards under Section 18, Article VII of the Article 134 of the RPC states:
Constitution are in place to cover such a situation, e.g., the martial law period is good only for 60
days; Congress may choose to revoke it even immediately after the proclamation is made; and, this Art. 134. Rebellion or insurrection; How committed. - The crime of rebellion or insurrection is
Court may investigate the factual background of the declaration.[169] committed by rising publicly and taking arms against the Government for the purpose of removing
from the allegiance to said Government or its laws, the territory of the Philippine Islands or any
Hence, the maxim falsus in uno, falsus in omnibus finds no application in this case. Falsities of part thereof, of any body of land, naval or other armed forces, depriving the Chief Executive or the
and/or inaccuracies in some of the facts stated in the proclamation and the written report are not Legislature, wholly or partially, of any of their powers or prerogatives.
enough reasons for the Court to invalidate the declaration and/or suspension as long as there are Thus, for rebellion to exist, the following elements must be present, to wit: "(1) there is a (a) public
other facts in the proclamation and the written Report that support the conclusion that there is an uprising and (b) taking arms against the Government; and (2) the purpose of the uprising or
actual invasion or rebellion and that public safety requires the declaration and/or suspension. movement is either (a) to remove from the allegiance to the Government or its laws: (i) the
territory of the Philippines or any part thereof; or (ii) any body of land, naval, or other armed
In sum, the Court's power to review is limited to the determination of whether the President in forces; or (b) to deprive the Chief Executive or Congress, wholly or partially, of any of their powers
declaring martial law and suspending the privilege of the writ of habeas corpus had sufficient and prerogatives."[175]
factual basis. Thus, our review would be limited to an examination on whether the President acted
within the bounds set by the Constitution, i.e., whether the facts in his possession prior to and at b) Probable cause is the allowable standard of proof for the President.
the time of the declaration or suspension are sufficient for him to declare martial law or suspend
the privilege of the writ of habeas corpus. In determining the existence of rebellion, the President only needs to convince himself that there
is probable cause or evidence showing that more likely than not a rebellion was committed or is
VIII. The parameters for determining the sufficiency of the factual basis for the declaration of being committed.[176] To require him to satisfy a higher standard of proof would restrict the
martial law and/or the suspension of the privilege of the writ of habeas corpus. exercise of his emergency powers. Along this line, Justice Carpio, in his Dissent in Fortun v.
President Macapagal-Arroyo, concluded that the President needs only to satisfy probable cause as
a) Actual invasion or rebellion, and public safety requirement. the standard of proof in determining the existence of either invasion or rebellion for purposes of
declaring martial law, and that probable cause is the most reasonable, most practical and most
Section 18, Article VII itself sets the parameters for determining the sufficiency of the factual basis expedient standard by which the President can fully ascertain the existence or non-existence of
for the declaration of martial law and/or the suspension of the privilege of the writ of habeas rebellion necessary for a declaration of martial law or suspension of the writ. This is because unlike
corpus, "namely (1) actual invasion or rebellion, and (2) public safety requires the exercise of such other standards of proof, which, in order to be met, would require much from the President and
power."[170] Without the concurrence of the two conditions, the President's declaration of martial therefore unduly restrain his exercise of emergency powers, the requirement of probable cause is
law and/or suspension of the privilege of the writ of habeas corpus must be struck down. much simpler. It merely necessitates an "average man [to weigh] the facts and circumstances
without resorting to the calibration of the rules of evidence of which he has no technical
As a general rule, a word used in a statute which has a technical or legal meaning, is construed to knowledge. He [merely] relies on common sense [and] x x x needs only to rest on evidence
have the same technical or legal meaning.[171] Since the Constitution did not define the term showing that, more likely than not, a crime has been committed x x x by the accused."[177]
"rebellion," it must be understood to have the same meaning as the crime of "rebellion" in the
Revised Penal Code (RPC).[172] To summarize, the parameters for determining the sufficiency of factual basis are as follows: 1)
actual rebellion or invasion; 2) public safety requires it; the first two requirements must concur;
During the July 29, 1986 deliberation of the Constitutional Commission of 1986, then and 3) there is probable cause for the President to believe that there is actual rebellion or invasion.
Commissioner Florenz D. Regalado alluded to actual rebellion as one defined under Article 134 of
the RPC: Having laid down the parameters for review, the Court shall now proceed to the core of the
controversy - whether Proclamation No. 216, Declaring a State of Martial Law and Suspending the
MR. DE LOS REYES. As I see it now, the Committee envisions actual rebellion and no longer Privilege of the Writ of Habeas Corpus in the whole of Mindanao, lacks sufficient factual basis.
imminent rebellion. Does the Committee mean that there should be actual shooting or actual
attack on the legislature or Malacaang, for example? Let us take for example a contemporary IX. There is sufficient factual basis for the declaration of martial law and the suspension of the
event - this Manila Hotel incident, everybody knows what happened. Would the Committee writ of habeas corpus.
consider that an actual act of rebellion?
At this juncture, it bears to emphasize that the purpose of judicial review is not the determination
MR. REGALADO. If we consider the definition of rebellion under Articles 134 and 135 of the of accuracy or veracity of the facts upon which the President anchored his declaration of martial
law or suspension of the privilege of the writ of habeas corpus; rather, only the sufficiency of the f) Capability of the Maute Group and other rebel groups to sow terror, and cause death and
factual basis as to convince the President that there is probable cause that rebellion exists. It must damage to property not only in Lanao del Sur but also in other parts of Mindanao;
also be reiterated that martial law is a matter of urgency and much leeway and flexibility should be
accorded the President. As such, he is not expected to completely validate all the information he and the Report[184] submitted to Congress:
received before declaring martial law or suspending the privilege of the writ of habeas corpus.
1. Zamboanga siege;[185]
We restate the elements of rebellion for reference:
2. Davao bombing;[186]
1. That there be (a) public uprising, and (b) taking up arms against the Government; and
3. Mamasapano carnage;[187]
2. That the purpose of the uprising or movement is either: (a) to remove from the allegiance to
said Government or its laws the territory of the Philippines or any part thereof, or any body of 4. Cotabato bombings;[188]
land, naval or other armed forces or (b) to deprive the Chief Executive or Congress, wholly or
partially, of any of their powers or prerogatives.[178] 5. Sultan Kudarat bombings;[189]
Petitioners concede that there is an armed public uprising in Marawi City.[179] However, they insist
that the armed hostilities do not constitute rebellion in the absence of the element of culpable 6. Sulu bombings;[190]
political purpose, i.e., the removal from the allegiance to the Philippine Government or its laws: (i)
the territory of the Philippines or any part thereof; or (ii) any body of land, naval, or other armed 7. Basilan bombings;[191]
forces; or (b) to deprive the Chief Executive or Congress, wholly or partially, of any of their powers
and prerogatives. 8. Attempt to capture Hapilon was confronted with armed resistance, by combined forces of ASG
and the Maute Group;[192]
The contention lacks merit.
9. Escalation of armed hostility against the government troops;[193]
a) Facts, events and information upon which the President anchored his decision to declare martial
law and suspend the privilege of the writ of habeas corpus. 10. Acts of violence directed not only against government authorities and establishments but
civilians as well;[194]
Since the President supposedly signed Proclamation No. 216 on May 23, 2017 at 10:00 PM,[180] the
Court will consider only those facts and/or events which were known to or have transpired on or 11. Takeover of major social, economic and political foundations which paralyzed Marawi City;[195]
before that time,
12. The object of the armed hostilities was to lay the groundwork for the establishment of a
consistent with the scope of judicial review. Thus, the following facts and/or events were deemed DAESH/ISIS wilayat or province;[196]
to have been considered by the President in issuing Proclamation No. 216, as plucked from and
extant in Proclamation No. 216 itself: 13. Maute Group has 263 active members, armed and combat ready;[197]

1. Proclamation No. 55 issued on September 4, 2016, declaring a state of national emergency on 14. Extensive networks linkages of the Maute Group with foreign and local armed groups;[198]
account of lawless violence in Mindanao;[181]
15. Adherence of the Maute Group to the ideals espoused by ISIS;[199]
2. Series of violent acts[182] committed by the Maute terrorist group including:
16. Publication of a video showing Maute Group's declaration of allegiance to ISIS;[200]
a) Attack on the military outpost in Butig, Lanao del Sur in February 2016, killing and wounding
several soldiers; 17. Foreign-based terrorist groups provide financial and logistical support to the Maute Group;[201]
b) Mass jailbreak in Marawi City in August 2016 of the arrested comrades of the Maute Group and
other detainees; 18. Events on May 23, 2017 in Marawi City, particularly:

3. On May 23, 2017:[183] a) at 2:00 PM, members and sympathizers of the Maute Group and ASG attacked various
government and privately-owned facilities;[202]
a) Takeover of a hospital in Marawi; b) at 4:00 PM, around fifty (50) armed criminals forcibly entered the Marawi City Jail; facilitated
b) Establishment of several checkpoints within Marawi; the escape of inmates; killed a member of PDEA; assaulted and disarmed on-duty personnel
c) Burning of certain government and private facilities; and/or locked them inside the cells; confiscated cellphones, personnel-issued firearms, and
d) Mounting casualties on the part of the government; vehicles;[203]
e) Hoisting the flag of ISIS in several areas; and c) by 4:30 PM, interruption of power supply; sporadic gunfights; city wide power outage by
evening;[204]
d) from 6:00 PM to 7:00 PM, Maute Group ambushed and burned the Marawi Police Station; throughout Marawi City; the decimation of a segment of the city population who resist; and the
commandeered a police car;[205] brazen display of DAESH flags constitute a clear, pronounced, and unmistakable intent to remove
e) BJMP personnel evacuated the Marawi City Jail and other affected areas;[206] Marawi City, and eventually the rest of Mindanao, from its allegiance to the Government."[225]
f) control over three bridges in Lanao del Sur, namely, Lilod, Bangulo, and Sauiaran, was taken by
the rebels;[207] 6) "There exists no doubt that lawless armed groups are attempting to deprive the President of his
g) road blockades and checkpoints set up by lawless armed groups at the Iligan-Marawi power, authority, and prerogatives within Marawi City as a precedent to spreading their control
junction;[208] over the entire Mindanao, in an attempt to undermine his control over executive departments,
h) burning of Dansalan College Foundation, Cathedral of Maria Auxiliadora, the nuns' quarters in bureaus, and offices in said area; defeat his mandate to ensure that all laws are faithfully executed;
the church, and the Shia Masjid Moncado Colony;[209] and remove his supervisory powers over local governments."[226]
i) taking of hostages from the church;[210]
j) killing of five faculty members of Dansalan College Foundation;[211] 7) "Law enforcement and other government agencies now face pronounced difficulty sending their
k) burning of Senator Ninoy Aquino College Foundation and Marawi Central Elementary Pilot reports to the Chief Executive due to the city-wide power outages. Personnel from the BJMP have
School;[212] been prevented from performing their functions. Through the attack and occupation of several
l) overrunning of Amai Pakpak Hospital;[213] hospitals, medical services in Marawi City have been adversely affected. The bridge and road
m) hoisting the ISIS flag in several areas;[214] blockades set up by the groups effectively deprive the government of its ability to deliver basic
n) attacking and burning of the Filipino-Libyan Friendship Hospital;[215] services to its citizens. Troop reinforcements have been hampered, preventing the government
o) ransacking of a branch of Landbank of the Philippines and commandeering an armored from restoring peace and order in the area. Movement by both civilians and government
vehicle;[216] personnel to and from the city is likewise hindered."[227]
p) reports regarding Maute Group's plan to execute Christians;[217]
q) preventing Maranaos from leaving their homes;[218] 8) "The taking up of arms by lawless armed groups in the area, with support being provided by
r) forcing young Muslims to join their group;[219] and foreign-based terrorists and illegal drug money, and their blatant acts of defiance which embolden
s) intelligence reports regarding the existence of strategic mass action of lawless armed groups in other armed groups in Mindanao, have resulted in the deterioration of public order and safety in
Marawi City, seizing public and private facilities, perpetrating killings of government personnel, Marawi City; they have likewise compromised the security of the entire Island of Mindanao."[228]
and committing armed uprising against and open defiance of the Government. [220]
9) "Considering the network and alliance-building activities among terrorist groups, local criminals,
b) The President's Conclusion and lawless armed men, the siege of Marawi City is a vital cog in attaining their long-standing goal:
absolute control over the entirety of Mindanao. These circumstances demand swift and decisive
After the assessment by the President of the aforementioned facts, he arrived at the following action to ensure the safety and security of the Filipino people and preserve our national
conclusions, as mentioned in Proclamation No. 216 and the Report: integrity."[229]

1) The Maute Group is "openly attempting to remove from the allegiance to the Philippine Thus, the President deduced from the facts available to him that there was an armed public
Government this part of Mindanao and deprive the Chief Executive of his powers and prerogatives uprising, the culpable purpose of which was to remove from the allegiance to the Philippine
to enforce the laws of the land and to maintain public order and safety in Mindanao, constituting Government a portion of its territory and to deprive the Chief Executive of any of his powers and
the crime of rebellion."[221] prerogatives, leading the President to believe that there was probable cause that the crime of
rebellion was and is being committed and that public safety requires the imposition of martial law
2) "[L]awless armed groups have taken up arms and committed public uprising against the duly and suspension of the privilege of the writ of habeas corpus.
constituted government and against the people of Mindanao, for the purpose of removing
Mindanao - starting with the City of Marawi, Lanao del Sur - from its allegiance to the Government A review of the aforesaid facts similarly leads the Court to conclude that the President, in issuing
and its laws and depriving the Chief Executive of his powers and prerogatives to enforce the laws Proclamation No. 216, had sufficient factual bases tending to show that actual rebellion exists. The
of the land and to maintain public order and safety in Mindanao, to the great damage, prejudice, President's conclusion, that there was an armed public uprising, the culpable purpose of which was
and detriment of the people therein and the nation as a whole."[222] the removal from the allegiance of the Philippine Government a portion of its territory and the
deprivation of the President from performing his powers and prerogatives, was reached after a
3) The May 23, 2017 events "put on public display the groups' clear intention to establish an tactical consideration of the facts. In fine, the President satisfactorily discharged his burden of
Islamic State and their capability to deprive the duly constituted authorities the President, proof.
foremost - of their powers and prerogatives."[223]
After all, what the President needs to satisfy is only the standard of probable cause for a valid
4) "These activities constitute not simply a display of force, but a clear attempt to establish the declaration of martial law and suspension of the privilege of the writ of habeas corpus. As Justice
groups' seat of power in Marawi City for their planned establishment of a DAESH wilayat or Carpio decreed in his Dissent in Fortun:
province covering the entire Mindanao."[224]
x x x [T]he Constitution does not compel the President to produce such amount of proof as to
5) "The cutting of vital lines for transportation and power; the recruitment of young Muslims to unduly burden and effectively incapacitate her from exercising such powers.
further expand their ranks and strengthen their force; the armed consolidation of their members
Definitely, the President need not gather proof beyond reasonable doubt, which is the standard of are false, inaccurate, simulated, and/or hyperbolic, does not persuade. As mentioned, the Court is
proof required for convicting an accused charged with a criminal offense. x x x not concerned about absolute correctness, accuracy, or precision of the facts because to do so
would unduly tie the hands of the President in responding to an urgent situation.
xxxx
Specifically, it alleges that the following facts are not true as shown by its counter-evidence:[231]
Proof beyond reasonable doubt is the highest quantum of evidence, and to require the President
to establish the existence of rebellion or invasion with such amount of proof before declaring FACTUAL STATEMENTS COUNTER-EVIDENCE
martial law or suspending the writ amounts to an excessive restriction on 'the President's power to Statements made by:
act as to practically tie her hands and disable her from effectively protecting the nation against (1) that the Maute group attacked Amai Pakpak (a) Dr. Amer Saber, Chief of the Hospital
threats to public safety.' Hospital and hoisted the DAESH flag there, (b) Health Secretary Paulyn Ubial;
among several locations. As of 0600H of 24 May (c) PNP Spokesperson Senior Supt. Dionardo
Neither clear and convincing evidence, which is employed in either criminal or civil cases, is 2017, members of the Maute Group were seen Carlos;
indispensable for a lawful declaration of martial law or suspension of the writ. This amount of guarding the entry gates of the Amai Pakpak (d) AFP Public Affairs Office Chief Co. Edgard
proof likewise unduly restrains the President in exercising her emergency powers, as it requires Hospital and that they held hostage the Arevalo; and
proof greater than preponderance of evidence although not beyond reasonable doubt. employees of the Hospital and took over the (e) Marawi City Mayor Majul Gandamra denying
PhilHealth office located thereat (Proclamation that the hospital was attacked by the Maute
Not even preponderance of evidence, which is the degree of proof necessary in civil cases, is No. 216 and Report); Group citing online news articles of Philstar,
demanded for a lawful declaration of martial law. Sunstar, Inquirer, and Bombo Radyo.[232]

xxxx

Weighing the superiority of the evidence on hand, from at least two opposing sides, before she
can act and impose martial law or suspend the writ unreasonably curtails the President's 2. that the Maute Group ambushed and burned Statements made by PNP Director General
emergency powers. the Marawi Police Station (Proclamation No. 216 Ronald dela Rosa and Marawi City Mayor Majul
and the Report); Gandamra in the online news reports of ABS-
Similarly, substantial evidence constitutes an unnecessary restriction on the President's use of her CBN News and CNN Philippines[233]denying that
emergency powers. Substantial evidence is the amount of proof required in administrative or the Maute group occupied the Marawi Police
quasi-judicial cases, or that amount of relevant evidence which a reasonable mind might accept as Station.
adequate to justify a conclusion.
3. that lawless armed groups likewise ransacked Statement made by the bank officials in the on-
I am of the view that probable cause of the existence of either invasion or rebellion suffices and the Landbank of the Philippines and line news article of Philstar[234] that the Marawi
satisfies the standard of proof for a valid declaration of martial law and suspension of the writ. commandeered one of its armored vehicles City branch was not ransacked but sustained
(Report); damages from the attacks.
Probable cause is the same amount of proof required for the filing of a criminal information by the
prosecutor and for the issuance of an arrest warrant by a judge. Probable cause has been defined
as a 'set of facts and circumstances as would lead a reasonably discreet and prudent man to
believe that the offense charged in the Information or any offense included therein has been Statements in the on-line news article
committed by the person sought to be arrested. 4. that the Marawi Central Elementary Pilot
of Philstar[235] made by the Marawi City Schools
School was burned (Proclamation No. 216 and
Division Assistant Superintendent Ana Alonto
In determining probable cause, the average man weighs the facts and circumstances without the Report);
denying that the school was burned and
resorting to the calibrations of the rules of evidence of which he has no technical knowledge. He Department of Education Assistant Secretary
relies on common sense. A finding of probable cause needs only to rest on evidence showing that, Tonisito Umali stating that they have not
more likely than not, a crime has been committed and that it was committed by the accused. received any report of damage.
Probable cause demands more than suspicion; it requires less than evidence that would justify
conviction.
5. that the Maute Group attacked various Statement in the on-line news article of
Probable cause, basically premised on common sense, is the most reasonable, most practical, and government facilities (Proclamation No. 216 and Inquirer[236] made by Marawi City Mayor Majul
most expedient standard by which the President can fully ascertain the existence or non-existence the Report). Gandamra stating that the ASG and the Maute
of rebellion, necessary for a declaration of martial law. x x x[230] Terror Groups have not taken over any
c) Inaccuracies, simulations, falsities, and hyperboles. government facility in Marawi City.

The allegation in the Lagman Petition that the facts stated in Proclamation No. 216 and the Report
However, the so-called counter-evidence were derived solely from unverified news articles on the Group were directed not only against government forces or establishments but likewise against
internet, with neither the authors nor the sources shown to have affirmed the contents thereof. It civilians and their properties.[242] In addition and in relation to the armed hostilities, bomb threats
was not even shown that efforts were made to secure such affirmation albeit the circumstances were issued;[243] road blockades and checkpoints were set up;[244] schools and churches were
proved futile. As the Court has consistently ruled, news articles are hearsay evidence, twice burned;[245] civilian hostages were taken and killed;[246] non-Muslims or Christians were
removed, and are thus without any probative value, unless offered for a purpose other than targeted;[247] young male Muslims were forced to join their group;[248] medical services and delivery
proving the truth of the matter asserted.[237] This pronouncement applies with equal force to the of basic services were hampered;[249] reinforcements of government troops and civilian movement
Cullamat Petition which likewise submitted online news articles[238] as basis for their claim of were hindered;[250] and the security of the entire Mindanao Island was compromised.[251]
insufficiency of factual basis.
These particular scenarios convinced the President that the atrocititts had already escalated to a
Again, it bears to reiterate that the maxim falsus in uno, falsus in omnibus finds no application in level that risked public safety and thus impelldd him to declare martial law and suspend the
these cases. As long as there are other facts in the proclamation and the written Report privilege of the writ of habeas corpus. In the last paragraph of his Report, the President declared:
indubitably showing the presenqe of an actual invasion or rebellion and that public safety requires
the declaration and/or suspension, the finding of sufficiency of factual basis, stands. While the government is presently conducting legitimate operations to address the on-going
rebellion, if not the seeds of invasion, public safety necessitates the continued implementation of
d) Ruling in Bedol v. Commission on Elections not applicable. martial law and the suspension of the privilege of the writ of habeas corpus in the whole of
Mindanao until such time that the rebellion is completely quelled.[252]
Petitioners, however, insist that in Bedol v. Commission on Elections,[239] news reports may be Based on the foregoing, we hold that the parameters for the declaration of martial law and
admitted on grounds of relevance, trustworthiness, and necessity. Petitioners' reliance on this case suspension of the privilege of the writ of habeas corpus have been properly and fully complied
is misplaced. The Court in Bedol made it clear that the doctrine of independent relevant with. Proclamation No. 216 has sufficient factual basis there being probable cause to believe that
statement, which is an exception to the hearsay rule, applies in cases "where only the fact that rebellion exists and that public safety requires the martial larv declaration and the suspension of
such statements were made is relevant, and the truth or falsity thereof is immaterial."[240] Here, the privilege of the writ of habeas corpus.
the question is not whether such statements were made by Saber, et al., but rather whether what
they said are true. Thus, contrary to the view of petitioners, the exception in Bedol finds no XI. Whole of Mindanao
application here.
a) The overriding and paramount concern of martial law is the protection of the security of the
e) There are other independent facts which support the finding that, more likely than not, nation and the good and safety of the public.
rebellion exists and that public safety requires it.
Considering the nation's and its people's traumatic experience of martial law under the Marcos
Moreover, the alleged false and/or inaccurate statements are just pieces and parcels of the regime, one would expect the framers of the 1987 Constitution to stop at nothing
Report; along with these alleged false data is an arsenal of other independent facts showing that from not resuscitating the law. Yet it would appear that the constitutional writers
more likely than not, actual rebellion exists, and public safety requires the declaration of martial entertained nodoubt about the necessity and practicality of such specie of extraordinary power
law or suspension of the privilege of the writ of habeas corpus. To be precise, the alleged false and thus, once again, bestowed on the Commander-in-Chief the power to declare martial law
and/or inaccurate statements are only five out of the several statements bulleted in the albeit in its diluted form.
President's Report. Notably, in the interpellation by Justice Francis H. Jardeleza during the second
day of the oral argument, petitioner Lagman admitted that he was not aware or that he had no Indeed, martial law and the suspension of the privilege of the writ of habeas corpus are necessary
personal knowledge of the other incidents cited.[241] As it thus stands, there is no question or for the protection of the security of the nation; suspension of the privilege of the writ of habeas
challenge with respect to the reliability of the other incidents, which by themselves are ample to corpus is "precautionary, and although it might [curtail] certain rights of individuals, [it] is for tlie
preclude the conclusion that the President's report is unreliable and that Proclamation No. 216 purpose of defending and protecting the security of the state or the entire country and our
was without sufficient factual basis. sovereign people".[253] Commissioner Ople referred to the suspension of the privilege of the writ
of habeas corpus as a "form of immobilization" or "as a means of immobilizing potential internal
Verily, there is no credence to petitioners' claim that the bases for the President's imposition of enemies" "especially in areas like Mindanao."[254]
martial law and suspension of the writ of habeas corpus were mostly inaccurate, simulated, false
and/or hyperbolic. Aside from protecting the security of the country, martial law also guarantees and promotes public
safety. It is worthy of mention that rebellion alone does not justify the declaration of martial law
X. Public safety requires the declaration of martial law and the suspension of the privilege of the or suspension of the privilege of the writ of habeas corpus; the public safety requirement must
writ of habeas corpus in the whole of Mindanao. likewise be present.

Invasion or rebellion alone may justify resort to the calling out power but definitely not the b) As Commander-in-Chief, the President receives vital, relevant, classified, and live information
declaration of martial law or suspension of the privilege of the writ of habeas corpus. For a which equip and assist him in making decisions.
declaration of martial law or suspension of the privilege of the writ of habeas corpus to be valid,
there must be a concurrence of actual rebellion or invasion and the public safety requirement. In In Parts IX and X, the Court laid down the arsenal of facts and events that formed the basis for
his Report, the President noted that the acts of violence perpetrated by the ASG and the Maute Proclamation No. 216. For the President, the totality of facts and events, more likely than not,
shows that actual rebellion exists and that public safety requires the declaration of martial law and a part thereof under martial law.
suspension of the privilege of the writ of habeas corpus. Otherwise stated, the President believes
that there is probable cause that actual rebellion exists and public safety warrants the issuance of This is both an acknowledgement and a recognition that it is the Executive Department,
Proclamation No. 216. In turn, the Court notes that the President, in arriving at such a conclusion, particularly the President as Commander-in-Chief, who is the repository of vital, classified, and live
relied on the facts and events included in the Report, which we find sufficient. information necessary for and relevant in calibrating the territorial application of martial law and
the suspension of the privilege of the writ of habeas corpus. It, too, is a concession that the
To be sure, the facts mentioned in the Proclamation and the Report are far from being exhaustive President has the tactical and military support, and thus has a more informed understanding of
or all-encompassing. At this juncture, it may not be amiss to state that as Commander-in-Chief, the what is happening on the ground. Thus, the Constitution imposed a limitation on the period of
President has possession of documents and information classified as "confidential", the contents application, which is 60 days, unless sooner nullified, revoked or extended, but not on the
of which cannot be included in the Proclamation or Report for reasons of national security. These territorial scope or area of coverage; it merely stated "the Philippines or any part thereof,"
documents may contain information detailing the position of government troops and rebels, stock depending on the assessment of the President.
of firearms or ammunitions, ground commands and operations, names of suspects and
sympathizers, etc. In fact, during the closed door session held by the Court, some information e) The Constitution has provided sufficient safeguards against possible abuses of Commander-in-
came to light, although not mentioned in the Proclamation or Report. But then again, the Chief's powers; further curtailment of Presidential powers should not only be discouraged but also
discretion whether to include the same in the Proclamation or Report is the judgment call of the avoided.
President. In fact, petitioners concede to this. During the oral argument, petitioner Lagman
admitted that "the assertion of facts [in the Proclamation and Report] is the call of the Considering the country's history, it is understandable that the resurgence of martial law would
President."[255] engender apprehensions among the citizenry. Even the Court as an institution cannot project a
stance of nonchalance. However, the importance of martial law in the context of our society
It is beyond cavil that the President can rely on intelligence reports and classified documents. "It is should outweigh one's prejudices and apprehensions against it. The significance of martial law
for the President as [C]ommander-in-[C]hief of the Armed Forces to appraise these [classified should not be undermined by unjustified fears and past experience. After all, martial law is critical
evidence or documents/] reports and be satisfied that the public safety demands the suspension of and crucial to the promotion of public safety, the preservation of the nation's sovereignty and
the writ."[256] Significantly, respect to these so-called classified documents is accorded even "when ultimately, the survival of our country. It is vital for the protection of the country not only against
[the] authors of or witnesses to these documents may not be revealed."[257] internal enemies but also against those enemies lurking from beyond our shores. As such, martial
law should not be cast aside, or its scope and potency limited and diluted, based on bias and
In fine, not only does the President have a wide array of information before him, he also has the unsubstantiated assumptions.
right, prerogative, and the means to access vital, relevant, and confidential data, concomitant with
his position as Commander-in-Chief of the Armed Forces. Conscious of these fears and apprehensions, the Constitution placed several safeguards which
effectively watered down the power to declate martial law. The 1987 Constitution "[clipped] the
c) The Court has no machinery or tool equal to that of the Commander-in-Chief to ably and properly powers of [the] Commander-in-Chief because of [the] experience with the previous
assess the ground conditions. regime."[261] Not only were the grounds limited to actual invasion or rebellion, but its duration was
likewise fixed at 60 days, unless sooner revoked, nullified, or extended; at the same time, it is
In contrast, the Court does not have the same resources available tp the President. However, this subject to the veto powers of the Court and Congress.
should not be considered as a constitutional lapse. On the contrary, this is in line with the function
of the Court, particularly in this instance, to determine the sufficiency of factual basis of Commissioner Monsod, who, incidentally, is a counsel for the Mohamad Petition, even exhorted
Proclamation No. 216. As thoroughly discussed in Part VIII, the determination by the Court of the his colleagues in the Constitutional Convention to look at martial law from a new perspective by
sufficiency of factual basis must be limited only to the facts and information mentioned in the elaborating on the sufficiency of the proposed safeguards:
Report and Proclamation. In fact, the Court, in David v. President Macapagal-Arroyo,[258] cautioned
not to "undertake an independent investigation beyond the pleadings." In this regard, "the Court MR. MONSOD. x x x
will have to rely on the fact-finding capabilities of the [E]xecutive [D]epartment;"[259] in turn, the
Executive Department will have to open its findings to the Court,[260] which it did during the closed Second, we have been given a spectre of non sequitur, that the mere declaration of martial law for
door sessioh last June 15, 2017. a fixed period not exceeding 60 days, which is subject to judicial review, is going to result in
numerous violations ofhuman rights, the predominance of the military forever and in untold
d) The 1987 Constitution grants to the President, as Commander-in-Chief the discretion to sufferings. Madam President, we are talking about invasion and rebellion. We may not have any
determine the territorial coverage or application of martial law or suspension of the privilege of the freedom to speak of after 60 days, if we put as a precondition the concurrence of Congress. That
writ of habeas corpus. might prevent the President from acting at that time in order to meet the problem. So I would like
to suggest that, perhaps, we should look at this in its proper perspective. We are only looking at a
Section 18, Article VII of the Constitution states that "[i]n case of invasion or rebellion, when the very specific case. We are only looking at a case of the first 60 days at its maximum. And we are
public safety requires it, [the President] may x x x suspend the privilege of writ of habeas corpus or looking at actual invasion and rebellion, and there are other safeguards in those cases.[262]
place the Philippines or any part thereof under martial law." Clearly, the Constitution grants to Even Bishop Bacani was convinced that the 1987 Constitution hrs enough safeguards against
the President the discretion to determine the territorial coverage of martial law and the presidential abuses and commission of human rights violations. In voting yes for the elimination of
suspension of the privilege of the writ of habeas corpus. He may put the entire Philippines or only
the requirement pf prior concurrence of Congress, Bishop Bacani stated, viz.: Philippine General Hospital (PGH) or the Manila Science High School (MSHS) could not be
discounted. There is no way of knowing that all participants in the rebellion went and stayed inside
BISHOP BACANI. Yes, just two sentences. The reason I vote yes is that despite my concern for the Court's compound.
human rights, I believe that a good President can also safeguard human rights and human lives as
well. And I do not want to unduly emasculate the powers of the President. x x x [263] Neither could it be validly argued that the armed contingent positioned in PGH or MSHS
Commissioner De los Reyes shared the same sentiment, to wit: is not engaged in rebellion because there is no publicity in their acts as, in fact, they were merely
lurking inside the compound of PGH and MSHS. However, it must be pointed out that for the crime
MR. DE LOS REYES. May I explain my vote, Madam President. of rebellion to be consummated, it is not required that all armed participants should congregate
in one place, in this case, the Court's compound, and publicly rise in arms against the government
x x x The power of the President to impose martial law is doubtless of a very high and delicate for the attainment of their culpable purpose. It suffices that a portionof the contingent gathered
nature. A free people are naturally jealous of the exercise of military power, and the power to and formed a mass or a crowd and engaged in an armyd public uprising against the government.
impose martial law is certainly felt to be one of no ordinary magnitude. But as presented by the Similarly, it cannot be validly concluded that the grounds on which the armed public uprising
Committee, there are many safeguards: 1) it is limited to 60 days; 2) Congress can revoke it; 3) the actually took place should be the measure of the extent, scope or range, of the actual rebellion.
Supreme Court can still review as to the sufficiency of factual basis; and 4) it does not suspend the This is logical since the other rebels positioned in PGH, MSHS, or elsewhere, whose participation
operation of the Constitution. To repeat what I have quoted when I interpellated Commissioner did not involve the publicity aspect of rebellion, may also be considered as engaging in the crime of
Monsod, it is said that the power to impose martial law is dangerous to liberty and may be rebellion.
abused. All powers may be abused if placed in unworthy hands. But it would be difficult, we
think, to point out any other hands in which this power will be more safe and at the same time Proceeding from the same illustration, suppose we say that the President, after finding probable
equally effectual. When citizens of the State are in arms against each other and the constituted cause that there exists actual rebellion that public safety requires it, declares martial law and
authorities are unable to execute the laws, the action of the President must be prompt or it is of suspends the writ of habeas corpus in the whole of Metro Manila, could we then say that the
little value. x x x[264] (Emphasis supplied) territorial coverage of the proclamation is too expansive?
At this juncture, it bears to stress that it was the collective sentiment of the framers of the 1987
Constitution that sufficient safeguards against possible misuse and abuse by the Commander-in- To answer this question, we revert back to the premise that the discretion to determine the
Chief of his extraordinary powers are already in place and that no further emasculation of the territorial scope of martial law lies with the President. The Constitution grants him the prerogative
presidential powers is called for in the guise of additional safeguards. The Constitution recognizes whether to put the entire Philippines or any part thereof under martial law. There is no
that any further curtailment, encumbrance, or emasculation of the presidential powers would not constitutional edict that martial law should be confined only in the particular place where the
generate any good among the three co-equal branches, and to the country and its citizens as a armed public uprising actually transpired. This is not only practical but also logical. Martial law is
whole. Thus: an urgent measure since at stake is the nation's territorial sovereignty and survival. As such, the
President has to respond quickly. After the rebellion in the Court's compound, he need not wait for
MR. OPLE. The reason for my concern, Madam President, is that when we put all of these another rebellion to be mounted in Quezon City before he could impose martial law thereat. If
encumbrances on the President and Commander-in-Chief during an actual invasion or rebellion, that is the case, then the President would have to wait until every remote corner in the country is
given an intractable Congress that may be dominated by opposition parties, we may be actually infested with rebels before he could declare martial law in the entire Philippines. For sure, this is
impelling the President to use the sword of Alexander to cut the Gordian knot by just declaring a not the scenario envisioned by the Constitution.
revolutionary government that sets him free to deal with the invasion or the insurrection. x x
x[265] (Emphasis supplied) Going back to the illustration above, although the President is not required to impose martial law
f) Rebellion and public safety; nature, scope, and range. only within the Court's compound because it is where the armed public uprising actually
transpired, he may do so if he sees fit. At the same time, however, he is not precluded from
It has been said that the "gravamen of the crime of rebellion is an armed public uprising against expanding the coverage of martial law beyond the Court's compound. After all, rebellion is not
the govemment;"[266] and that by nature, "rebellion is x x x a crime of masses or multitudes, confined within predetermined bounds.
involving crowd action, that cannot be confined a priori, within predetermined bounds."[267] We
understand this to mean that the precise extent or range of the rebellion could not be measured Public safety, which is another component element for the declaration of martial law, "involves
by exact metes and bounds. the prevention of and protection from events that could endanger the safety of the general public
from significant danger, injury/harm, or damage, such as crimes or disasters."[268] Public safety is
To illustrate: A contingent armed with high-powered firearms publicly assembled in Padre Faura, an abstract term; it does not take any physical form. Plainly, its range, extent or scope could not be
Ennita, Manila where the Court's compound is situated. They overpowered the guards, entered physically measured by metes and bounds.
the Court's premises, and hoisted the ISIS flag. Their motive was political, i.e., they want to remove
from the allegiance to the Philippine government a part of the territory of the Philippines, Perhaps another reason why the territorial scope of martial law should not necessarily be limited
particularly the Court's compound and establish it as an ISIS-territory. to the particular vicinity where the armed public uprising actually transpired, is because of the
unique characteristic of rebellion as a crime. "The crime of rebellion consists of manyacts. It is a
Based on the foregoing illustration, and vis-a-vis the nature of the crime of rebellion, could we vast movement of men and a complex net of intrigues and plots. Acts committed in furtherance of
validly say that the rebellion is confined only within the Court's compound? Definitely not. The rebellion[,] though crimes in themselves[,] are deemed absorbed in one single crime of
possibility that there are other rebels positioned in the nearby buildings or compound of the rebellion."[269] Rebellion absorbs "other acts committed in its pursuance".[270] Direct
assault,[271] murder,[272] homicide,[273] arson,[274] robbery,[275] and kidnapping,[276] just to name a their present endeavor in Mindanao. Until now the Court is in a quandary and can only speculate
few, are absorbed in the crime of rebellion if committed in furtherance of rebelliop; "[i]t cannot be whether the 60-day lifespan of Proclamation No. 216 could outlive the present hostilities in
made a basis of a separate charge."[277] Jurisprudence also teaches that not only common crimes Mindanao. It is on this score that the Court should give the President sufficient leeway to address
may be absorbed in rebellion but also "offenses under special laws [such as Presidential Decree the peace and order problem in Mindanao.
No. 1829][278] which are perpetrated in furtherance of the political offense".[279] "All crimes,
whether punishable under a special law or general law, which are mere components or Thus, considering the current situation, it will not serve any purpose if the President is goaded into
ingredients, or committed in furtherance thereof, become absorbed in the crime of rebellion and using "the sword of Alexander to cut the Gordian knot"[282] by attempting to impose another
cannot be isolated and charged as separate crimes in themselves."[280] encumbrance; after all, "the declaration of martial law or the suspension of the privilege of the
writ of habeas corpus is essentially an executive act."[283]
Thus, by the theory of absorption, the crime of murder committed n Makati City, if committed in
furtherance of the crime of rebellion being hypothetically staged in Padre Faura, Ermita, Manila, is Some sectors, impelled perhaps by feelings of patriotism, may wish to subdue, rein in, or give the
stripped of its common complexion and is absorbed in the crime of rebellion. This all the more President a nudge, so to speak, as some sort of a reminder of the nation's experience under the
makes it difficult to confine the application of martial law only to the place where the armed public Marcos-styled martial law. However, it is not fair to judge President Duterte based on the ills some
uprising is actually taking place. In the illustration above, Padre Faura could only be the nerve of us may have experienced during the Marcos-martial law era. At this point, the Court quotes the
center of the rebellion but at the same time rebellion is also happening in Makati City. insightful discourse of Commissioner Ople:

In fine, it is difficult, if not impossible, to fix the territorial scope of martial law in direct MR. OPLE. x x x
proportion to the "range" of actual rebellion and public safety simply because rebellion and public
safety have no fixed physiqal dimensions. Their transitory and abstract nature defies precise Madam President, there is a tendency to equate patriotism with rendering the executive branch of
measurements; hence, the determination of the territorial scope of mart al law could only be the government impotent, as though by reducing drastically the powers of the executive, we are
drawn from arbitrary, not fixed, variables. The Constitution must have considered these limitations rendering a service to human welfare. I think it is also important to understand that the
when it granted tte President wide leeway and flexibility in determining the territorial scope of extraordinary measures contemplated in the Article on the Executive pertain to a practical state of
martial law. war existing in this country when national security will become a common bond of patriotism of all
Filipinos, especially if it is an actual invasion or an actual rebellion, and the President may have to
Moreover, the President's duty to maintain peace and public safety is not limited only to the place be given a minimum flexibility to cope with such unprecedented threats to the survival of a nation.
where there is actual rebellion; it extends to other areas where the present hostilities are in I think the Commission has done so but at the same time has not, in any manner, shunned the task
danger of spilling over. It is not intended merely to prevent the escape of lawless elements from of putting these powers under a whole system of checks and balances, including the possible
Marawi City, but also to avoid enemy reinforcements and to cut their supply lines coming from revocation at any time of a proclamation of martial law by the Congress, and in any case a definite
different parts of Mindanao. Thus, limiting the proclamation and/or suspension to the place where determination of these extraordinary powers, subject only to another extension to be determined
there is actual rebellion would not only defeat the purpose of declaring martial law, it will make by Congress in the event that it is necessary to do so because the emergency persists.
the exercise thereof ineffective and useless.
So, I think this Article on the Executive for which I voted is completely responsible; it is attuned
g) The Court must stay within the confines of its power. to the freedom and the rights of the citizenry. It does not render the presidency impotent and, at
the same time, it allows for a vigorous representation of the people through their Congress
The Court can only act within the confines of its power. For the Court to overreach is to infringe when an emergency measure is in force and effect.[284]
upon another's territory. Clearly, the power to determine the scope of territorial application h) Several local armed groups have formed linkages aimed at committing rebellion and acts in
belongs to the President. "The Court cannot indulge in judicial legislation without violating the furtherance thereof in the whole of Mindanao.
principle of separation of powers, and, hence, undermining the foundation of our republican
system."[281] With a predominantly Muslim population, Marawi City is "the only Islamic City of the
South."[285] On April 15, 1980, it was conferred the official title of "Islamic City of Marawi."[286] The
To reiterate, the Court is not equipped with the competence and logistical machinery to determine city's first name, "Dansalan," "was derived from the word 'dansal', meaning a destination point or
the strategical value of other places in the military's efforts to quell the rebellion and restore rendezvous. Literally, it also means arrival or coming."[287] Marawi lies in the heart of Mindanao. In
peace. It would be engaging in an act of adventurism if it dares to embark on a mission of fact, the Kilometer Zero marker in Mindanao is found in Marawi City thereby making Marawi City
deciphering the territorial metes and bounds of martial law. To be blunt about it, hours after the the point of reference of all roads in Mindanao.
proclamation of martial law none of the members of this Court could have divined that more than
ten thousand souls would be forced to evacuate to Iligan and Cagayan de Oro and that the military Thus, there is reasonable basis to believe that Marawi is only the staging point of the rebellion,
would have to secure those places also; none of us could have predicted that Cayamora Maute both for symbolic and strategic reasons. , Marawi may not be the target but the whole of
would be arrested in Davao City or that his wife Ominta Romato Maute would be apprehended in Mindanao. As mentioned in the Report, "[l]awless armed groups have historically used provinces
Masiu, Lanao del Sur; and, none of us had an inkling that the Bangsamoro Islamic Freedom adjoining Marawi City as escape routes, supply lines, and backdoor passages;"[288] there is also the
Fighters (BIFF) would launch an attack in Cotabato City. The Court has no military background and plan to establish a wilayatin Mindanao by staging the siege of Marawi. The report that prior to
technical expertise to predict that. In the same manner, the Court lacks the technical capability to May 23, 2017, Abdullah Maute had already dispatched some of his men to various places in
determine which part of Mindanao would best serve as forward operating base of the military in Mindanao, such as Marawi, Iligan, and Cagayan de Oro for bombing operations, carnapping, and
the murder of military and police personnel,[289] must also be considered. Indeed, there is some In determining what crime was committed, we have to look into the main objective of the
semblance of truth to the contention that Marawi is only the start, and Mindanao the end. malefactors. If it is political, such as for the purpose of severing the allegiance of Mindanao to the
Philippine Government to establish a wilayat therein, the crime is rebellion. If, on the other hand,
Other events also show that the atrocities were not concentrated in Marawi City. Consider these: the primary objective is to sow and create a condition of widespread and extraordinary fear and
panic among the populace in order to coerce the government to give in to an unlawful demand,
a. On January 13, 2017, an improvised explosive device (IED) exploded in Barangay Campo the crime is terrorism. Here, we have already explained and ruled that the President did not err in
Uno, Lamita City, Basilan. A civilian was killed while another was wounded.[290] believing that what is going on in Marawi City is one contemplated under the crime of rebellion.
b. On January 19, 2017, the ASG kidnapped three Indonesians near Bakungan Island,
Taganak, Tawi-Tawi.[291] In any case, even assuming that the insurgency in Marawi City can also be characterized as
c. On January 29, 2017, the ASG detonated an IED in Barangay Danapah, Albarka, Basilan terrorism, the same will not in any manner affect Proclamation No. 216. Section 2 of Republic Act
resulting in the death of two children and the wounding of three others.[292] (RA) No. 9372, otherwise known as the Human Security Act of 2007 expressly provides that
d. From March to May 2017, there were eleven (11) separate instances of IED explosions "[n]othing in this Act shall be interpreted as a curtailment, restriction or diminution of
by the BIFF in Mindanao. These resulted in the death and wounding of several constitutionally recognized powers of the executive branch of the government." Thus, as long as
personalities.[293] the President complies with all the requirements of Section 18, Article VII, the existence of
e. On February 26, 2017, the ASG beheaded its kidnap victim, Juergen Kantner in Sulu.[294] terrorism cannot prevent him from exercising his extraordinary power of proclaiming martial law
f. On April 11, 2017, the ASG infiltrated Inabaga, Bohol resulting in firefights between or suspending the privilege of the writ of habeas corpus. After all, the extraordinary powers of the
rebels and government troops.[295] President are bestowed on him by the Constitution. No act of Congress can, therefore, curtail or
g. On April 13, 2017, the ASG beheaded Filipino kidnap victim Noel Besconde.[296] diminish such powers.
h. On April 20, 2017, the ASG kidnapped SSg. Anni Siraji artd beheaded him three days
later.[297] Besides, there is nothing in Art. 134 of the RPC and RA 9372 which states that rebellion and
terrorism are mutuality exclusive of each other or that they cannot co-exist together. RA 9372
does not expressly or impliedly repeal Art. 134 of the RPC. And while rebellion is one of the
There were also intelligence reports from the military about offensives committed by the ASG and predicate crimes of terrorism, one cannot absorb the other as they have different elements. [300]
other local rebel groups. All these suggest th t the rebellion in Marawi has already spilled over to
other parts of Mindanao. Verily, the Court upholds the validity of the declaration of martial law and suspension of the
privilege of the writ of habeas corpus in the entire Mindanao region.
Moreover, considering the widespread atrocities in Mindanao and tbe linkages established among
rebel groups, the armed uprising that was initially staged in Marawi cannot be justified as confined At the end of the day, however ardently and passionately we may believe in the validity or
only to Marawi. The Court therefore will not simply disregard the events that happened during the correctness of the varied and contentious causes or principles that we espouse, advocate or
Davao City bombing, the Mamasapano massacre, the Zamboanga City siege, and the countless champion, let us not forget that at this point in time we, the Filipino people, are confronted with a
bombings in Cotabato, Sultan Kudarat, Sulu, and Basilan, among others.[298] The Court cannot crisis of such magnitude and proportion that we all need to summon the spirit of unity and act as
simply take the battle of Marawi in isolation. As a crime without predetermined bounds, the one undivided nation, if we are to overcome and prevail in the struggle at hand.
President has reasonable basis to believe that the declaration of martial law, as well as the
suspension of the privilege of the writ of habeas corpus in the whole of Mindanao, is most Let us face up to the fact that the siege in Marawi City has entered the second month and only God
necessary, effective, and called for by the circumstances. or Allah knows when it would end. Let us take notice of the fact that the casualties of the war are
mounting. To date, 418 have died. Out of that were 303 Maute rebels as against 71 government
i) Terrorism neither negates nor absorbs rebellion. troops and 44 civilians.

It is also of judicial notice that the insurgency in Mindanao has be n ongoing for decades. While Can we not sheathe our swords and pause for a while to bury our dead, including our differences
some groups have sought legal and peaceful means, others have resorted to violent extremism and prejudices?
and terrorism. Rebellion may be subsumed under the crime of terrorism, which has a broader
scope covering a wide range of predicate crimes. In fact, rebellion is only one of the various means WHEREFORE, the Court FINDS sufficient factual bases for the issuance of Proclamation No. 216
by which terrorism can be committed.[299] However, while the scope of terrorism may be and DECLARES it as CONSTITUTIONAL. Accordingly, the consolidated Petitions are
comprehensive, its purpose is distinct and well-defined. The objective of a "terrorist" is to sow and hereby DISMISSED.
create a condition of widespread fear among the populace in order to coerce the government to
give in to an unlawful demand. This condition of widespread fear is traditionally achieved through SO ORDERED.
bombing, kidnapping, mass killing, and beheading, among others. In contrast, the purpose of
rebellion, as previously discussed, is political, i.e., (a) to remove from the allegiance to the
Philippine Government or its laws: (i) the territory of the Philippines or any part thereof; (ii) any
body of land, naval, or armed forces; or (b) to deprive the Chief Executive or Congress, wholly or
partially, of any of their powers and prerogatives.
[ G.R. No. 231671, July 25, 2017]
On May 23, 2017, President Duterte issued Proclamation No. 216, declaring a state of martial law
and suspending the privilege of the writ of habeas corpus in the Mindanao group of islands on the
ALEXANDER A. PADILLA, RENE A.V. SAGUISAG, CHRISTIAN S. MONSOD, LORETTA ANN P. ROSALES,
grounds of rebellion and necessity of public safety pursuant to Article VII, Section 18 of the 1987
RENE B. GOROSPE, AND SENATOR LEILA M. DE LIMA, PETITIONERS, VS. CONGRESS OF THE
Constitution.
PHILIPPINES, CONSISTING OF THE SENATE OF THE PHILIPPINES, AS REPRESENTED BY SENATE
PRESIDENT AQUILINO "KOKO" PIMENTEL III, AND THE HOUSE OF REPRESENTATIVES, AS
Within forty-eight (48) hours after the proclamation, or on May 25, 2017, and while the Congress
REPRESENTED BY HOUSE SPEAKER PANTALEON D. ALVAREZ, RESPONDENTS.
was in session, President Duterte transmitted his "Report relative to Proclamation No. 216 dated
23 May 2017" (Report) to the Senate, through Senate President Pimentel, and the House of
[G.R. No. 231694]
Representatives, through House Speaker Pantaleon D. Alvarez (House Speaker Alvarez).
FORMER SEN. WIGBERTO E. TAADA, BISHOP EMERITUS DEOGRACIAS S. IIGUEZ, BISHOP
According to President Duterte's Proclamation No. 216 and his Report to the Congress, the
BRODERICK PABILLO, BISHOP ANTONIO R. TOBIAS, MO. ADELAIDA YGRUBAY, SHAMAH BULANGIS
declaration of a state of martial law and the suspension of the privilege of the writ of habeas
AND CASSANDRA D. DELURIA, PETITIONERS, VS. CONGRESS OF THE PHILIPPINES, CONSISTING OF
corpus in the whole of Mindanao ensued from the series of armed attacks, violent acts, and
THE SENATE AND THE HOUSE OF REPRESENTATIVES, AQUILINO "KOKO" PIMENTEL III, PRESIDENT,
atrocities directed against civilians and government authorities, institutions, and establishments
SENATE OF THE PHILIPPINES, AND PANTALEON D. ALVAREZ, SPEAKER, HOUSE OF THE
perpetrated by the Abu Sayyaf and Maute terrorist groups, in complicity with other local and
REPRESENTATIVES, RESPONDENTS.
foreign armed affiliates, who have pledged allegiance to the Islamic State of Iraq and Syria (ISIS), to
sow lawless violence, terror, and political disorder over the said region for the ultimate purpose of
LEONARDO-DE CASTRO, J.: establishing a DAESH wilayah or Islamic Province in Mindanao.
These consolidated petitions under consideration essentially assail the failure and/or refusal of Representatives from the Executive Department, the military, and other security officials of the
respondent Congress of the Philippines (the Congress), composed of the Senate and the House of government were thereafter invited, on separate occasions, by the Senate and the House of
Representatives, to convene in joint session and therein deliberate on Proclamation No. 216 issued Representatives for a conference briefing regarding the circumstances, details, and updates
on May 23, 2017 by President Rodrigo Roa Duterte (President Duterte). Through Proclamation No. surrounding the President's proclamation and report.
216, President Duterte declared a state of martial law and suspended the privilege of the writ
of habeas corpus in the whole of Mindanao for a period not exceeding sixty (60) days effective On May 29, 2017, the briefing before the Senate was conducted, which lasted for about four (4)
from the date of the proclamation's issuance. hours, by Secretary of National Defense Delfin N. Lorenza (Secretary Lorenzana), National Security
Adviser and Director General of the National Security Council Hermogenes C. Esperon, Jr.
In the Petition for Mandamus of Alexander A. Padilla (Padilla), Rene A.V. Saguisag (Saguisag), (Secretary Esperon), and Chief of Staff of the Armed Forces of the Philippines (AFP) General
Christian S. Monsod (Monsod), Loretta Ann P. Rosales (Rosales), Rene B. Gorospe (Gorospe), and Eduardo M. Ao (General Ao). The following day, May 30, 2017, the Senate deliberated on these
Senator Leila M. De Lima (Senator De Lima), filed on June 6, 2017 and docketed as G.R. No. 231671 proposed resolutions: (a) Proposed Senate (P.S.) Resolution No. 388, [3]which expressed support for
(the Padilla Petition), petitioners seek a ruling from the Court directing the Congress to convene in President Duterte's Proclamation No. 216; and (b) P.S. Resolution No. 390, [4] which called for the
joint session to deliberate on Presidential Proclamation No. 216, and to vote thereon.[1] convening in joint session of the Senate and the House of Representatives to deliberate on
President Duterte's Proclamation No. 216.
In the Petition for Certiorari and Mandamus of former Senator Wigberto E. Taada (Taada),
Bishop Emeritus Deogracias Iiguez (Bishop Iiguez), Bishop Broderick Pabillo (Bishop Pabillo), P.S. Resolution No. 388 was approved, after receiving seventeen (17) affirmative votes as against
Bishop Antonio Tobias (Bishop Tobias), Mo. Adelaida Ygrubay (Mo. Ygrubay), Shamah Bulangis five (5) negative votes, and was adopted as Senate Resolution No. 49[5] entitled "Resolution
(Bulangis), and Cassandra D. Deluria (Deluria), filed on June 7, 2017 and docketed as G.R. No. Expressing the Sense of the Senate Not to Revoke, at this Time, Proclamation No. 216, Series of
231694 (the Taada Petition), petitioners entreat the Court to: (a) declare the refusal of the 2017, Entitled 'Declaring a State of Martial Law and Suspending the Privilege of the Writ of Habeas
Congress to convene in joint session for the purpose of considering Proclamation No. 216 to be in Corpus in the Whole of Mindanao.'"[6]
grave abuse of discretion amounting to a lack or excess of jurisdiction; and (b) issue a writ
of mandamus directing the Congress to convene in joint session for the aforementioned P.S. Resolution No. 390, on the other hand, garnered only nine (9) votes from the senators who
purpose.[2] were in favor of it as opposed to twelve (12) votes from the senators who were against its
approval and adoption.[7]
Respondent Congress, represented by the Office of the Solicitor General (OSG), filed
its Consolidated Comment on June 27, 2017. Respondents Senate of the Philippines and Senate On May 31, 2017, the House of Representatives, having previously constituted itself as a
President Aquilino "Koko" Pimentel III (Senate President Pimentel), through the Office of the Committee of the Whole House,[8] was briefed by Executive Secretary Salvador C. Medialdea
Senate Legal Counsel, separately filed their Consolidated Comment (Ex Abudanti Cautela) on June (Executive Secretary Medialdea), Secretary Lorenzana, and other security officials for about six (6)
29, 2017. hours. After the closed-door briefing, the House of Representatives resumed its regular meeting
and deliberated on House Resolution No. 1050 entitled "Resolution Expressing the Full Support of
the House of Representatives to President Rodrigo Duterte as it Finds No Reason to Revoke
ANTECEDENT FACTS Proclamation No. 216, Entitled 'Declaring a State of Martial Law and Suspending the Privilege of
the Writ of Habeas Corpus in the Whole of Mindanao.'"[9] The House of Representatives proceeded
to divide its members on the matter of approving said resolution through viva voce voting. The Petitioners claim that there is an actual case or controversy in this instance and that their case is
result shows that the members who were in favor of passing the subject resolution secured the ripe for adjudication. According to petitioners, the resolutions separately passed by the Senate and
majority vote.[10] the House of Representatives, which express support as well as the intent not to revoke President
Duterte's Proclamation No. 216, injure their rights "to a proper [and] mandatory legislative review
The House of Representatives also purportedly discussed the proposal calling for a joint session of of the declaration of martial law" and that the continuing failure of the Congress to convene in
the Congress to deliberate and vote on President Duterte's Proclamation No. 216. After the joint session similarly causes a continuing injury to their rights.[13]
debates, however, the proposal was rejected.[11]
Petitioners also allege that, as citizens and taxpayers, they all have locus standi in their "assertion
These series of events led to the filing of the present consolidated petitions. of a public right" which they have been deprived of when the Congress refused and/or failed to
convene in joint session to deliberate on President Duterte's Proclamation No. 216. Senator De
Lima adds that she, together with the other senators who voted in favor of the resolution to
THE PARTIES' ARGUMENTS convene the Congress jointly, were even effectively denied the opportunity to perform their
constitutionally-mandated duty, under Article VII, Section 18 of the Constitution, to deliberate on
The Padilla Petition the said proclamation of the President in a joint session of the Congress.[14]

Petitioners in G.R. No. 231671 raise the question of "[w]hether Congress is required to convene in On the propriety of resorting to the remedy of mandamus, petitioners posit that "the duty of
joint session, deliberate, and vote jointly under Article VII, [Section] 18 of the Constitution" and Congress to convene in joint session upon the proclamation of martial law or the suspension of the
submit the following arguments in support of their petition: privilege of the writ of habeas corpus does not require the exercise of discretion." Such mandate
upon the Congress is allegedly a purely ministerial act which can be compelled through a writ
[I] THE PETITION SATISFIES THE REQUISITES FOR THE EXERCISE OF THE HONORABLE COURT'S of mandamus.[15]
POWER OF JUDICIAL REVIEW.
As for the substantive issue, it is the primary contention of petitioners that a plain reading of
[i] THERE IS AN ACTUAL CASE OR CONTROVERSY. Article VII, Section 18 of the Constitution shows that the Congress is required to convene in joint
session to review Proclamation No. 216 and vote as a single deliberative body. The performance of
PETITIONERS, AS PART OF THE PUBLIC AND AS TAXPAYERS, POSSESS LEGAL STANDING TO FILE the constitutional obligation is allegedly mandatory, not discretionary.[16]
[ii]
THIS PETITION.
According to petitioners, the discretionary nature of the phrase "may revoke such proclamation or
PETITIONER (DE LIMA], AS MEMBER OF CONGRESS, HAS LEGAL STANDING TO FILE THIS suspension" under Article VII, Section 18 of the Constitution allegedly pertain to the power of the
[iii]
PETITION. Congress to revoke but not to its obligation to jointly convene and vote which, they stress, is
mandatory. To require the Congress to convene only when it exercises the power to revoke is
[iv] THE CASE AND THE ISSUE INVOLVED ARE RIPE FOR JUDICIAL DETERMINATION. purportedly absurd since the Congress, without convening in joint session, cannot know
beforehand whether a majority vote in fact exists to effect a revocation.[17]
[II] THE PLAIN TEXT OF THE CONSTITUTION, SUPPORTED BY THE EXPRESS INTENT OF THE
FRAMERS, AND CONFIRMED BY THE SUPREME COURT, REQUIRES THAT CONGRESS CONVENE IN Petitioners claim that in Fortun v. Macapagal-Arroyo,[18] this Court described the "duty" of the
JOINT SESSION TO DELIBERATE AND VOTE AS A SINGLE DELIBERATIVE BODY. Congress to convene in joint session as "automatic." The convening of the Congress in joint session
when former President Gloria Macapagal-Arroyo (President Macapagal-Arroyo) declared martial
THE PLAIN TEXT OF THE CONSTITUTION REQUIRES THAT CONGRESS CONVENE IN JOINT law and suspended the privilege of the writ of habeas corpus in Maguindanao was also a legislative
[i]
SESSION. precedent where the Congress clearly recognized its duty to convene in joint session.[19]

THE EXPRESS INTENT OF THE FRAMERS IS FOR CONGRESS TO CONVENE IN JOINT SESSION TO The mandate upon the Congress to convene jointly is allegedly intended by the 1986
[ii]
DELIBERATE AND VOTE AS A SINGLE DELIBERATIVE BODY. Constitutional Commission (ConCom) to serve as a protection against potential abuses in the
exercise of the President's power to declare martial law and suspend the privilege of the writ
THE SUPREME COURT CONFIRMED IN FORTUN v. GMA THAT CONGRESS HAS THE of habeas corpus. It is "a mechanism purposely designed by the Constitution to compel Congress
[iii]
"AUTOMATIC DUTY" TO CONVENE IN JOINT SESSION. to review the propriety of the President's action x x x [and] meant to contain martial law powers
within a democratic framework for the preservation of democracy, prevention of abuses, and
LEGISLATIVE PRECEDENT ALSO RECOGNIZES CONGRESS' DUTY TO CONVENE IN JOINT protection of the people."[20]
[iv]
SESSION.
The Taada Petition
[III] THE REQUIREMENT TO ACT AS A SINGLE DELIBERATIVE BODY UNDER ARTICLE VII, [SECTION] 18
OF THE CONSTITUTION IS A MANDATORY, MINISTERIAL CONSTITUTIONAL DUTY OF CONGRESS, The petitioners in G.R. No. 231694 chiefly opine that:
WHICH CAN BE COMPELLED BY MANDAMUS.[12]
I. A PLAIN READING OF THE 1987 CONSTITUTION LEADS TO THE INDUBITABLE the position of their legislators with respect to this matter of the highest national interest."[26]
CONCLUSION THAT A JOINT SESSION OF CONGRESS TO REVIEW A DECLARATION OF
MARTIAL LAW BY THE PRESIDENT IS MANDATORY. Petitioners add that a public, transparent, and deliberative process is purportedly necessary to
allay the people's fears against "executive overreach." This concern allegedly cannot be addressed
II. FAILURE TO CONVENE A JOINT SESSION DEPRIVES LAWMAKERS OF A DELIBERATIVE AND by briefings in executive sessions given by representatives of the Executive Branch to both Houses
INTERROGATORY PROCESS TO REVIEW MARTIAL LAW. of the Congress.[27]

III. FAILURE TO CONVENE A JOINT SESSION DEPRIVES THE PUBLIC OF TRANSPARENT Petitioners further postulate that, based on the deliberations of the Members of the ConCom, the
PROCEEDINGS WITHIN WHICH TO BE INFORMED OF THE FACTUAL BASES OF MARTIAL phrase "voting jointly" under Article VII, Section 18 was intended to mean that a joint session is a
LAW AND THE INTENDED PARAMETERS OF ITS IMPLEMENTATION. procedural requirement, necessary for the Congress to decide whether to revoke, affirm, or even
extend the declaration of martial law.[28]
IV. THE FRAMERS OF THE CONSTITUTION INTENDED THAT A JOINT SESSION OF CONGRESS
BE CONVENED IMMEDIATELY AFTER THE DECLARATION OF MARTIAL LAW.[21] Consolidation of Respondents' Comments

Similar to the contentions in the Padilla Petition, petitioners maintain that they have sufficiently Respondents assert firmly that there is no mandatory duty on their part to "vote jointly," except in
shown all the essential requisites in order for this Court to exercise its power of judicial review, in cases of revocation or extension of the proclamation of martial law or the suspension of the
that: (1) an actual case or controversy exists; (2) they possess the standing to file this case; (3) the privilege of the writ of habeas corpus.[29] In the absence of such duty, the non-convening of the
constitutionality of a governmental act has been raised at the earliest possible opportunity; and (4) Congress in joint session does not pose any actual case or controversy that may be the subject of
the constitutionality of the said act is the very lis mota of the petition. judicial review.[30]Additionally, respondents argue that the petitions raise a political question over
which the Court has no jurisdiction.
According to petitioners, there is an actual case or controversy because the failure and/or refusal
of the Congress to convene jointly deprived legislators of a venue within which to raise a motion Petitioners' avowal that they are citizens and taxpayers is allegedly inadequate to clothe them
for revocation (or even extension) of President Duterte's Proclamation No. 216 and the public of with locus standi. Generalized interests, albeit accompanied by the assertion of a public right, do
an opportunity to be properly informed as to the bases and particulars thereof. [22] not establish locus standi. Petitioners must show that they have a direct and personal interest in
the Congress' failure to convene in joint session, which they failed to present herein. A taxpayer's
Petitioners likewise claim to have legal standing to sue as citizens and taxpayers. Nonetheless, they suit is likewise proper only when there is an exercise of the spending or taxing power of the
submit that the present case calls for the Court's liberality in the appreciation of their locus Congress. However, in these cases, the funds used in the implementation of martial law in
standi given the fact that their petition presents "a question of first impression - one of paramount Mindanao are taken from those funds already appropriated by the Congress. Senator De Lima's
importance to the future of our democracy - as well as the extraordinary nature of Martial Law averment of her locus standi as an incumbent member of the legislature similarly lacks merit.
itself."[23] Insofar as the powers of the Congress are not impaired, there is no prejudice to each Member
thereof; and even assuming arguendo that the authority of the Congress is indeed compromised,
Petitioners contend that the convening of the Congress in joint session, whenever the President Senator De Lima still does not have standing to file the present petition for mandamus because it
declares martial law or suspends the privilege of the writ of habeas corpus, is a public right and is not shown that she has been allowed to participate in the Senate sessions during her
duty mandated by the Constitution. The writ of mandamus is, thus, the "proper recourse for incarceration. She cannot, therefore, claim that she has suffered any direct injury from the non-
citizens who seek to enforce a public right and to compel the performance of a public duty, convening of the Congress in joint session.[31]
especially when the public right involved is mandated by the Constitution."[24]
Respondents further contend that the constitutional right to information, as enshrined under
For this group of petitioners, the Members of the Congress gravely abused their discretion for their Article III, Section 7 of the Constitution, is not absolute. Matters affecting national security are
refusal to convene in joint session, underscoring that "[w]hile a writ of mandamus will not considered as a valid exception to the right to information of the public. For this reason, the
generally lie from one branch of the government to a coordinate branch, or to compel the petitioners' and the public's right to participate in the deliberations of the Congress regarding the
performance of a discretionary act, this admits of certain exceptions, such as in instances of gross factual basis of a martial law declaration may be restricted in the interest of national security and
abuse of discretion, manifest injustice, or palpable excess of authority, when there is no other public safety.[32]
plain, speedy and adequate remedy."[25]
Respondents allege that petitioners failed to present an appropriate case for mandamus to
As to the merits, petitioners assert that the convening of the Congress in joint session after the lie. Mandamus will only issue when the act to be compelled is a clear legal duty or a ministerial
declaration of martial law is mandatory under Article VII, Section 18 of the Constitution, whether duty imposed by law upon the defendant or respondent to perform the act required that the law
or not the Congress is in session or there is intent to revoke. It is their theory that a joint session specifically enjoins as a duty resulting from office, trust, or station.[33]
should be a deliberative process in which, after debate and discussion, legislators can come to an
informed decision as to the factual and legal bases for the declaration of martial law. Moreover, According to respondents, it is erroneous to assert that it is their ministerial duty to convene in
"legislators who wish to revoke the martial law proclamation should have the right to put that vote joint session whenever martial law is proclaimed or the privilege of the writ of habeas corpus is
on historical record in joint session and, in like manner, the public should have the right to know suspended in the absence of a clear and specific constitutional or legal provision. In fact, Article VII,
Section 18 does not use the words "joint session" at all, much less impose the convening of such
joint session upon the proclamation of martial law or the suspension of the privilege of the writ On July 22, 2017, the Congress convened in joint session and, with two hundred sixty-one (261)
of habeas corpus. What the Constitution requires is joint voting when the action of the Congress is votes in favor versus eighteen (18) votes against, overwhelmingly approved the extension of the
to revoke or extend the proclamation or suspension.[34] proclamation of martial law and the suspension of the privilege of the writ of habeas corpus in
Mindanao until December 31, 2017.
Indeed, prior concurrence of the Congress is not constitutionally required for the effectivity of the
proclamation or suspension. Quoting from the deliberations of the framers of the Constitution STATEMENT OF THE ISSUES
pertaining to Article VII, Section 18, the Congress points out that it was the intention of the said
framers to grant the President the power to declare martial law or suspend the privilege of the After a meticulous consideration of the parties' submissions, we synthesize them into the following
writ of habeas corpusfor a period not exceeding sixty (60) days without the concurrence of the fundamental issues:
Congress. There is absolutely nothing under the Constitution that mandates the Congress to
convene in joint session when their intention is merely to discuss, debate, and/or review the I. Whether or not the Court has jurisdiction over the subject matter of these consolidated
factual and legal basis for the proclamation. That is why the phrase "voting jointly" is limited only petitions;
in case the Congress intends to revoke the proclamation.[35] In a situation where the Congress is
not in session, the Constitution simply provides that the Congress must convene in accordance II. Whether or not the petitions satisfy the requisites for the Court's exercise of its power
with its rules but does not state that it must convene in joint session. Respondents further refer to of judicial review;
the proper procedure for the holding of joint sessions.
III. Whether or not the Congress has the mandatory duty to convene jointly upon the
Respondents brush aside as mere obiter dictum the Court's pronouncement in the Fortun case that President's proclamation of martial law or the suspension of the privilege of the writ
it is the duty of the Congress to convene upon the declaration of martial law. That whether or not of habeas corpus under Article VII, Section 18 of the 1987 Constitution; and
the Congress should convene in joint session in instances where it is not revoking the proclamation
was not an issue in that case. Moreover, the factual circumstances in the Fortun case are entirely IV. Whether or not a writ of mandamus or certiorari may be issued in the present cases.
different from the present cases. The Congress then issued a concurrent resolution calling for the
convening of a joint session as the intention- at least as far as the Senate was concerned - was to
revoke the proclamation of martial law and the suspension of the privilege of the writ of habeas THE COURT'S RULING
corpus in Maguindanao. The Fortun case then cannot be considered a legislative precedent of an
"automatic convening of a joint session by the Congress upon the President's proclamation of The Court's jurisdiction over these consolidated petitions
martial law."[36]
The principle of separation of powers
Respondents argue that the remedy of certiorari is likewise unavailing. To justify judicial
intervention, the abuse of discretion must be so patent and gross as to amount to an evasion of a The separation of powers doctrine is the backbone of our tripartite system of government. It is
positive duty or to a virtual refusal to perform a duty enjoined by law or to act at all in implicit in the manner that our Constitution lays out in separate and distinct Articles the powers
contemplation of law, as where the power is exercised in an arbitrary and despotic manner by and prerogatives of each co-equal branch of government. In Belgica v. Ochoa,[41] this Court had the
reason of passion or hostility.[37] The Congress has the duty to convene and vote jointly only in two opportunity to restate:
(2) instances, as respondents have already explained. The Congress had even issued their
respective resolutions expressing their support to, as well as their intent not to revoke, President The principle of separation of powers refers to the constitutional demarcation of the three
Duterte's Proclamation No. 216. There then can be no evasion of a positive duty or a virtual refusal fundamental powers of government In the celebrated words of Justice Laurel in Angara v. Electoral
to perform a duty on the part of the Congress if there is no duty to begin with.[38] Commission, it means that the "Constitution has blocked out with deft strokes and in bold lines,
allotment of power to the executive, the legislative and the judicial departments of the
Respondents respectfully remind the Court to uphold the "constitutional demarcation of the three government" To the legislative branch of government, through Congress, belongs the power to
fundamental powers of government."[39] The Court may not intervene in the internal affairs of the make laws; to the executive branch of government, through the President, belongs the power to
Legislature and it is not within the province of the courts to direct the Congress how to do its work. enforce laws; and to the judicial branch of government, through the Court, belongs the power to
Respondents stress that this Court cannot direct the Congress to convene in joint session without interpret laws. Because the three great powers have been, by constitutional design, ordained in
violating the basic principle of the separation of powers.[40] this respect, "[e]ach department of the government has exclusive cognizance of matters within its
jurisdiction, and is supreme within its own sphere." Thus, "the legislature has no authority to
Subsequent Events execute or construe the law, the executive has no authority to make or construe the law, and the
judiciary has no power to make or execute the law." The principle of separation of powers and its
On July 14, 2017, petitioners in G.R. No. 231671, the Padilla Petition, filed a Manifestation, calling concepts of autonomy and independence stem from the notion that the powers of government
the attention of the Court to the imminent expiration of the sixty (60)-day period of validity of must be divided to avoid concentration of these powers in any one branch; the division, it is
Proclamation No. 216 on July 22, 2017. Despite the lapse of said sixty (60)-day period, petitioners hoped, would avoid any single branch from lording its power over the other branches or the
exhort the Court to still resolve the instant cases for the guidance of the Congress, State actors, citizenry. To achieve this purpose, the divided power must be wielded by co-equal branches of
and all Filipinos. government that are equally capable of independent action in exercising their respective
mandates. Lack of independence would result in the inability of one branch of government to
check the arbitrary or self-interest assertions of another or others. (Emphases supplied, citations advocate, a former Secretary of Justice, Chairperson of the Commission on Human Rights, and a
omitted.) taxpayer.
Contrary to respondents' protestations, the Court's exercise of jurisdiction over these petitions
cannot be deemed as an unwarranted intrusion into the exclusive domain of the Legislature. On the other hand, in G.R. No. 231694, while petitioner Taada sues in his capacity as a Filipino
Bearing in mind that the principal substantive issue presented in the cases at bar is the proper citizen and former legislator, his co-petitioners (Bishop Iiguez, Bishop Pabillo, Bishop Tobias, Mo.
interpretation of Article VII, Section 18 of the 1987 Constitution, particularly regarding the duty of Ygrubay, Bulangis, and Deluria) all sue in their capacity as Filipino citizens.
the Congress to vote jointly when the President declares martial law and/or suspends the privilege
of the writ of habeas corpus, there can be no doubt that the Court may take jurisdiction over the Respondents insist that none of the petitioners have legal standing, whether as a citizen, taxpayer,
petitions. It is the prerogative of the Judiciary to declare "what the law is."[42] It is worth repeating or legislator, to file the present cases.
here that:
The Court has consistently held that locus standi is a personal and substantial interest in a case
[W]hen the judiciary mediates to allocate constitutional boundaries, it does not assert any such that the party has sustained or will sustain direct injury as a result of the challenged
superiority over the other departments; it does not in reality nullify or invalidate an act of the governmental act. The question is whether the challenging party alleges such personal stake in the
legislature, but only asserts the solemn and sacred obligation assigned to it by the outcome of the controversy so as to assure the existence of concrete adverseness that would
Constitution to determine conflicting claims of authority under the Constitution and to establish sharpen the presentation of issues and illuminate the court in ruling on the constitutional question
for the parties in an actual controversy the rights which that instrument secures and guarantees to posed.[49]
them.[43] (Emphases supplied.)
Political question doctrine Petitioners satisfy these standards.

Corollary to respondents' invocation of the principle of separation of powers, they argue that The Court has recognized that every citizen has the right, if not the duty, to interfere and see that a
these petitions involve a political question in which the Court may not interfere. It is true that the public offense be properly pursued and punished, and that a public grievance be
Court continues to recognize questions of policy as a bar to its exercise of the power of judicial remedied.[50] When a citizen exercises this "public right" and challenges a supposedly illegal or
review.[44] However, in a long line of cases,[45] we have given a limited application to the political unconstitutional executive or legislative action, he represents the public at large, thus, clothing
question doctrine. him with the requisite locus standi. He may not sustain an injury as direct and adverse as
compared to others but it is enough that he sufficiently demonstrates in his petition that he is
In The Diocese of Bacolod v. Commission on Elections,[46] we emphasized that the Court's judicial entitled to protection or relief from the Court in the vindication of a public right.[51]
power as conferred by the Constitution has been expanded to include "the duty of the courts of
justice to settle actual controversies involving rights which are legally demandable and Verily, legal standing is grounded on the petitioner's personal interest in the controversy. A citizen
enforceable, and to determine whether or not there has been a grave abuse of discretion who files a petition before the court asserting a public right satisfies the requirement of personal
amounting to lack or excess of jurisdiction on the part of any branch or instrumentality of the interest simply because the petitioner is a member of the general public upon which the right is
Government." Further, in past cases, the Court has exercised its power of judicial review noting vested.[52] A citizen's personal interest in a case challenging an allegedly unconstitutional act lies in
that the requirement of interpreting the constitutional provision involved the legality and not his interest and duty to uphold and ensure the proper execution of the law.[53]
the wisdom of a manner by which a constitutional duty or power was exercised.[47]
The present petitions have been filed by individuals asserting that the Senate and the House of
In Association of Medical Clinics for Overseas Workers, Inc. (AMCOW) v. GCC Approved Medical Representatives have breached an allegedly constitutional duty to convene in joint session to
Centers Association, Inc.,[48] we explained the rationale behind the Court's deliberate on Presidential Proclamation No. 216. The citizen-petitioners' challenge of a purportedly
expanded certiorari jurisdiction. Citing former Chief Justice and Constitutional Commissioner unconstitutional act in violation of a public right, done in behalf of the general public, gives them
Roberto R. Concepcion in his sponsorship speech for Article VIII, Section 1 of the Constitution, we legal standing.
reiterated that the courts cannot hereafter evade the duty to settle matters, by claiming that such
matters constitute a political question. On the other hand, Senator De Lima questions the Congress' failure to convene in joint session to
deliberate on Proclamation No. 216, which, according to the petitioners, is the legislature's
Existence of the requisites for judicial review constitutional duty.

Petitioners' legal standing We have ruled that legislators have legal standing to ensure that the constitutional prerogatives,
powers, and privileges of the Members of the Congress remain inviolate.[54] Thus, they are allowed
Petitioners in G.R. No. 231671 allege that they are suing in the following capacities: (1) Padilla as a to question the validity of any official action - or in these cases, inaction - which, to their mind,
member of the legal profession representing victims of human rights violations, and a taxpayer; (2) infringes on their prerogatives as legislators.[55]
Saguisag as a human rights lawyer, former member of the Philippine Senate, and a taxpayer; (3)
Monsod as a framer of the Philippine Constitution and member of the 1986 ConCom, and a Actual case or controversy
taxpayer; (4) Rosales as a victim of human rights violations committed under martial law declared
by then President Ferdinand E. Marcos, and a taxpayer; (5) Gorospe as a lawyer and a taxpayer; It is long established that the power of judicial review is limited to actual cases or controversies.
and (6) Senator De Lima as an incumbent Member of the Philippine Senate, a human rights There is an actual case or controversy where there is a conflict of legal rights, an assertion of
opposite legal claims, where the contradiction of the rights can be interpreted and enforced on the
basis of existing law and jurisprudence.[56] At present, the Commander-in-Chief still possesses the power to suspend the privilege of the writ
of habeas corpus and to proclaim martial law. However, these executive powers are now subject
There are two conflicting claims presented before the Court: on the one hand, the petitioners' to the review of both the legislative and judicial branches. This check-and-balance mechanism was
assertion that the Congress has the mandatory duty to convene in joint session to deliberate on installed in the 1987 Constitution precisely to prevent potential abuses of these executive
Proclamation No. 216; and, on the other, the respondents' view that so convening in joint session prerogatives.
is discretionary on the part of the Congress.
Inasmuch as the present petitions raise issues concerning the Congress' role in our government's
Petitioners seek relief through a writ of mandamus and/or certiorari. Mandamus is a remedy system of checks and balances, these are matters of paramount public interest or issues of
granted by law when any tribunal, corporation, board, officer, or person unlawfully neglects the transcendental importance deserving the attention of the Court in view of their seriousness,
performance of an act which the law specifically enjoins as a duty resulting from an office, trust, or novelty, and weight as precedents.[63]
station, or unlawfully excludes another from the use or enjoyment of a right or office to which
such other is entitled.[57] Certiorari, as a special civil action, is available only if: (1) it is directed Mootness
against a tribunal, board, or officer exercising judicial or quasi-judicial functions; (2) the tribunal,
board, or officer acted without or in excess of jurisdiction or with grave abuse of discretion The Court acknowledges that the main relief prayed for in the present petitions (i.e., that the
amounting to lack or excess of jurisdiction; and (3) there is no appeal nor any plain, speedy, and Congress be directed to convene in joint session and therein deliberate whether to affirm or
adequate remedy in the ordinary course of law.[58] With respect to the Court, however, certiorari is revoke Proclamation No. 216) may arguably have been rendered moot by: (a) the lapse of the
broader in scope and reach, and it may be issued to correct errors of jurisdiction committed not original sixty (60) days that the President's martial law declaration and suspension of the privilege
only by a tribunal, corporation, board, or officer exercising judicial, quasijudicial, or ministerial of the writ of habeas corpus were effective under Proclamation No. 216; (b) the subsequent
functions, but also to set right, undo, and restrain any act of grave abuse of discretion amounting extension by the Congress of the proclamation of martial law and the suspension of the privilege
to lack or excess of jurisdiction by any branch or instrumentality of the Government, even if the of the writ of habeas corpus over the whole of Mindanao after convening in joint session on July
latter does not exercise judicial, quasi-judicial or ministerial functions.[59] 22, 2017; and (c) the Court's own decision in Lagman v. Medialdea,[64] wherein we ruled on the
sufficiency of the factual bases for Proclamation No. 216 under the original period stated therein.
As the present petitions allege an omission on the part of the Congress that constitutes neglect of
their constitutional duties, the petitions make a prima facie case for mandamus, and an actual case In David v. Macapagal-Arroyo, the jurisprudential rules regarding mootness were succinctly
or controversy ripe for adjudication exists. When an act or omission of a branch of government is summarized, thus:
seriously alleged to have infringed the Constitution, it becomes not only the right but, in fact, the
duty of the judiciary to settle the dispute.[60] A moot and academic case is one that ceases to present a justiciable controversy by virtue of
supervening events, so that a declaration thereon would be of no practical use or value. Generally,
Respondents aver that the Congress cannot be compelled to do something that is discretionary on courts decline jurisdiction over such case or dismiss it on ground of mootness.
their part nor could they be guilty of grave abuse of discretion in the absence of any mandatory
obligation to jointly convene on their part to affirm the President's proclamation of martial law. xxxx
Thus, petitioners are not entitled to the reliefs prayed for in their petitions
for mandamus and/or certiorari; consequently, no actual case or controversy exists. The "moot and academic" principle is not a magical formula that can automatically dissuade the
courts in resolving a case. Courts will decide cases, otherwise moot and academic, if: first, there is
There is no merit to respondents' position. a grave violation of the Constitution; second, the exceptional character of the situation and the
paramount public interest is involved; third, when constitutional issue raised requires
For the Court to exercise its power of judicial review and give due course to the petitions, it is formulation of controlling principles to guide the bench, the bar, and the public; and fourth, the
sufficient that the petitioners set forth their material allegations to make out a prima facie case case is capable of repetition yet evading review.[65] (Emphasis supplied, citations omitted.)
for mandamus or certiorari.[61] Whether the petitioners are actually and ultimately entitled to the It cannot be gainsaid that there are compelling and weighty reasons for the Court to proceed with
reliefs prayed for is exactly what is to be determined by the Court after careful consideration of the resolution of these consolidated petitions on the merits. As explained in the preceding
the parties' pleadings and submissions. discussion, these cases involve a constitutional issue of transcendental significance and novelty. A
definitive ruling from this Court is imperative not only to guide the Bench, the Bar, and the public
Liberality in cases of transcendental importance but, more importantly, to clarify the parameters of congressional conduct required by the 1987
Constitution, in the event of a repetition of the factual precedents that gave rise to these cases.
In any case, it is an accepted doctrine that the Court may brush aside procedural technicalities and,
nonetheless, exercise its power of judicial review in cases of transcendental importance. The duty of the Congress to vote jointly under Article VII, Section 18

There are marked differences between the Chief Executive's military powers, including the power We now come to the crux of the present petitions - the issue of whether or not under Article VII,
to declare martial law, as provided under the present Constitution, in comparison to that granted Section 18 of the 1987 Constitution, it is mandatory for the Congress to automatically convene in
in the 1935 Constitution. Under the 1935 Constitution,[62] such powers were seemingly limitless, joint session in the event that the President proclaims a state of martial law and/or suspends the
unrestrained, and purely subject to the President's wisdom and discretion. privilege of the writ of habeas corpus in the Philippines or any part thereof.
privilege of the writ of habeas corpus and grants the Congress the power to revoke, as well as
The Court answers in the negative. The Congress is not constitutionally mandated to convene in extend, the proclamation and/or suspension; and vests upon the Judiciary the power to review the
joint session except to vote jointly to revoke the President's declaration or suspension. sufficiency of the factual basis tor such proclamation and/or suspension.

By the language of Article VII, Section 18 of the 1987 Constitution, the Congress. is only required to There are four provisions in Article VII, Section 18 of the 1987 Constitution specifically pertaining
vote jointly to revoke the President's proclamation of martial law and/or suspension of the to the role of the Congress when the President proclaims martial law and/or suspends the privilege
privilege of the writ of habeas corpus. of the writ of habeas corpus, viz.:

Article VII, Section 18 of the 1987 Constitution fully reads: a. Within forty-eight (48) hours from the proclamation of martial law or the suspension of the
privilege of the writ of habeas corpus, the President shall submit a report in person or in writing to
Sec. 18. The President shall be the Commander-in-Chief of all armed forces of the Philippines and the Congress;
whenever it becomes necessary, he may call out such armed forces to prevent or suppress lawless
violence, invasion or rebellion. In case of invasion or rebellion, when the public safety requires it, b. The Congress, voting jointly, by a vote of at least a majority of all its Members in regular or
he may, for a period not exceeding sixty days, suspend the privilege of the writ of habeas corpus or special session, may revoke such proclamation or suspension, which revocation shall not be set
place the Philippines or any part thereof under martial law. Within forty-eight hours from the aside by the President;
proclamation of martial law or the suspension of the privilege of the writ of habeas corpus, the
President shall submit a report in person or in writing to the Congress. The Congress, voting c. Upon the initiative of the President, the Congress may, in the same manner. extend such
jointly, by a vote of at least a majority of all its Members in regular or special session, may proclamation or suspension for a period to he determined by the Congress, if the invasion or
revoke such proclamation or suspension which revocation shall not be set aside by the rebellion shall persist; and
President. Upon the initiative of the President, the Congress may, in the same manner, extend
such proclamation or suspension for a period to be determined by the Congress, if the invasion d. The Congress, if not in session, shall within twenty-four hours (24) following such proclamation
or rebellion shall persist and public safety requires it. or suspension, convene in accordance with its rules without need of call.
There is no question herein that the first provision was complied with, as within forty-eight (48)
The Congress, if not in session, shall, within twenty-four hours following such proclamation or hours from the issuance on May 23, 2017 by President Duterte of Proclamation No. 216, declaring
suspension, convene in accordance with its rules without need of a call. a state of martial law and suspending the privilege of the writ of habeas corpus in Mindanao,
copies of President Duterte's Report relative to Proclamation No. 216 was transmitted to and
The Supreme Court may review, in an appropriate proceeding filed by any citizen, the sufficiency received by the Senate and the House of Representatives on May 25, 2017.
of the factual basis of the proclamation of martial law or the suspension of the privilege of the writ
or the extension thereof, and must promulgate its decision thereon within thirty days from its The Court will not touch upon the third and fourth provisions as these concern factual
filing. circumstances which are not availing in the instant petitions. The petitions at bar involve the initial
proclamation of martial law and suspension of the privilege of the writ of habeas corpus, and not
A state of martial law does not suspend the operation of the Constitution, nor supplant the their extension; and the 17th Congress was still in session[68] when President Duterte issued
functioning of the civil courts or legislative assemblies, nor authorize the conferment of jurisdiction Proclamation No. 216 on May 23, 2017.
on military courts and agencies over civilians where civil courts are able to function, nor
automatically suspend the privilege of the writ. It is the second provision that is under judicial scrutiny herein: "The Congress, voting jointly, by a
vote of at least a majority of all its Members in regular or special session, may revoke such
The suspension of the privilege of the writ shall apply only to persons judicially charged for proclamation or suspension, which revocation shall not be set aside by the President."
rebellion or offenses inherent in or directly connected with invasion.
A cardinal rule in statutory construction is that when the law is clear and free from any doubt or
During the suspension of the privilege of the writ, any person thus arrested or detained shall be ambiguity, there is no room for construction or interpretation. There is only room for application.
judicially charged within three days, otherwise he shall be released. (Emphasis supplied.) According to the plain-meaning rule or verba legis, when the statute is clear, plain, and free from
Outside explicit constitutional limitations, the Commander-in-Chief clause in Article VII, Section 18 ambiguity, it must be given its literal meaning and applied without attempted interpretation. It is
of the 1987 Constitution vests on the President, as Commander-in-Chief, absolute authority over expressed in the maxims index animi sermo or "speech is the index of intention[,]" and verba legis
the persons and actions of the members of the armed forces, [66] in recognition that the President, non est recedendum or "from the words of a statute there should be no departure."[69]
as Chief Executive, has the general responsibility to promote public peace, and as Commander-in-
Chief, the more specific duty to prevent and suppress rebellion and lawless violence. [67] However, In Funa v. Chairman Villar,[70] the Court also applied the verba legis rule in constitutional
to safeguard against possible abuse by the President of the exercise of his power to proclaim construction, thus:
martial law and/or suspend the privilege of the writ of habeas corpus, the 1987 Constitution,
through the same provision, institutionalized checks and balances on the President's power The rule is that if a statute or constitutional provision is clear, plain and free from ambiguity, it
through the two other co-equal and independent branches of government, i.e., the Congress and must he given its literal meaning and applied without attempted interpretation. This is known as
the Judiciary. In particular, Article VII, Section 18 of the 1987 Constitution requires the President to the plain meaning rule enunciated by the maxim verba legis non est recedendum, or from the
submit a report to the Congress after his proclamation of martial law and/or suspension of the words of a statute there should be no departure.
mind the object sought to be accomplished by its adoption, and the evils, if any, sought to be
The primary source whence to ascertain constitutional intent or purpose is the language of the prevented or remedied. A doubtful provision will be examined in the light of the history of the
provision itself. If possible, the words in the Constitution must be given their ordinary meaning, times, and the condition and circumstances under which the Constitution was framed. The object
save where technical terms are employed. J.M. Tuason & Co., Inc. v. Land Tenure is to ascertain the reason which induced the framers of the Constitution to enact the particular
Administration illustrates the verbal legis rule in this wise: provision and the purpose sought to be accomplished thereby, in order to construe the whole as
to make the words consonant to that reason and calculated to effect that purpose.
We look to the language of the document itself in our search for its meaning. We do not of course However, in the same Decision, the Court issued the following caveat:
stop there, but that is where we begin. It is to he assumed that the words in which constitutional
provisions are couched express the objective sought to be attained. They are to be given their While it is permissible in this jurisdiction to consult the debates and proceedings of the
ordinary meaning except where technical terms are employed in which case the significance thus constitutional convention in order to arrive at the reason and purpose of the resulting
attached to them prevails. As the Constitution is not primarily a lawyer's document, it being Constitution, resort thereto may be had only when other guides fail as said proceedings are
essential for the rule of law to obtain that it should ever be present in the people's powerless to vary the terms of the Constitution when the meaning is clear. Debates in the
consciousness, its language as much as possible should be understood in the sense they have in constitutional convention "are of value as showing the views of the individual members, and as
common use. What it says according to the text of the provision to be construed compels indicating the reasons for their votes, but they give us no light as to the views' of the large majority
acceptance and negates the power of the courts to alter it, based on the postulate that the who did not talk, much less of the mass of our fellow citizens whose votes at the polls gave that
framers and the people mean what they say. Thus there are cases where the need for construction instrument the force of fundamental law. We think it safer to construe the constitution from
is reduced to a minimum. (Emphases supplied.) what appears upon its face." The proper interpretation therefore depends more on how it was
The provision in question is clear, plain, and unambiguous. In its literal and ordinary meaning, the understood by the people adopting it than in the framer's understanding thereof.[74] (Emphasis
provision grants the Congress the power to revoke the President's proclamation of martial law or supplied.)
the suspension of the privilege of the writ of habeas corpus and prescribes how the Congress may As the Court established in its preceding discussion, the clear meaning of the relevant provision in
exercise such power, i.e., by a vote of at least a majority of all its Members, voting jointly, in a Article VII, Section 18 of the 1987 Constitution is that the Congress is only required to vote jointly
regular or special session. The use of the word "may" in the provision - such that "[t]he Congress x on the revocation of the President's proclamation of martial law and/or suspension of the privilege
x x may revoke such proclamation or suspension x x x" - is to be construed as permissive and of the writ of habeas corpus. Based on the Civil Liberties Union case, there is already no need to
operating to confer discretion on the Congress on whether or not to revoke,[71] but in order to look beyond the plain language of the provision and decipher the intent of the framers of the 1987
revoke, the same provision sets the requirement that at least a majority of the Members of the Constitution. Nonetheless, the deliberations on Article VII, Section 18 of the 1986 ConCom does
Congress, voting jointly, favor revocation. not reveal a manifest intent of the framers to make it mandatory for the Congress to convene in
joint session following the President's proclamation and/or suspension, so it could deliberate as a
It is worthy to stress that the provision does not actually refer to a "joint session." While it may be single body, regardless of whether its Members will concur in or revoke the President's
conceded, subject to the discussions below, that the phrase "voting jointly" shall already be proclamation and/or suspension.
understood to mean that the joint voting will be done "in joint session," notwithstanding the
absence of clear language in the Constitution,[72] still, the requirement that "[t]he Congress, voting What is evident in the deliberations of the 1986 ConCom were the framers' intentions to (a)
jointly, by a vote of at least a majority of all its Members in regular or special session, x x x" remove the requirement of prior concurrence by the Congress for the effectivity of the President's
explicitly applies only to the situation when the Congress revokes the President's proclamation of proclamation of martial law and/or suspension of the privilege of the writ of habeas corpus; and
martial law and/or suspension of the privilege of the writ of habeas corpus. Simply put, the (b) grant to the Congress the discretionary power to revoke the President's proclamation and/or
provision only requires Congress to vote jointly on the revocation of the President's proclamation suspension by a vote of at least a majority of its Members, voting jointly.
and/or suspension.
As the Commander-in-Chief clause was initially drafted, the President's suspension of the privilege
Hence, the plain language of the subject constitutional provision does not support the petitioners' of the writ of habeas corpus required the prior concurrence of at least a majority of all the
argument that it is obligatory for the Congress to convene in joint session following the President's members of the Congress to be effective. The first line read, "The President shall be the
proclamation of martial law and/or suspension of the privilege of the writ of habeas corpus, under commander-in-chief of all the armed forces of the Philippines and, whenever it becomes
all circumstances. necessary, he may call out such armed forces to prevent or suppress lawless violence, invasion or
rebellion[;]" and the next line, "In case of invasion or rebellion, when the public safety requires it,
The deliberations of the 1986 ConCom reveal the framers' specific intentions to (a) remove the he may, for a period not exceeding sixty days, and, with the concurrence of at least a majority of
requirement of prior concurrence of the Congress for the effectivity of the President's all the members of the Congress, suspend the privilege of the writ of habeas corpus."[75]
proclamation of martial law and/or suspension of the privilege of the writ of habeas corpus; and
(b) grant to the Congress the discretionary power to revoke the President's proclamation and/or The Commissioners, however, extensively debated on whether or not there should be prior
suspension by a vote of at least a majority of its Members, voting jointly. concurrence by the Congress, and the exchanges below present the considerations for both sides:

The Court recognized in Civil Liberties Union v. The Executive Secretary[73] that: MR. NATIVIDAD. First and foremost, we agree with the Commissioner's thesis that in the first
imposition of martial law there is no need for concurrence of the majority of the Members of
A fool proof yardstick in constitutional construction is the intention underlying the provision under Congress because the provision says "in case of actual invasion and rebellion." If there is actual
consideration. Thus, it has been held that the Court in construing a Constitution should bear in invasion and rebellion, as Commissioner Crispino de Castro said, there is need for immediate
response because there is an attack. Second, the fact of securing a concurrence may be impractical interpretation is a situation of actual invasion or rebellion. In these situations, the President has to
because the roads might be blocked or barricaded. They say that in case of rebellion, one cannot act quickly. Secondly, this declaration has a time fuse. It is only good for a maximum of 60 days. At
even take his car and go to the Congress, which is possible because the roads are blocked or the end of 60 days, it automatically terminates. Thirdly, the right of the judiciary to inquire into the
barricaded. And maybe if the revolutionaries are smart, they would have an individual team for sufficiency of the factual basis of the proclamation always exists, even during those first 60 days.
each and every Member of the Congress so he would not be able to respond to a call for a session.
So the requirement of an initial concurrence of the majority of all the Members of the Congress in MR. SUAREZ. Given our traumatic experience during the past administration, if we give exclusive
case of an invasion or rebellion might be impractical as I can see it. right to the President to determine these factors, especially the existence of an invasion or
rebellion and the second factor of determining whether the public safety requires it or not, may I
Second, Section 15 states that the Congress may revoke the declaration or lift the suspension. call the attention of the Gentleman to what happened to us during the past administration.
Proclamation No. 1081 was issued by Ferdinand E. Marcos in his capacity as President of the
And third, the matter of declaring martial law is already a justiciable question and no longer a Philippines by virtue of the powers vested upon him purportedly under Article VII, Section 10(2) of
political one in that it is subject to judicial review at any point in time. So on that basis, I agree that the Constitution, wherein he made this predicate under the "Whereas" provision.
there is no need for concurrence as a prerequisite to declare martial law or to suspend the
privilege of the writ of habeas corpus. x x x Whereas, the rebellion and armed action undertaken by these lawless elements of the
Communists and other armed aggrupations organized to overthrow the Republic of the Philippines
xxxx by armed violence and force have assumed the magnitude of an actual state of war against our
people and the Republic of the Philippines.
MR. SUAREZ. x x x And may I also call the attention of the Gentleman to General Order No.3, also promulgated by
Ferdinand E. Marcos, in his capacity as Commander-in-Chief of all the Armed Forces of the
The Commissioner is suggesting that in connection with Section 15, we delete the phrase "and, Philippines and pursuant to Proclamation No. 1081 dated September 21, 1972 wherein he said,
with the concurrence of at least a majority of all the Members of the Congress..." among other things:

MR. PADILLA. That is correct especially for the initial suspension of the privilege of the writ Whereas, martial law having been declared because of wanton destruction of lives and properties,
of habeas corpus or also the declaration of martial law. widespread lawlessness and anarchy and chaos and disorder now prevailing throughout the
country, which condition has been brought about by groups of men who are actively engaged in a
MR. SUAREZ. So in both instances, the Commissioner is suggesting that this would be an exclusive criminal conspiracy to seize political and state power in the Philippines in order to take over the
prerogative of the President? government by force and violence, the extent of which has now assumed the proportion of an
actual war against our people and the legitimate government...
MR. PADILLA. At least initially, for a period of 60 days. But even that period of 60 days may be And he gave all reasons in order to suspend the privilege of the writ of habeas corpus and declare
shortened by the Congress or the Senate because the next sentence says that the Congress or the martial law in our country without justifiable reason. Would the Gentleman still insist on the
Senate may even revoke the proclamation. deletion of the phrase "and, with the concurrence of at least a majority of all the members of the
Congress"?
xxxx
MR. MONSOD. Yes, Madam President, in the case of Mr. Marcos he is undoubtedly an aberration
MR. MONSOD. x x x in our history and national consciousness. But given the possibility that there would be another
Marcos, our Constitution now has sufficient safeguards. As I said, it is not really true, as the
We are back to Section 15, page 7, lines 1 and 2. I just want to reiterate my previous proposal to Gentleman has mentioned, that there is an exclusive right to determine the factual bases
amend by deletion the phrase "and, with the concurrence of at least a majority of all the members because the paragraph beginning on line 9 precisely tells us that the Supreme Court may review,
of Congress." in an appropriate proceeding filed by any citizen, the sufficiency of the factual basis of the
proclamation of martial law or the suspension of the privilege of the writ or the extension thereof
xxxx and must promulgate its decision on the same within 30 days from its filing.

MR. SUAREZ. x x x I believe that there are enough safeguards. The Constitution is supposed to balance the interests
of the country. And here we are trying to balance the public interest in case of invasion or rebellion
The Commissioner is proposing a very substantial amendment because this means that he is as against the rights of citizens. And I am saying that there are enough safeguards, unlike in 1972
vesting exclusively unto the President the right to determine the factors which may lead to the when Mr. Marcos was able to do all those things mentioned.
declaration of martial law and the suspension of the writ of habeas corpus. I suppose he has strong
and compelling reasons in seeking to delete this particular phrase. May we be informed of his good MR. SUAREZ. Will that prevent a future President from doing what Mr. Marcos had done?
and substantial reasons?
MR. MONSOD. There is nothing absolute in this world, and there may be another Marcos. What
MR. MONSOD. This situation arises in cases of invasion or rebellion. And in previous interpellations we are looking for are safeguards that are reasonable and, I believe, adequate at this point. On the
regarding this phrase, even during the discussions on the Bill of Rights, as I understand it, the other hand, in case of invasion or rebellion, even during the first 60 days when the intention
here is to protect the country in that situation, it would be unreasonable to ask that there revoke the President's proclamation and/or suspension, that is, "voting jointly, by a vote of at least
should be a concurrence on the part of the Congress, which situation is automatically terminated a majority of all its Members in regular or special session."
at the end of such 60 days.
The ConCom deliberations on this particular provision substantially revolved around whether the
xxxx two Houses will have to vote jointly or separately to revoke the President's proclamation of martial
law and/or suspension of the privilege of the writ of habeas corpus; but as the Court reiterates, it
MR. SUAREZ. Would the Gentleman not feel more comfortable if we provide for a legislative check is undisputedly for the express purpose of revoking the President's proclamation and/or
on this awesome power of the Chief Executive acting as Commander-in-Chief? suspension.

MR. MONSOD. I would be less comfortable if we have a presidency that cannot act under those Based on the ConCom deliberations, pertinent portions of which are reproduced hereunder, the
conditions. underlying reason for the requirement that the two Houses of the Congress will vote jointly is to
avoid the possibility of a deadlock and to facilitate the process of revocation of the President's
MR. SUAREZ. But he can act with the concurrence of the proper or appropriate authority. proclamation of martial law and/or suspension of the privilege of the writ of habeas corpus:

MR. MONSOD. Yes. But when those situations arise, it is very unlikely that the concurrence of MR. MONSOD. Madam President, I want to ask the Committee a clarifying question on line 4 of
Congress would be available; and, secondly, the President will be able to act quickly in order to page 7 as to whether the meaning here is that the majority of all the Members of each House vote
deal with the circumstances. separately. Is that the intent of this phrase?

MR. SUAREZ. So, we would be subordinating actual circumstances to expediency. xxxx

MR. MONSOD. I do not believe it is expediency when one is trying to protect the country in the FR. BERNAS. We would like a little discussion on that because yesterday we already removed the
event of an invasion or a rebellion. necessity for concurrence of Congress for the initial imposition of martial law. If we require the
Senate and the House of Representatives to vote separately for purposes of revoking the
MR. SUAREZ. No. But in both instances, we would be seeking to protect not only the country but imposition of martial law, that will make it very difficult for Congress to revoke the imposition of
the rights of simple citizens. We have to balance these interests without sacrificing the security of martial law and the suspension of the privilege of the writ of habeas corpus. That is just thinking
the State. aloud. To balance the fact that the President acts unilaterally, then the Congress voting as one
body and not separately can revoke the declaration of martial law or the suspension of the
MR. MONSOD. I agree with the Gentleman that is why in the Article on the Bill of Rights, which privilege of the writ of habeas corpus.
was approved on Third Reading, the safeguards and the protection of the citizens have been
strengthened. And on line 21 of this paragraph, I endorsed the proposed amendment of MR. MONSOD. In other words, voting jointly.
Commissioner Padilla. We are saying that those who are arrested should be judicially charged
within five days; otherwise, they shall be released. So, there are enough safeguards. FR. BERNAS. Jointly, yes.

MR. SUAREZ. These are safeguards after the declaration of martial law and after the suspension xxxx
of the writ ofhabeas corpus.
MR. RODRIGO. May I comment on the statement made by Commissioner Bernas? I was a Member
MR. MONSOD. That is true.[76] (Emphases supplied.) of the Senate for 12 years. Whenever a bicameral Congress votes, it is always separately.
Ultimately, twenty-eight (28) Commissioners voted to remove the requirement for prior
concurrence by the Congress for the effectivity of the President's proclamation of martial law For example, bills coming from the Lower House are voted upon by the Members of the House.
and/or suspension of the privilege of the writ of habeas corpus, against only twelve (12) Then they go up to the Senate and voted upon separately. Even on constitutional amendments,
Commissioners who voted to retain it. where Congress meets in joint session, the two Houses vote separately.

As the result of the foregoing, the 1987 Constitution does not provide at all for the manner of Otherwise, the Senate will be useless; it will be sort of absorbed by the House considering that the
determination and expression of concurrence (whether prior or subsequent) by the Congress in Members of the Senate are completely outnumbered by the Members of the House. So, I believe
the President's proclamation of martial law and/or suspension of the privilege of the writ that whenever Congress acts, it must be the two Houses voting separately.
of habeas corpus. In the instant cases, both Houses of the Congress separately passed resolutions,
in accordance with their respective rules of procedure, expressing their support for President If the two Houses vote "jointly," it would mean mixing the 24 Senators with 250 Congressmen. This
Duterte's Proclamation No. 216. would result in the Senate being absorbed and controlled by the House. This violates the purpose
of having a Senate.
In contrast, being one of the constitutional safeguards against possible abuse by the President of
his power to proclaim martial law and/or suspend the privilege of the writ of habeas corpus, the FR. BERNAS. I quite realize that that is the practice and, precisely, in proposing this, I am
1987 Constitution explicitly provides for how the Congress may exercise its discretionary power to consciously proposing this as an exception to this practice because of the tremendous effect on
the nation when the privilege of the writ of habeas corpus is suspended and then martial law is
imposed. Since we have allowed the President to impose martial law and suspend the privilege of MR. RODRIGO. I was the one who proposed that the two Houses vote separately because if they
the writ of habeas corpus unilaterally, we should make it a little more easy for Congress to vote jointly, the Senators are absolutely outnumbered. It is insulting to the intelligence of the
reverse such actions for the sake of protecting the rights of the people. Senators to join a session where they know they are absolutely outnumbered. Remember that the
Senators are elected at large by the whole country. The Senate is a separate Chamber. The
MR. RODRIGO. Maybe the way it can be done is to vest this function in just one of the Chambers - Senators have a longer term than the Members of the House; they have a six-year term. They are a
to the House alone or to the Senate alone. But to say, "by Congress," both House and Senate continuing Senate. Out of 24, twelve are elected every year. So, if they will participate at all, the
"voting" jointly is practically a vote by the House. Senate must vote separately. That is the practice everywhere where there are two chambers. But
as I said, between having a joint session of the Senate and the House voting jointly where it is
FR. BERNAS. I would be willing to say just the vote of the House. practically the House that will decide alone, the lesser of two evils is just to let the House decide
alone instead of insulting the Senators by making them participate in a charade.
MR. RODRIGO. That is less insulting to the Senate. However, there are other safeguards. For
example, if, after 60 days the Congress does not act, the effectiveness of the declaration of martial MR. REGALADO. May the Committee seek this clarification from Commissioner Rodrigo? This
law or the suspension of the privilege of the writ ceases. Furthermore, there is recourse to the voting is supposed to revoke the proclamation of martial law. If the two Houses vote separately
Supreme Court. and a majority is obtained in the House of Representatives for the revocation of the proclamation
of martial law but that same majority cannot he obtained in the Senate voting separately, what
FR. BERNAS. I quite realize that there is this recourse to the Supreme Court and there is a time would be the situation?
limit, but at the same time because of the extraordinary character of this event when martial law is
imposed, I would like to make it easier for the representatives of the people to review this very MR. RODRIGO. Then the proclamation of martial law or the suspension continues for almost two
significant action taken by the President. months. After two months, it stops. Besides, there is recourse to the Supreme Court.

MR. RODRIGO. Between the Senate being absorbed and controlled by the House numerically and MR. REGALADO. Therefore, that arrangement would be very difficult for the legislative since they
the House voting alone, the lesser of two evils is the latter. are voting separately and, for lack of majority in one of the Houses they are precluded from
revoking that proclamation. They will just, therefore, have to wait until the lapse of 60 days.
xxxx
MR. RODRIGO. It might be difficult, yes. But remember, we speak of the Members of Congress who
MR. GUINGONA. x x x are elected by the people. Let us not forget that the President is also elected by the people. Are we
forgetting that the President is elected by the people? We seem to distrust all future Presidents
In connection with the inquiry of Commissioner Monsod, and considering the statements made by just became one President destroyed our faith by his declaration of martial law. I think we are
Commissioner Rodrigo, I would like to say, in reply to Commissioner Bernas, that perhaps because overreacting. Let us not judge all Presidents who would henceforth be elected by the Filipino
of necessity, we might really have to break tradition. Perhaps it would be better to give this people on the basis of the abuses made by that one President. Of course, we must be on guard;
function of revoking the proclamation of martial law or the suspension of the writ or extending but let us not overreact.
the same to the House of Representatives, instead of to the Congress. I feel that even the Senators
would welcome this because they would feel frustrated by the imbalance in the number between Let me make my position clear. I am against the proposal to make the House and the Senate vote
the Senators and the Members of the House of Representatives. jointly. That is an insult to the Senate.

Anyway, Madam President, we have precedents or similar cases. For example, under Section 24 of xxxx
the committee report on the Legislative, appropriation, revenue or tariff bills, and bills authorizing
increase of public debt are supposed to originate exclusively in the House of Representatives. MR. RODRIGO. Will the Gentleman yield to a question?
Besides, we have always been saying that it is the Members of the House of Representatives who
are mostly in touch with the people since they represent the various districts of our country. MR. MONSOD. Yes, Madam President.

xxxx MR. RODRIGO. So, in effect, if there is a joint session composed of 250 Members of the House plus
24 Members of the Senate, the total would be 274. The majority would be one-half plus one.
MR. MONSOD. I would prefer to have the vote of both Houses because this is a very serious
question that must be fully discussed. By limiting it alone to the House of Representatives, then we MR. MONSOD. So, 148 votes.
lose the benefit of the advice and opinion of the Members of the Senate. I would prefer that they
would be in joint session, but I would agree with Father Bernas that they should not be voting MR. RODRIGO. And the poor Senators would be absolutely absorbed and outnumbered by the 250
separately as part of the option. I think they should be voting jointly, so that, in effect, the Members of the House. Is that it?
Senators will have only one vote. But at least we have the benefit of their advice.
MR. MONSOD. Yes, that is one of the implications of the suggestion and the amendment is being
xxxx made nonetheless because there is a higher objective or value which is to prevent a deadlock that
would enable the President to continue the full 60 days in case one House revokes and the other
House does not. xxxx

The proposal also allows the Senators to participate fully in the discussions and whether we like it MR. DE CASTRO. What is the rationale of the amendment?
or not, the Senators have very large persuasive powers because of their prestige and their national
vote. MR. RODRIGO. It is intended to avoid that very extraordinary and awkward provision which would
make the 24 Senators meet jointly with 250 Members of the House and make them vote jointly.
MR. RODRIGO. So, the Senators will have the "quality votes" but Members of the House will have What I mean is, the 24 Senators, like a drop in the bucket, are absorbed numerically by the 250
the "quantity votes." Is that it? Members of the House.

MR. MONSOD. The Gentleman is making an assumption that they will vote against each other. I xxxx
believe that they will discuss, probably in joint session and vote on it; then the consensus will be
clear. MR. SARMIENTO. Madam President, we need the wisdom of the Senators. What is at stake is the
future of our country - human rights and civil liberties. If we separate the Senators, then we
xxxx deprive the Congressmen of the knowledge and experience of these 24 men. I think we should
forget the classification of "Senators" or "Congressmen." We should all work together to restore
MR. NOLLEDO. Madam President, the purpose of the amendment is really to set forth a limitation democracy in our country. So we need the wisdom of 24 Senators.
because we have to avoid a stalemate. For example, the Lower House decides that the declaration
of martial law should be revoked, and that later on, the Senate sitting separately decides that it MR. RODRIGO. Madam President, may I just answer. This advice of the 24 Senators can be sought
should not be revoked. It becomes inevitable that martial law shall continue even if there should because they are in the same building. Anyway, the provision, with the amendment of
be no factual basis for it. Commissioner Monsod, does not call for a joint session. It only says: "the Congress, by a vote of
at least a majority of all its Members in regular or special session" - it does not say "joint session."
MR. OPLE. Madam President, if this amendment is adopted, we will be held responsible for a So, I believe that if the Members of the House need the counsel of the Senators, they can always
glaring inconsistency in the Constitution to a degree that it distorts the bicameral system that we call on them, they can invite them.[78](Emphasis supplied.)
have agreed to adopt. I reiterate: If there are deadlocks, it is the responsibility of the presidential The proposed amendment was not adopted, however, as only five (5) Commissioners voted in its
leadership, together with the leaders of both Houses, to overcome them.[77] (Emphases supplied.) favor and twenty-five (25) Commissioners voted against it. Thus, the power to revoke the
When the matter was put to a vote, twenty-four (24) Commissioners voted for the two Houses of President's proclamation of martial law and/or suspension of the privilege of the writ of habeas
the Congress "voting jointly" in the revocation of the President's proclamation of martial law corpus still lies with both Houses of the Congress, voting jointly, by a vote of at least a majority of
and/or suspension of the privilege of the writ of habeas corpus, and thirteen (13) Commissioners all its Members.
opted for the two Houses "voting separately."
Significantly, the Commissioners only settled the manner of voting by the Congress, i.e., "voting
Yet, there was another attempt to amend the provision by requiring just the House of jointly, by a vote of at least a majority of all its Members," in order to revoke the President's
Representatives, not the entire Congress, to vote on the revocation of the President's proclamation of martial law and/or suspension of the privilege of the writ of habeas corpus, but
proclamation of martial law and/or suspension of the privilege of the writ of habeas corpus: they did not directly take up and specify in Article VII, Section 18 of the 1987 Constitution that the
voting shall be done during a joint session of both Houses of the Congress. In fact, Commissioner
MR. RODRIGO. Madam President, may I propose an amendment? Francisco A. Rodrigo expressly observed that the provision does not call for a joint session. That
the Congress will vote on the revocation of the President's proclamation and/or suspension in a
xxxx joint session can only be inferred from the arguments of the Commissioners who pushed for the
"voting jointly" amendment that the Members of the House of Representatives will benefit from
MR. RODRIGO. On Section 15, page 7, line 4, I propose to change the word "Congress" to HOUSE the advice, opinion, and/or wisdom of the Senators, which will be presumably shared during a
OF REPRESENTATIVES so that the sentence will read: "The HOUSE OF REPRESENTATIVES, by a vote joint session of both Houses. Such inference is far from a clear mandate for the Congress to
of at least a majority of all its Members in regular or special session, may revoke such proclamation automatically convene in joint session, under all circumstances, when the President proclaims
or suspension or extend the same if the invasion or rebellion shall persist and public safety martial law and/or suspends the privilege of the writ of habeas corpus, even when Congress does
requires it." not intend to revoke the President's proclamation and/or suspension.

FR. BERNAS. Madam President, the proposed amendment is really a motion for reconsideration. There was no obligation on the part of the Congress herein to convene in joint session as the
We have already decided that both Houses will vote jointly. Therefore, the proposed amendment, provision on revocation under Article VII, Section 18 of the 1987 Constitution did not even come
in effect, asks for a reconsideration of that vote in order to give it to the House of Representatives. into operation in light of the resolutions, separately adopted by the two Houses of the Congress in
accordance with their respective rules of procedure, expressing support for President Duterte's
MR. RODRIGO. Madam President, the opposite of voting jointly is voting separately. If my Proclamation No. 216.
amendment were to vote separately, then, yes, it is a motion for reconsideration. But this is
another formula. The provision in Article VII, Section 18 of the 1987 Constitution requiring the Congress to vote
jointly in a joint session is specifically for the purpose of revocation of the President's proclamation 6. On 29 May 2017, about 3:30 p.m., a closed door briefing was conducted by Secretary Lorenzana,
of martial law and/or suspension of the privilege of the writ of habeas corpus. In the petitions at Secretary Esperon and other security officials for the Senators to brief them about the
bar, the Senate and House of Representatives already separately adopted resolutions expressing circumstances surrounding the declaration of martial law and to inform them about details about
support for President Duterte's Proclamation No. 216. Given the express support of both Houses of the President's Report. The briefing lasted for about four (4) hours. After the briefing, the Senators
the Congress for Proclamation No. 216, and their already evident lack of intent to revoke the had a caucus to determine what could be publicly revealed.
same, the provision in Article VII, Section 18 of the 1987 Constitution on revocation did not even
come into operation and, therefore, there is no obligation on the part of the Congress to convene 7. On the same day, 29 May 2017, the House of Representatives resolved to constitute itself as a
in joint session. Committee of the Whole on 31 May 2017 to consider the President's Report.

Practice and logic dictate that a collegial body will first hold a meeting among its own members to 8. On 30 May 2017, two (2) resolutions were introduced in the Senate about the proclamation of
get a sense of the opinions of its individual members and, if possible and necessary, reach an martial law. The first one was P.S. Resolution No. 388 (hereinafter, "P.S.R. No. 388") introduced by
official stance, before convening with another collegial body. This is exactly what the two Houses Senators Sotto, Pimentel, Recto, Angara, Binay, Ejercito, Gatchalian, Gordon, Honasan, Lacson,
of the Congress did in these cases. Legarda, Pacquiao, Villanueva, Villar and Zubiri which was entitled, "Expressing the Sense of the
Senate, Supporting the Proclamation No. 216 dated May 23, 2017, entitled "Declaring a State of
The two Houses of the Congress, the Senate and the House of Representatives immediately took Martial Law and Suspending the Privilege of the Writ of Habeas Corpus in the Whole of Mindanao"
separate actions on President Duterte's proclamation of martial law and suspension of the and Finding no Cause to revoke the Same." The second one was P.S. Resolution No. 390
privilege of the writ of habeas corpus in Mindanao through Proclamation No. 216, in accordance (hereinafter, "P.S.R. No. 390") introduced by Senators Pangilinan, Drilon, Hontiveros, Trillanes,
with their respective rules of procedure. The Consolidated Comment (Ex Abudanti Cautela), filed by Aquino and De Lima which was entitled, "Resolution to Convene Congress in Joint Session and
the Senate and Senate President Pimentel, recounted in detail the steps undertaken by both Deliberate on Proclamation No. 216 dated 23 May 2017 entitled, "Declaring a State of Martial Law
Houses of the Congress as regards Proclamation No. 216, to wit: and Suspending the Privilege of the Writ of Habeas Corpus in the Whole of Mindanao." x x x

2. On the date of the President's declaration of martial law and the suspension of the privilege of 9. Discussions were made on the two (2) proposed resolutions during the plenary deliberations of
the writ of habeas corpus, Congress was in session (from May 2, to June 2, 2017), in its First the Senate on 30 May 2017. The first resolution to be discussed was P.S.R. No. 388. During the
Regular Session of the 17th Congress, as evidenced by its Legislative Calendar, otherwise known as deliberations, amendments were introduced to it and after the amendments and the debates,
Calendar of Session as contained in Concurrent Resolution No. 3 of both the Senate and the House P.S.R. No. 388 was voted upon and it was adopted by a vote of seventeen (17) affirmative votes
of Representatives. x x x and five (5) negative votes. The amended, substituted and approved version of P.S.R. No. 388,
which was then renamed Resolution No. 49, states as follows:
3. During the plenary session of the Senate on the following day, 24 May 2017, privilege speeches
and discussions had already been made about the declaration of martial law and the suspension of RESOLUTION NO. 49
the privilege of the writ of habeas corpus. This prompted Senator Franklin M. Drilon to move to
invite the Secretary of National Defense, the National Security Adviser and the Chief of Staff of the RESOLUTION EXPRESSING THE SENSE OF THE SENATE NOT TO REVOKE, AT THIS TIME,
Armed Forces of the Philippines to brief the senators in closed session on what transpired in PROCLAMATION NO. 216, SERIES OF 2017, ENTITLED, "DECLARING A STATE OF MARTIAL LAW AND
Mindanao. Submitted to a vote and there being no objection, the Senate approved the motion. x x SUSPENDING THE PRIVILEGE OF THE WRIT OF HABEAS CORPUS IN THE WHOLE OF MINDANAO."
x
WHEREAS, the 1987 Philippine Constitution, Article VII, Section 18, provides that:
4. On 25 May 2017, the President furnished the Senate and the House of Representatives, through
Senate President Aquilino "Koko" Pimentel III and Speaker Pantaleon D. Alvarez, respectively, with "... in case of invasion or rebellion, when the public safety requires it, he (President) may, for a
copies of his report (hereinafter, the "Report") detailing the factual and legal basis for his period not exceeding sixty days, suspend the privilege of the writ of habeas corpus or place the
declaration of martial law and the suspension of the privilege of the writ of habeas corpus in Philippines or any part thereof under martial law...";
Mindanao. WHEREAS, President Rodrigo Roa Duterte issued Proclamation No. 216, series of 2017, entitled
"Declaring a State of Martial Law and Suspending the Privilege of the Writ of Habeas Corpus in the
5. On or about 25 May 2017, invitation letters were issued and sent by the Senate Secretary, Atty. Whole of Mindanao," on May 23, 2017 (the "Proclamation");
Lutgardo B. Barbo to the following officials requesting them to attend a briefing for the Senators
on 29 May 2017 at 3:00 p.m. at the Senators' Lounge at the Senate in a closed door session to WHEREAS, pursuant to his duty under the Constitution, on May 25, 2017, and within forth-eight
describe what transpired in Mindanao which was the basis of the declaration of martial law in hours after the issuance of the Proclamation, President Duterte submitted to the Senate his report
Mindanao: (a) Secretary Delfin N. Lorenzana, Secretary of National Defense (hereinafter, on the factual and legal basis of the Proclamation;
"Secretary Lorenzana"); (b) Secretary Hermogenes C. Esperon, Jr., National Security Adviser and
Director General of the National Security Council (hereinafter, "Secretary Esperon"); and (c) WHEREAS, on May 29, 2017, the Senators were briefed by the Department of National Defense
General Eduardo M. Ao, Chief of Staff of the Armed Forces of the Philippines (hereinafter, "Gen. (DND), the Armed Forces of the Philippines (AFP), and by the National Security Council (NSC) on
Ao"). The said letters stated that the Senators requested that the President's Report be explained the factual circumstances surrounding the Proclamation as well as the updates on the situation in
and that more details be given about the same. x x x Mindanao;
WHEREAS, on the basis of the information received by the Senators, the Senate is convinced that hold a joint session, and only thereafter can the Houses adopt the rules to be observed for that
President Duterte declared martial law and suspended the privilege of the writ of habeas corpus in particular joint session:
the whole of Mindanao because actual rebe11ion exists and that the public safety requires it;
It must be stated that the Senate and the House of Representatives have their own respective
WHEREAS, the Senate, at this time, agrees that there is no compelling reason to revoke Rules, i.e., the Rules of the Senate and the Rules of the House of Representatives. There is no
Proclamation No. 216, series of 2017; general body of Rules applicable to a joint session of Congress. Based on parliamentary practice
and procedure, the Senate and House of Representatives only adopt Rules for a joint session on
WHEREAS, the Proclamation does not suspend the operation of the Constitution, which among an ad hoc basis but only after both Houses have already agreed to convene in a joint session
others, guarantees respect for human rights and guards against any abuse or violation thereof: through a Concurrent Resolution. The Rules for a Joint Session for a particular purpose
Now, therefore, be it become functus officioafter the purpose of the joint session has been achieved. Examples of
these Rules for a Joint Session are (1) the Rules of the Joint Public Session of Congress on
Resolved, as it is hereby resolved, To express the sense of the Senate, that there is no compelling Canvassing the Votes Cast for Presidential and Vice-Presidential Candidates in the May 9, 2016
reason to revoke Proclamation No. 216, series of 2017 at this time. Election adopted on 24 May 2016; and (2) the Rules of the Joint Session of Congress on
Proclamation No. 1959 (Proclaiming a State of Martial Law and Suspending the Privilege of the
Adopted. x x x" Writ of Habeas Corpus in the Province of Maguindanao, Except for Certain Areas) adopted on 09
xxxx December 2009. The only time that the Senate and the House of Representatives do not adopt
Rules for a joint session is when they convene on the fourth Monday of July for its regular session
10. Immediately thereafter, P.S.R. No. 390 was also deliberated upon. After a prolonged to receive or listen to the State of the Nation Address of the President and even then, they adopt a
discussion, a vote was taken on it and nine (9) senators were in favor and twelve (12) were against. Concurrent Resolution to do so.
As such, P.S.R. No. 390 calling for a joint session of Congress was not adopted. x x x
The usual procedure for having a .ioint session is for both Houses to first adopt a Concurrent
11. In the meantime, on 31 May 2017, the House of Representatives acting as a Committee of the Resolution to hold a joint session. This is achieved by either of two (2) ways: (1) both the Senate
Whole was briefed for about six (6) hours by officials of the government led by Executive Secretary and the House of Representatives simultaneously adopting the Concurrent Resolution - an
Salvador C. Medialdea (hereinafter, "Executive Secretary Medialdea"), Secretary Lorenzana and example would be when the two (2) Houses inform the President that they are ready to receive
other security officials on the factual circumstances surrounding the President's declaration of his State of the Nation Address or (2) For one (1) House to pass its own resolution and to send it
martial law and on the statements contained in the President's Report. During the evening of the to the other House for the latter's concurrence. Once the joint session of both Houses is actually
same day, a majority of the House of Representatives passed Resolution No. 1050 entitled, convened, it is only then that the Senate and the House of Representatives jointly adopt the Rules
"Resolution Expressing the Full Support of the House of Representatives to President Rodrigo Roa for the joint session. x x x[80](Emphases supplied.)
Duterte As It Finds No Reason to Revoke Proclamation No. 216 Entitled, 'Declaring A State of With neither Senate nor the House of Representatives adopting a concurrent resolution, no joint
Martial Law and Suspending the Privilege of the Writ of Habeas Corpus in the Whole of session by the two Houses of the Congress can be had in the present cases.
Mindanao.'" In the same deliberations, it was likewise proposed that the House of Representatives
call for a joint session of Congress to deliberate and vote on the President's declaration of martial The Court is bound to respect the rules of the Congress, a co-equal and independent branch of
law and the suspension of the privilege of the writ of habeas corpus. However, after debates, the government. Article VI, Section 16(3) of the 1987 Constitution states that "[e]ach House shall
proposal was not carried. x x x.[79] determine the rules of its proceedings." The provision has been traditionally construed as a grant
It cannot be disputed then that the Senate and House of Representatives placed President of full discretionary authority to the Houses of Congress in the formulation, adoption, and
Duterte's Proclamation No. 216 under serious review and consideration, pursuant to their power promulgation of its rules; and as such, the exercise of this power is generally exempt from judicial
to revoke such a proclamation vested by the Constitution on the Congress. Each House timely took supervision and interference.[81] Moreover, unless there is a clear showing by strong and
action by accepting and assessing the President's Report, inviting over and interpellating executive convincing reasons that they conflict with the Constitution, "all legislative acts are clothed with an
officials, and deliberating amongst their fellow Senators or Representatives, before finally voting in armor of constitutionality particularly resilient where such acts follow a long-settled and well-
favor of expressing support for President Duterte's Proclamation No. 216 and against calling for a established practice by the Legislature."[82] Nothing in this Decision should be presumed to give
joint session with the other House. The prompt actions separately taken by the two Houses of the precedence to the rules of the Houses of the Congress over the provisions of the Constitution. This
Congress on President Duterte's Proclamation No. 216 belied all the purported difficulties and Court simply holds that since the Constitution does not regulate the manner by which the
delays such procedures would cause as raised in the Concurring and Dissenting Opinion of Congress may express its concurrence to a Presidential proclamation of martial law and/or
Associate Justice Marvic M.V.F. Leonen (Justice Leonen). As earlier pointed out, there is no suspension of the privilege of the writ of habeas corpus, the Houses of the Congress have the
constitutional provision governing concurrence by the Congress in the President's proclamation of discretion to adopt rules of procedure as they may deem appropriate for that purpose.
martial law and/or suspension of the privilege of the writ of habeas corpus, and absent a specific
mandate for the Congress to hold a joint session in the event of concurrence, then whether or not The Court highlights the particular circumstance herein that both Houses of Congress already
to hold a joint session under such circumstances is completely within the discretion of the separately expressed support for President Duterte's Proclamation No. 216, so revocation was
Congress. not even a possibility and the provision on revocation under Article VII, Section 18 of the 1987
Constitution requiring the Congress to vote jointly in a joint session never came into operation. It
The Senate and Senate President Pimentel explained in their Consolidated Comment (Ex Abudanti will be a completely different scenario if either of the Senate or the House of Representatives, or
Cautela), that, by practice, the two Houses of the Congress must adopt a concurrent resolution to if both Houses of the Congress, resolve/s to revoke the President's proclamation of martial law
and/or suspension of the privilege of the writ of habeas corpus, in which case, Article VII, Section
18 of the 1987 Constitution shall apply and the Congress must convene in joint session to vote First, the provision specially addresses the situation when the President proclaims martial law
jointly on the revocation of the proclamation and/or suspension. Given the foregoing parameters and/or suspends the privilege of the writ of habeas corpus while the Congress is in recess. To
in applying Article VII, Section 18 of the 1987 Constitution, Justice Leonen's concern, expressed in ensure that the Congress will be able to act swiftly on the proclamation and/or suspension, the
his Concurring and Dissenting Opinion, that a deadlock may result in the future, is completely 1987 Constitution provides that it should convene within twenty-four (24) hours without need for
groundless. call. It is a whole different situation when the Congress is still in session as it can readily take up
the proclamation and/or suspension in the course of its regular sessions, as what happened in
The legislative precedent referred to by petitioners actually supports the position of the Court in these cases. Second, the provision only requires that the Congress convene without call, but it
the instant cases. On December 4, 2009, then President Macapagal-Arroyo issued Proclamation does not explicitly state that the Congress shall already convene in joint session. In fact, the
No. 1959, entitled "Proclaiming a State of Martial Law and Suspending the Privilege of the Writ of provision actually states that the Congress "convene in accordance with its rules," which can only
Habeas Corpus in the Province of Maguindanao, except for Certain Areas." The Senate, on mean the respective rules of each House as there are no standing rules for joint sessions.
December 14, 2009, adopted Resolution No. 217, entitled "Resolution Expressing the Sense of the And third, it cannot be said herein that the Congress failed to convene immediately to act on
Senate that the Proclamation of Martial Law in the Province of Maguindanao is Contrary to the Proclamation No. 216. Both Houses of the Congress promptly took action on Proclamation No.
Provisions of the 1987 Constitution." Consequently, the Senate and the House of Representatives 216, with the Senate already issuing invitations to executive officials even prior to receiving
adopted Concurrent Resolutions, i.e., Senate Concurrent Resolution No. 14 and House Concurrent President Duterte's Report, except that the two Houses of the Congress acted separately. By
Resolution No. 33, calling both Houses of the Congress to convene in joint session on December 9, initially undertaking separate actions on President Duterte's Proclamation No. 216 and making
2009 at 4:00 p.m. at the Session Hall of the House of Representatives to deliberate on their respective determination of whether to support or revoke said Proclamation, the Senate and
Proclamation No. 1959. It appears then that the two Houses of the Congress in 2009 also initially the House of Representatives were only acting in accordance with their own rules of procedure
took separate actions on President Macapagal-Arroyo's Proclamation No. 1959, with the Senate and were not in any way remiss in their constitutional duty to guard against a baseless or
eventually adopting Resolution No. 217, expressing outright its sense that the proclamation of unjustified proclamation of martial law and/or suspension of the privilege of the writ of habeas
martial law was unconstitutional and necessarily implying that such proclamation should be corpus by the President.
revoked. With one of the Houses favoring revocation, and in observation of the established
practice of the Congress, the two Houses adopted concurrent resolutions to convene in joint There is likewise no basis for petitioners' assertion that without a joint session, the public cannot
session to vote on the revocation of Proclamation No. 1959. hold the Senators and Representatives accountable for their respective positions on President
Duterte's Proclamation No. 216. Senate records completely chronicled the deliberations and the
For the same reason, the Fortun case cannot be deemed a judicial precedent for the present cases. voting by the Senators on Senate Resolution No. 49 (formerly P.S. Resolution No. 388) and P.S.
The factual background of the Fortuncase is not on all fours with these cases. Once more, the Resolution No. 390. While it is true that the House of Representatives voted on House Resolution
Court points out that in the Fortun case, the Senate expressed through Resolution No. 217 its No. 1050 viva voce, this is only in accordance with its rules. Per the Rules of the House of
objection to President Macapagal-Arroyo's Proclamation No. 1959 for being unconstitutional, and Representatives:
both the Senate and the House of Representatives adopted concurrent resolutions to convene in
joint session for the purpose of revoking said proclamation; while in the cases at bar, the Senate RULE XV
and the House of Representatives adopted Senate Resolution No. 49 and House Resolution No. Voting
1050, respectively, which expressed support for President Duterte's Proclamation No. 216, and
both Houses of the Congress voted against calling for a joint session. In addition, the fundamental Sec. 115. Manner of Voting. - The Speaker shall rise and state the motion or question that is being
issue in the Fortun case was whether there was factual basis for Proclamation No. 1959 and not put to a vote in clear, precise and simple language. The Speaker shall say "as many as are in favor,
whether it was mandatory for the Congress to convene in joint session; and even before the (as the question may be) say 'aye'". After the affirmative vote is counted, the Speaker shall say "as
Congress could vote on the revocation of Proclamation No. 1959 and the Court could resolve many as are opposed, (as the question may be) say 'nay'".
the Fortun case, President Macapagal-Arroyo already issued Proclamation No. 1963 on December
12, 2009, entitled "Proclaiming the Termination of the State of Martial Law and the Restoration of If the Speaker doubts the result of the voting or a motion to divide the House is carried, the House
the Privilege of the Writ of Habeas Corpus in the Province of Maguindanao." Furthermore, the shall divide. The Speaker shall ask those in favor to rise, to be followed by those against. If still in
word "automatic" in the Fortun case referred to the duty or power of the Congress to review the doubt of the outcome or a count by tellers is demanded, the Speaker shall name one (1) Member
proclamation of martial law and/or suspension of the privilege of the writ of habeas corpus, rather from each side of the question to count the Members in the affirmative and those in the negative.
than the joint session ofCongress.[83] After the count is reported, the Speaker shall announce the result.

Petitioners invoke the following provision also in Article VII, Section 18 of the 1987 Constitution: An abstention shall not be counted as a vote. Unless otherwise provided by the Constitution or by
"The Congress, if not in session, shall, within twenty-four hours following such proclamation or these rules, a majority of those voting, there being a quorum, shall decide the issue.
suspension convene in accordance with its rules without call." Petitioners reason that if the
Congress is not in session, it is constitutionally mandated to convene within twenty-four (24) hours Sec. 116. Nominal Voting. - Upon motion of a Member, duly approved by one-fifth (115) of the
from the President's proclamation of martial law and/or suspension of the privilege of the writ Members present, there being a quorum, nominal voting on any question may be called. In case of
of habeas corpus, then it is with all the more reason required to convene immediately if in session. nominal voting, the Secretary General shall call, in alphabetical order, the nan1es of the Members
who shall state their vote as their names are called.
The Court is not persuaded.
Sec. 117. Second Call on Nominal Voting. - A second call on nominal voting shall be made to allow SEC. 127. Executive sessions shall be held whenever a Senator so requests it and his petition has
Members who did not vote during the first call to vote. Members who fail to vote during the been duly seconded, or when the security of the State or public interest so requires. Thereupon,
second call shall no longer be allowed to vote. the President shall order that the public be excluded from the gallery and the doors of the session
Since no one moved for nominal voting on House Resolution No. 1050, then the votes of the hall be closed.
individual Representatives cannot be determined. It does not render though the proceedings
unconstitutional or invalid. The Senator who presented the motion shall then explain the reasons which he had for submitting
the same.
The Congress did not violate the right of the public to information when it did not convene in joint
session. The minutes of the executive sessions shall be recorded in a separate book. (Emphasis supplied)
From afore-quoted rules, it is clear that matters affecting the security of the state are
The Court is not swayed by petitioners' argument that by not convening in joint sessionthe considered confidential and must be discussed and deliberated upon in an executive session,
Congress violated the public's right to information because as records show, the Congress still excluding the public therefrom.
conducted deliberations on President Duterte's Proclamation No. 216, albeit separately; and the
public's right to information on matters of national security is not absolute. When such matters are That these matters are considered confidential is in accordance with settled jurisprudence that, in
being taken up in the Congress, whether in separate or joint sessions, the Congress has discretion the exercise of their right to information, the government may withhold certain types of
in the manner the proceedings will be conducted. information from the public such as state secrets regarding military, diplomatic, and other
national security matters.[85] The Court has also ruled that the Congress' deliberative process,
Petitioners contend that the Constitution requires a public deliberation process on the including information discussed and deliberated upon in an executive session, [86] may be kept out
proclamation of martial law: one that is conducted via a joint session and by a single body. They of the public's reach.
insist that the Congress must be transparent, such that there is an "open and robust debate,"
where the evaluation of the proclamation's factual bases and subsequent implementation shall be The Congress not only recognizes the sensitivity of these matters but also endeavors to preserve
openly discussed and where each member's position on the issue is heard and made known to the their confidentiality. In fact, Rule XLVII, Section 128[87] of the Rules of the Senate expressly
public. establishes a secrecy ban prohibiting all its members, including Senate officials and employees,
from divulging any of the confidential matters taken up by the Senate. A Senator found to have
The petitioners' insistence on the conduct of a "joint session" contemplates a mandatory joint violated this ban faces the possibility of expulsion from his office. [88] This is consistent with the
Congressional session where public viewing is allowed. Ethical Standards Act[89] that prohibits public officials and employees from using or divulging
"confidential or classified information officially known to them by reason of their office and not
However, based on their internal rules, each House has the discretion over the manner by which made available to the public."[90]
Congressional proceedings are to be conducted. Verily, sessions are generally open to the
public,[84] but each House may decide to hold an executive session due to the confidential nature Certainly, the factual basis of the declaration of martial law involves intelligence information,
of the subject matter to be discussed and deliberated upon. military tactics, and other sensitive matters that have an undeniable effect on national security.
Thus, to demand Congress to hold a public session during which the legislators shall openly discuss
Rule XI of the Rules of the House of Representatives provides: these matters, all the while under public scrutiny, is to effectively compel them to make sensitive
information available to everyone, without exception, and to breach the recognized policy of
Section 82. Sessions Open to the Public. - Sessions shall be open to the public. However, when the preserving these matters' confidentiality, at the risk of being sanctioned, penalized, or expelled
security of the State or the dignity of the House or any of its Members are affected by any motion from Congress altogether.
or petition being considered, the House may hold executive sessions.
That these are the separate Rules of the two Houses of the Congress does not take away from
Guests and visitors in the galleries are prohibited from using their cameras and video recorders. their persuasiveness and applicability in the event of a joint session. Since both Houses separately
Cellular phones and other similar electronic devices shall be put in silent mode. recognize the policy of preserving the confidentiality of national security matters, then in all
likelihood, they will consistently observe the same in a joint session. The nature of these matters
Section 83. Executive Sessions. - When the House decides to hold an executive session, the Speaker as confidential is not affected by the composition of the body that will deliberate upon it - whether
shall direct the galleries and hallways to be cleared and the doors closed. Only the Secretary it be the two Houses of the Congress separately or in joint session.
General, the Sergeantat-Arms and other persons specifically authorized by the House shall be
admitted to the executive session. They shall preserve the confidentiality of everything read or Also, the petitioners' theory that a regular session must be preferred over a mere briefing for
discussed in the session. (Emphasis supplied.) purposes of ensuring that the executive and military officials are placed under oath does not have
Rule XLVII of the Rules of the Senate similarly sets forth the following: merit. The Senate Rules of Procedure Governing Inquiries In Aid of Legislation[91]require that all
witnesses at executive sessions or public hearings who testify as to matters of fact shall give such
SEC. 126. The executive sessions of the Senate shall be held always behind closed doors. In such testimony under oath or affirmation. The proper implementation of this rule is within the Senate's
sessions, only the Secretary, the Sergeant-at-Arms, and/or such other persons as may be competence, which is beyond the Court's reach.
authorized by the Senate may be admitted to the session hall.
Propriety of the issuance of a writ of mandamus or certiorari
that to pray in one petition for the issuance of both a writ of mandamus and a writ of certiorari for
For mandamus to lie, there must be compliance with Rule 65, Section 3, Rules of Court, to wit: the very same act - which, in the Taada Petition, the non-convening by the two Houses of the
Congress in joint session - is contradictory, as the former involves a mandatory duty which the
SECTION 3. Petition for mandamus. - When any tribunal, corporation, board, officer or person government branch or instrumentality must perform without discretion, while the latter
unlawfully neglects the performance of an act which the law specifically enjoins as a duty resulting recognizes discretion on the part of the government branch or instrumentality but which was
from an office, trust, or station, or unlawfully excludes another from the use and enjoyment of a exercised arbitrarily or despotically. Nevertheless, if the Court is to adjudge the petition
right or office to which such other is entitled, and there is no other plain, speedy and adequate for certiorari alone, it still finds the same to be without merit. To reiterate, the two Houses of the
remedy in the ordinary course of law, the person aggrieved thereby may file a verified petition in Congress decided to no longer hold a joint session only after deliberations among their Members
the proper court, alleging the facts with certainty and praying that judgment be rendered and putting the same to vote, in accordance with their respective rules of procedure. Premises
commanding the respondent, immediately or at some other time to be specified by the court, to considered, the Congress did not gravely abuse its discretion when it did not jointly convene upon
do the act required to be done to protect the rights of the petitioner, and to pay the damages the President's issuance of Proclamation No. 216 prior to expressing its concurrence thereto.
sust;rined by the petitioner by reason of the wrongful acts of the respondent.
Jurisprudence has laid down the following requirements for a petition for mandamus to prosper: WHEREFORE, the petitions are DISMISSED for lack of merit.

[T]hus, a petition for mandamus will prosper if it is shown that the subject thereof is a ministerial SO ORDERED.
act or duty, and not purely discretionary on the part of the board, officer or person, and that
the petitioner has a welldefined, clear and certain right to warrant the grant thereof.

The difference between a ministerial and discretionary act has long been established. A purely
ministerial act or duty is one which an officer or tribunal performs in a given state of facts, in a
prescribed manner, in obedience to the mandate of a legal authority, without regard to or the
exercise of his own judgment upon the propriety or impropriety of the act done. If the law
imposes a duty upon a public officer and gives him the right to decide how or when the duty
shall be performed, such duty is discretionary and not ministerial. The duty is ministerial only
when the discharge of the same requires neither the exercise of official discretion or
judgment.[92] (Emphases added.)
It is essential to the issuance of a writ of mandamus that petitioner should have a clear legal right
to the thing demanded and it must be the imperative duty of the respondent to perform the act
required. Mandamus never issues in doubtful cases. While it may not be necessary that the
ministerial duty be absolutely expressed, it must however, be clear. The writ neither confers
powers nor imposes duties. It is simply a command to exercise a power already possessed and to
perform a duty already imposed.[93]

Although there are jurisprudential examples of the Court issuing a writ of mandamus to compel
the fulfillment of legislative duty,[94] we must distinguish the present controversy with those
previous cases. In this particular instance, the Court has no authority to compel the Senate and the
House of Representatives to convene in joint session absent a clear ministerial duty on its part to
do so under the Constitution and in complete disregard of the separate actions already undertaken
by both Houses on Proclamation No. 216, including their respective decisions to no longer hold a
joint session, considering their respective resolutions not to revoke said Proclamation.

In the same vein, there is no cause for the Court to grant a writ of certiorari.

As earlier discussed, under the Court's expanded jurisdiction, a petition tor certiorari is a proper
remedy to question the act of any branch or instrumentality of the government on the ground of
grave abuse of discretion amounting to lack or excess of jurisdiction by any branch or
instrumentality of the government, even if the latter does not exercise judicial, quasi-judicial or
ministerial functions.[95] Grave abuse of discretion implies such capricious and whimsical exercise
of judgment as to be equivalent to lack or excess of jurisdiction; in other words, power is exercised
in an arbitrary or despotic manner by reason of passion, prejudice, or personal hostility; and such
exercise is so patent or so gross as to amount to an evasion of a positive duty or to a virtual refusal
either to perform the duty enjoined or to act at all in contemplation of law.[96] It bears to mention
IN THE MATTER OF THE PETITION FOR THE WRIT OF G.R. No. 191805
WHEREFORE, the petition for writ of amparo and writ of habeas data
AMPARO AND HABEAS DATA IN FAVOR OF NORIEL H.
is GRANTED.
RODRIGUEZ,
Respondents Gen. Victor S. Ibrado, Lt. Gen. Delfin Bangit, Maj. Gen.
NORIEL H. RODRIGUEZ, Nestor Z. Ochoa, PCSupt. Ameto G. Tolentino, PSSupt. Jude W. Santos, Col.
Petitioner, Remigio M. De Vera, Lt. Col. Laurence E. Mina and 1Lt. Ryan S. Matutina, or
their replacements in their official posts if they have already vacated the same,
are ORDERED to furnish this Court within five (5) days from notice of this
- versus - decision, official or unofficial reports pertaining to petitioner covering but not
limited to intelligence reports, operation reports and provost marshal reports
GLORIA MACAPAGAL-ARROYO, GEN. VICTOR S. IBRADO, prior to, during and subsequent to September 6, 2009 made by the 5th Infantry
PDG JESUS AME VERSOZA, LT. GEN. DELFIN BANGIT, MAJ. Division, Philippine Army, its branches and subsidiaries, including the
17th Infantry Battalion, Philippine Army.
GEN. NESTOR Z. OCHOA, P/CSUPT. AMETO G. TOLENTINO,
P/SSUPT. JUDE W. SANTOS, COL. REMIGIO M. DE VERA, an The above-named respondents are also DIRECTED to refrain from
officer named MATUTINA, LT. COL. MINA, CALOG, GEORGE using the said reports in any transaction or operation of the military.
PALACPAC under the name HARRY, ANTONIO CRUZ, Necessarily, the afore-named respondents are ORDERED to expunge from the
records of the military all documents having any reference to petitioner.
ALDWIN BONG PASICOLAN and VINCENT CALLAGAN,
Respondents. Likewise, the afore-named respondents, as well as respondents Police
Director General Jesus Ame Versoza, Antonio Cruz, Aldwin Pasicolan and Vicente
x - - - - - - - - - - - - - - - - - - - - - - - - -x Callagan are DIRECTED to ensure that no further violation of petitioners rights to
IN THE MATTER OF THE PETITION FOR THE WRIT OF AMPARO G.R. No. 193160 life, liberty and security is committed against the latter or any member of his
family.
AND HABEAS DATA IN FAVOR OF NORIEL H. RODRIGUEZ,
November 15, 2011 The petition is DISMISSED with respect to President Gloria
POLICE DIR. GEN. JESUS A. VERSOZA, P/SSUPT. JUDE W. Macapagal-Arroyo on account of her presidential immunity from suit. Similarly,
SANTOS, BGEN. REMEGIO M. DE VERA, 1ST LT. RYAN S. the petition is DISMISSED with respect to respondents Calog and George
Palacpac or Harry for lack of merit.
MATUTINA, LT. COL. LAURENCE E. MINA, ANTONIO C. CRUZ,
ALDWIN C. PASICOLAN and VICENTE A. CALLAGAN, Petitioners prayer for issuance of a temporary protection order and
Petitioners, inspection order is DENIED.

- versus -
Noriel Rodriguez (Rodriguez) is petitioner in G.R. No. 191805 and respondent in G.R. No.
NORIEL H. RODRIGUEZ, 193160. He is a member of Alyansa Dagiti Mannalon Iti Cagayan (Kagimungan), a peasant
Respondent.
organization affiliated with Kilusang Magbubukid ng Pilipinas (KMP).
x - - - - - - - - - - - - - - - - - - - - - - - - - - - - - - - - - - - - - - - - - - - - - - - - - - -x

DECISION On the other hand, Gloria Macapagal-Arroyo (former President Arroyo), Police Director
General (PDG.) Jesus A. Verzosa, Police Senior Superintendent (P/SSupt.) Jude W. Santos, Brigadier
SERENO, J.:
General (Brig. Gen.) Remegio M. De Vera, First Lieutenant (1 st Lt.) Ryan S. Matutina, Lieutenant

Before this Court are two consolidated cases, namely, (1) Petition for Partial Review on Colonel (Lt. Col.) Laurence E. Mina, Antonio C. Cruz (Cruz), Aldwin C. Pasicolan (Pasicolan) and

Certiorari dated 20 April 2010 (G.R. No. 191805), and (2) Petition for Review on Certiorari dated 19 Vicente A. Callagan (Callagan) are respondents in G.R. No. 191805 and petitioners in G.R. No.

August 2010 (G.R. No. 193160).[1] Both Petitions assail the 12 April 2010 Decision of the Court of 193160. At the time the events relevant to the present Petitions occurred, former President Arroyo

Appeals, the dispositive portion of which reads: was the President of the Philippines. PDG. Verzosa, P/SSupt. Santos, Brig. Gen. De Vera, 1 st Lt.
Matutina and Lt. Col. Mina were officers of the Philippine National Police (PNP). Cruz, Pasicolan and Fearing that the food might be poisoned, he refused to eat anything. He slept on the papag while
Callagan were Special Investigators of the Commission on Human Rights (CHR) in Region II. being tied to it at the waist.[6]

Antecedent Facts On 8 September 2009, the men forced Rodriguez into a vehicle, which brought them to Bugey and
Mission. While passing houses along the way, the men asked him if his contacts lived in those
Rodriguez claims that the military tagged KMP as an enemy of the State under the Oplan
houses. When he failed to answer, a soldier pointed a gun to his head and threatened to kill him and
Bantay Laya, making its members targets of extrajudicial killings and enforced disappearances. [2]
his family. Because he remained silent, the soldiers beat him and tied him up. The vehicle returned
to the military camp at past 1:00 p.m., where he was again subjected to tactical interrogation about
On 6 September 2009, at 5:00 p.m., Rodriguez had just reached Barangay Tapel, Cagayan onboard a
the location of an NPA camp and his alleged NPA comrades. He suffered incessant mauling every
tricycle driven by Hermie Antonio Carlos (Carlos), when four men forcibly took him and forced him
time he failed to answer.[7]
into a car. Inside the vehicle were several men in civilian clothes, one of whom was holding a .45
caliber pistol. Subsequently, three more persons arrived, and one of them carried a gun at his
At dawn on 9 September 2009, soldiers armed with rifles took Rodriguez and made him their guide
side. Two men boarded the car, while the others rode on the tricycle.[3]
on their way to an NPA camp in Birao. Accompanying them was a man named Harry, who, according
to the soldiers, was an NPA member who had surrendered to the military. Harry pointed to
The men tied the hands of Rodriguez, ordered him to lie on his stomach, sat on his back and started
Rodriguez and called him a member of the NPA. He also heard Harry tell the soldiers that the latter
punching him. The car travelled towards the direction of Sta. Teresita-Mission and moved around
knew the area well and was acquainted with a man named Elvis. The soldiers loaded Rodriguez into
the area until about 2:00 a.m. During the drive, the men forced Rodriguez to confess to being a
a military truck and drove to Tabbak, Bugey. While he was walking with the soldiers, he noticed a
member of the New Peoples Army (NPA), but he remained silent. The car then entered a place that
soldier with the name tag Matutina, who appeared to be an official because the other soldiers
appeared to be a military camp. There were soldiers all over the area, and there was a banner with
addressed him as sir.[8]
the word Bravo written on it. Rodriguez later on learned that the camp belonged to the 17 th Infantry
Battalion of the Philippine Army.[4]
Upon reaching Birao on foot, the soldiers looked for and was able to locate a certain Elvis and told
him that Rodriguez had identified his whereabouts location. The soldiers forced Rodriguez to
Rodriguez was brought to a canteen, where six men confronted him, ordering him to
convince Elvis to disclose the location of the NPA camp. They brought the two to the mountains,
confess to his membership in the NPA. Due to his exhaustion, he unintentionally fell asleep. As a
where both were threatened with death. When the soldiers punched Elvis, Rodriguez told them that
result, the men hit him on the head to wake him up. After the interrogation, two of the men guarded
he would reveal the location of the NPA camp if they let Elvis go home. They finally released Elvis
him, but did not allow him to sleep.[5]
around 3:00 p.m. that day. The soldiers and Rodriguez spent the next three nights in the

In the morning of 7 September 2009, the men tied the hands of Rodriguez, blindfolded him and mountains.[9]

made him board a vehicle. While they were in transit, the soldiers repeatedly hit him in the head
On 12 September 2009, the soldiers again hit Rodriguez and forced him to identify the location of
and threatened to kill him. When the car stopped after about ten minutes, the soldiers brought him
the NPA camp. He was blindfolded and warned to get ready because they would beat him up again
to a room, removed his blindfold, and forced him to confess to being a member of the NPA. During
in the military camp. Upon arrival therein, they brought him to the same room where he had first
the interrogation, the soldiers repeatedly hit him on the head. Thereafter, he was detained inside
been detained, and two soldiers mauled him again. They repeatedly punched and kicked him. In the
the room for the entire day. The soldiers tied his stomach to a papag, and gave him rice and viand.
afternoon, they let him rest and gave him an Alaxan tablet. Thereafter, he fell asleep due to over-
fatigue and extreme body pain. The soldiers, however, hit him again. After giving him a pen and a Rodriguez was brought to another military camp, where he was ordered to sign a piece of paper
piece of paper, they ordered him to write down his request for rice from the people. When he stating that he was a surrenderee and was never beaten up. Scared and desperate to end his ordeal,
refused, the soldiers maltreated him once more.[10] he signed the paper and was warned not to report anything to the media.[17]

On 13 September 2009, the soldiers forced Rodriguez to sign documents declaring that he had Around 6:00 a.m. on 17 September 2009, the soldiers instructed petitioner to take a bath. They gave
surrendered in an encounter in Cumao, and that the soldiers did not shoot him because he became him a pair of jeans and perfume. While he was having breakfast, the two soldiers guarding him
a military asset in May. When he refused to sign the document, he received another beating. Thus, repeatedly reminded him not to disclose to the media his experience in the camp and to say instead
he was compelled to sign, but did so using a different signature to show that he was merely that he had surrendered to the military.[18]
coerced.[11]
At 9:00 a.m. on the same day, the mother and the brother of Rodriguez arrived surrounded by
The soldiers showed Rodriguez photographs of different persons and asked him if he knew the men several men. His mother, Wilma Rodriguez (Wilma), talked to Lt. Col. Mina. Rodriguez heard one of
appearing therein. When he told them that he did not recognize the individuals on the photos, the the soldiers tell Wilma that he had surrendered to the military and had long been its asset. His
soldiers instructed him to write down the name of his school and organization, but he declined. The brother, Rodel Rodriguez (Rodel), informed him that the men accompanying them were from the
soldiers then wrote something on the paper, making it appear that he was the one who had written CHR, namely, Pasicolan, Cruz and Callagan. Upon seeing Rodriguez, Cruz instructed him to lift up his
it, and forced him to sign the document. The soldiers took photographs of him while he was signing. shirt, and one of the CHR employees took photographs of his bruises.[19]
Afterwards, the soldiers forced him down, held his hands, and sat on his feet. He did not only receive
A soldier tried to convince Wilma to let Rodriguez stay in the camp for another two weeks to
another beating, but was also electrocuted. The torture lasted for about an hour.[12]
supposedly prevent the NPA from taking revenge on him. Respondent Calog also approached
At 11:00 p.m. on 15 September 2009, the soldiers brought Rodriguez to a military operation in the Rodriguez and Rodel and asked them to become military assets. Rodel refused and insisted that they
mountains, where he saw Matutina again. They all spent the night there.[13] take Rodriguez home to Manila. Again, the soldiers reminded them to refrain from facing the media.
The soldiers also told them that the latter will be taken to the Tuguegarao Airport and guarded until
In the morning of 16 September 2009, the soldiers and Rodriguez started their descent. When they
they reached home.[20]
stopped, the soldiers took his photograph and asked him to name the location of the NPA camp.
Thereafter, they all returned to the military camp. The soldiers asked him to take a bath and wear a Rodriguez and his family missed their flight. Subsequently, the soldiers accompanied them to the
white polo shirt handed to him. He was then brought to the Enrile Medical Center, where Dr. Juliet CHR office, where Rodriguez was made to sign an affidavit stating that he was neither abducted nor
Ramil (Dr. Ramil) examined him.[14] When the doctor asked him why he had bruises and contusions, tortured. Afraid and desperate to return home, he was forced to sign the document. Cruz advised
he lied and told her that he sustained them when he slipped, as he noticed a soldier observing him. him not to file a case against his abductors because they had already freed him. The CHR personnel
Dr. Ramils medical certificate indicated that he suffered from four hematomas in the epigastric area, then led him and his family to the CHR Toyota Tamaraw FX service vehicle. He noticed that a vehicle
chest and sternum.[15] with soldiers on board followed them.[21]

Back at the camp, the soldiers let Rodriguez eat with several military officials and took pictures of The Tamaraw FX pulled over and respondent 1st Lt. Matutina boarded the vehicle. Upon reaching a mall
him while he was eating with them. They also asked him to point to a map in front of him and again in Isabela, Rodriguez, his family, Callagan, 1st Lt. Matutina and two other soldiers transferred to an
took his photograph. Later, they told him that he would finally see his mother. [16] orange Toyota Revo with plate number WTG 579. Upon reaching the boundary of Nueva Ecija and
Nueva Viscaya, 1st Lt. Matutina alighted and called Rodriguez to a diner. A certain Alan approached
Rodriguez and handed him a cellphone with a SIM card. The latter and his family then left and resumed d. Ordering respondents to produce documents submitted to them regarding
their journey back home.[22] any report on Rodriguez, including operation reports and provost marshall reports of the
5th Infantry Division, the Special Operations Group of the Armed Forces of the Philippines
Rodriguez reached his house in Sta. Ana, Manila at 3:00 a.m. on 18 September 2010. Callagan and two
(AFP), prior to, on and subsequent to 6 September 2009.
soldiers went inside the house, and took photographs and a video footage thereof. The soldiers explained
that the photos and videos would serve as evidence of the fact that Rodriguez and his family were able to e. Ordering records pertinent or in any way connected to Rodriguez, which are
arrive home safely. Despite Rodriguezs efforts to confront the soldiers about their acts, they still continued in the custody of respondents, to be expunged, disabused, and forever barred from being
and only left thirty minutes later.[23] used.[27]

On 19 September 2009, Dr. Reginaldo Pamugas, a physician trained by the International On 15 December 2009, we granted the respective writs after finding that the petition
Committee on Torture and Rehabilitation, examined Rodriguez and issued a Medical Certificate stating sufficiently alleged that Rodriguez had been abducted, tortured and later released by members of
that the latter had been a victim of torture.[24] the 17th Infantry Battalion of the Philippine Army.[28] We likewise ordered respondents therein to file
a verified return on the writs on or before 22 December 2009 and to comment on the petition on or
Around 7:00 a.m. on 3 November 2010, Rodriguez and his girlfriend, Aileen Hazel Robles, noticed
before 4 January 2010.[29] Finally, we directed the Court of Appeals to hear the petition on 4 January
that several suspicious-looking men followed them at the Metro Rail Transit (MRT), in the streets
2010 and decide on the case within 10 days after its submission for decision.[30]
and on a jeepney.[25]
During the initial hearing on 4 January 2010, the Court of Appeals required the parties to
On 7 December 2009, Rodriguez filed before this Court a Petition for the Writ of Amparo and
submit affidavits and other pieces of evidence at the next scheduled hearing on 27 January 2010. [31]
Petition for the Writ of Habeas Data with Prayers for Protection Orders, Inspection of Place, and
Production of Documents and Personal Properties dated 2 December 2009.[26] The petition was filed On 8 January 2010, respondents therein, through the Office of the Solicitor General (OSG), filed their
against former President Arroyo, Gen. Ibrado, PDG. Versoza, Lt. Gen. Bangit, Major General (Maj. Return of the Writ, which was likewise considered as their comment on the petition. [32] In their
Gen.) Nestor Z. Ochoa, P/CSupt. Tolentino, P/SSupt. Santos, Col. De Vera, 1 st Lt. Matutina, Calog, Return, respondents therein alleged that Rodriguez had surrendered to the military on 28 May 2009
George Palacpac (Palacpac), Cruz, Pasicolan and Callagan. The petition prayed for the following after he had been put under surveillance and identified as Ka Pepito by former rebels. [33] According
reliefs: to his military handlers, Corporal (Cpl.) Rodel B. Cabaccan and Cpl. Julius P. Navarro, Rodriguez was
a former member of the NPA operating in Cagayan Valley.[34] Wanting to bolt from the NPA, he told
a. The issuance of the writ of amparo ordering respondents to desist from
Cpl. Cabaccan and Cpl. Navarro that he would help the military in exchange for his protection.[35]
violating Rodriguezs right to life, liberty and security.
Upon his voluntary surrender on 28 May 2009, Rodriguez was made to sign an Oath of
b. The issuance of an order to enjoin respondents from doing harm to or
Loyalty and an Agents Agreement/Contract, showing his willingness to return to society and become
approaching Rodriguez, his family and his witnesses.
a military asset.[36] Since then, he acted as a double agent, returning to the NPA to gather
information.[37] However, he feared that his NPA comrades were beginning to suspect him of being
c. Allowing the inspection of the detention areas of the Headquarters of Bravo
an infiltrator.[38] Thus, with his knowledge and consent, the soldiers planned to stage a sham
Co., 5th Infantry Division, Maguing, Gonzaga, Cagayan and another place near where
abduction to erase any suspicion about him being a double agent. [39]Hence, the abduction subject
Rodriguez was brought.
of the instant petition was conducted.[40]
a. The Court of Appeals erred in not granting the Interim Relief for
Meanwhile, Cruz, Pasicolan and Callagan filed a Consolidated Return of the Writ dated 15 January temporary protection order.
2010,[41] alleging that they had exercised extraordinary diligence in locating Rodriguez, facilitating his
b. The Court of Appeals erred in saying: (H)owever, given the nature
safe turnover to his family and securing their journey back home to Manila. More specifically, they of the writ of amparo, which has the effect of enjoining the commission by
alleged that, on 16 September 2009, after Wilma sought their assistance in ascertaining the respondents of violation to petitioners right to life, liberty and security, the
safety of petitioner is ensured with the issuance of the writ, even in the absence
whereabouts of her son, Cruz made phone calls to the military and law enforcement agencies to of an order preventing respondent from approaching petitioner.
determine his location.[42] Cruz was able to speak with Lt. Col. Mina, who confirmed that Rodriguez was
c. The Court of Appeals erred in not finding that respondent Gloria
in their custody.[43] This information was transmitted to CHR Regional Director Atty. Jimmy P. Baliga. Macapagal Arroyo had command responsibility.[53]

He, in turn, ordered Cruz, Pasicolan and Callagan to accompany Wilma to the 17th Infantry Division.[44] On the other hand, respondents therein, in their Comment dated 30 July 2010, averred:

When the CHR officers, along with Wilma and Rodel, arrived at the 17th Infantry Battalion a. The Court of Appeals properly dropped then President Gloria
Macapagal Arroyo as a party-respondent, as she may not be sued in any case
at Masin, Alcala, Cagayan, Brigade Commander Col. de Vera and Battalion Commander Lt. Col. Mina during her tenure of office or actual incumbency.
alleged that Rodriguez had become one of their assets, as evidenced by the Summary on the
b. Petitioner had not presented any adequate and competent
Surrender of Noriel Rodriguez and the latters Contract as Agent. [45] The CHR officers observed his evidence, much less substantial evidence, to establish his claim that public
respondents had violated, were violating or threatening to violate his rights to
casual and cordial demeanor with the soldiers.[46] In any case, Cruz asked him to raise his shirt to see life, liberty and security, as well as his right to privacy. Hence, he was not
if he had been subjected to any maltreatment. Cruz and Pasicolan did not see any traces of entitled to the privilege of the writs of amparo and habeas data or to the
corresponding interim reliefs (i.e. inspection order, production order and
torture. Thereafter, Rodriguez was released to his family, and they were made to sign a certification temporary protection order) provided under the rule on the writ
of amparo and the rule on the writ of habeas data.[54]
to this effect. During the signing of the document, herein CHR officers did not witness any threat,
intimidation or force employed against Rodriguez or his family. [47]

On 19 August 2010, PDG. Verzosa, P/SSupt. Santos, BGen. De Vera, 1st Lt. Matutina, Lt.
During their journey back to the home of Rodriguez, the CHR officers observed that he
Col. Mina, Cruz, Pasicolan and Callagan filed a Petition for Review on Certiorari, seeking the reversal
was very much at ease with his military escorts, especially with 1 st Lt. Matutina.[48]Neither was there
of the 12 April 2010 Decision of the Court of Appeals.[55] They alleged that Rodriguez
any force or intimidation when the soldiers took pictures of his house, as the taking of photographs
Has not presented any adequate and competent evidence, must less
was performed with Wilmas consent.[49] substantial evidence, to establish his claim that petitioners have violated, are
violating or threatening with violation his rights to life, liberty and security, as
well as his right to privacy; hence, he is not entitled to the privilege of the writs
During the hearing on 27 January 2010, the parties agreed to file additional affidavits and
of amparo and habeas data and their corresponding interim reliefs (i.e.,
position papers and to have the case considered submitted for decision after the filing of these inspection order, production order and temporary protection order) provided
under the Rule on the Writ of Amparo and the Rule on the Writ of Habeas
pleadings.[50] Data.[56]

On 12 April 2010, the Court of Appeals rendered its assailed Decision.[51] Subsequently, on 28 April
In ascertaining whether the Court of Appeals committed reversible error in issuing its
2010, respondents therein filed their Motion for Reconsideration.[52] Before the Court of Appeals assailed Decision and Resolution, the following issues must be resolved:
could resolve this Motion for Reconsideration, Rodriguez filed the instant Petition for Partial Review
on Certiorari (G.R. No. 191805), raising the following assignment of errors: I. Whether the interim reliefs prayed for by Rodriguez may be granted after the
writs of amparo and habeas data have already been issued in his favor.
II. Whether former President Arroyo should be dropped as a respondent on the basis complained of; or (c) in case the database or information contains erroneous data or information,
of the presidential immunity from suit. order its deletion, destruction or rectification.[67]

First issue: Grant of interim reliefs


III. Whether the doctrine of command responsibility can be used in amparo and
habeas data cases.
In the petition in G.R. No. 191805, Rodriguez prays for the issuance of a temporary
protection order. It must be underscored that this interim relief is only available before final
IV. Whether the rights to life, liberty and property of Rodriguez were violated or
judgment. Section 14 of the Rule on the Writ of Amparo clearly provides:
threatened by respondents in G.R. No. 191805.
Interim Reliefs. Upon filing of the petition or at anytime before final
At the outset, it must be emphasized that the writs of amparo and habeas data were judgment, the court, justice or judge may grant any of the following reliefs:

promulgated to ensure the protection of the peoples rights to life, liberty and security. [57] The rules Temporary Protection Order. The court, justice or judge, upon
motion or motu proprio, may order that the petitioner or the aggrieved party
on these writs were issued in light of the alarming prevalence of extrajudicial killings and enforced
and any member of the immediate family be protected in a government agency
disappearances.[58] The Rule on the Writ of Amparo took effect on 24 October 2007,[59] and the Rule or by an accredited person or private institution capable of keeping and
securing their safety. If the petitioner is an organization, association or
on the Writ of Habeas Data on 2 February 2008.[60] institution referred to in Section 3(c) of this Rule, the protection may be
extended to the officers involved.
The writ of amparo is an extraordinary and independent remedy that provides rapid
The Supreme Court shall accredit the persons and private
judicial relief, as it partakes of a summary proceeding that requires only substantial evidence to institutions that shall extend temporary protection to the petitioner or the
aggrieved party and any member of the immediate family, in accordance with
make the appropriate interim and permanent reliefs available to the petitioner. [61] It is not an action guidelines which it shall issue.
to determine criminal guilt requiring proof beyond reasonable doubt, or liability for damages
The accredited persons and private institutions shall comply with the
requiring preponderance of evidence, or administrative responsibility requiring substantial evidence rules and conditions that may be imposed by the court, justice or judge.
that will require full and exhaustive proceedings.[62] Rather, it serves both preventive and curative
(a) Inspection Order. The court, justice or judge, upon
roles in addressing the problem of extrajudicial killings and enforced disappearances. [63] It is verified motion and after due hearing, may order any person in possession or
control of a designated land or other property, to permit entry for the purpose
preventive in that it breaks the expectation of impunity in the commission of these offenses, and it of inspecting, measuring, surveying, or photographing the property or any
is curative in that it facilitates the subsequent punishment of perpetrators by inevitably leading to relevant object or operation thereon.

subsequent investigation and action.[64] The motion shall state in detail the place or places to be inspected. It
shall be supported by affidavits or testimonies of witnesses having personal
knowledge of the enforced disappearance or whereabouts of the aggrieved
Meanwhile, the writ of habeas data provides a judicial remedy to protect a persons right
party.
to control information regarding oneself, particularly in instances where such information is being
If the motion is opposed on the ground of national security or of the
collected through unlawful means in order to achieve unlawful ends.[65] As an independent and privileged nature of the information, the court, justice or judge may conduct a
summary remedy to protect the right to privacy especially the right to informational privacy [66] the hearing in chambers to determine the merit of the opposition.

proceedings for the issuance of the writ of habeas data does not entail any finding of criminal, civil The movant must show that the inspection order is necessary to
establish the right of the aggrieved party alleged to be threatened or violated.
or administrative culpability. If the allegations in the petition are proven through substantial
evidence, then the Court may (a) grant access to the database or information; (b) enjoin the act The inspection order shall specify the person or persons authorized
to make the inspection and the date, time, place and manner of making the
inspection and may prescribe other conditions to protect the constitutional
rights of all parties. The order shall expire five (5) days after the date of its It does not determine guilt nor pinpoint criminal culpability for the
issuance, unless extended for justifiable reasons. disappearance; rather, it determines responsibility, or at least accountability,
for the enforced disappearance for purposes of imposing the appropriate
(b) Production Order. The court, justice, or judge, upon remedies to address the disappearance. Responsibility refers to the extent the
verified motion and after due hearing, may order any person in possession, actors have been established by substantial evidence to have participated in
custody or control of any designated documents, papers, books, accounts, whatever way, by action or omission, in an enforced disappearance, as a
letters, photographs, objects or tangible things, or objects in digitized or measure of the remedies this Court shall craft, among them, the directive to
electronic form, which constitute or contain evidence relevant to the petition file the appropriate criminal and civil cases against the responsible parties in
or the return, to produce and permit their inspection, copying or the proper courts. Accountability, on the other hand, refers to the measure of
photographing by or on behalf of the movant. remedies that should be addressed to those who exhibited involvement in the
enforced disappearance without bringing the level of their complicity to the
The motion may be opposed on the ground of national security or of level of responsibility defined above; or who are imputed with
the privileged nature of the information, in which case the court, justice or knowledge relating to the enforced disappearance and who carry the burden
judge may conduct a hearing in chambers to determine the merit of the of disclosure; or those who carry, but have failed to discharge, the burden of
opposition. extraordinary diligence in the investigation of the enforced disappearance. In
all these cases, the issuance of the Writ of Amparo is justified by our primary
The court, justice or judge shall prescribe other conditions to protect goal of addressing the disappearance, so that the life of the victim is preserved
the constitutional rights of all the parties. and his liberty and security are restored.[70] (Emphasis supplied.)

(c) Witness Protection Order. The court, justice or judge,


upon motion or motu proprio, may refer the witnesses to the Department of
Justice for admission to the Witness Protection, Security and Benefit Program, Thus, in the case at bar, the Court of Appeals, in its Decision[71] found respondents in G.R. No.
pursuant to Republic Act No. 6981. 191805 with the exception of Calog, Palacpac or Harry to be accountable for the violations of Rodriguezs

The court, justice or judge may also refer the witnesses to other right to life, liberty and security committed by the 17th Infantry Battalion, 5th Infantry Division of the
government agencies, or to accredited persons or private institutions capable
Philippine Army. [72] The Court of Appeals dismissed the petition with respect to former President Arroyo
of keeping and securing their safety. (Emphasis supplied)
on account of her presidential immunity from suit. Rodriguez contends, though, that she should remain a
respondent in this case to enable the courts to determine whether she is responsible or accountable
We held in Yano v. Sanchez[68] that [t]hese provisional reliefs are intended to assist the
therefor. In this regard, it must be clarified that the Court of Appeals rationale for dropping her from
court before it arrives at a judicious determination of the amparo petition. Being interim reliefs, they
the list of respondents no longer stands since her presidential immunity is limited only to her
can only be granted before a final adjudication of the case is made. In any case, it must be underscored
incumbency.
that the privilege of the writ of amparo, once granted, necessarily entails the protection of the
aggrieved party. Thus, since we grant petitioner the privilege of the writ of amparo, there is no need to In Estrada v. Desierto,[73] we clarified the doctrine that a non-sitting President does not
issue a temporary protection order independently of the former. The order restricting respondents enjoy immunity from suit, even for acts committed during the latters tenure. We emphasize our
from going near Rodriguez is subsumed under the privilege of the writ. ruling therein that courts should look with disfavor upon the presidential privilege of immunity,
especially when it impedes the search for truth or impairs the vindication of a right, to wit:
Second issue: Presidential immunity from suit
We reject [Estradas] argument that he cannot be prosecuted for the
reason that he must first be convicted in the impeachment proceedings. The
It bears stressing that since there is no determination of administrative, civil or criminal impeachment trial of petitioner Estrada was aborted by the walkout of the
liability in amparo and habeas data proceedings, courts can only go as far as ascertaining prosecutors and by the events that led to his loss of the presidency. Indeed, on
February 7, 2001, the Senate passed Senate Resolution No. 83 Recognizing that
responsibility or accountability for the enforced disappearance or extrajudicial killing. As we held the Impeachment Court is Functus Officio. Since the Impeachment Court is now
functus officio, it is untenable for petitioner to demand that he should first be
in Razon v. Tagitis:[69]
impeached and then convicted before he can be prosecuted. The plea if
granted, would put a perpetual bar against his prosecution. Such a submission that the President was not subject to judicial process and that he should first
has nothing to commend itself for it will place him in a better situation than a be impeached and removed from office before he could be made amenable to
non-sitting President who has not been subjected to impeachment judicial proceedings. The claim was rejected by the US Supreme Court. It
proceedings and yet can be the object of a criminal prosecution. To be sure, concluded that when the ground for asserting privilege as to subpoenaed
the debates in the Constitutional Commission make it clear that when materials sought for use in a criminal trial is based only on the generalized
impeachment proceedings have become moot due to the resignation of the interest in confidentiality, it cannot prevail over the fundamental demands of
President, the proper criminal and civil cases may already be filed against him, due process of law in the fair administration of criminal justice. In the 1982 case
viz: of Nixon v. Fitzgerald, the US Supreme Court further held that the immunity of
the President from civil damages covers only official acts. Recently, the US
xxx xxx xxx Supreme Court had the occasion to reiterate this doctrine in the case of Clinton
v. Jones where it held that the US President's immunity from suits for money
Mr. Aquino. On another point, if an damages arising out of their official acts is inapplicable to unofficial
impeachment proceeding has been filed against the conduct.[74](Emphasis supplied)
President, for example, and the President resigns before
judgment of conviction has been rendered by the
impeachment court or by the body, how does it affect the
impeachment proceeding? Will it be necessarily dropped? Further, in our Resolution in Estrada v. Desierto,[75] we reiterated that the presidential
immunity from suit exists only in concurrence with the presidents incumbency:
Mr. Romulo. If we decide the purpose of
impeachment to remove one from office, then his Petitioner stubbornly clings to the contention that he is entitled to
resignation would render the case moot and academic. absolute immunity from suit. His arguments are merely recycled and we need
However, as the provision says, the criminal and civil not prolong the longevity of the debate on the subject. In our Decision, we
aspects of it may continue in the ordinary courts. exhaustively traced the origin of executive immunity in our jurisdiction and its
bends and turns up to the present time. We held that given the intent of the
This is in accord with our ruling in In Re: Saturnino Bermudez 1987 Constitution to breathe life to the policy that a public office is a public
that incumbent Presidents are immune from suit or from being brought to trust, the petitioner, as a non-sitting President, cannot claim executive
court during the period of their incumbency and tenure but not beyond. xxx immunity for his alleged criminal acts committed while a sitting
President. Petitioner's rehashed arguments including their thinly disguised
We now come to the scope of immunity that can be claimed by new spins are based on the rejected contention that he is still President, albeit,
petitioner as a non-sitting President. The cases filed against petitioner Estrada a President on leave. His stance that his immunity covers his entire term of
are criminal in character. They involve plunder, bribery and graft and office or until June 30, 2004 disregards the reality that he has relinquished the
corruption. By no stretch of the imagination can these crimes, especially presidency and there is now a new de jure President.
plunder which carries the death penalty, be covered by the alleged mantle of
immunity of a non-sitting president. Petitioner cannot cite any decision of this Petitioner goes a step further and avers that even a non-sitting
Court licensing the President to commit criminal acts and wrapping him with President enjoys immunity from suit during his term of office. He buttresses his
post-tenure immunity from liability. It will be anomalous to hold that position with the deliberations of the Constitutional Commission, viz:
immunity is an inoculation from liability for unlawful acts and omissions. The
rule is that unlawful acts of public officials are not acts of the State and the Mr. Suarez. Thank you.
officer who acts illegally is not acting as such but stands in the same footing
as any other trespasser. The last question is with reference to the
Committee's omitting in the draft proposal the immunity
Indeed, a critical reading of current literature on executive immunity provision for the President. I agree with Commissioner
will reveal a judicial disinclination to expand the privilege especially when it Nolledo that the Committee did very well in striking out
impedes the search for truth or impairs the vindication of a right. In the 1974 this second sentence, at the very least, of the original
case of US v. Nixon, US President Richard Nixon, a sitting President, was provision on immunity from suit under the 1973
subpoenaed to produce certain recordings and documents relating to his Constitution. But would the Committee members not
conversations with aids and advisers. Seven advisers of President Nixon's agree to a restoration of at least the first sentence that
associates were facing charges of conspiracy to obstruct justice and other the president shall be immune from suit during his tenure,
offenses which were committed in a burglary of the Democratic National considering that if we do not provide him that kind of an
Headquarters in Washington's Watergate Hotel during the 1972 presidential immunity, he might be spending all his time facing
campaign. President Nixon himself was named an unindicted co-conspirator.
President Nixon moved to quash the subpoena on the ground, among others,
litigations, as the President-in-exile in Hawaii is now facing
litigations almost daily? committed by subordinate members of the armed forces or other persons subject to their control
in international wars or domestic conflict.[78] Although originally used for ascertaining criminal
Fr. Bernas:
complicity, the command responsibility doctrine has also found application in civil cases for human
The reason for the omission is that we consider rights abuses.[79] In the United States, for example, command responsibility was used in Ford v.
it understood in present jurisprudence that during his
tenure he is immune from suit. Garcia and Romagoza v. Garcia civil actions filed under the Alien Tort Claims Act and the Torture
Victim Protection Act.[80] This development in the use of command responsibility in civil proceedings
Mr. Suarez:
shows that the application of this doctrine has been liberally extended even to cases not criminal in
So there is no need to express it here.
nature. Thus, it is our view that command responsibility may likewise find application in proceedings
Fr. Bernas: seeking the privilege of the writ of amparo. As we held in Rubrico:

There is no need. It was that way before. The It may plausibly be contended that command responsibility, as legal
only innovation made by the 1973 Constitution was to basis to hold military/police commanders liable for extra-legal killings, enforced
make that explicit and to add other things. disappearances, or threats, may be made applicable to this jurisdiction on the
theory that the command responsibility doctrine now constitutes a principle
Mr. Suarez: of international law or customary international law in accordance with the
incorporation clause of the Constitution.
On the understanding, I will not press for any
more query, madam President. If command responsibility were to be invoked and applied to these
I thank the Commissioner for the clarification. proceedings, it should, at most, be only to determine the author who, at the
first instance, is accountable for, and has the duty to address, the
Petitioner, however, fails to distinguish between term and tenure. The disappearance and harassments complained of, so as to enable the Court to
term means the time during which the officer may claim to hold the office as of devise remedial measures that may be appropriate under the premises to
right, and fixes the interval after which the several incumbents shall succeed one protect rights covered by the writ of amparo. As intimated earlier, however,
another. The tenure represents the term during which the incumbent actually the determination should not be pursued to fix criminal liability on respondents
holds office. The tenure may be shorter than the term for reasons within or preparatory to criminal prosecution, or as a prelude to administrative
beyond the power of the incumbent. From the deliberations, the intent of the disciplinary proceedings under existing administrative issuances, if there be
framers is clear that the immunity of the president from suit is concurrent only any.[81] (Emphasis supplied.)
with his tenure and not his term.[76] (Emphasis supplied)

Precisely in the case at bar, the doctrine of command responsibility may be used to
Applying the foregoing rationale to the case at bar, it is clear that former President Arroyo
determine whether respondents are accountable for and have the duty to address the abduction of
cannot use the presidential immunity from suit to shield herself from judicial scrutiny that would assess
Rodriguez in order to enable the courts to devise remedial measures to protect his rights. Clearly,
whether, within the context of amparo proceedings, she was responsible or accountable for the abduction
nothing precludes this Court from applying the doctrine of command responsibility
of Rodriguez.
in amparo proceedings to ascertain responsibility and accountability in extrajudicial killings and

Third issue: Command responsibility in amparo proceedings enforced disappearances. In this regard, the Separate Opinion of Justice Conchita Carpio-Morales
in Rubrico is worth noting, thus:

To attribute responsibility or accountability to former President Arroyo, Rodriguez That proceedings under the Rule on the Writ of Amparo do not
determine criminal, civil or administrative liability should not abate the
contends that the doctrine of command responsibility may be applied. As we explained in Rubrico
applicability of the doctrine of command responsibility. Taking Secretary of
v. Arroyo,[77] command responsibility pertains to the responsibility of commanders for crimes National Defense v. Manalo and Razon v. Tagitis in proper context, they do not
preclude the application of the doctrine of command responsibility to Amparo
cases. This Separate Opinion was reiterated in the recently decided case of Boac v.
Cadapan,[83] likewise penned by Justice Carpio-Morales, wherein this Court ruled:
Manalo was actually emphatic on the importance of the right to
security of person and its contemporary signification as a guarantee of Rubrico categorically denies the application of command
protection of ones rights by the government. It further stated that protection responsibility in amparo cases to determine criminal liability. The Court
includes conducting effective investigations, organization of the government maintains its adherence to this pronouncement as far as amparo cases are
apparatus to extend protection to victims of extralegal killings or enforced concerned.
disappearances, or threats thereof, and/or their families, and bringing
offenders to the bar of justice. Rubrico, however, recognizes a preliminary yet limited application
Tagitis, on the other hand, cannot be more categorical on the of command responsibility in amparo cases to instances of determining
application, at least in principle, of the doctrine of command responsibility: the responsible or accountable individuals or entities that are duty-bound to
abate any transgression on the life, liberty or security of the aggrieved party.
Given their mandates, the PNP and PNP-CIDG
officials and members were the ones who were remiss in If command responsibility were to be invoked
their duties when the government completely failed to and applied to these proceedings, it should, at most, be
exercise the extraordinary diligence that the Amparo Rule only to determine the author who, at the first instance,
requires. We hold these organizations accountable is accountable for, and has the duty to address, the
through their incumbent Chiefs who, under this disappearance and harassments complained of, so as to
Decision, shall carry the personal responsibility of seeing enable the Court to devise remedial measures that may
to it that extraordinary diligence, in the manner the be appropriate under the premises to protect rights
Amparo Rule requires, is applied in addressing the covered by the writ of amparo. As intimated earlier,
enforced disappearance of Tagitis. however, the determination should not be pursued to fix
criminal liability on respondents preparatory to criminal
Neither does Republic Act No. 9851 emasculate the applicability of prosecution, or as a prelude to administrative disciplinary
the command responsibility doctrine to Amparo cases. The short title of the proceedings under existing administrative issuances, if
law is the Philippine Act on Crimes Against International Humanitarian Law, there be any.
Genocide, and Other Crimes Against Humanity. Obviously, it should, as it did,
only treat of superior responsibility as a ground for criminal responsibility for In other words, command responsibility may be loosely applied
the crimes covered. Such limited treatment, however, is merely in keeping with in amparo cases in order to identify those accountable individuals that have
the statutes purpose and not intended to rule out the application of the the power to effectively implement whatever processes an amparo court
doctrine of command responsibility to other appropriate cases. would issue. In such application, the amparo court does not impute criminal
responsibility but merely pinpoint the superiors it considers to be in the best
Indeed, one can imagine the innumerable dangers of insulating high- position to protect the rights of the aggrieved party.
ranking military and police officers from the coverage of reliefs available under
the Rule on the Writ of Amparo. The explicit adoption of the doctrine of Such identification of the responsible and accountable superiors
command responsibility in the present case will only bring Manalo and Tagitis may well be a preliminary determination of criminal liability which, of course,
to their logical conclusion. is still subject to further investigation by the appropriate government
agency. (Emphasis supplied.)
In fine, I submit that the Court should take this opportunity to state
what the law ought to be if it truly wants to make the Writ of Amparo an
effective remedy for victims of extralegal killings and enforced disappearances
or threats thereof. While there is a genuine dearth of evidence to hold As earlier pointed out, amparo proceedings determine (a) responsibility, or the extent the
respondents Gen. Hermogenes Esperon and P/Dir. Gen. Avelino Razon actors have been established by substantial evidence to have participated in whatever way, by
accountable under the command responsibility doctrine, the ponencias
hesitant application of the doctrine itself is replete with implications abhorrent action or omission, in an enforced disappearance, and (b) accountability, or the measure of remedies
to the rationale behind the Rule on the Writ of Amparo.[82] (Emphasis supplied.)
that should be addressed to those (i) who exhibited involvement in the enforced disappearance
without bringing the level of their complicity to the level of responsibility defined above; or (ii) who
are imputed with knowledge relating to the enforced disappearance and who carry the burden of
disclosure; or (iii) those who carry, but have failed to discharge, the burden of extraordinary Responsibility in all Government Offices, particularly at all Levels of Command in the Philippine
diligence in the investigation of the enforced disappearance. Thus, although there is no National Police and other Law Enforcement Agencies (E.O. 226).[88] Under E.O. 226, a government
determination of criminal, civil or administrative liabilities, the doctrine of command responsibility official may be held liable for neglect of duty under the doctrine of command responsibility if he has
may nevertheless be applied to ascertain responsibility and accountability within these foregoing knowledge that a crime or offense shall be committed, is being committed, or has been committed
definitions. by his subordinates, or by others within his area of responsibility and, despite such knowledge, he
did not take preventive or corrective action either before, during, or immediately after its

a. Command responsibility of the President commission.[89]Knowledge of the commission of irregularities, crimes or offenses is presumed when
(a) the acts are widespread within the government officials area of jurisdiction; (b) the acts have

Having established the applicability of the doctrine of command responsibility been repeatedly or regularly committed within his area of responsibility; or (c) members of his

in amparo proceedings, it must now be resolved whether the president, as commander-in-chief of immediate staff or office personnel are involved.[90]

the military, can be held responsible or accountable for extrajudicial killings and enforced
Meanwhile, as to the issue of failure to prevent or punish, it is important to note that as the
disappearances. We rule in the affirmative.
commander-in-chief of the armed forces, the president has the power to effectively command, control

To hold someone liable under the doctrine of command responsibility, the following and discipline the military.[91]

elements must obtain:


b. Responsibility or accountability of former President Arroyo

a. the existence of a superior-subordinate relationship between the accused as


superior and the perpetrator of the crime as his subordinate; The next question that must be tackled is whether Rodriguez has proven through
substantial evidence that former President Arroyo is responsible or accountable for his
b. the superior knew or had reason to know that the crime was about to be or abduction.We rule in the negative.
had been committed; and
Rodriguez anchors his argument on a general allegation that on the basis of the Melo
c. the superior failed to take the necessary and reasonable measures to prevent Commission and the Alston Report, respondents in G.R. No. 191805 already had knowledge of and
the criminal acts or punish the perpetrators thereof.[84] information on, and should have known that a climate of enforced disappearances had been
perpetrated on members of the NPA.[92] Without even attaching, or at the very least, quoting these
The president, being the commander-in-chief of all armed forces,[85] necessarily possesses
reports, Rodriguez contends that the Melo Report points to rogue military men as the perpetrators.
control over the military that qualifies him as a
While the Alston Report states that there is a policy allowing enforced disappearances and pins the
superior within the purview of the command responsibility doctrine. [86]
blame on the President, we do not automatically impute responsibility to former President Arroyo
for each and every count of forcible disappearance.[93]Aside from Rodriguezs general averments,
On the issue of knowledge, it must be pointed out that although international tribunals
there is no piece of evidence that could establish her responsibility or accountability for his
apply a strict standard of knowledge, i.e., actual knowledge, such may nonetheless be established
abduction. Neither was there even a clear attempt to show that she should have known about the
through circumstantial evidence.[87] In the Philippines, a more liberal view is adopted and superiors
violation of his right to life, liberty or security, or that she had failed to investigate, punish or prevent
may be charged with constructive knowledge. This view is buttressed by the enactment of Executive
it.
Order No. 226, otherwise known as the Institutionalization of the Doctrine of Command
Fourth Issue: Responsibility or accountability of respondents in G.R. No. 191805 More particularly, the fact of Rodriguezs abduction was corroborated by Carlos in
his Sinumpaang Salaysay dated 16 September 2009,[99] wherein he recounted in detail the
The doctrine of totality of evidence in amparo cases was first laid down in this Courts
circumstances surrounding the victims capture.
ruling in Razon,[94] to wit:

The fair and proper rule, to our mind, is to consider all the pieces of As regards the allegation of torture, the respective Certifications of Dr. Ramil and Dr.
evidence adduced in their totality, and to consider any evidence otherwise Pamugas validate the physical maltreatment Rodriguez suffered in the hands of the soldiers of the
inadmissible under our usual rules to be admissible if it is consistent with the
admissible evidence adduced. In other words, we reduce our rules to the most 17th Infantry Battalion, 5th Infantry Division. According to the Certification dated 12 October 2009
basic test of reason i.e., to the relevance of the evidence to the issue at hand executed by Dr. Ramil,[100] she examined Rodriguez in the Alfonso Ponce Enrile Memorial District
and its consistency with all other pieces of adduced evidence. Thus, even
hearsay evidence can be admitted if it satisfies this basic minimum Hospital on 16 September 2009 and arrived at the following findings:
test.[95] (Emphasis supplied.)
FACE
- 10cm healed scar face right side
- 2cm healed scar right eyebrow (lateral area)
In the case at bar, we find no reason to depart from the factual findings of the Court of - 2cm healed scar right eye brow (median area)
Appeals, the same being supported by substantial evidence. A careful examination of the records of - 4cm x 2cm hematoma anterior chest at the sternal area right side
- 3cm x 2cm hematoma sternal area left side
this case reveals that the totality of the evidence adduced by Rodriguez indubitably prove the - 6cm x 1cm hematoma from epigastric area to ant. chest left side
- 6cm x 1cm hematoma from epigastric area to ant. chest right side
responsibility and accountability of some respondents in G.R. No. 191805 for violating his right to
- Multiple healed rashes (brownish discoloration) both forearm
life, liberty and security. - Multiple healed rashes (brownish discoloration)
- both leg arm
- hip area/lumbar area[101]
a. The totality of evidence proved by substantial evidence the responsibility or
accountability of respondents for the violation of or threat to Rodriguez right to
life, liberty and security Dr. Pamugas performed a separate medical examination of Rodriguez on 19 September
2009, the results of which confirmed that the injuries suffered by the latter were inflicted through
torture. Dr. Pamugas thus issued a Medical Report dated 23 September 2009,[102] explicitly stating
After a careful examination of the records of these cases, we are convinced that the Court
that Rodriguez had been tortured during his detention by the military, to wit:
of Appeals correctly found sufficient evidence proving that the soldiers of the 17 thInfantry Battalion,
5th Infantry Division of the military abducted Rodriguez on 6 September 2009, and detained and X. Interpretation of Findings

tortured him until 17 September 2009. The above physical and psychological findings sustained by the
subject are related to the torture and ill-treatment done to him. The multiple
Rodriguezs Sinumpaang Salaysay dated 4 December 2009 was a meticulous and circular brown to dark brown spots found on both legs and arms were due to
the insect bites that he sustained when he was forced to join twice in the
straightforward account of his horrific ordeal with the military, detailing the manner in which he was military operations. The abrasions could also be due to the conditions related
during military operations. The multiple pin-point blood spots found on his left
captured and maltreated on account of his suspected membership in the NPA.[96] His narration of
ear is a result of an unknown object placed inside his left ear. The areas of
his suffering included an exhaustive description of his physical surroundings, personal circumstances tenderness he felt during the physical examination were due to the
overwhelming punching and kicking on his body. The occasional difficulty of
and perceived observations. He likewise positively identified respondents 1 st Lt. Matutina and Lt. sleeping is a symptom experience (sic) by the subject as a result of the
Col. Mina to be present during his abduction, detention and torture, [97] and respondents Cruz, psychological trauma he encountered during his detention.

Pasicolan and Callagan as the CHR representatives who appeared during his release.[98] XI. Conclusions and Recommendations
33. Na sa kasalukuhan, hanggang ngayon ay nag-aalala pa ako sa paa
The physical injuries and psychological trauma suffered by the (sic) sa kaligtasan ng aming buong pamilya, lalo na kay Noriel; xxx[105]
subject are secondary to the torture and ill-treatment done to him while in
detention for about 11 days. The physical injuries sustained by the subject, of
which the age is compatible with the alleged date of
infliction (sic).[103] (Emphasis supplied.) Also, Rodel made the following supporting averments in his Sinumpaang Salaysay dated
3 December 2009:[106]

In assessing the weight of the Certifications, the Court of Appeals correctly relied on the 24. Na nang makita ko si Noriel, hindi sya makalakad ng diretso,
hinang-hina sya, malaki ang ipinayat at nanlalalim ang mga mata;
medical finding that the injuries suffered by Rodriguez matched his account of the maltreatment
25. Na nang makita ko ang aking kapatid ay nakaramdam ako ng awa
inflicted on him by the soldiers of the 17th Infantry Battalion, 5th Infantry Division of the Philippine
dahil nakilala ko syang masigla at masayahin;
Army. Further, the kind of injuries he sustained showed that he could not have sustained them from
26. Na ilang minuto lang ay binulugan nya ako ng Kuya, ilabas mo ako
merely falling, thus making respondents claim highly implausible. dito, papatayin nila ako.

27. Na sinabihan kami ni Lt. Col. Mina na baka pwedeng maiwan pa


Despite these medical findings that overwhelmingly supported and lent credibility to the
ng dalwang linggo ang aking kapatid sa kanila para raw ma-train sya.
allegations of Rodriguez in his Sinumpaang Salaysay, respondents in G.R. No. 191805 still stubbornly
clung to their argument that he was neither abducted nor detained. Rather, they claimed that he 28. Na hindi kami pumayag ng aking nanay; xxx[107]
was a double agent, whose relationship with the military was at all times congenial. This contention
cannot be sustained, as it is far removed from ordinary human experience.
Moreover, the Court of Appeals likewise aptly pointed out the illogical, if not outrightly

If it were true that Rodriguez maintained amicable relations with the military, then he contradictory, contention of respondents in G.R. No. 191805 that while Rodriguez had complained

should have unhesitatingly assured his family on 17 September 2009 that he was among friends. of his exhaustion from his activities as a member of the CPP-NPA, he nevertheless willingly

Instead, he vigorously pleaded with them to get him out of the military facility. In fact, in volunteered to return to his life in the NPA to become a double-agent for the military. The lower

the Sinumpaang Salaysay dated 4 December 2009[104] Wilma executed, she made the following court ruled in this manner:

averments: In the Return of the Writ, respondent AFP members alleged that
petitioner confided to his military handler, Cpl. Navarro, that petitioner could
18. Na nang Makita ko ang aking anak ay nakaramdam ako sa kanya no longer stand the hardships he experienced in the wilderness, and that he
ng awa dahil sa mukha syang pagod at malaki ang kanyang ipinayat. wanted to become an ordinary citizen again because of the empty promises of
the CPP-NPA. However, in the same Return, respondents state that petitioner
19. Na niyakap ko sya at sa aming pagkakayakap ay binulungan agreed to become a double agent for the military and wanted to re-enter the
nya ako na wag ko syang iiwan sa lugar na iyon; CPP-NPA, so that he could get information regarding the movement directly
from the source. If petitioner was tired of life in the wilderness and desired to
xxx xxx xxx become an ordinary citizen again, it defies logic that he would agree to
become an undercover agent and work alongside soldiers in the mountains
23. Na sinabihan ako ng mga sundalo na kung pwede daw ay maiwan or the wilderness he dreads to locate the hideout of his alleged NPA
muna ng dalawang linggo sa kampo ako at si Noriel para daw matrain pa si comrades.[108] (Emphasis supplied.)
Noriel sa loob ng kampo;

24. Na hindi ako pumayag na maiwan ang aking anak;


Furthermore, the appellate court also properly ruled that aside from the abduction,
xxx xxx xxx
detention and torture of Rodriguez, respondents, specifically 1 st Lt. Matutina, had violated and
threatened the formers right to security when they made a visual recording of his house, as well as
the photos of his relatives, to wit:

In the videos taken by the soldiers one of whom was respondent proceedings, responsibility may refer to the participation of the respondents, by action
Matutina in the house of petitioner on September 18, 2009, the soldiers even
went as far as taking videos of the photos of petitioners relatives hung on the or omission, in enforced disappearance.[112] Accountability, on the other hand, may attach to
wall of the house, as well as videos of the innermost part of the house. This respondents who are imputed with knowledge relating to the enforced disappearance and
Court notes that 1Lt. Matutina, by taking the said videos, did not merely
intend to make proofs of the safe arrival of petitioner and his family in their who carry the burden of disclosure; or those who carry, but have failed to discharge, the burden of
home. 1Lt. Matutina also desired to instill fear in the minds of petitioner and
extraordinary diligence in the investigation of the enforced disappearance.[113]
his family by showing them that the sanctity of their home, from then on, will
not be free from the watchful eyes of the military, permanently captured
through the medium of a seemingly innocuous cellhpone video camera. The In this regard, we emphasize our ruling in Secretary of National Defense v. Manalo[114] that
Court cannot and will not condone such act, as it intrudes into the very core of
petitioners right to security guaranteed by the fundamental law.[109] (Emphasis the right to security of a person includes the positive obligation of the government to ensure the
supplied.)
observance of the duty to investigate, viz:

Third, the right to security of person is a guarantee of protection of


Taken in their totality, the pieces of evidence adduced by Rodriguez, as well as the one's rights by the government. In the context of the writ of Amparo, this right
is built into the guarantees of the right to life and liberty under Article III,
contradictory defenses presented by respondents in G.R. No. 191805, give credence to his claim that Section 1 of the 1987 Constitution and the right to security of person (as
freedom from threat and guarantee of bodily and psychological integrity)
he had been abducted, detained and tortured by soldiers belonging to the 17 th Infantry Battalion,
under Article III, Section 2. The right to security of person in this third sense is
5th Infantry Division of the military. a corollary of the policy that the State guarantees full respect for human rights
under Article II, Section 11 of the 1987 Constitution. As the government is the
chief guarantor of order and security, the Constitutional guarantee of the rights
It must be pointed out, however, that as to respondents Cruz, Pasicolan and Callagan, to life, liberty and security of person is rendered ineffective if government does
not afford protection to these rights especially when they are under
there was no substantial evidence to show that they violated, or threatened with violation,
threat. Protection includes conducting effective investigations, organization
Rodriguezs right to life, liberty and security. Despite the dearth of evidence to show the CHR officers of the government apparatus to extend protection to victims of extralegal
killings or enforced disappearances (or threats thereof) and/or their families,
responsibility or accountability, this Court nonetheless emphasizes its criticism as regards their and bringing offenders to the bar of justice. The Inter-American Court of
capacity to recognize torture or any similar form of abuse. The CHR, being constitutionally mandated Human Rights stressed the importance of investigation in the Velasquez
Rodriguez Case, viz:
to protect human rights and investigate violations thereof,[110]should ensure that its officers are well- (The duty to investigate) must be undertaken
equipped to respond effectively to and address human rights violations. The actuations of in a serious manner and not as a mere formality
preordained to be ineffective. An investigation must have
respondents unmistakably showed their insufficient competence in facilitating and ensuring the safe an objective and be assumed by the State as its own legal
duty, not as a step taken by private interests that depends
release of Rodriguez after his ordeal.
upon the initiative of the victim or his family or upon their
offer of proof, without an effective search for the truth by
b. The failure to conduct a fair and effective investigation amounted to a violation of or the government.

threat to Rodriguez right to life, liberty and security xxx xxx xxx

Similarly, the European Court of Human Rights (ECHR) has


The Rule on the Writ of Amparo explicitly states that the violation of or threat to the right interpreted the right to security not only as prohibiting the State from
arbitrarily depriving liberty, but imposing a positive duty on the State to afford
to life, liberty and security may be caused by either an act or an omission of a public protection of the right to liberty. The ECHR interpreted the right to security of
official.[111] Moreover, in the context of amparo person under Article 5(1) of the European Convention of Human Rights in the
leading case on disappearance of persons, Kurt v. Turkey. In this case, surrounding petitioners abduction and disappearance. Instead, PDG Verzosa
the claimant's son had been arrested by state authorities and had not been disclaims accountability by merely stating that petitioner has no cause of action
seen since. The family's requests for information and investigation regarding against him. Palpable, however, is the lack of any effort on the part of PDG
his whereabouts proved futile. The claimant suggested that this was a Verzosa to effectively and aggressively investigate the violations of petitioners
violation of her son's right to security of person. The ECHR ruled, viz: right to life, liberty and security by members of the 17 th Infantry Battalion,
17th Infantry Division, Philippine Army.[116] (Emphasis supplied.)
... any deprivation of liberty must not only have
been effected in conformity with the substantive and
procedural rules of national law but must equally be in
keeping with the very purpose of Article 5, namely to Clearly, the absence of a fair and effective official investigation into the claims of
protect the individual from arbitrariness... Having Rodriguez violated his right to security, for which respondents in G.R. No. 191805 must be held
assumed control over that individual it is incumbent on
the authorities to account for his or her whereabouts. For responsible or accountable.
this reason, Article 5 must be seen as requiring the
authorities to take effective measures to safeguard
against the risk of disappearance and to conduct a Nevertheless, it must be clarified that Rodriguez was unable to establish any responsibility
prompt effective investigation into an arguable claim or accountability on the part of respondents P/CSupt. Tolentino, P/SSupt. Santos, Calog and
that a person has been taken into custody and has not
been seen since.[115] (Emphasis supplied) Palacpac. Respondent P/CSupt. Tolentino had already retired when the abduction and torture of
Rodriguez was perpetrated, while P/SSupt. Santos had already been reassigned and transferred to
the National Capital Regional Police Office six months before the subject incident occurred.
In the instant case, this Court rules that respondents in G.R. No. 191805 are responsible
Meanwhile, no sufficient allegations were maintained against respondents Calog and Palacpac.
or accountable for the violation of Rodriguezs right to life, liberty and security on account of their
abject failure to conduct a fair and effective official investigation of his ordeal in the hands of the
From all the foregoing, we rule that Rodriguez was successful in proving through
military. Respondents Gen. Ibrado, PDG. Verzosa, Lt. Gen. Bangit, Maj. Gen. Ochoa, Col. De Vera and
substantial evidence that respondents Gen. Ibrado, PDG. Verzosa, Lt. Gen. Bangit, Maj. Gen. Ochoa,
Lt. Col. Mina only conducted a perfunctory investigation, exerting no efforts to take Ramirezs
Brig. Gen. De Vera, 1st Lt. Matutina, and Lt. Col. Mina were responsible and accountable for the
account of the events into consideration. Rather, these respondents solely relied on the reports and
violation of Rodriguezs rights to life, liberty and security on the basis of (a) his abduction, detention
narration of the military. The ruling of the appellate court must be emphasized:
and torture from 6 September to 17 September 2009, and (b) the lack of any fair and effective official
In this case, respondents Ibrado, Verzosa, Bangit, Tolentino, Santos, investigation as to his allegations. Thus, the privilege of the writs of amparo and habeas data must
De Vera, and Mina are accountable, for while they were charged with the
investigation of the subject incident, the investigation they conducted and/or be granted in his favor. As a result, there is no longer any need to issue a temporary protection
relied on is superficial and one-sided. The records disclose that the military, in order, as the privilege of these writs already has the effect of enjoining respondents in G.R. No.
investigating the incident complained of, depended on the Comprehensive
Report of Noriel Rodriguez @Pepito prepared by 1Lt. Johnny Calub for the 191805 from violating his rights to life, liberty and security.
Commanding Officer of the 501st Infantry Brigade, 5th Infantry Division,
Philippine Army. Such report, however, is merely based on the narration of the
It is also clear from the above discussion that despite (a) maintaining former President
military. No efforts were undertaken to solicit petitioners version of the subject
incident and no witnesses were questioned regarding the alleged abduction of Arroyo in the list of respondents in G.R. No. 191805, and (b) allowing the application of the command
petitioner.
responsibility doctrine to amparo and habeas data proceedings, Rodriguez failed to prove through
Respondent PDG Verzosa, as Chief of the PNP, is accountable substantial evidence that former President Arroyo was responsible or accountable for the violation
because Section 24 of Republic Act No. 6975, otherwise known as the PNP Law,
specifies the PNP as the governmental office with the mandate to investigate of his rights to life, liberty and property. He likewise failed to prove through substantial evidence the
and prevent crimes, effect the arrest of criminal offenders, bring offenders to
accountability or responsibility of respondents Maj. Gen. Ochoa, Cruz, Pasicolan and Callagan.
justice and assist in their prosecution. In this case, PDG Verzosa failed to order
the police to conduct the necessary investigation to unmask the mystery
WHEREFORE, we resolve to GRANT the Petition for Partial Review in G.R. No. 191805
and DENY the Petition for Review in G.R. No. 193160. The Decision of the Court of Appeals is
hereby AFFIRMED WITH MODIFICATION.

The case is dismissed with respect to respondents former President Gloria Macapagal-
Arroyo, P/CSupt. Ameto G. Tolentino, and P/SSupt. Jude W. Santos, Calog, George Palacpac, Antonio
Cruz, Aldwin Pasicolan and Vicent Callagan for lack of merit.

This Court directs the Office of the Ombudsman (Ombudsman) and the Department of
Justice (DOJ) to take the appropriate action with respect to any possible liability or liabilities, within
their respective legal competence, that may have been incurred by respondents Gen. Victor Ibrado,
PDG. Jesus Verzosa, Lt. Gen. Delfin Bangit, Maj. Gen. Nestor Ochoa, Brig. Gen. Remegio De Vera,
1st Lt. Ryan Matutina, and Lt. Col. Laurence Mina. The Ombudsman and the DOJ are ordered to
submit to this Court the results of their action within a period of six months from receipt of this
Decision.

In the event that herein respondents no longer occupy their respective posts, the
directives mandated in this Decision and in the Court of Appeals are enforceable against the
incumbent officials holding the relevant positions. Failure to comply with the foregoing shall
constitute contempt of court.

SO ORDERED.
SALVACION A. MONSANTO, petitioner, vs. FULGENCIO S. FACTORAN, JR., respondent. in People v. Lising, Crim. Case No. 6675, October 4, 1985, that acquittal, not
absolute pardon, of a former public officer is the only ground for
FERNAN, C.J.: G.R. No. 78239 February 9, 1989 reinstatement to his former position and entitlement to payment of his
salaries, benefits and emoluments due to him during the period of his
suspension pendente lite.
The principal question raised in this petition for review is whether or not a public officer, who has
been granted an absolute pardon by the Chief Executive, is entitled to reinstatement to her former
position without need of a new appointment. In fact, in such a situation, the former public official must secure a
reappointment before he can reassume his former position. ...

In a decision rendered on March 25, 1983, the Sandiganbayan convicted petitioner Salvacion A.
Monsanto (then assistant treasurer of Calbayog City) and three other accused, of the complex Anent the civil liability of Monsanto, the Revised Penal Code expressly
crime of estafa thru falsification of public documents and sentenced them to imprisonment of four provides that "a pardon shall in no case exempt the culprit from payment of
(4) years, two (2) months and one (1) day of prision correccional as minimum, to ten (10) years and the civil indemnity imposed upon him by the sentence." (Sec. 36, par. 2).
one (1) day of prision mayor as maximum, and to pay a fine of P3,500. They were further ordered
to jointly and severally indemnify the government in the sum of P4,892.50 representing the IN VIEW OF THE FOREGOING, this Office holds that Salvacion A. Monsanto is
balance of the amount defrauded and to pay the costs proportionately. not entitled to an automatic reinstatement on the basis of the absolute
pardon granted her but must secure an appointment to her former position
Petitioner Monsanto appealed her conviction to this Court which subsequently affirmed the same. and that, notwithstanding said absolute pardon, she is liable for the civil
She then filed a motion for reconsideration but while said motion was pending, she was extended liability concomitant to her previous conviction. 3
on December 17, 1984 by then President Marcos absolute pardon which she accepted on
December 21, 1984. Her subsequent motion for reconsideration having been denied, petitioner filed the present
petition in her behalf We gave due course on October 13, 1987.
By reason of said pardon, petitioner wrote the Calbayog City treasurer requesting that she be
restored to her former post as assistant city treasurer since the same was still vacant. Petitioner's basic theory is that the general rules on pardon cannot apply to her case by reason of
the fact that she was extended executive clemency while her conviction was still pending appeal in
Petitioner's letter-request was referred to the Ministry of Finance for resolution in view of the this Court. There having been no final judgment of conviction, her employment therefore as
provision of the Local Government Code transferring the power of appointment of treasurers from assistant city treasurer could not be said to have been terminated or forfeited. In other words,
the city governments to the said Ministry. In its 4th Indorsement dated March 1, 1985, the Finance without that final judgment of conviction, the accessory penalty of forfeiture of office did not
Ministry ruled that petitioner may be reinstated to her position without the necessity of a new attach and the status of her employment remained "suspended." More importantly, when pardon
appointment not earlier than the date she was extended the absolute pardon. It also directed the was issued before the final verdict of guilt, it was an acquittal because there was no offense to
city treasurer to see to it that the amount of P4,892.50 which the Sandiganbayan had required to speak of. In effect, the President has declared her not guilty of the crime charged and has
be indemnified in favor of the government as well as the costs of the litigation, be satisfied. 1 accordingly dismissed the same. 4

Seeking reconsideration of the foregoing ruling, petitioner wrote the Ministry on April 17, 1985 It is well to remember that petitioner had been convicted of the complex crime of estafa thru
stressing that the full pardon bestowed on her has wiped out the crime which implies that her falsification of public documents and sentenced to imprisonment of four years, two months and
service in the government has never been interrupted and therefore the date of her reinstatement one day of prision correccional as minimum, to ten years and one day of prision mayor as
should correspond to the date of her preventive suspension which is August 1, 1982; that she is maximum. The penalty of prision mayor carries the accessory penalties of temporary absolute
entitled to backpay for the entire period of her suspension; and that she should not be required to disqualification and perpetual special disqualification from the right of suffrage, enforceable during
pay the proportionate share of the amount of P4,892.50. 2 the term of the principal penalty. 5 Temporary absolute disqualification bars the convict from
public office or employment, such disqualification to last during the term of the sentence. 6 Even if
the offender be pardoned, as to the principal penalty, the accessory penalties remain unless the
The Ministry of Finance, however, referred petitioner's letter to the Office of the President for same have been expressly remitted by the pardon. 7 The penalty of prision correccional carries, as
further review and action. On April 15, 1986, said Office, through Deputy Executive Secretary one of its accessory penalties, suspension from public office. 8
Fulgenio S. Factoran, Jr. held:
The propositions earlier advanced by petitioner reveal her inadequate understanding of the nature
We disagree with both the Ministry of Finance and the petitioner because, as of pardon and its legal consequences. This is not totally unexpected considering that the
borne out by the records, petitioner was convicted of the crime for which she authorities on the subject have not been wholly consistent particularly in describing the effects of
was accused. In line with the government's crusade to restore absolute pardon.
honesty in public service, this Office adopts, as a juridical guide (Miranda v.
Imperial, 77 Phil. 1966), the Resolution of the Sandiganbayan, 2nd Division,
The benign mercy of pardon is of British origin, conceived to temper the gravity of the King's A pardon reaches both the punishment prescribed for the offense and the
wrath. But Philippine jurisprudence on the subject has been largely influenced by American case guilt of the offender; and when the pardon is full, it releases the punishment
law. and blots out of existence the guilt, so that in the eye of the law the offender
is as innocent as if he had never committed the offense. If granted before
Pardon is defined as "an act of grace, proceeding from the power entrusted with the execution of conviction, it prevents any of the penalties and disabilities, consequent upon
the laws, which exempts the individual, on whom it is bestowed, from the punishment the law conviction, from attaching; if granted after conviction, it removes the
inflicts for a crime he has committed. It is the private, though official act of the executive penalties and disabilities and restores him to all his civil rights; it makes him,
magistrate, delivered to the individual for whose benefit it is intended, and not communicated as it were, a new man, and gives him a new credit and capacity. 14
officially to the Court. ... A pardon is a deed, to the validity of which delivery is essential, and
delivery is not complete without acceptance." 8-a Such generalities have not been universally accepted, recognized or approved. 15 The modern
trend of authorities now rejects the unduly broad language of the Garland case (reputed to be
At the time the antecedents of the present case took place, the pardoning power was governed by perhaps the most extreme statement which has been made on the effects of a pardon). To our
the 1973 Constitution as amended in the April 7, 1981 plebiscite. The pertinent provision reads: mind, this is the more realistic approach. While a pardon has generally been regarded as blotting
out the existence of guilt so that in the eye of the law the offender is as innocent as though he
never committed the offense, it does not operate for all purposes. The very essence of a pardon is
The President may, except in cases of impeachment, grant reprieves, forgiveness or remission of guilt. Pardon implies guilt. It does not erase the fact of the commission
commutations and pardons, remit fines and forfeitures, and with the of the crime and the conviction thereof. It does not wash out the moral stain. It involves
concurrence of the Batasang Pambansa, grant amnesty. 9 forgiveness and not forgetfulness. 16

The 1981 amendments had deleted the earlier rule that clemency could be extended only upon The better considered cases regard full pardon (at least one not based on the offender's
final conviction, implying that clemency could be given even before conviction. Thus, petitioner's innocence) as relieving the party from all the punitive consequences of his criminal act, including
unconditional pardon was granted even as her appeal was pending in the High Court. It is worth the disqualifications or disabilities based on the finding of guilt. 17 But it relieves him from nothing
mentioning that under the 1987 Constitution, the former limitation of final conviction was more. "To say, however, that the offender is a "new man", and "as innocent as if he had never
restored. But be that as it may, it is our view that in the present case, it is not material when the committed the offense;" is to ignore the difference between the crime and the criminal. A person
pardon was bestowed, whether before or after conviction, for the result would still be the same. adjudged guilty of an offense is a convicted criminal, though pardoned; he may be deserving of
Having accepted the pardon, petitioner is deemed to have abandoned her appeal and her punishment, though left unpunished; and the law may regard him as more dangerous to society
unreversed conviction by the Sandiganbayan assumed the character of finality. than one never found guilty of crime, though it places no restraints upon him following his
conviction." 18
Having disposed of that preliminary point, we proceed to discuss the effects of a full and absolute
pardon in relation to the decisive question of whether or not the plenary pardon had the effect of A pardon looks to the future. It is not retrospective. 19 It makes no amends for the past. It affords
removing the disqualifications prescribed by the Revised Penal Code. no relief for what has been suffered by the offender. It does not impose upon the government any
obligation to make reparation for what has been suffered. "Since the offense has been established
In Pelobello v. Palatino, 10 We find a reiteration of the stand consistently adopted by the courts on by judicial proceedings, that which has been done or suffered while they were in force is presumed
the various consequences of pardon: "... we adopt the broad view expressed in Cristobal v. to have been rightfully done and justly suffered, and no satisfaction for it can be required." 20 This
Labrador, G.R. No. 47941, December 7, 1940, that subject to the limitations imposed by the would explain why petitioner, though pardoned, cannot be entitled to receive backpay for lost
Constitution, the pardoning power cannot be restricted or controlled by legislative action; that an earnings and benefits.
absolute pardon not only blots out the crime committed but removes all disabilities resulting from
the conviction. ... (W)e are of the opinion that the better view in the light of the constitutional Petitioner maintains that when she was issued absolute pardon, the Chief Executive declared her
grant in this jurisdiction is not to unnecessarily restrict or impair the power of the Chief Executive not guilty of the crime for which she was convicted. In the case of State v. Hazzard, 21 we find this
who, after an inquiry into the environmental facts, should be at liberty to atone the rigidity of the strong observation: "To assume that all or even a major number of pardons are issued because of
law to the extent of relieving completely the party ... concerned from the accessory and resultant innocence of the recipients is not only to indict our judicial system, but requires us to assume that
disabilities of criminal conviction. which we all know to be untrue. The very act of forgiveness implies the commission of wrong, and
that wrong has been established by the most complete method known to modern civilization.
The Pelobello v. Palatino and Cristobal v. Labrador cases, 11 and several others 12 show the Pardons may relieve from the disability of fines and forfeitures attendant upon a conviction, but
unmistakable application of the doctrinal case of Ex Parte Garland, 13 whose sweeping they cannot erase the stain of bad character, which has been definitely fixed. 22
generalizations to this day continue to hold sway in our jurisprudence despite the fact that much
of its relevance has been downplayed by later American decisions. In this ponencia, the Court wishes to stress one vital point: While we are prepared to concede that
pardon may remit all the penal consequences of a criminal indictment if only to give meaning to
Consider the following broad statements: the fiat that a pardon, being a presidential prerogative, should not be circumscribed by legislative
action, we do not subscribe to the fictitious belief that pardon blots out the guilt of an individual
and that once he is absolved, he should be treated as if he were innocent. For whatever may have SO ORDERED.
been the judicial dicta in the past, we cannot perceive how pardon can produce such "moral
changes" as to equate a pardoned convict in character and conduct with one who has constantly
maintained the mark of a good, law-abiding citizen.

Pardon cannot mask the acts constituting the crime. These are "historical" facts which, despite the
public manifestation of mercy and forgiveness implicit in pardon, "ordinary, prudent men will take
into account in their subsequent dealings with the actor." 23

Pardon granted after conviction frees the individual from all the penalties and legal disabilities and
restores him to all his civil rights. But unless expressly grounded on the person's innocence (which
is rare), it cannot bring back lost reputation for honesty, integrity and fair dealing. 24 This must be
constantly kept in mind lest we lose track of the true character and purpose of the privilege.

Thus, notwithstanding the expansive and effusive language of the Garland case, we are in full
agreement with the commonly-held opinion that pardon does not ipso facto restore a convicted
felon to public office necessarily relinquished or forfeited by reason of the conviction 25 although
such pardon undoubtedly restores his eligibility for appointment to that office. 26

The rationale is plainly evident Public offices are intended primarily for the collective protection,
safety and benefit of the common good. They cannot be compromised to favor private interests.
To insist on automatic reinstatement because of a mistaken notion that the pardon virtually
acquitted one from the offense of estafa would be grossly untenable. A pardon, albeit full and
plenary, cannot preclude the appointing power from refusing appointment to anyone deemed to
be of bad character, a poor moral risk, or who is unsuitable by reason of the pardoned conviction.

For petitioner Monsanto, this is the bottom line: the absolute disqualification or ineligibility from
public office forms part of the punishment prescribed by the Revised Penal Code for estafa thru
falsification of public documents. It is clear from the authorities referred to that when her guilt and
punishment were expunged by her pardon, this particular disability was likewise removed.
Henceforth, petitioner may apply for reappointment to the office which was forfeited by reason of
her conviction. And in considering her qualifications and suitability for the public post, the facts
constituting her offense must be and should be evaluated and taken into account to determine
ultimately whether she can once again be entrusted with public funds. Stated differently, the
pardon granted to petitioner has resulted in removing her disqualification from holding public
employment but it cannot go beyond that. To regain her former post as assistant city treasurer,
she must re-apply and undergo the usual procedure required for a new appointment.

Finally, petitioner has sought exemption from the payment of the civil indemnity imposed upon
her by the sentence. The Court cannot oblige her. Civil liability arising from crime is governed by
the Revised Penal Code. It subsists notwithstanding service of sentence, or for any reason the
sentence is not served by pardon, amnesty or commutation of sentence. Petitioner's civil liability
may only be extinguished by the same causes recognized in the Civil Code, namely: payment, loss
of the thing due, remission of the debt, merger of the rights of creditor and debtor, compensation
and novation. 27

WHEREFORE, the assailed resolution of former Deputy Executive Secretary Fulgencio S. Factoran,
Jr., dated April 15, 1986, is AFFIRMED. No costs.
G.R. No. 206666 January 21, 2015 Moreover, in accordance with Section 2 of Republic Act No. 7080, as amended by Republic Act No.
7659, the Court hereby declares the forfeiture in favor of the government of the following:
ATTY. ALICIA RISOS-VIDAL, Petitioner, ALFREDO S. LIM Petitioner-Intervenor,
vs. (1) The total amount of Five Hundred Forty[-]Two Million Seven Hundred Ninety[-]One
COMMISSION ON ELECTIONS and JOSEPH EJERCITO ESTRADA, Respondents. Thousand Pesos (545,291,000.00), with interest and income earned, inclusive of the
amount of Two Hundred Million Pesos (200,000,000.00), deposited in the name and
DECISION account of the Erap Muslim Youth Foundation.

LEONARDO-DE CASTRO, J.: (2) The amount of One Hundred Eighty[-]Nine Million Pesos (189,000,000.00), inclusive
of interests and income earned, deposited in the Jose Velarde account.

Before the Court are (1) a Petition for Certiorari filed under Rule 64, in relation to Rule 65, both of
the Revised Rules of Court, by Atty. Alicia Risos-Vidal (Risos-Vidal), which essentially prays for the (3) The real property consisting of a house and lot dubbed as "Boracay Mansion" located
issuance of the writ of certiorari annulling and setting aside the April 1, 20131 and April 23, at #100 11th Street, New Manila, Quezon City.
20132 Resolutions of the Commission on Elections (COMELEC), Second Division and En bane,
respectively, in SPA No. 13-211 (DC), entitled "Atty. Alicia Risos-Vidal v. Joseph Ejercito Estrada" for The cash bonds posted by accused Jose "Jinggoy" Estrada and Atty. Edward S. Serapio are hereby
having been rendered with grave abuse of discretion amounting to lack or excess of jurisdiction; ordered cancelled and released to the said accused or their duly authorized representatives upon
and (2) a Petition-in-Intervention3 filed by Alfredo S. Lim (Lim), wherein he prays to be declared the presentation of the original receipt evidencing payment thereof and subject to the usual
2013 winning candidate for Mayor of the City of Manila in view of private respondent former accounting and auditing procedures. Likewise, the hold-departure orders issued against the said
President Joseph Ejercito Estradas (former President Estrada) disqualification to run for and hold accused are hereby recalled and declared functus oficio.4
public office.
On October 25, 2007, however, former President Gloria Macapagal Arroyo (former President
The Facts Arroyo) extended executive clemency, by way of pardon, to former President Estrada. The full text
of said pardon states:
The salient facts of the case are as follows:
MALACAAN PALACE
On September 12, 2007, the Sandiganbayan convicted former President Estrada, a former MANILA
President of the Republic of the Philippines, for the crime of plunder in Criminal Case No. 26558,
entitled "People of the Philippines v. Joseph Ejercito Estrada, et al." The dispositive part of the By the President of the Philippines
graft courts decision reads:
PARDON
WHEREFORE, in view of all the foregoing, judgment is hereby rendered in Criminal Case No. 26558
finding the accused, Former President Joseph Ejercito Estrada, GUILTY beyond reasonable doubt of WHEREAS, this Administration has a policy of releasing inmates who have reached the age of
the crime of PLUNDER, defined in and penalized by Republic Act No. 7080, as amended. On the seventy (70),
other hand, for failure of the prosecution to prove and establish their guilt beyond reasonable
doubt, the Court finds the accused Jose "Jinggoy" Estrada and Atty. Edward S. Serapio NOT GUILTY
of the crime of plunder, and accordingly, the Court hereby orders their ACQUITTAL. WHEREAS, Joseph Ejercito Estrada has been under detention for six and a half years,

The penalty imposable for the crime of plunder under Republic Act No. 7080, as amended by WHEREAS, Joseph Ejercito Estrada has publicly committed to no longer seek any elective position
Republic Act No. 7659, is Reclusion Perpetua to Death. There being no aggravating or mitigating or office,
circumstances, however, the lesser penalty shall be applied in accordance with Article 63 of the
Revised Penal Code. Accordingly, the accused Former President Joseph Ejercito Estrada is hereby IN VIEW HEREOF and pursuant to the authority conferred upon me by the Constitution, I hereby
sentenced to suffer the penalty of Reclusion Perpetua and the accessory penalties of civil grant executive clemency to JOSEPH EJERCITO ESTRADA, convicted by the Sandiganbayan of
interdiction during the period of sentence and perpetual absolute disqualification. Plunder and imposed a penalty of Reclusion Perpetua. He is hereby restored to his civil and
political rights.
The period within which accused Former President Joseph Ejercito Estrada has been under
detention shall be credited to him in full as long as he agrees voluntarily in writing to abide by the The forfeitures imposed by the Sandiganbayan remain in force and in full, including all writs and
same disciplinary rules imposed upon convicted prisoners. processes issued by the Sandiganbayan in pursuance hereof, except for the bank account(s) he
owned before his tenure as President.
Upon acceptance of this pardon by JOSEPH EJERCITO ESTRADA, this pardon shall take effect. relied on Section 40 of the Local Government Code (LGC), in relation to Section 12 of the Omnibus
Election Code (OEC), which state respectively, that:
Given under my hand at the City of Manila, this 25th Day of October, in the year of Our Lord, two
thousand and seven. Sec. 40, Local Government Code:

Gloria M. Arroyo (sgd.) SECTION 40. Disqualifications.- The following persons are disqualified from running for any elective
local position:
By the President:
(a) Those sentenced by final judgment for an offense involving moral turpitude or for an
IGNACIO R. BUNYE (sgd.) offense punishable by one (1) year or more of imprisonment, within two (2) years after
Acting Executive Secretary5 serving sentence; (b) Those removed from office as a result of an administrative case;

On October 26, 2007, at 3:35 p.m., former President Estrada "received and accepted" 6 the pardon (c) Those convicted by final judgment for violating the oath of allegiance to the Republic;
by affixing his signature beside his handwritten notation thereon.
(d) Those with dual citizenship;
On November 30, 2009, former President Estrada filed a Certificate of Candidacy7 for the position
of President. During that time, his candidacy earned three oppositions in the COMELEC: (1) SPA (e) Fugitives from justice in criminal or nonpolitical cases here or abroad;
No. 09-024 (DC), a "Petition to Deny Due Course and Cancel Certificate of Candidacy" filed by Rev.
Elly Velez B. Lao Pamatong, ESQ; (2) SPA No. 09-028 (DC), a petition for "Disqualification as (f) Permanent residents in a foreign country or those who have acquired the right to
Presidential Candidate" filed by Evilio C. Pormento (Pormento); and (3) SPA No. 09-104 (DC), a reside abroad and continue to avail of the same right after the effectivity of this Code;
"Petition to Disqualify Estrada Ejercito, Joseph M.from Running as President due to Constitutional and
Disqualification and Creating Confusion to the Prejudice of Estrada, Mary Lou B" filed by Mary Lou
Estrada. In separate Resolutions8 dated January 20, 2010 by the COMELEC, Second Division,
however, all three petitions were effectively dismissed on the uniform grounds that (i) the (g) The insane or feeble minded. (Emphasis supplied.)
Constitutional proscription on reelection applies to a sitting president; and (ii) the pardon granted
to former President Estrada by former President Arroyo restored the formers right to vote and be Sec. 12, Omnibus Election Code:
voted for a public office. The subsequent motions for reconsideration thereto were denied by the
COMELEC En banc. Section 12. Disqualifications. - Any person who has been declared by competent authority insane
or incompetent, or has been sentenced by final judgmentfor subversion, insurrection, rebellion, or
After the conduct of the May 10, 2010 synchronized elections, however, former President Estrada for any offense for which he has been sentenced to a penalty of more than eighteen months or for
only managed to garner the second highest number of votes. a crime involving moral turpitude, shall be disqualified to be a candidate and to hold any public
office, unless he has been given plenary pardon or granted amnesty. (Emphases supplied.)
Of the three petitioners above-mentioned, only Pormento sought recourse to this Court and filed a
petition for certiorari, which was docketed as G.R. No. 191988, entitled "Atty. Evilio C. Pormento v. In a Resolution dated April 1, 2013,the COMELEC, Second Division, dismissed the petition for
Joseph ERAP Ejercito Estrada and Commission on Elections." But in a Resolution9 dated August disqualification, the fallo of which reads:
31, 2010, the Court dismissed the aforementioned petition on the ground of mootness considering
that former President Estrada lost his presidential bid. WHEREFORE, premises considered, the instant petition is hereby DISMISSED for utter lack of
merit.12
On October 2, 2012, former President Estrada once more ventured into the political arena, and
filed a Certificate of Candidacy,10 this time vying for a local elective post, that ofthe Mayor of the The COMELEC, Second Division, opined that "[h]aving taken judicial cognizance of the consolidated
City of Manila. resolution for SPA No. 09-028 (DC) and SPA No. 09-104 (DC) and the 10 May 2010 En Banc
resolution affirming it, this Commission will not be labor the controversy further. Moreso, [Risos-
On January 24, 2013, Risos-Vidal, the petitioner in this case, filed a Petition for Disqualification Vidal] failed to present cogent proof sufficient to reverse the standing pronouncement of this
against former President Estrada before the COMELEC. The petition was docketed as SPA No. 13- Commission declaring categorically that [former President Estradas] right to seek public office has
211 (DC). Risos Vidal anchored her petition on the theory that "[Former President Estrada] is been effectively restored by the pardon vested upon him by former President Gloria M. Arroyo.
Disqualified to Run for Public Office because of his Conviction for Plunder by the Sandiganbayan in Since this Commission has already spoken, it will no longer engage in disquisitions of a settled
Criminal Case No. 26558 entitled People of the Philippines vs. Joseph Ejercito Estrada Sentencing matter lest indulged in wastage of government resources."13
Him to Suffer the Penalty of Reclusion Perpetuawith Perpetual Absolute Disqualification." 11 She
The subsequent motion for reconsideration filed by Risos-Vidal was denied in a Resolution dated The Issue
April 23, 2013.
Though raising five seemingly separate issues for resolution, the petition filed by Risos-Vidal
On April 30, 2013, Risos-Vidal invoked the Courts jurisdiction by filing the present petition. She actually presents only one essential question for resolution by the Court, that is, whether or not
presented five issues for the Courts resolution, to wit: the COMELEC committed grave abuse of discretion amounting to lack or excess of jurisdiction in
ruling that former President Estrada is qualified to vote and be voted for in public office as a result
I. RESPONDENT COMELEC COMMITTED GRAVE ABUSE OF DISCRETION AMOUNTING TO of the pardon granted to him by former President Arroyo.
LACK OR EXCESS OF JURISDICTION IN HOLDING THAT RESPONDENT ESTRADAS PARDON
WAS NOT CONDITIONAL; In her petition, Risos-Vidal starts her discussion by pointing out that the pardon granted to former
President Estrada was conditional as evidenced by the latters express acceptance thereof. The
II. RESPONDENT COMELEC COMMITTED GRAVE ABUSE OF DISCRETION AMOUNTING TO "acceptance," she claims, is an indication of the conditional natureof the pardon, with the
LACK OR EXCESS OF JURISDICTION IN NOT FINDING THAT RESPONDENT ESTRADA IS condition being embodied in the third Whereas Clause of the pardon, i.e., "WHEREAS, Joseph
DISQUALIFIED TO RUN AS MAYOR OF MANILA UNDER SEC. 40 OF THE LOCAL Ejercito Estrada has publicly committed to no longer seek any elective position or office." She
GOVERNMENTCODE OF 1991 FOR HAVING BEEN CONVICTED OF PLUNDER, AN OFFENSE explains that the aforementioned commitment was what impelled former President Arroyo to
INVOLVING MORAL TURPITUDE; pardon former President Estrada, without it, the clemency would not have been extended. And
any breach thereof, that is, whenformer President Estrada filed his Certificate of Candidacy for
President and Mayor of the City of Manila, he breached the condition of the pardon; hence, "he
III. RESPONDENT COMELEC COMMITTED GRAVE ABUSE OF DISCRETION AMOUNTING TO ought to be recommitted to prison to serve the unexpired portion of his sentence x x x and
LACK OR EXCESS OF JURISDICTION IN DISMISSING THE PETITION FOR DISQUALIFICATION disqualifies him as a candidate for the mayoralty [position] of Manila." 16
ON THE GROUND THAT THE CASE INVOLVES THE SAME OR SIMILAR ISSUES IT ALREADY
RESOLVED IN THE CASES OF "PORMENTO VS. ESTRADA", SPA NO. 09-028 (DC) AND IN
"RE: PETITION TO DISQUALIFY ESTRADA EJERCITO, JOSEPH M. FROM RUNNING AS Nonetheless, Risos-Vidal clarifies that the fundamental basis upon which former President Estrada
PRESIDENT, ETC.," SPA NO. 09-104 (DC); mustbe disqualified from running for and holding public elective office is actually the proscription
found in Section 40 of the LGC, in relation to Section 12 ofthe OEC. She argues that the crime of
plunder is both an offense punishable by imprisonment of one year or more and involving moral
IV. RESPONDENT COMELEC COMMITTED GRAVE ABUSE OF DISCRETION AMOUNTING TO turpitude; such that former President Estrada must be disqualified to run for and hold public
LACK OR EXCESS OF JURISDICTION IN NOT RULING THAT RESPONDENT ESTRADAS elective office.
PARDON NEITHER RESTORED HIS RIGHT OF SUFFRAGE NOR REMITTED HIS PERPETUAL
ABSOLUTE DISQUALIFICATION FROM SEEKING PUBLIC OFFICE; and
Even with the pardon granted to former President Estrada, however, Risos-Vidal insists that the
same did not operate to make available to former President Estrada the exception provided under
V. RESPONDENT COMELEC COMMITTED GRAVE ABUSE OF DISCRETION AMOUNTING TO Section 12 of the OEC, the pardon being merely conditional and not absolute or plenary.
LACK OR EXCESS OF JURISDICTION IN NOT HAVING EXERCISED ITS POWER TO MOTU Moreover, Risos-Vidal puts a premium on the ostensible requirements provided under Articles 36
PROPRIO DISQUALIFY RESPONDENT ESTRADA IN THE FACE OF HIS PATENT and 41 of the Revised Penal Code, to wit:
DISQUALIFICATION TO RUN FOR PUBLIC OFFICE BECAUSE OF HIS PERPETUAL AND
ABSOLUTE DISQUALIFICATION TO SEEK PUBLIC OFFICE AND TO VOTE RESULTING FROM
HIS CRIMINAL CONVICTION FOR PLUNDER.14 ART. 36. Pardon; its effects. A pardon shall not work the restoration of the right to hold
publicoffice, or the right of suffrage, unless such rights be expressly restored by the terms of the
pardon.
While this case was pending beforethe Court, or on May 13, 2013, the elections were conducted as
scheduled and former President Estrada was voted into office with 349,770 votes cast in his favor.
The next day, the local board of canvassers proclaimed him as the duly elected Mayor of the City A pardon shall in no case exempt the culprit from the payment of the civil indemnity imposed
of Manila. upon him by the sentence.

On June 7, 2013, Lim, one of former President Estradas opponents for the position of Mayor, xxxx
moved for leave to intervene in this case. His motion was granted by the Court in a
Resolution15 dated June 25, 2013. Lim subscribed to Risos-Vidals theory that former President ART. 41. Reclusion perpetua and reclusion temporal Their accessory penalties. The penalties of
Estrada is disqualified to run for and hold public office as the pardon granted to the latter failed to reclusion perpetua and reclusion temporal shall carry with them that of civil interdiction for life or
expressly remit his perpetual disqualification. Further, given that former President Estrada is during the period of the sentence as the case may be, and that of perpetual absolute
disqualified to run for and hold public office, all the votes obtained by the latter should be disqualification which the offender shall suffer even though pardoned as to the principal penalty,
declared stray, and, being the second placer with 313,764 votes to his name, he (Lim) should be unless the same shall have been expressly remitted in the pardon. (Emphases supplied.)
declared the rightful winning candidate for the position of Mayor of the City of Manila.
She avers that in view of the foregoing provisions of law, it is not enough that a pardon makes a For his part, former President Estrada presents the following significant arguments to defend his
general statement that such pardon carries with it the restoration of civil and political rights. By stay in office: that "the factual findings of public respondent COMELEC, the Constitutional body
virtue of Articles 36 and 41, a pardon restoring civil and political rights without categorically mandated to administer and enforce all laws relative to the conduct of the elections, [relative to
making mention what specific civil and political rights are restored "shall not work to restore the the absoluteness of the pardon, the effects thereof, and the eligibility of former President Estrada
right to hold public office, or the right of suffrage; nor shall it remit the accessory penalties of civil to seek public elective office] are binding [and conclusive] on this Honorable Supreme Court;" that
interdiction and perpetual absolute disqualification for the principal penalties of reclusion he "was granted an absolute pardon and thereby restored to his full civil and political rights,
perpetua and reclusion temporal."17 In other words, she considers the above constraints as including the right to seek public elective office such as the mayoral (sic) position in the City of
mandatory requirements that shun a general or implied restoration of civil and political rights in Manila;" that "the majority decision in the case of Salvacion A. Monsanto v. Fulgencio S. Factoran,
pardons. Jr.,which was erroneously cited by both Vidal and Lim as authority for their respective claims, x x x
reveal that there was no discussion whatsoever in the ratio decidendi of the Monsanto case as to
Risos-Vidal cites the concurring opinions of Associate Justices Teodoro R. Padilla and Florentino P. the alleged necessity for an expressed restoration of the right to hold public office in the pardon
Feliciano in Monsanto v. Factoran, Jr.18 to endorse her position that "[t]he restoration of the right as a legal prerequisite to remove the subject perpetual special disqualification;" that moreover,
to hold public office to one who has lost such right by reason of conviction in a criminal case, but the "principal question raised in this Monsanto case is whether or not a public officer, who has
subsequently pardoned, cannot be left to inference, no matter how intensely arguable, but must been granted an absolute pardon by the Chief Executive, is entitled to reinstatement toher former
be statedin express, explicit, positive and specific language." position without need of a new appointment;" that his "expressed acceptance [of the pardon] is
not proof that the pardon extended to [him] is conditional and not absolute;" that this case is a
mere rehash of the casesfiled against him during his candidacy for President back in 2009-2010;
Applying Monsantoto former President Estradas case, Risos-Vidal reckons that "such express that Articles 36 and 41 of the Revised Penal Code "cannot abridge or diminish the pardoning
restoration is further demanded by the existence of the condition in the [third] [W]hereas [C]lause power of the President expressly granted by the Constitution;" that the text of the pardon granted
of the pardon x x x indubitably indicating that the privilege to hold public office was not restored to him substantially, if not fully, complied with the requirement posed by Article 36 of the Revised
to him."19 Penal Code as it was categorically stated in the said document that he was "restored to his civil and
political rights;" that since pardon is an act of grace, it must be construed favorably in favor of the
On the other hand, the Office ofthe Solicitor General (OSG) for public respondent COMELEC, grantee;25 and that his disqualification will result in massive disenfranchisement of the hundreds of
maintains that "the issue of whether or not the pardon extended to [former President Estrada] thousands of Manileos who voted for him.26
restored his right to run for public office had already been passed upon by public respondent
COMELEC way back in 2010 via its rulings in SPA Nos. 09-024, 09-028 and 09-104, there is no The Court's Ruling
cogent reason for it to reverse its standing pronouncement and declare [former President Estrada]
disqualified to run and be voted as mayor of the City of Manila in the absence of any new
argument that would warrant its reversal. To be sure, public respondent COMELEC correctly The petition for certiorari lacks merit.
exercised its discretion in taking judicial cognizance of the aforesaid rulings which are known toit
and which can be verified from its own records, in accordance with Section 2, Rule 129 of the Rules Former President Estrada was granted an absolute pardon that fully restored allhis civil and
of Court on the courts discretionary power to take judicial notice of matters which are of public political rights, which naturally includes the right to seek public elective office, the focal point of
knowledge, orare capable of unquestionable demonstration, or ought to be known to them this controversy. The wording of the pardon extended to former President Estrada is complete,
because of their judicial functions."20 unambiguous, and unqualified. It is likewise unfettered by Articles 36 and 41 of the Revised Penal
Code. The only reasonable, objective, and constitutional interpretation of the language of the
Further, the OSG contends that "[w]hile at first glance, it is apparent that [former President pardon is that the same in fact conforms to Articles 36 and 41 of the Revised Penal Code. Recall
Estradas] conviction for plunder disqualifies him from running as mayor of Manila under Section that the petition for disqualification filed by Risos-Vidal against former President Estrada, docketed
40 of the [LGC], the subsequent grant of pardon to him, however, effectively restored his right to as SPA No. 13-211 (DC), was anchored on Section 40 of the LGC, in relation to Section 12 of the
run for any public office."21 The restoration of his right to run for any public office is the exception OEC, that is, having been convicted of a crime punishable by imprisonment of one year or more,
to the prohibition under Section 40 of the LGC, as provided under Section 12 of the OEC. As to the and involving moral turpitude, former President Estrada must be disqualified to run for and hold
seeming requirement of Articles 36 and 41 of the Revised Penal Code, i.e., the express public elective office notwithstanding the fact that he is a grantee of a pardon that includes a
restoration/remission of a particular right to be stated in the pardon, the OSG asserts that "an statement expressing "[h]e is hereby restored to his civil and political rights." Risos-Vidal theorizes
airtight and rigid interpretation of Article 36 and Article 41 of the [RPC] x x x would be stretching that former President Estrada is disqualified from running for Mayor of Manila inthe May 13, 2013
too much the clear and plain meaning of the aforesaid provisions."22 Lastly, taking into Elections, and remains disqualified to hold any local elective post despite the presidential pardon
consideration the third Whereas Clause of the pardon granted to former President Estrada, the extended to him in 2007 by former President Arroyo for the reason that it (pardon) did not
OSG supports the position that it "is not an integral part of the decree of the pardon and cannot expressly provide for the remission of the penalty of perpetual absolute disqualification,
therefore serve to restrict its effectivity."23 particularly the restoration of his (former President Estrada) right to vote and bevoted upon for
public office. She invokes Articles 36 and 41 of the Revised Penal Code as the foundations of her
theory.
Thus, the OSG concludes that the "COMELEC did not commit grave abuse of discretion amounting
to lack or excess of jurisdiction in issuing the assailed Resolutions."24
It is insisted that, since a textual examination of the pardon given to and accepted by former corruption" that would be enumerated and defined by Congress through the enactment of a law.
President Estrada does not actually specify which political right is restored, it could be inferred that The following is the pertinent portion lifted from the Record of the Commission (Vol. II):
former President Arroyo did not deliberately intend to restore former President Estradas rights of
suffrage and to hold public office, orto otherwise remit the penalty of perpetual absolute MR. ROMULO. I ask that Commissioner Tan be recognized to introduce an amendment on the
disqualification. Even if her intention was the contrary, the same cannot be upheld based on the same section.
pardons text.

THE PRESIDENT. Commissioner Tan is recognized.


The pardoning power of the President cannot be limited by legislative action.

SR. TAN. Madam President, lines 7 to 9 state:


The 1987 Constitution, specifically Section 19 of Article VII and Section 5 of Article IX-C, provides
that the President of the Philippines possesses the power to grant pardons, along with other acts
of executive clemency, to wit: However, the power to grant executive clemency for violations of corrupt practices laws may be
limited by legislation.

Section 19. Except in cases of impeachment, or as otherwise provided in this Constitution, the
President may grant reprieves, commutations, and pardons, and remit fines and forfeitures, after I suggest that this be deletedon the grounds that, first, violations of corrupt practices may include
conviction by final judgment. a very little offense like stealing 10; second, which I think is more important, I get the impression,
rightly or wrongly, that subconsciously we are drafting a constitution on the premise that all our
future Presidents will bebad and dishonest and, consequently, their acts will be lacking in wisdom.
He shall also have the power to grant amnesty with the concurrence of a majority of all the Therefore, this Article seems to contribute towards the creation of an anti-President Constitution
Members of the Congress. or a President with vast responsibilities but no corresponding power except to declare martial law.
Therefore, I request that these lines be deleted.
xxxx
MR. REGALADO. Madam President,may the Committee react to that?
Section 5. No pardon, amnesty, parole, or suspension of sentence for violation of election laws,
rules, and regulations shall be granted by the President without the favorable recommendation of THE PRESIDENT. Yes, please.
the Commission.

MR. REGALADO. This was inserted here on the resolution of Commissioner Davide because of the
It is apparent from the foregoing constitutional provisions that the only instances in which the fact that similar to the provisions on the Commission on Elections, the recommendation of that
President may not extend pardon remain to be in: (1) impeachment cases; (2) cases that have not Commission is required before executive clemency isgranted because violations of the election
yet resulted in a final conviction; and (3) cases involving violations of election laws, rules and laws go into the very political life of the country.
regulations in which there was no favorable recommendation coming from the COMELEC.
Therefore, it can be argued that any act of Congress by way of statute cannot operate to delimit
the pardoning power of the President. With respect to violations of our Corrupt Practices Law, we felt that it is also necessary to have
that subjected to the same condition because violation of our Corrupt Practices Law may be of
such magnitude as to affect the very economic systemof the country. Nevertheless, as a
In Cristobal v. Labrador27 and Pelobello v. Palatino,28 which were decided under the 1935 compromise, we provided here that it will be the Congress that will provide for the classification as
Constitution,wherein the provision granting pardoning power to the President shared similar to which convictions will still require prior recommendation; after all, the Congress could take into
phraseology with what is found in the present 1987 Constitution, the Court then unequivocally account whether or not the violation of the Corrupt Practices Law is of such magnitude as to affect
declared that "subject to the limitations imposed by the Constitution, the pardoning power cannot the economic life of the country, if it is in the millions or billions of dollars. But I assume the
be restricted or controlled by legislative action." The Court reiterated this pronouncement in Congress in its collective wisdom will exclude those petty crimes of corruption as not to require
Monsanto v. Factoran, Jr.29 thereby establishing that, under the present Constitution, "a pardon, any further stricture on the exercise of executive clemency because, of course, there is a whale of
being a presidential prerogative, should not be circumscribed by legislative action." Thus, it is a difference if we consider a lowly clerk committing malversation of government property or funds
unmistakably the long-standing position of this Court that the exercise of the pardoning power is involving one hundred pesos. But then, we also anticipate the possibility that the corrupt practice
discretionary in the President and may not be interfered with by Congress or the Court, except of a public officer is of such magnitude as to have virtually drained a substantial portion of the
only when it exceeds the limits provided for by the Constitution. treasury, and then he goes through all the judicial processes and later on, a President who may
have close connections with him or out of improvident compassion may grant clemency under
This doctrine of non-diminution or non-impairment of the Presidents power of pardon by acts of such conditions. That is why we left it to Congress to provide and make a classification based on
Congress, specifically through legislation, was strongly adhered to by an overwhelming majority of substantial distinctions between a minor act of corruption or an act of substantial proportions. SR.
the framers of the 1987 Constitution when they flatly rejected a proposal to carve out an TAN. So, why do we not just insert the word GROSS or GRAVE before the word "violations"?
exception from the pardoning power of the President in the form of "offenses involving graft and
MR. REGALADO. We feel that Congress can make a better distinction because "GRAVE" or "GROSS" Originally, my limitation was, it should be with the concurrence of the convicting court, but the
can be misconstrued by putting it purely as a policy. Committee left it entirely to the legislature to formulate the mechanics at trying, probably, to
distinguish between grave and less grave or serious cases of violation of the Anti-Graft and Corrupt
MR. RODRIGO. Madam President. Practices Act. Perhaps this is now the best time, since we have strengthened the Article on
Accountability of Public Officers, to accompany it with a mandate that the Presidents right to
grant executive clemency for offenders or violators of laws relating to the concept of a public
THE PRESIDENT. Commissioner Rodrigo is recognized. office may be limited by Congress itself.

MR. RODRIGO. May I speak in favor of the proposed amendment? MR. SARMIENTO. Madam President.

THE PRESIDENT. Please proceed. THE PRESIDENT. Commissioner Sarmiento is recognized.

MR. RODRIGO. The power to grant executive clemency is essentially an executive power, and that MR. SARMIENTO. May I briefly speak in favor of the amendment by deletion.
is precisely why it is called executive clemency. In this sentence, which the amendment seeks to
delete, an exception is being made. Congress, which is the legislative arm, is allowed to intrude
into this prerogative of the executive. Then it limits the power of Congress to subtract from this Madam President, over and over again, we have been saying and arguing before this
prerogative of the President to grant executive clemency by limiting the power of Congress to only Constitutional Commission that we are emasculating the powers of the presidency, and this
corrupt practices laws. There are many other crimes more serious than these. Under this provision to me is another clear example of that. So, I speak against this provision. Even the 1935
amendment, Congress cannot limit the power of executive clemency in cases of drug addiction and and the 1973 Constitutions do not provide for this kind of provision.
drug pushing which are very, very serious crimes that can endanger the State; also, rape with
murder, kidnapping and treason. Aside from the fact that it is a derogation of the power of the I am supporting the amendment by deletion of Commissioner Tan.
President to grant executive clemency, it is also defective in that it singles out just one kind of
crime. There are far more serious crimes which are not included. MR. ROMULO. Commissioner Tingson would like to be recognized.

MR. REGALADO. I will just make one observation on that. We admit that the pardoning power is THE PRESIDENT. Commissioner Tingson is recognized.
anexecutive power. But even in the provisions on the COMELEC, one will notice that
constitutionally, it is required that there be a favorable recommendation by the Commission on
Elections for any violation of election laws. MR. TINGSON. Madam President, I am also in favor of the amendment by deletion because I am in
sympathy with the stand of Commissioner Francisco "Soc" Rodrigo. I do believe and we should
remember that above all the elected or appointed officers of our Republic, the leader is the
At any rate, Commissioner Davide, as the principal proponent of that and as a member of the President. I believe that the country will be as the President is, and if we systematically emasculate
Committee, has explained in the committee meetings we had why he sought the inclusion of this the power of this presidency, the time may come whenhe will be also handcuffed that he will no
particular provision. May we call on Commissioner Davide to state his position. longer be able to act like he should be acting.

MR. DAVIDE. Madam President. So, Madam President, I am in favor of the deletion of this particular line.

THE PRESIDENT. Commissioner Davide is recognized. MR. ROMULO. Commissioner Colayco would like to be recognized.

MR. DAVIDE. I am constrained to rise to object to the proposal. We have just approved the Article THE PRESIDENT. Commissioner Colayco is recognized.
on Accountability of Public Officers. Under it, it is mandated that a public office is a public trust,
and all government officers are under obligation to observe the utmost of responsibility, integrity,
loyalty and efficiency, to lead modest lives and to act with patriotism and justice. MR. COLAYCO. Thank you very much, Madam President.

In all cases, therefore, which would go into the verycore of the concept that a public office is a I seldom rise here to object to or to commend or to recommend the approval of proposals, but
public trust, the violation is itself a violation not only of the economy but the moral fabric of public now I find that the proposal of Commissioner Tan is worthy of approval of this body.
officials. And that is the reason we now want that if there is any conviction for the violation of the
Anti-Graft and Corrupt Practices Act, which, in effect, is a violation of the public trust character of Why are we singling out this particular offense? There are other crimes which cast a bigger blot on
the public office, no pardon shall be extended to the offender, unless some limitations are the moral character of the public officials.
imposed.
Finally, this body should not be the first one to limit the almost absolute power of our Chief VOTING
Executive in deciding whether to pardon, to reprieve or to commute the sentence rendered by the
court. THE PRESIDENT. As many as are in favor of the proposed amendment of Commissioner Tan to
delete the last sentence of Section 17 appearing on lines 7, 8 and 9, please raise their hand.
I thank you. (Several Members raised their hand.)

THE PRESIDENT. Are we ready to vote now? As many as are against, please raise their hand. (Few Members raised their hand.)

MR. ROMULO. Commissioner Padilla would like to be recognized, and after him will be The results show 34 votes in favor and 4 votes against; the amendment is approved.30 (Emphases
Commissioner Natividad. supplied.)

THE PRESIDENT. Commissioner Padilla is recognized. The proper interpretation of Articles

MR. PADILLA. Only one sentence, Madam President. The Sandiganbayan has been called the Anti- 36 and 41 of the Revised Penal Code.
Graft Court, so if this is allowed to stay, it would mean that the Presidents power togrant pardon
or reprieve will be limited to the cases decided by the Anti-Graft Court, when as already stated, The foregoing pronouncements solidify the thesis that Articles 36 and 41 of the Revised Penal
there are many provisions inthe Revised Penal Code that penalize more serious offenses. Code cannot, in any way, serve to abridge or diminish the exclusive power and prerogative of the
President to pardon persons convicted of violating penal statutes.
Moreover, when there is a judgment of conviction and the case merits the consideration of the
exercise of executive clemency, usually under Article V of the Revised Penal Code the judge will The Court cannot subscribe to Risos-Vidals interpretation that the said Articles contain specific
recommend such exercise of clemency. And so, I am in favor of the amendment proposed by textual commands which must be strictly followed in order to free the beneficiary of presidential
Commissioner Tan for the deletion of this last sentence in Section 17. grace from the disqualifications specifically prescribed by them.

THE PRESIDENT. Are we ready to vote now, Mr. Floor Leader? Again, Articles 36 and 41 of the Revised Penal Code provides:

MR. NATIVIDAD. Just one more. ART. 36. Pardon; its effects. A pardon shall not work the restoration of the right to hold
publicoffice, or the right of suffrage, unless such rights be expressly restored by the terms of the
THE PRESIDENT. Commissioner Natividad is recognized. pardon.

MR. NATIVIDAD. I am also against this provision which will again chip more powers from the A pardon shall in no case exempt the culprit from the payment of the civil indemnity imposed
President. In case of other criminals convicted in our society, we extend probation to them while in upon him by the sentence.
this case, they have already been convicted and we offer mercy. The only way we can offer mercy
to them is through this executive clemency extended to them by the President. If we still close this xxxx
avenue to them, they would be prejudiced even worse than the murderers and the more vicious
killers in our society. I do not think they deserve this opprobrium and punishment under the new
Constitution. ART. 41. Reclusion perpetua and reclusion temporal Their accessory penalties. The penalties of
reclusion perpetua and reclusion temporal shall carry with them that of civil interdiction for life or
during the period of the sentence as the case may be, and that of perpetual absolute
I am in favor of the proposed amendment of Commissioner Tan. disqualification which the offender shall suffer even though pardoned as to the principal penalty,
unless the same shall have been expressly remitted in the pardon. (Emphases supplied.)
MR. ROMULO. We are ready tovote, Madam President.
A rigid and inflexible reading of the above provisions of law, as proposed by Risos-Vidal, is
THE PRESIDENT. Is this accepted by the Committee? unwarranted, especially so if it will defeat or unduly restrict the power of the President to grant
executive clemency.
MR. REGALADO. The Committee, Madam President, prefers to submit this to the floor and also
because of the objection of the main proponent, Commissioner Davide. So we feel that the It is well-entrenched in this jurisdiction that where the words of a statute are clear, plain, and free
Commissioners should vote on this question. from ambiguity, it must be given its literal meaning and applied without attempted interpretation.
Verba legis non est recedendum. From the words of a statute there should be no departure.31 It is
this Courts firm view that the phrase in the presidential pardon at issue which declares that A close scrutiny of the text of the pardon extended to former President Estrada shows that both
former President Estrada "is hereby restored to his civil and political rights" substantially complies the principal penalty of reclusion perpetua and its accessory penalties are included in the pardon.
with the requirement of express restoration. The first sentence refers to the executive clemency extended to former President Estrada who was
convicted by the Sandiganbayan of plunder and imposed a penalty of reclusion perpetua. The
The Dissent of Justice Marvic M.V.F. Leonen agreed with Risos Vidal that there was no express latter is the principal penalty pardoned which relieved him of imprisonment. The sentence that
remission and/or restoration of the rights of suffrage and/or to hold public office in the pardon followed, which states that "(h)e is hereby restored to his civil and political rights," expressly
granted to former President Estrada, as required by Articles 36 and 41 of the Revised Penal Code. remitted the accessory penalties that attached to the principal penalty of reclusion perpetua.
Hence, even if we apply Articles 36 and 41 of the Revised Penal Code, it is indubitable from the
textof the pardon that the accessory penalties of civil interdiction and perpetual absolute
Justice Leonen posits in his Dissent that the aforementioned codal provisions must be followed by disqualification were expressly remitted together with the principal penalty of reclusion perpetua.
the President, as they do not abridge or diminish the Presidents power to extend clemency. He
opines that they do not reduce the coverage of the Presidents pardoning power. Particularly, he
states: In this jurisdiction, the right toseek public elective office is recognized by law as falling under the
whole gamut of civil and political rights.

Articles 36 and 41 refer only to requirements of convention or form. They only provide a
procedural prescription. They are not concerned with areas where or the instances when the Section 5 of Republic Act No. 9225,34 otherwise known as the "Citizenship Retention and
President may grant pardon; they are only concerned with how he or she is to exercise such power Reacquisition Act of 2003," reads as follows:
so that no other governmental instrumentality needs to intervene to give it full effect.
Section 5. Civil and Political Rights and Liabilities. Those who retain or reacquire Philippine
All that Articles 36 and 41 do is prescribe that, if the President wishes to include in the pardon the citizenship under this Act shall enjoy full civil and political rights and be subject to all attendant
restoration of the rights of suffrage and to hold public office, or the remission of the accessory liabilities and responsibilities under existing laws of the Philippines and the following conditions:
penalty of perpetual absolute disqualification,he or she should do so expressly. Articles 36 and 41 (1) Those intending to exercise their right of suffrage must meet the requirements under Section 1,
only ask that the President state his or her intentions clearly, directly, firmly, precisely, and Article V of the Constitution, Republic Act No. 9189, otherwise known as "The Overseas Absentee
unmistakably. To belabor the point, the President retains the power to make such restoration or Voting Act of 2003" and other existing laws;
remission, subject to a prescription on the manner by which he or she is to state it. 32
(2) Those seeking elective public office in the Philippines shall meet the qualifications for
With due respect, I disagree with the overbroad statement that Congress may dictate as to how holding such public office as required by the Constitution and existing laws and, at the
the President may exercise his/her power of executive clemency. The form or manner by which time of the filing of the certificate of candidacy, make a personal and sworn
the President, or Congress for that matter, should exercise their respective Constitutional powers renunciation of any and all foreign citizenship before any public officer authorized to
or prerogatives cannot be interfered with unless it is so provided in the Constitution. This is the administer an oath;
essence of the principle of separation of powers deeply ingrained in our system of government
which "ordains that each of the three great branches of government has exclusive cognizance of (3) Those appointed to any public office shall subscribe and swear an oath of allegiance
and is supreme in matters falling within its own constitutionally allocated sphere."33 Moreso, this to the Republic of the Philippines and its duly constituted authorities prior to their
fundamental principle must be observed if noncompliance with the form imposed by one branch assumption of office: Provided, That they renounce their oath of allegiance to the
on a co-equal and coordinate branch will result into the diminution of an exclusive Constitutional country where they took that oath; (4) Those intending to practice their profession in
prerogative. the Philippines shall apply with the proper authority for a license or permit to engage in
such practice; and
For this reason, Articles 36 and 41 of the Revised Penal Code should be construed in a way that will
give full effect to the executive clemency granted by the President, instead of indulging in an (5) That right to vote or be elected or appointed to any public office in the Philippines
overly strict interpretation that may serve to impair or diminish the import of the pardon which cannot be exercised by, or extended to, those who:
emanated from the Office of the President and duly signed by the Chief Executive himself/herself.
The said codal provisions must be construed to harmonize the power of Congress to define crimes (a) are candidates for or are occupying any public office in the country of
and prescribe the penalties for such crimes and the power of the President to grant executive which theyare naturalized citizens; and/or
clemency. All that the said provisions impart is that the pardon of the principal penalty does
notcarry with it the remission of the accessory penalties unless the President expressly includes
said accessory penalties in the pardon. It still recognizes the Presidential prerogative to grant (b) are in active service as commissioned or non commissioned officers in the
executive clemency and, specifically, to decide to pardon the principal penalty while excluding its armed forces of the country which they are naturalized citizens. (Emphases
accessory penalties or to pardon both. Thus, Articles 36 and 41 only clarify the effect of the pardon supplied.)
so decided upon by the President on the penalties imposedin accordance with law.
No less than the International Covenant on Civil and Political Rights, to which the Philippines is a 41 of the Revised Penal Code that will in effect require the President to use a statutorily prescribed
signatory, acknowledges the existence of said right. Article 25(b) of the Convention states: Article language in extending executive clemency, even if the intent of the President can otherwise be
25 deduced from the text or words used in the pardon. Furthermore, as explained above, the pardon
here is consistent with, and not contrary to, the provisions of Articles 36 and 41.
Every citizen shall have the right and the opportunity, without any of the distinctions mentioned in
Article 2 and without unreasonable restrictions: The disqualification of former President Estrada under Section 40 of the LGC in relation to Section
12 of the OEC was removed by his acceptance of the absolute pardon granted to him.
xxxx
Section 40 of the LGC identifies who are disqualified from running for any elective local position.
(b) To vote and to be electedat genuine periodic elections which shall be by universal and equal Risos-Vidal argues that former President Estrada is disqualified under item (a), to wit:
suffrage and shall be held by secret ballot, guaranteeing the free expression of the will of the
electors[.] (Emphasis supplied.) (a) Those sentenced by final judgment for an offense involving moral turpitude or for an offense
punishable by one (1) year or more of imprisonment, within two (2) years after serving sentence[.]
Recently, in Sobejana-Condon v. Commission on Elections,35 the Court unequivocally referred to (Emphasis supplied.)
the right to seek public elective office as a political right, to wit:
Likewise, Section 12 of the OEC provides for similar prohibitions, but it provides for an exception,
Stated differently, it is an additional qualification for elective office specific only to Filipino citizens to wit:
who re-acquire their citizenship under Section 3 of R.A. No. 9225. It is the operative act that
restores their right to run for public office. The petitioners failure to comply there with in Section 12. Disqualifications. x x x unless he has been given plenary pardon or granted amnesty.
accordance with the exact tenor of the law, rendered ineffectual the Declaration of Renunciation (Emphasis supplied.)
of Australian Citizenship she executed on September 18, 2006. As such, she is yet to regain her
political right to seek elective office. Unless she executes a sworn renunciation of her Australian As earlier stated, Risos-Vidal maintains that former President Estradas conviction for plunder
citizenship, she is ineligible to run for and hold any elective office in the Philippines. (Emphasis disqualifies him from running for the elective local position of Mayor of the City of Manila under
supplied.) Section 40(a) of the LGC. However, the subsequent absolute pardon granted to former President
Estrada effectively restored his right to seek public elective office. This is made possible by reading
Thus, from both law and jurisprudence, the right to seek public elective office is unequivocally Section 40(a) of the LGC in relation to Section 12 of the OEC.
considered as a political right. Hence, the Court reiterates its earlier statement that the pardon
granted to former President Estrada admits no other interpretation other than to mean that, upon While it may be apparent that the proscription in Section 40(a) of the LGC is worded in absolute
acceptance of the pardon granted tohim, he regained his FULL civil and political rights including terms, Section 12 of the OEC provides a legal escape from the prohibition a plenary pardon or
the right to seek elective office. amnesty. In other words, the latter provision allows any person who has been granted plenary
pardon or amnesty after conviction by final judgment of an offense involving moral turpitude, inter
On the other hand, the theory of Risos-Vidal goes beyond the plain meaning of said penal alia, to run for and hold any public office, whether local or national position.
provisions; and prescribes a formal requirement that is not only unnecessary but, if insisted upon,
could be in derogation of the constitutional prohibition relative to the principle that the exercise of Take notice that the applicability of Section 12 of the OEC to candidates running for local elective
presidential pardon cannot be affected by legislative action. positions is not unprecedented. In Jalosjos, Jr. v. Commission on Elections,37 the Court
acknowledged the aforementioned provision as one of the legal remedies that may be availed of
Risos-Vidal relied heavily on the separate concurring opinions in Monsanto v. Factoran, Jr. 36 to to disqualify a candidate in a local election filed any day after the last day for filing of certificates of
justify her argument that an absolute pardon must expressly state that the right to hold public candidacy, but not later than the date of proclamation.38 The pertinent ruling in the Jalosjos case is
office has been restored, and that the penalty of perpetual absolute disqualification has been quoted as follows:
remitted.
What is indisputably clear is that false material representation of Jalosjos is a ground for a petition
This is incorrect. under Section 78. However, since the false material representation arises from a crime penalized
by prision mayor, a petition under Section 12 ofthe Omnibus Election Code or Section 40 of the
Her reliance on said opinions is utterly misplaced. Although the learned views of Justices Teodoro Local Government Code can also be properly filed. The petitioner has a choice whether to anchor
R. Padilla and Florentino P. Feliciano are to be respected, they do not form partof the controlling his petition on Section 12 or Section 78 of the Omnibus Election Code, or on Section 40 of the Local
doctrine nor to be considered part of the law of the land. On the contrary, a careful reading of the Government Code. The law expressly provides multiple remedies and the choice of which remedy
majority opinion in Monsanto, penned by no less than Chief Justice Marcelo B. Fernan, reveals no to adopt belongs to petitioner.39 (Emphasis supplied.)
statement that denotes adherence to a stringent and overly nuanced application of Articles 36 and
The third preambular clause of the pardon did not operate to make the pardon conditional. Where the scope and import of the executive clemency extended by the President is in issue, the
Court must turn to the only evidence available to it, and that is the pardon itself. From a detailed
Contrary to Risos-Vidals declaration, the third preambular clause of the pardon, i.e., "[w]hereas, review ofthe four corners of said document, nothing therein gives an iota of intimation that the
Joseph Ejercito Estrada has publicly committed to no longer seek any elective position or office," third Whereas Clause is actually a limitation, proviso, stipulation or condition on the grant of the
neither makes the pardon conditional, nor militate against the conclusion that former President pardon, such that the breach of the mentioned commitment not to seek public office will result ina
Estradas rights to suffrage and to seek public elective office have been restored. revocation or cancellation of said pardon. To the Court, what it is simply is a statement of fact or
the prevailing situation at the time the executive clemency was granted. It was not used as a
condition to the efficacy orto delimit the scope of the pardon.
This is especially true as the pardon itself does not explicitly impose a condition or limitation,
considering the unqualified use of the term "civil and political rights"as being restored.
Jurisprudence educates that a preamble is not an essential part of an act as it is an introductory or Even if the Court were to subscribe to the view that the third Whereas Clausewas one of the
preparatory clause that explains the reasons for the enactment, usually introduced by the word reasons to grant the pardon, the pardon itself does not provide for the attendant consequence of
"whereas."40 Whereas clauses do not form part of a statute because, strictly speaking, they are not the breach thereof. This Court will be hard put to discern the resultant effect of an eventual
part of the operative language of the statute.41 In this case, the whereas clause at issue is not an infringement. Just like it will be hard put to determine which civil or political rights were restored if
integral part of the decree of the pardon, and therefore, does not by itself alone operate to make the Court were to take the road suggested by Risos-Vidal that the statement "[h]e is hereby
the pardon conditional or to make its effectivity contingent upon the fulfilment of the restored to his civil and political rights" excludes the restoration of former President Estradas
aforementioned commitment nor to limit the scope of the pardon. rights to suffrage and to hold public office. The aforequoted text ofthe executive clemency granted
does not provide the Court with any guide asto how and where to draw the line between the
included and excluded political rights.
On this matter, the Court quotes with approval a relevant excerpt of COMELEC Commissioner
Maria Gracia Padacas separate concurring opinion in the assailed April 1, 2013 Resolution of the
COMELEC in SPA No. 13-211 (DC), which captured the essence of the legal effect of preambular Justice Leonen emphasizes the point that the ultimate issue for resolution is not whether the
paragraphs/whereas clauses, viz: pardon is contingent on the condition that former President Estrada will not seek janother elective
public office, but it actually concerns the coverage of the pardon whether the pardon granted to
former President Estrada was so expansive as to have restored all his political rights, inclusive of
The present dispute does not raise anything which the 20 January 2010 Resolution did not the rights of suffrage and to hold public office. Justice Leonen is of the view that the pardon in
conclude upon. Here, Petitioner Risos-Vidal raised the same argument with respect to the 3rd question is not absolute nor plenary in scope despite the statement that former President Estrada
"whereas clause" or preambular paragraph of the decree of pardon. It states that "Joseph Ejercito is "hereby restored to his civil and political rights," that is, the foregoing statement restored to
Estrada has publicly committed to no longer seek any elective position or office." On this former President Estrada all his civil and political rights except the rights denied to him by the
contention, the undersigned reiterates the ruling of the Commission that the 3rd preambular unremitted penalty of perpetual absolute disqualification made up of, among others, the rights of
paragraph does not have any legal or binding effect on the absolute nature of the pardon suffrage and to hold public office. He adds that had the President chosen to be so expansive as to
extended by former President Arroyo to herein Respondent. This ruling is consistent with the include the rights of suffrage and to hold public office, she should have been more clear on her
traditional and customary usage of preambular paragraphs. In the case of Echegaray v. Secretary intentions.
of Justice, the Supreme Court ruled on the legal effect of preambular paragraphs or whereas
clauses on statutes. The Court stated, viz.:
However, the statement "[h]e is hereby restored to his civil and political rights," to the mind of the
Court, iscrystal clear the pardon granted to former President Estrada was absolute, meaning, it
Besides, a preamble is really not an integral part of a law. It is merely an introduction to show its was not only unconditional, it was unrestricted in scope, complete and plenary in character, as the
intent or purposes. It cannot be the origin of rights and obligations. Where the meaning of a term "political rights"adverted to has a settled meaning in law and jurisprudence.
statute is clear and unambiguous, the preamble can neither expand nor restrict its operation much
less prevail over its text.
With due respect, I disagree too with Justice Leonen that the omission of the qualifying word "full"
can be construed as excluding the restoration of the rights of suffrage and to hold public office.
If former President Arroyo intended for the pardon to be conditional on Respondents promise There appears to be no distinction as to the coverage of the term "full political rights" and the
never to seek a public office again, the former ought to have explicitly stated the same in the text term "political rights" used alone without any qualification. How to ascribe to the latter term the
of the pardon itself. Since former President Arroyo did not make this an integral part of the decree meaning that it is "partial" and not "full" defies ones understanding. More so, it will be extremely
of pardon, the Commission is constrained to rule that the 3rd preambular clause cannot be difficult to identify which of the political rights are restored by the pardon, when the text of the
interpreted as a condition to the pardon extended to former President Estrada. 42 (Emphasis latter is silent on this matter. Exceptions to the grant of pardon cannot be presumed from the
supplied.) absence of the qualifying word "full" when the pardon restored the "political rights" of former
President Estrada without any exclusion or reservation.
Absent any contrary evidence, former President Arroyos silence on former President Estradas
decision torun for President in the May 2010 elections against, among others, the candidate of the Therefore, there can be no other conclusion but to say that the pardon granted to former
political party of former President Arroyo, after the latters receipt and acceptance of the pardon President Estrada was absolute in the absence of a clear, unequivocal and concrete factual basis
speaks volume of her intention to restore him to his rights to suffrage and to hold public office. upon which to anchor or support the Presidential intent to grant a limited pardon.
To reiterate, insofar as its coverageis concerned, the text of the pardon can withstand close
scrutiny even under the provisions of Articles 36 and 41 of the Revised Penal Code.

The COMELEC did not commit grave abuse of discretion amounting to lack or excess of jurisdiction
in issuing the assailed Resolutions.

In light of the foregoing, contrary to the assertions of Risos-Vidal, the COMELEC did not commit
grave abuse of discretion amounting to lack or excess of jurisdiction in issuing the assailed
Resolutions.

The Court has consistently held that a petition for certiorariagainst actions of the COMELEC is
confined only to instances of grave abuse of discretion amounting to patentand substantial denial
of due process, because the COMELEC is presumed to be most competent in matters falling within
its domain.43

As settled in jurisprudence, grave abuse of discretion is the arbitrary exercise of power due to
passion, prejudice or personal hostility; or the whimsical, arbitrary, or capricious exercise of power
that amounts to an evasion or refusal to perform a positive duty enjoined by law or to act at all in
contemplation of law. For an act to be condemned as having been done with grave abuse of
discretion, such an abuse must be patent and gross.44

The arguments forwarded by Risos-Vidal fail to adequately demonstrate any factual or legal bases
to prove that the assailed COMELEC Resolutions were issued in a "whimsical, arbitrary or
capricious exercise of power that amounts to an evasion orrefusal to perform a positive duty
enjoined by law" or were so "patent and gross" as to constitute grave abuse of discretion.

On the foregoing premises and conclusions, this Court finds it unnecessary to separately discuss
Lim's petition-in-intervention, which substantially presented the same arguments as Risos-Vidal's
petition.

WHEREFORE, the petition for certiorari and petition-inintervention are DISMISSED. The Resolution
dated April 1, 2013 of the Commission on Elections, Second Division, and the Resolution dated
April 23, 2013 of the Commission on Elections, En bane, both in SPA No. 13-211 (DC), are
AFFIRMED.

SO ORDERED.
G.R. No. L-1278 January 21, 1949 WHEREAS, the fact that such acts were committed in furtherance of the resistance to
the enemy is not a valid defense under the laws of the Philippines;
LORETO BARRIOQUINTO and NORBERTO JIMENEZ, petitioners, vs.
ENRIQUE A. FERNANDEZ, ANTONIO BELMONTE and FELICISIMO OCAMPO, as Commissioners of WHEREAS, the persons so accused should not be regarded as criminals but rather as
the Fourteenth Guerrilla Amnesty Commission, respondents. patriots and heroes who have rendered invaluable service to the nation; and

FERIA, J.: WHEREAS, it is desirable that without the least possible delay, these persons be freed
form the indignity and the jeopardy to which they are now being subjected;
This is a special action of mandamus instituted by the petitioners against the respondents who
composed the 14th Guerrilla Amnesty Commission, to compel the latter to act and decide whether NOW, THEREFORE, I Manuel Roxas, President of the Philippines in accordance with the
or not the petitioners are entitled to the benefits of amnesty. provisions of Article VII, section 10, paragraph 6 of the Constitution, do hereby declare
and proclaim an amnesty inn favor of al persons who committed any act penalized
Petitioners Norberto Jimenez and Loreto Barrioquinto were charged with the crime of murder. As under the Revised Penal Code in furtherance of the resistance to the enemy or against
the latter had not yet been arrested the case proceeded against the former, and after trial Court of persons aiding in the war effort of the enemy, and committed during the period from
First Instance of Zamboanga sentenced Jimenez to life imprisonment. Before the period for December 8, 1941 to the date when each particular area of the Philippines was actually
perfecting an appeal had expired, the defendant Jimenez became aware of the Proclamation No. 8, liberated from the enemy control and occupation. This amnesty shall not apply to
dated September 7, 1946, which grants amnesty in favor of all persons who may be charged with crimes against chastity or to acts committed from purely personal motives.
an act penalized under the Revised Penal Code in furtherance of the resistance to the enemy or
against persons aiding in the war efforts of the enemy, and committed during the period from It is further proclaimed and declared that in order to determine who among those
December 8, 1941, to the date when particular area of the Philippines where the offense was against whom charges have been filed before the courts of the Philippines or against
actually committed was liberated from enemy control and occupation, and said Jimenez decided whom charges may be filed in the future, come within the terms of this amnesty,
to submit his case to the Guerrilla Amnesty Commission presided by the respondents herein, and Guerrilla Amnesty Commissions, simultaneously to be established , shall examine the
the other petitioner Loreto Barrioquinto, who had then been already apprehended, did the same. facts and circumstance surrounding each case and, if necessary, conduct summary
hearings of witnesses both for the complainant and the accused. These Commissions
After a preliminary hearing had started, the Amnesty Commission, prescribed by the respondents, shall decided each case and, upon finding that it falls within the terms of this
issued on January 9, 1947, an order returning the cases of the petitioners to the Court of First proclamation, the Commissions shall so declare and this amnesty shall immediately be
Instance of Zamboanga, without deciding whether or not they are entitled to the benefits of he effective as to the accused, who shall forthwith be released or discharged.
said Amnesty Proclamation, on the ground that inasmuch as neither Barrioquinto nor Jimenez
have admitted having committed the offense, because Barrioquinto alleged that it was Hipolito The theory of the respondents, supported by the dissenting opinion, is predicated on a wrong
Tolentino who shot and killed the victim, they cannot invoke the benefits of amnesty. conception of the nature or character of an amnesty. Amnesty must be distinguished from pardon.

The Amnesty Proclamation of September 7, 1946, issued by the President with the concurrence of Pardon is granted by the Chief Executive and as such it is a private act which must be pleaded and
Congress of the Philippines, reads in part as follows: proved by the person pardoned, because the courts take no notice thereof; while amnesty by
Proclamation of the Chief Executive with the concurrence of Congress, and it is a public act of
WHEREAS, since the inception of the war until the liberation of the different areas which the courts should take judicial notice. Pardon is granted to one after conviction; while
comprising the territory of the Philippines, volunteer armed forces of Filipinos and for of amnesty is granted to classes of persons or communities who may be guilty of political offenses,
other nationalities operated as guerrillas and other patriotic individuals and groups generally before or after the institution of the criminal prosecution and sometimes after
pursued activities in opposition to the forces and agents of the Japanese Empire in the conviction. Pardon looks forward and relieves the offender from the consequences of an offense
invasion and occupation of the Philippines; of which he has been convicted, that is, it abolished or forgives the punishment, and for that
reason it does ""nor work the restoration of the rights to hold public office, or the right of suffrage,
unless such rights be expressly restored by the terms of the pardon," and it "in no case exempts
WHEREAS, members of such forces, in their determined efforts to resist the enemy, and the culprit from the payment of the civil indemnity imposed upon him by the sentence" article 36,
to bring about his ultimate defeat, committed acts penalized under the Revised Penal Revised Penal Code). while amnesty looks backward and abolishes and puts into oblivion the
Code; offense itself, it so overlooks and obliterates the offense with which he is charged that the person
released by amnesty stands before the law precisely as though he had committed no offense.
WHEREAS, charges have been presented in the courts against many members of these (section 10[6], Article VII, Philippine Constitution; State vs. Blalock, 62 N.C., 242, 247; In re Briggs,
resistance forces, for such acts; 135 N.C., 118; 47 S.E. 402., 403; Ex parte Law, 35 GA., 285, 296; State ex rel AnheuserBusch
Brewing Ass'n. vs. Eby, 170 Mo., 497; 71 S.W 52, 61; Burdick vs United States, N.Y., 35 S. Ct., 267;
271; 236 U.S., 79; 59 Law. ed., 476.)
In view of the foregoing, we are of the opinion and so hold that, in order to entitle a person to the There is no necessity for an accused to admit his responsibility for the commission of a criminal act
benefits of the Amnesty Proclamation of September 7, 1946, it is not necessary that he should, as before a court of Amnesty Commission may investigate and extend or not to him the benefits of
a condition precedent or sine qua non, admit having committed the criminal act or offense with amnesty. The fact that he pleads not guilty or that he has not committed the act with which he is
which he is charged and allege the amnesty as a defense; it is sufficient that the evidence either of charged, does not necessarily prove that he is not guilty thereof. Notwithstanding his denial, the
the complainant or the accused, shows that the offense committed comes within the terms of said evidence for the prosecution or complainant may show the contrary, as it is generally the case in
Amnesty Proclamation. Hence, it is not correct to say that "invocation of the benefits of amnesty is criminal proceedings, and what should in such a case be determined is whether or not the offense
in the nature of a plea of confession and avoidance." Although the accused does not confess the committed is of political character. The plea of not having committed the offense made by an
imputation against him, he may be declared by the courts or the Amnesty Commissions entitled to accused simply means that he can not be convicted of the offense charged because he is not guilty
the benefits. For, whether or not he admits or confesses having committed the offense with which thereof, and, even if the evidence would show that he is, because he has committed it in
he is charged, the Commissions should, if necessary or requested by the interested party, conduct furtherance of the resistance to the enemy or against persons a ding in the war efforts of the
summary hearing of the witnesses both for the complainants and the accused, on whether he has enemy, and not for purely political motives.
committed the offense in furtherance of the resistance to the enemy, or against persons aiding in
the war efforts of the enemy, and decide whether he is entitled to the benefits of amnesty and to According to Administrative Order No. 11 of October 2, 1946, creating the Amnesty Commissions,
be "regarded as a patriot or hero who have rendered invaluable services to the nation,," or not, in issued by the President of the Philippines, cases pending in the Courts of First Instance of the
accordance with the terms of the Amnesty Proclamation. since the Amnesty Proclamation is a province in which the accused claims the benefits of Amnesty Proclamation, and cases already
public act, the courts as well as the Amnesty Commissions created thereby should take notice of decided by said courts but not yet elevated on appeal to the appellate courts, shall be passed upon
the terms of said Proclamation and apply the benefits granted therein to cases coming within their and decided by the respective Amnesty Commission, and cases pending appeal shall be passed
province or jurisdiction, whether pleaded or claimed by the person charged with such offenses or upon by the Seventh Amnesty Commission. Under the theory of the respondents and the writer
not, if the evidence presented show that the accused is entitled to said benefits. oft he dissenting opinion, the Commissions should refuse to comply with the directive of said
Administrative Order, because is almost all cases pending in the Court of First Instance, and all
The right to the benefits of amnesty, once established by the evidence presented either by the those pending appeal form the sentence of said courts, the defendants must not have pleaded
complainant or prosecution, or by the defense, can not be waived, because it is of public interest guilty or admitted having committed the offense charged for otherwise, they would not or could
that a person who is regarded by the Amnesty Proclamation which has the force of a law, not only not have appealed from the judgment of the Courts of First Instance. To hold that a Amnesty
as innocent, for he stands in the eyes of the law as if he had never committed any punishable Commission should not proceed to the investigation and act and decide whether the offense with
offense because of the amnesty, but as a patriot or hero, can not be punishment as a criminal. Just which an accused was charged comes within the Amnesty Proclamation if he does not admit or
as the courts of justice can not convict a person who, according to the evidence, has committed an confess having committed it would be to defeat the purpose for which the Amnesty Proclamation
act not punishable by law, although he confesses being guilty thereof, so also and a fortiori they was issued and the Amnesty Commission were established. If the courts have to proceed to the
can not convict a person considered by law not a criminal, but as a patriot and hero, for having trail or hearing of a case and decide whether the offense committed by the defendant comes
rendered invaluable services to the nation inn committing such an act. within the terms of the Amnesty Proclamation although the defendant has plead not guilty, there
is no reason why the Amnesty Commissions can not do so. Where a defendant to admit or confess
While it is true that the evidence must show that the offense charged was against chastity and was having committed the offense or being responsible therefor before he can invoke the benefit of
committed in furtherance of the resistance against the enemy, for otherwise, it is to be naturally amnesty, as there is no law which makes such admission or confession not admissible as evidence
presumed that is has been committed for purely personal motive, it is nonetheless true that against him in the courts of justices in case the Amnesty Commission finds that the offense does
though the motive as a mental impulse is state of mind or subjective, it need not be testified to be not come within the terms of the Amnesty Proclamation, nobody or few would take the risk of
the defendant himself at his arraignment or hearing of the case. Generally the motive for the submitting their case to said Commission.
commission of an offense is established by the testimony of witnesses on the acts or statements of
the accused before or immediately after the commission of the offense, deeds or words hat may Besides, in the present case, the allegation of Loreto Barrioquinto that the offended party or victim
express it or from which his motive or reason for committing it may be inferred. The statement of was shot and killed by Agapito Hipolito , does not necessarily bar the respondents from finding,
testimony of a defendant at the time of arraignment or the hearing of the case about said motive, after the summary hearing of the witnesses for the complaints and the accused, directed in the
can not generally be considered and relied on, specially if there is evidence to the contrary, as the said Amnesty Proclamation and Administrative Order No. 11, that the petitioners are responsible
true expression of the reason o motive he had at the time of committing the offense. Because such for the killing of the victim, either as principals by cooperation, inducement or conspiration, or as
statements or testimony may be an afterthought or colored by the interest he may have to suit his accessories before as well as after the fact, but that they are entitled to the benefits of amnesty,
defense or the purpose for which he intends to achieve with such declaration. Hence it does not because they were members of the same group of guerrilleros who killed the victim in furtherance
stand to reason and logic to say, as the dissenting opinion avers, that unless the defendant admits of the resistance to the enemy or against persons aiding in the war efforts of the enemy.
at the investigation or hearing having committed the offense with which he is charged, and states
that he did it in furtherance of the resistance to the enemy, and not for purely personal motive, it Wherefore, the respondents are hereby ordered to immediately proceed to hear and decide the
is impossible for the court of Commission to verify the motive for the commission of the offense, application for amnesty of petitioners Barrioquinto and Jimenez, unless amnesty of petitioners
because only the accused could explain of the offense, because only the accused could explain his Barrioquinto and Jimenez, unless the courts have in the meantime already decided, expressly and
belief and intention or the motive of committing the offense. finally, the question whether or not they are entitled to the benefits of the Amnesty Proclamation
No. 8 of September 7, 1946. So ordered.
G.R. No. 212426 The petitions1 before this Court question the constitutionality of the Enhanced Defense
Cooperation Agreement (EDCA) between the Republic of the Philippines and the United States of
RENE A.V. SAGUISAG, WIGBERTO E. TAADA, FRANCISCO "DODONG" NEMENZO, JR., SR. MARY America (U.S.). Petitioners allege that respondents committed grave abuse of discretion
JOHN MANANZAN, PACIFICO A. AGABIN, ESTEBAN "STEVE" SALONGA, H. HARRY L. ROQUE, JR., amounting to lack or excess of jurisdiction when they entered into EDCA with the U.S.,2 claiming
EVALYN G. URSUA, EDRE U. OLALIA, DR. CAROL PAGADUAN-ARAULLO, DR. ROLAND SIMBULAN, that the instrument violated multiple constitutional provisions. 3 In reply, respondents argue that
AND TEDDY CASIO, Petitioners, petitioners lack standing to bring the suit. To support the legality of their actions, respondents
vs. invoke the 1987 Constitution, treaties, and judicial precedents.4
EXECUTIVE SECRETARY PAQUITO N. OCHOA, JR., DEPARTMENT OF NATIONAL DEFENSE
SECRETARY VOLTAIRE GAZMIN, DEPARTMENT OF FOREIGN AFFAIRS SECRETARY ALBERT DEL A proper analysis of the issues requires this Court to lay down at the outset the basic parameters
ROSARIO, JR., DEPARTMENT OF BUDGET AND MANAGEMENT SECRETARY FLORENCIO ABAD, of the constitutional powers and roles of the President and the Senate in respect of the above
AND ARMED FORCES OF THE PHILIPPINES CHIEF OF STAFF GENERAL EMMANUEL T. issues. A more detailed discussion of these powers and roles will be made in the latter portions.
BAUTISTA, Respondents.
I. BROAD CONSTITUTIONAL CONTEXT OF THE POWERS OF THE PRESIDENT: DEFENSE, FOREIGN
x-----------------------x RELATIONS, AND EDCA

G.R. No. 212444 A. The Prime Duty of the State and the Consolidation of Executive Power in the President

BAGONG ALYANSANG MAKABAYAN (BAYAN), REPRESENTED BY ITS SECRETARY GENERAL Mataimtim kong pinanunumpaan (o pinatotohanan) na tutuparin ko nang buong katapatan at
RENATO M. REYES, JR., BAYAN MUNA PARTY-LIST REPRESENTATIVES NERI J. COLMENARES AND sigasig ang aking mga tungkulin bilang Pangulo (o Pangalawang Pangulo o Nanunungkulang
CARLOS ZARATE, GABRIELA WOMEN'S PARTY-LIST REPRESENTATIVES LUZ ILAGAN AND Pangulo) ng Pilipinas, pangangalagaan at ipagtatanggol ang kanyang Konstitusyon, ipatutupad
EMERENCIANA DE JESUS, ACT TEACHERS PARTY-LIST REPRESENTATIVE ANTONIO L. TINIO, ang mga batas nito, magiging makatarungan sa bawat tao, at itatalaga ang aking sarili sa
ANAKPAWIS PARTY-LIST REPRESENTATIVE FERNANDO HICAP, KABATAAN PARTY-LIST paglilingkod sa Bansa. Kasihan nawa aka ng Diyos.
REPRESENTATIVE TERRY RIDON, MAKABAYANG KOALISYON NG MAMAMAYAN (MAKABAYAN),
REPRESENTED BY SATURNINO OCAMPO AND LIZA MAZA, BIENVENIDO LUMBERA, JOEL C. - Panunumpa sa Katungkulan ng Pangulo ng Pilipinas ayon sa Saligang Batas5
LAMANGAN, RAFAEL MARIANO, SALVADOR FRANCE, ROGELIO M. SOLUTA, AND CLEMENTE G.
BAUTISTA, Petitioners,
vs. The 1987 Constitution has "vested the executive power in the President of the Republic of the
DEPARTMENT OF NATIONAL DEFENSE (DND) SECRETARY VOLTAIRE GAZMIN, DEPARTMENT OF Philippines."6 While the vastness of the executive power that has been consolidated in the person
FOREIGN AFFAIRS SECRETARY ALBERT DEL ROSARIO, EXECUTIVE SECRETARY PAQUITO N. of the President cannot be expressed fully in one provision, the Constitution has stated the prime
OCHOA, JR., ARMED FORCES OF THE PHILIPPINES CHIEF OF STAFF GENERAL EMMANUEL T. duty of the government, of which the President is the head:
BAUTISTA, DEFENSE UNDERSECRETARY PIO LORENZO BATINO, AMBASSADOR LOURDES
YPARRAGUIRRE, AMBASSADOR J. EDUARDO MALAYA, DEPARTMENT OF JUSTICE The prime duty of the Government is to serve and protect the people. The Government may call
UNDERSECRETARY FRANCISCO BARAAN III, AND DND ASSISTANT SECRETARY FOR STRATEGIC upon the people to defend the State and, in the fulfillment thereof, all citizens may be required,
ASSESSMENTS RAYMUND JOSE QUILOP AS CHAIRPERSON AND MEMBERS, RESPECTIVELY, OF under conditions provided by law, to render personal military or civil service.7 (Emphases supplied)
THE NEGOTIATING PANEL FOR THE PHILIPPINES ON EDCA, Respondents.
B. The duty to protect the territory and the citizens of the Philippines, the power to call upon the
x-----------------------x people to defend the State, and the President as Commander-in-Chief

KILUSANG MAYO UNO, REPRESENTED BY ITS CHAIRPERSON, ELMER LABOG, CONFEDERATION The duty to protect the State and its people must be carried out earnestly and effectively
FOR UNITY, RECOGNITION AND ADVANCEMENT OF GOVERNMENT EMPLOYEES (COURAGE), throughout the whole territory of the Philippines in accordance with the constitutional provision
REPRESENTED BY ITS NATIONAL PRESIDENT FERDINAND GAITE, NATIONAL FEDERATION OF on national territory. Hence, the President of the Philippines, as the sole repository of executive
LABOR UNIONS-KILUSANG MAYO UNO, REPRESENTED BY ITS NATIONAL PRESIDENT JOSELITO power, is the guardian of the Philippine archipelago, including all the islands and waters embraced
USTAREZ, NENITA GONZAGA, VIOLETA ESPIRITU, VIRGINIA FLORES, AND ARMANDO TEODORO, therein and all other territories over which it has sovereignty or jurisdiction. These territories
JR., Petitioners-in-Intervention, consist of its terrestrial, fluvial, and aerial domains; including its territorial sea, the seabed, the
RENE A.Q. SAGUISAG, JR., Petitioner-in-Intervention. subsoil, the insular shelves, and other submarine areas; and the waters around, between, and
connecting the islands of the archipelago, regardless of their breadth and dimensions.8
DECISION
To carry out this important duty, the President is equipped with authority over the Armed Forces
SERENO, J.: of the Philippines (AFP),9 which is the protector of the people and the state. The AFP's role is to
secure the sovereignty of the State and the integrity of the national territory.10 In addition, the Clearly, the power to defend the State and to act as its representative in the international sphere
Executive is constitutionally empowered to maintain peace and order; protect life, liberty, and inheres in the person of the President. This power, however, does not crystallize into absolute
property; and promote the general welfare.11 discretion to craft whatever instrument the Chief Executive so desires. As previously mentioned,
the Senate has a role in ensuring that treaties or international agreements the President enters
In recognition of these powers, Congress has specified that the President must oversee, ensure, into, as contemplated in Section 21 of Article VII of the Constitution, obtain the approval of two-
and reinforce our defensive capabilities against external and internal threats12 and, in the same thirds of its members.
vein, ensure that the country is adequately prepared for all national and local emergencies arising
from natural and man-made disasters.13 Previously, treaties under the 1973 Constitution required ratification by a majority of the Batasang
Pambansa,19except in instances wherein the President "may enter into international treaties or
To be sure, this power is limited by the Constitution itself. To illustrate, the President may call out agreements as the national welfare and interest may require."20 This left a large margin of
the AFP to prevent or suppress instances of lawless violence, invasion or rebellion, 14 but not discretion that the President could use to bypass the Legislature altogether. This was a departure
suspend the privilege of the writ of habeas corpus for a period exceeding 60 days, or place the from the 1935 Constitution, which explicitly gave the President the power to enter into treaties
Philippines or any part thereof under martial law exceeding that same span. In the exercise of only with the concurrence of two-thirds of all the Members of the Senate.21 The 1987 Constitution
these powers, the President is also duty-bound to submit a report to Congress, in person or in returned the Senate's power22 and, with it, the legislative's traditional role in foreign affairs.23
writing, within 48 hours from the proclamation of martial law or the suspension of the privilege of
the writ of habeas corpus; and Congress may in turn revoke the proclamation or suspension. The The responsibility of the President when it comes to treaties and international agreements under
same provision provides for the Supreme Court's review of the factual basis for the proclamation the present Constitution is therefore shared with the Senate. This shared role, petitioners claim, is
or suspension, as well as the promulgation of the decision within 30 days from filing. bypassed by EDCA.

C. The power and duty to conduct foreign relations II. HISTORICAL ANTECEDENTS OF EDCA

The President also carries the mandate of being the sole organ in the conduct of foreign A. U.S. takeover of Spanish colonization and its military bases, and the transition to Philippine
relations.15 Since every state has the capacity to interact with and engage in relations with other independence
sovereign states,16 it is but logical that every state must vest in an agent the authority to represent
its interests to those other sovereign states. The presence of the U.S. military forces in the country can be traced to their pivotal victory in the
1898 Battle of Manila Bay during the Spanish-American War.24 Spain relinquished its sovereignty
The conduct of foreign relations is full of complexities and consequences, sometimes with life and over the Philippine Islands in favor of the U.S. upon its formal surrender a few months later. 25 By
death significance to the nation especially in times of war. It can only be entrusted to that 1899, the Americans had consolidated a military administration in the archipelago. 26
department of government which can act on the basis of the best available information and can
decide with decisiveness. x x x It is also the President who possesses the most comprehensive and When it became clear that the American forces intended to impose colonial control over the
the most confidential information about foreign countries for our diplomatic and consular officials Philippine Islands, General Emilio Aguinaldo immediately led the Filipinos into an all-out war
regularly brief him on meaningful events all over the world. He has also unlimited access to ultra- against the U.S.27 The Filipinos were ultimately defeated in the Philippine-American War, which
sensitive military intelligence data. In fine, the presidential role in foreign affairs is dominant and lasted until 1902 and led to the downfall of the first Philippine Republic.28 The Americans
the President is traditionally accorded a wider degree of discretion in the conduct of foreign henceforth began to strengthen their foothold in the country.29 They took over and expanded the
affairs. The regularity, nay, validity of his actions are adjudged under less stringent standards, lest former Spanish Naval Base in Subic Bay, Zambales, and put up a cavalry post called Fort
their judicial repudiation lead to breach of an international obligation, rupture of state relations, Stotsenberg in Pampanga, now known as Clark Air Base.30
forfeiture of confidence, national embarrassment and a plethora of other problems with equally
undesirable consequences.17
When talks of the eventual independence of the Philippine Islands gained ground, the U.S.
manifested the desire to maintain military bases and armed forces in the country.31 The U.S.
The role of the President in foreign affairs is qualified by the Constitution in that the Chief Congress later enacted the Hare-Hawes-Cutting Act of 1933, which required that the proposed
Executive must give paramount importance to the sovereignty of the nation, the integrity of its constitution of an independent Philippines recognize the right of the U.S. to maintain the latter's
territory, its interest, and the right of the sovereign Filipino people to self-determination.18 In armed forces and military bases.32 The Philippine Legislature rejected that law, as it also gave the
specific provisions, the President's power is also limited, or at least shared, as in Section 2 of U.S. the power to unilaterally designate any part of Philippine territory as a permanent military or
Article II on the conduct of war; Sections 20 and 21 of Article VII on foreign loans, treaties, and naval base of the U.S. within two years from complete independence. 33
international agreements; Sections 4(2) and 5(2)(a) of Article VIII on the judicial review of
executive acts; Sections 4 and 25 of Article XVIII on treaties and international agreements entered
into prior to the Constitution and on the presence of foreign military troops, bases, or facilities. The U.S. Legislature subsequently crafted another law called the Tydings-McDuffie Act or the
Philippine Independence Act of 1934. Compared to the old Hare-Hawes-Cutting Act, the new law
provided for the surrender to the Commonwealth Government of "all military and other
D. The relationship between the two major presidential functions and the role of the Senate reservations" of the U.S. government in the Philippines, except "naval reservations and refueling
stations."34 Furthermore, the law authorized the U.S. President to enter into negotiations for the To further strengthen their defense and security relationship,59 the Philippines and the U.S. next
adjustment and settlement of all questions relating to naval reservations and fueling stations entered into the MDT in 1951. Concurred in by both the Philippine60 and the U.S.61 Senates, the
within two years after the Philippines would have gained independence.35 Under the Tydings- treaty has two main features: first, it allowed for mutual assistance in maintaining and developing
McDuffie Act, the U.S. President would proclaim the American withdrawal and surrender of their individual and collective capacities to resist an armed attack;62 and second, it provided for
sovereignty over the islands 10 years after the inauguration of the new government in the their mutual self-defense in the event of an armed attack against the territory of either
Philippines.36 This law eventually led to the promulgation of the 1935 Philippine Constitution. party.63 The treaty was premised on their recognition that an armed attack on either of them
would equally be a threat to the security of the other.64
The original plan to surrender the military bases changed.37 At the height of the Second World
War, the Philippine and the U.S. Legislatures each passed resolutions authorizing their respective C. Current legal regime on the presence of U.S. armed forces in the country
Presidents to negotiate the matter of retaining military bases in the country after the planned
withdrawal of the U.S.38 Subsequently, in 1946, the countries entered into the Treaty of General In view of the impending expiration of the 1947 MBA in 1991, the Philippines and the U.S.
Relations, in which the U.S. relinquished all control and sovereignty over the Philippine negotiated for a possible renewal of their defense and security relationship.65 Termed as the
Islands, except the areas that would be covered by the American military bases in the Treaty of Friendship, Cooperation and Security, the countries sought to recast their military ties by
country.39 This treaty eventually led to the creation of the post-colonial legal regime on which providing a new framework for their defense cooperation and the use of Philippine
would hinge the continued presence of U.S. military forces until 1991: the Military Bases installations.66 One of the proposed provisions included an arrangement in which U.S. forces would
Agreement (MBA) of 1947, the Military Assistance Agreement of 1947, and the Mutual Defense be granted the use of certain installations within the Philippine naval base in Subic.67 On 16
Treaty (MDT) of 1951.40 September 1991, the Senate rejected the proposed treaty.68

B. Former legal regime on the presence of U.S. armed forces in the territory of an independent The consequent expiration of the 1947 MBA and the resulting paucity of any formal agreement
Philippines (1946-1991) dealing with the treatment of U.S. personnel in the Philippines led to the suspension in 1995 of
large-scale joint military exercises.69In the meantime, the respective governments of the two
Soon after the Philippines was granted independence, the two countries entered into their first countries agreed70 to hold joint exercises at a substantially reduced level.71 The military
military arrangement pursuant to the Treaty of General Relations - the 1947 MBA.41 The Senate arrangements between them were revived in 1999 when they concluded the first Visiting Forces
concurred on the premise of "mutuality of security interest,"42 which provided for the presence Agreement (VFA).72
and operation of 23 U.S. military bases in the Philippines for 99 years or until the year 2046. 43 The
treaty also obliged the Philippines to negotiate with the U.S. to allow the latter to expand the As a "reaffirm[ation] [of the] obligations under the MDT,"73 the VFA has laid down the regulatory
existing bases or to acquire new ones as military necessity might require. 44 mechanism for the treatment of U.S. military and civilian personnel visiting the country.74 It
contains provisions on the entry and departure of U.S. personnel; the purpose, extent, and
A number of significant amendments to the 1947 MBA were made.45 With respect to its duration, limitations of their activities; criminal and disciplinary jurisdiction; the waiver of certain claims; the
the parties entered into the Ramos-Rusk Agreement of 1966, which reduced the term of the treaty importation and exportation of equipment, materials, supplies, and other pieces of property
from 99 years to a total of 44 years or until 1991.46 Concerning the number of U.S. military bases in owned by the U.S. government; and the movement of U.S. military vehicles, vessels, and aircraft
the country, the Bohlen-Serrano Memorandum of Agreement provided for the return to the into and within the country.75 The Philippines and the U.S. also entered into a second counterpart
Philippines of 17 U.S. military bases covering a total area of 117,075 hectares.47 Twelve years later, agreement (VFA II), which in turn regulated the treatment of Philippine military and civilian
the U.S. returned Sangley Point in Cavite City through an exchange of notes.48 Then, through the personnel visiting the U.S.76 The Philippine Senate concurred in the first VFA on 27 May 1999.77
Romulo-Murphy Exchange of Notes of 1979, the parties agreed to the recognition of Philippine
sovereignty over Clark and Subic Bases and the reduction of the areas that could be used by the Beginning in January 2002, U.S. military and civilian personnel started arriving in Mindanao to take
U.S. military.49 The agreement also provided for the mandatory review of the treaty every five part in joint military exercises with their Filipino counterparts.78 Called Balikatan, these exercises
years.50 In 1983, the parties revised the 1947 MBA through the Romualdez-Armacost involved trainings aimed at simulating joint military maneuvers pursuant to the MDT.79
Agreement.51 The revision pertained to the operational use of the military bases by the U.S.
government within the context of Philippine sovereignty,52 including the need for prior
consultation with the Philippine government on the former' s use of the bases for military combat In the same year, the Philippines and the U.S. entered into the Mutual Logistics Support
operations or the establishment of long-range missiles.53 Agreement to "further the interoperability, readiness, and effectiveness of their respective military
forces"80 in accordance with the MDT, the Military Assistance Agreement of 1953, and the
VFA.81 The new agreement outlined the basic terms, conditions, and procedures for facilitating the
Pursuant to the legislative authorization granted under Republic Act No. 9, 54 the President also reciprocal provision of logistics support, supplies, and services between the military forces of the
entered into the 1947 Military Assistance Agreement55 with the U.S. This executive agreement two countries.82 The phrase "logistics support and services" includes billeting, operations support,
established the conditions under which U.S. military assistance would be granted to the construction and use of temporary structures, and storage services during an approved activity
Philippines,56 particularly the provision of military arms, ammunitions, supplies, equipment, under the existing military arrangements.83 Already extended twice, the agreement will last until
vessels, services, and training for the latter's defense forces.57 An exchange of notes in 1953 made 2017.84
it clear that the agreement would remain in force until terminated by any of the parties.58
D. The Enhanced Defense Cooperation Agreement Senate. Before this Court may begin to analyze the constitutionality or validity of an official act of a
coequal branch of government, however, petitioners must show that they have satisfied all the
EDCA authorizes the U.S. military forces to have access to and conduct activities within certain essential requisites for judicial review.93
"Agreed Locations" in the country. It was not transmitted to the Senate on the executive's
understanding that to do so was no longer necessary.85 Accordingly, in June 2014, the Department Distinguished from the general notion of judicial power, the power of judicial review specially
of Foreign Affairs (DFA) and the U.S. Embassy exchanged diplomatic notes confirming the refers to both the authority and the duty of this Court to determine whether a branch or an
completion of all necessary internal requirements for the agreement to enter into force in the two instrumentality of government has acted beyond the scope of the latter's constitutional
countries.86 powers.94 As articulated in Section 1, Article VIII of the Constitution, the power of judicial review
involves the power to resolve cases in which the questions concern the constitutionality or validity
According to the Philippine government, the conclusion of EDCA was the result of intensive and of any treaty, international or executive agreement, law, presidential decree, proclamation, order,
comprehensive negotiations in the course of almost two years.87 After eight rounds of instruction, ordinance, or regulation.95 In Angara v. Electoral Commission, this Court exhaustively
negotiations, the Secretary of National Defense and the U.S. Ambassador to the Philippines signed discussed this "moderating power" as part of the system of checks and balances under the
the agreement on 28 April 2014.88 President Benigno S. Aquino III ratified EDCA on 6 June Constitution. In our fundamental law, the role of the Court is to determine whether a branch of
2014.89 The OSG clarified during the oral arguments90 that the Philippine and the U.S. governments government has adhered to the specific restrictions and limitations of the latter's power:96
had yet to agree formally on the specific sites of the Agreed Locations mentioned in the
agreement. The separation of powers is a fundamental principle in our system of government. It obtains not
through express provision but by actual division in our Constitution. Each department of the
Two petitions for certiorari were thereafter filed before us assailing the constitutionality of EDCA. government has exclusive cognizance of matters within its jurisdiction, and is supreme within its
They primarily argue that it should have been in the form of a treaty concurred in by the Senate, own sphere. But it does not follow from the fact that the three powers are to be kept separate
not an executive agreement. and distinct that the Constitution intended them to be absolutely unrestrained and independent of
each other. The Constitution has provided for an elaborate system of checks and balances to
secure coordination in the workings of the various departments of the government. x x x. And the
On 10 November 2015, months after the oral arguments were concluded and the parties ordered judiciary in turn, with the Supreme Court as the final arbiter, effectively checks the other
to file their respective memoranda, the Senators adopted Senate Resolution No. (SR) 105.91 The departments in the exercise of its power to determine the law, and hence to declare executive
resolution expresses the "strong sense"92 of the Senators that for EDCA to become valid and and legislative acts void if violative of the Constitution.
effective, it must first be transmitted to the Senate for deliberation and concurrence.

xxxx
III. ISSUES

As any human production, our Constitution is of course lacking perfection and perfectibility, but as
Petitioners mainly seek a declaration that the Executive Department committed grave abuse of much as it was within the power of our people, acting through their delegates to so provide, that
discretion in entering into EDCA in the form of an executive agreement. For this reason, we cull the instrument which is the expression of their sovereignty however limited, has established a
issues before us: republican government intended to operate and function as a harmonious whole, under a
system of checks and balances, and subject to specific limitations and restrictions provided in
A. Whether the essential requisites for judicial review are present the said instrument. The Constitution sets forth in no uncertain language the restrictions and
limitations upon governmental powers and agencies. If these restrictions and limitations are
B. Whether the President may enter into an executive agreement on foreign military transcended it would be inconceivable if the Constitution had not provided for a mechanism by
bases, troops, or facilities which to direct the course of government along constitutional channels, for then the distribution
of powers would be mere verbiage, the bill of rights mere expressions of sentiment, and the
principles of good government mere political apothegms. Certainly, the limitations and
C. Whether the provisions under EDCA are consistent with the Constitution, as well as restrictions embodied in our Constitution are real as they should be in any living constitution. x x x.
with existing laws and treaties In our case, this moderating power is granted, if not expressly, by clear implication from section 2
of article VIII of [the 1935] Constitution.
IV. DISCUSSION
The Constitution is a definition of the powers of government. Who is to determine the nature,
A. Whether the essential requisites for judicial review have been satisfied scope and extent of such powers? The Constitution itself has provided for the instrumentality of
the judiciary as the rational way. And when the judiciary mediates to allocate constitutional
boundaries, it does not assert any superiority over the other departments; it does not in reality
Petitioners are hailing this Court's power of judicial review in order to strike down EDCA for
nullify or invalidate an act of the legislature, but only asserts the solemn and sacred obligation
violating the Constitution. They stress that our fundamental law is explicit in prohibiting the
assigned to it by the Constitution to determine conflicting claims of authority under the
presence of foreign military forces in the country, except under a treaty concurred in by the
Constitution and to establish for the parties in an actual controversy the rights which that
instrument secures and guarantees to them. This is in truth all that is involved in what is termed constitutionality is the lis mota of the case.106 Of these four, the first two conditions will be the
"judicial supremacy" which properly is the power of judicial review under the Constitution. x x x focus of our discussion.
x. (Emphases supplied)
1. Petitioners have shown the presence of an actual case or controversy.
The power of judicial review has since been strengthened in the 1987 Constitution. The scope of
that power has been extended to the determination of whether in matters traditionally considered The OSG maintains107 that there is no actual case or controversy that exists, since the Senators
to be within the sphere of appreciation of another branch of government, an exercise of discretion have not been deprived of the opportunity to invoke the privileges of the institution they are
has been attended with grave abuse.97 The expansion of this power has made the political representing. It contends that the nonparticipation of the Senators in the present petitions only
question doctrine "no longer the insurmountable obstacle to the exercise of judicial power or the confirms that even they believe that EDCA is a binding executive agreement that does not require
impenetrable shield that protects executive and legislative actions from judicial inquiry or their concurrence.
review."98

It must be emphasized that the Senate has already expressed its position through SR
This moderating power, however, must be exercised carefully and only if it cannot be completely 105.108 Through the Resolution, the Senate has taken a position contrary to that of the OSG. As the
avoided. We stress that our Constitution is so incisively designed that it identifies the spheres of body tasked to participate in foreign affairs by ratifying treaties, its belief that EDCA infringes upon
expertise within which the different branches of government shall function and the questions of its constitutional role indicates that an actual controversy - albeit brought to the Court by non-
policy that they shall resolve.99 Since the power of judicial review involves the delicate exercise of Senators, exists.
examining the validity or constitutionality of an act of a coequal branch of government, this Court
must continually exercise restraint to avoid the risk of supplanting the wisdom of the
constitutionally appointed actor with that of its own.100 Moreover, we cannot consider the sheer abstention of the Senators from the present proceedings
as basis for finding that there is no actual case or controversy before us. We point out that the
focus of this requirement is the ripeness for adjudication of the matter at hand, as opposed to its
Even as we are left with no recourse but to bare our power to check an act of a coequal branch of being merely conjectural or anticipatory.109 The case must involve a definite and concrete issue
government - in this case the executive - we must abide by the stringent requirements for the involving real parties with conflicting legal rights and legal claims admitting of specific relief
exercise of that power under the Constitution. Demetria v. Alba101 and Francisco v. House of through a decree conclusive in nature.110 It should not equate with a mere request for an opinion
Representatives102 cite the "pillars" of the limitations on the power of judicial review as enunciated or advice on what the law would be upon an abstract, hypothetical, or contingent state of
in the concurring opinion of U.S. Supreme Court Justice Brandeis in Ashwander v. Tennessee Valley facts.111 As explained in Angara v. Electoral Commission:112
Authority.103 Francisco104 redressed these "pillars" under the following categories:

[The] power of judicial review is limited to actual cases and controversies to be exercised after
1. That there be absolute necessity of deciding a case full opportunity of argument by the parties, and limited further to the constitutional question
raised or the very lis mota presented. Any attempt at abstraction could only lead to dialectics and
2. That rules of constitutional law shall be formulated only as required by the facts of barren legal questions and to sterile conclusions of wisdom, justice or expediency of legislation.
the case More than that, courts accord the presumption of constitutionality to legislative enactments, not
only because the legislature is presumed to abide by the Constitution but also because
3. That judgment may not be sustained on some other ground the judiciary in the determination of actual cases and controversies must reflect the wisdom and
justice of the people as expressed through their representatives in the executive and legislative
departments of the government. (Emphases supplied)
4. That there be actual injury sustained by the party by reason of the operation of the
statute
We find that the matter before us involves an actual case or controversy that is already ripe for
adjudication. The Executive Department has already sent an official confirmation to the U.S.
5. That the parties are not in estoppel Embassy that "all internal requirements of the Philippines x x x have already been complied
with."113 By this exchange of diplomatic notes, the Executive Department effectively performed
6. That the Court upholds the presumption of constitutionality the last act required under Article XII(l) of EDCA before the agreement entered into force. Section
25, Article XVIII of the Constitution, is clear that the presence of foreign military forces in the
(Emphases supplied) country shall only be allowed by virtue of a treaty concurred in by the Senate. Hence, the
performance of an official act by the Executive Department that led to the entry into force of an
executive agreement was sufficient to satisfy the actual case or controversy requirement.
These are the specific safeguards laid down by the Court when it exercises its power of judicial
review.105 Guided by these pillars, it may invoke the power only when the following four stringent
requirements are satisfied: (a) there is an actual case or controversy; (b) petitioners possess locus 2. While petitioners Saguisag et. al., do not have legal standing, they nonetheless raise issues
standi; (c) the question of constitutionality is raised at the earliest opportunity; and (d) the issue of involving matters of transcendental importance.
The question of locus standi or legal standing focuses on the determination of whether those All obligations under this Agreement are subject to the availability of appropriated
assailing the governmental act have the right of appearance to bring the matter to the court for funds authorized for these purposes. (Emphases supplied)
adjudication.114 They must show that they have a personal and substantial interest in the case,
such that they have sustained or are in immediate danger of sustaining, some direct injury as a This provision means that if the implementation of EDCA would require the disbursement of public
consequence of the enforcement of the challenged governmental act.115 Here, "interest" in the funds, the money must come from appropriated funds that are specifically authorized for this
question involved must be material - an interest that is in issue and will be affected by the official purpose. Under the agreement, before there can even be a disbursement of public funds, there
act - as distinguished from being merely incidental or general.116 Clearly, it would be insufficient to must first be a legislative action. Until and unless the Legislature appropriates funds for EDCA, or
show that the law or any governmental act is invalid, and that petitioners stand to suffer in some unless petitioners can pinpoint a specific item in the current budget that allows expenditure
indefinite way.117 They must show that they have a particular interest in bringing the suit, and that under the agreement, we cannot at this time rule that there is in fact an appropriation or a
they have been or are about to be denied some right or privilege to which they are lawfully disbursement of funds that would justify the filing of a taxpayers' suit.
entitled, or that they are about to be subjected to some burden or penalty by reason of the act
complained of.118 The reason why those who challenge the validity of a law or an international
agreement are required to allege the existence of a personal stake in the outcome of the Petitioners Bayan et al. also claim129 that their co-petitioners who are party-list representatives
controversy is "to assure the concrete adverseness which sharpens the presentation of issues upon have the standing to challenge the act of the Executive Department, especially if it impairs the
which the court so largely depends for illumination of difficult constitutional questions."119 constitutional prerogatives, powers, and privileges of their office. While they admit that there is no
incumbent Senator who has taken part in the present petition, they nonetheless assert that they
also stand to sustain a derivative but substantial injury as legislators. They argue that under the
The present petitions cannot qualify as citizens', taxpayers', or legislators' suits; the Senate as a Constitution, legislative power is vested in both the Senate and the House of Representatives;
body has the requisite standing, but considering that it has not formally filed a pleading to join the consequently, it is the entire Legislative Department that has a voice in determining whether or
suit, as it merely conveyed to the Supreme Court its sense that EDCA needs the Senate's not the presence of foreign military should be allowed. They maintain that as members of the
concurrence to be valid, petitioners continue to suffer from lack of standing. Legislature, they have the requisite personality to bring a suit, especially when a constitutional
issue is raised.
In assailing the constitutionality of a governmental act, petitioners suing as citizens may dodge the
requirement of having to establish a direct and personal interest if they show that the act affects a The OSG counters130 that petitioners do not have any legal standing to file the suits concerning the
public right.120 In arguing that they have legal standing, they claim121 that the case they have filed lack of Senate concurrence in EDCA. Respondent emphasizes that the power to concur in treaties
is a concerned citizen's suit. But aside from general statements that the petitions involve the and international agreements is an "institutional prerogative" granted by the Constitution to the
protection of a public right, and that their constitutional rights as citizens would be violated, they Senate. Accordingly, the OSG argues that in case of an allegation of impairment of that power, the
fail to make any specific assertion of a particular public right that would be violated by the injured party would be the Senate as an institution or any of its incumbent members, as it is the
enforcement of EDCA. For their failure to do so, the present petitions cannot be considered by Senate's constitutional function that is allegedly being violated.
the Court as citizens' suits that would justify a disregard of the aforementioned requirements.

The legal standing of an institution of the Legislature or of any of its Members has already been
In claiming that they have legal standing as taxpayers, petitioners122 aver that the implementation recognized by this Court in a number of cases.131 What is in question here is the alleged
of EDCA would result in the unlawful use of public funds. They emphasize that Article X(1) refers to impairment of the constitutional duties and powers granted to, or the impermissible intrusion
an appropriation of funds; and that the agreement entails a waiver of the payment of taxes, fees, upon the domain of, the Legislature or an institution thereof.132 In the case of suits initiated by the
and rentals. During the oral arguments, however, they admitted that the government had not yet legislators themselves, this Court has recognized their standing to question the validity of any
appropriated or actually disbursed public funds for the purpose of implementing the official action that they claim infringes the prerogatives, powers, and privileges vested by the
agreement.123 The OSG, on the other hand, maintains that petitioners cannot sue as Constitution in their office.133 As aptly explained by Justice Perfecto in Mabanag v. Lopez Vito:134
taxpayers.124Respondent explains that EDCA is neither meant to be a tax measure, nor is it directed
at the disbursement of public funds.
Being members of Congress, they are even duty bound to see that the latter act within the
bounds of the Constitution which, as representatives of the people, they should uphold, unless
A taxpayer's suit concerns a case in which the official act complained of directly involves the illegal they are to commit a flagrant betrayal of public trust. They are representatives of the sovereign
disbursement of public funds derived from taxation.125 Here, those challenging the act must people and it is their sacred duty to see to it that the fundamental law embodying the will of the
specifically show that they have sufficient interest in preventing the illegal expenditure of public sovereign people is not trampled upon. (Emphases supplied)
money, and that they will sustain a direct injury as a result of the enforcement of the assailed
act.126 Applying that principle to this case, they must establish that EDCA involves the exercise by
Congress of its taxing or spending powers.127 We emphasize that in a legislators' suit, those Members of Congress who are challenging the
official act have standing only to the extent that the alleged violation impinges on their right to
participate in the exercise of the powers of the institution of which they are
We agree with the OSG that the petitions cannot qualify as taxpayers' suits. We emphasize that a members.135 Legislators have the standing "to maintain inviolate the prerogatives, powers, and
taxpayers' suit contemplates a situation in which there is already an appropriation or a privileges vested by the Constitution in their office and are allowed to sue to question the validity
disbursement of public funds.128 A reading of Article X(l) of EDCA would show that there has been of any official action, which they claim infringes their prerogatives as legislators."136 As legislators,
neither an appropriation nor an authorization of disbursement of funds. The cited provision reads:
they must clearly show that there was a direct injury to their persons or the institution to which instrumentality of the government; and (3) the lack of any other party that has a more direct and
they belong.137 specific interest in raising the present questions.141

As correctly argued by respondent, the power to concur in a treaty or an international agreement An exhaustive evaluation of the memoranda of the parties, together with the oral arguments,
is an institutional prerogative granted by the Constitution to the Senate, not to the entire shows that petitioners have presented serious constitutional issues that provide ample
Legislature. In Pimentel v. Office of the Executive Secretary, this Court did not recognize the justification for the Court to set aside the rule on standing. The transcendental importance of the
standing of one of the petitioners therein who was a member of the House of Representatives. The issues presented here is rooted in the Constitution itself. Section 25, Article XVIII thereof, cannot
petition in that case sought to compel the transmission to the Senate for concurrence of the be any clearer: there is a much stricter mechanism required before foreign military troops,
signed text of the Statute of the International Criminal Court. Since that petition invoked the facilities, or bases may be allowed in the country. The DFA has already confirmed to the U.S.
power of the Senate to grant or withhold its concurrence in a treaty entered into by the Executive Embassy that "all internal requirements of the Philippines x x x have already been complied
Department, only then incumbent Senator Pimentel was allowed to assert that authority of the with."142 It behooves the Court in this instance to take a liberal stance towards the rule on standing
Senate of which he was a member. and to determine forthwith whether there was grave abuse of discretion on the part of the
Executive Department.
Therefore, none of the initial petitioners in the present controversy has the standing to maintain
the suits as legislators. We therefore rule that this case is a proper subject for judicial review.

Nevertheless, this Court finds that there is basis for it to review the act of the Executive for the B. Whether the President may enter into an executive agreement on foreign military
following reasons. bases, troops, or facilities

In any case, petitioners raise issues involving matters of transcendental importance. C. Whether the provisions under EDCA are consistent with the Constitution, as well as
with existing laws and treaties
Petitioners138 argue that the Court may set aside procedural technicalities, as the present petition
tackles issues that are of transcendental importance. They point out that the matter before us is Issues B and C shall be discussed together infra.
about the proper exercise of the Executive Department's power to enter into international
agreements in relation to that of the Senate to concur in those agreements. They also assert that 1. The role of the President as the executor of the law includes the duty to defend the State, for
EDCA would cause grave injustice, as well as irreparable violation of the Constitution and of the which purpose he may use that power in the conduct of foreign relations
Filipino people's rights.

Historically, the Philippines has mirrored the division of powers in the U.S. government. When the
The OSG, on the other hand, insists139 that petitioners cannot raise the mere fact that the present Philippine government was still an agency of the Congress of the U.S., it was as an agent entrusted
petitions involve matters of transcendental importance in order to cure their inability to comply with powers categorized as executive, legislative, and judicial, and divided among these three
with the constitutional requirement of standing. Respondent bewails the overuse of great branches.143 By this division, the law implied that the divided powers cannot be exercised
"transcendental importance" as an exception to the traditional requirements of constitutional except by the department given the power.144
litigation. It stresses that one of the purposes of these requirements is to protect the Supreme
Court from unnecessary litigation of constitutional questions.
This divide continued throughout the different versions of the Philippine Constitution and
specifically vested the supreme executive power in the Governor-General of the Philippines,145 a
In a number of cases,140 this Court has indeed taken a liberal stance towards the requirement of position inherited by the President of the Philippines when the country attained independence.
legal standing, especially when paramount interest is involved. Indeed, when those who challenge One of the principal functions of the supreme executive is the responsibility for the faithful
the official act are able to craft an issue of transcendental significance to the people, the Court execution of the laws as embodied by the oath of office.146 The oath of the President prescribed by
may exercise its sound discretion and take cognizance of the suit. It may do so in spite of the the 1987 Constitution reads thus:
inability of the petitioners to show that they have been personally injured by the operation of a
law or any other government act.
I do solemnly swear (or affirm) that I will faithfully and conscientiously fulfill my duties as
President (or Vice-President or Acting President) of the Philippines, preserve and defend its
While this Court has yet to thoroughly delineate the outer limits of this doctrine, we emphasize Constitution, execute its laws, do justice to every man, and consecrate myself to the service of the
that not every other case, however strong public interest may be, can qualify as an issue of Nation. So help me God. (In case of affirmation, last sentence will be omitted.) 147 (Emphases
transcendental importance. Before it can be impelled to brush aside the essential requisites for supplied)
exercising its power of judicial review, it must at the very least consider a number of factors: (1)
the character of the funds or other assets involved in the case; (2) the presence of a clear case of
disregard of a constitutional or statutory prohibition by the public respondent agency or This Court has interpreted the faithful execution clause as an obligation imposed on the President,
and not a separate grant of power.148 Section 1 7, Article VII of the Constitution, expresses this
duty in no uncertain terms and includes it in the provision regarding the President's power of government ill executed, whatever it may be in theory, must be in practice a bad government."
control over the executive department, viz: The mistakes of State governments need not be repeated here.

The President shall have control of all the executive departments, bureaus, and offices. He shall xxxx
ensure that the laws be faithfully executed.
Every other consideration to one side, this remains certain - The Congress of the United States
The equivalent provisions in the next preceding Constitution did not explicitly require this oath clearly intended that the Governor-General's power should be commensurate with his
from the President. In the 1973 Constitution, for instance, the provision simply gives the President responsibility. The Congress never intended that the Governor-General should be saddled with the
control over the ministries.149 A similar language, not in the form of the President's oath, was responsibility of administering the government and of executing the laws but shorn of the power
present in the 1935 Constitution, particularly in the enumeration of executive functions. 150 By to do so. The interests of the Philippines will be best served by strict adherence to the basic
1987, executive power was codified not only in the Constitution, but also in the Administrative principles of constitutional government.
Code:151
In light of this constitutional duty, it is the President's prerogative to do whatever is legal and
SECTION 1. Power of Control. - The President shall have control of all the executive departments, necessary for Philippine defense interests. It is no coincidence that the constitutional provision on
bureaus, and offices. He shall ensure that the laws be faithfully executed. (Emphasis supplied) the faithful execution clause was followed by that on the President's commander-in-chief
powers,164 which are specifically granted during extraordinary events of lawless violence, invasion,
Hence, the duty to faithfully execute the laws of the land is inherent in executive power and is or rebellion. And this duty of defending the country is unceasing, even in times when there is no
intimately related to the other executive functions. These functions include the faithful execution state of lawlesss violence, invasion, or rebellion. At such times, the President has full powers to
of the law in autonomous regions;152 the right to prosecute crimes;153 the implementation of ensure the faithful execution of the laws.
transportation projects;154 the duty to ensure compliance with treaties, executive agreements and
executive orders;155 the authority to deport undesirable aliens;156 the conferment of national It would therefore be remiss for the President and repugnant to the faithful-execution clause of
awards under the President's jurisdiction;157 and the overall administration and control of the the Constitution to do nothing when the call of the moment requires increasing the military's
executive department.158 defensive capabilities, which could include forging alliances with states that hold a common
interest with the Philippines or bringing an international suit against an offending state.
These obligations are as broad as they sound, for a President cannot function with crippled hands,
but must be capable of securing the rule of law within all territories of the Philippine Islands and The context drawn in the analysis above has been termed by Justice Arturo D. Brion's Dissenting
be empowered to do so within constitutional limits. Congress cannot, for instance, limit or take Opinion as the beginning of a "patent misconception."165 His dissent argues that this approach
over the President's power to adopt implementing rules and regulations for a law it has enacted.159 taken in analyzing the President's role as executor of the laws is preceded by the duty to preserve
and defend the Constitution, which was allegedly overlooked.166
More important, this mandate is self-executory by virtue of its being inherently executive in
nature.160 As Justice Antonio T. Carpio previously wrote,161 In arguing against the approach, however, the dissent grossly failed to appreciate the nuances of
the analysis, if read holistically and in context. The concept that the President cannot function with
[i]f the rules are issued by the President in implementation or execution of self-executory crippled hands and therefore can disregard the need for Senate concurrence in treaties167 was
constitutional powers vested in the President, the rule-making power of the President is not a never expressed or implied. Rather, the appropriate reading of the preceding analysis shows that
delegated legislative power. The most important self-executory constitutional power of the the point being elucidated is the reality that the President's duty to execute the laws and protect
President is the President's constitutional duty and mandate to "ensure that the laws be faithfully the Philippines is inextricably interwoven with his foreign affairs powers, such that he must resolve
executed." The rule is that the President can execute the law without any delegation of power issues imbued with both concerns to the full extent of his powers, subject only to the limits
from the legislature. supplied by law. In other words, apart from an expressly mandated limit, or an implied limit by
virtue of incompatibility, the manner of execution by the President must be given utmost
deference. This approach is not different from that taken by the Court in situations with fairly
The import of this characteristic is that the manner of the President's execution of the law, even similar contexts.
if not expressly granted by the law, is justified by necessity and limited only by law, since the
President must "take necessary and proper steps to carry into execution the law."162 Justice
George Malcolm states this principle in a grand manner:163 Thus, the analysis portrayed by the dissent does not give the President authority to bypass
constitutional safeguards and limits. In fact, it specifies what these limitations are, how these
limitations are triggered, how these limitations function, and what can be done within the sphere
The executive should be clothed with sufficient power to administer efficiently the affairs of state. of constitutional duties and limitations of the President.
He should have complete control of the instrumentalities through whom his responsibility is
discharged. It is still true, as said by Hamilton, that "A feeble executive implies a feeble execution
of the government. A feeble execution is but another phrase for a bad execution; and a Justice Brion's dissent likewise misinterprets the analysis proffered when it claims that the foreign
relations power of the President should not be interpreted in isolation. 168 The analysis itself
demonstrates how the foreign affairs function, while mostly the President's, is shared in several on the basis of the best available information and can decide with
instances, namely in Section 2 of Article II on the conduct of war; Sections 20 and 21 of Article VII decisiveness .... It is also the President who possesses the most
on foreign loans, treaties, and international agreements; Sections 4(2) and 5(2)(a) of Article VIII on comprehensive and the most confidential information about foreign countries
the judicial review of executive acts; Sections 4 and 25 of Article XVIII on treaties and international for our diplomatic and consular officials regularly brief him on meaningful
agreements entered into prior to the Constitution and on the presence of foreign military troops, events all over the world. He has also unlimited access to ultra-sensitive
bases, or facilities. military intelligence data. In fine, the presidential role in foreign affairs is
dominant and the President is traditionally accorded a wider degree of
In fact, the analysis devotes a whole subheading to the relationship between the two major discretion in the conduct of foreign affairs. The regularity, nay, validity of his
presidential functions and the role of the Senate in it. actions are adjudged under less stringent standards, lest their judicial
repudiation lead to breach of an international obligation, rupture of state
relations, forfeiture of confidence, national embarrassment and a plethora
This approach of giving utmost deference to presidential initiatives in respect of foreign affairs is of other problems with equally undesirable consequences.169 (Emphases
not novel to the Court. The President's act of treating EDCA as an executive agreement is not the supplied)
principal power being analyzed as the Dissenting Opinion seems to suggest. Rather, the
preliminary analysis is in reference to the expansive power of foreign affairs. We have long treated
this power as something the Courts must not unduly restrict. As we stated recently in Vinuya v. Understandably, this Court must view the instant case with the same perspective and
Romulo: understanding, knowing full well the constitutional and legal repercussions of any judicial
overreach.

To be sure, not all cases implicating foreign relations present political questions, and courts
certainly possess the authority to construe or invalidate treaties and executive agreements. 2. The plain meaning of the Constitution prohibits the entry of foreign military bases, troops or
However, the question whether the Philippine government should espouse claims of its nationals facilities, except by way of a treaty concurred in by the Senate - a clear limitation on the
against a foreign government is a foreign relations matter, the authority for which is demonstrably President's dual role as defender of the State and as sole authority in foreign relations.
committed by our Constitution not to the courts but to the political branches. In this case, the
Executive Department has already decided that it is to the best interest of the country to waive all Despite the President's roles as defender of the State and sole authority in foreign relations, the
claims of its nationals for reparations against Japan in the Treaty of Peace of 1951. The wisdom of 1987 Constitution expressly limits his ability in instances when it involves the entry of foreign
such decision is not for the courts to question. Neither could petitioners herein assail the said military bases, troops or facilities. The initial limitation is found in Section 21 of the provisions on
determination by the Executive Department via the instant petition for certiorari. the Executive Department: "No treaty or international agreement shall be valid and effective
unless concurred in by at least two-thirds of all the Members of the Senate." The specific limitation
In the seminal case of US v. Curtiss-Wright Export Corp., the US Supreme Court held that "[t]he is given by Section 25 of the Transitory Provisions, the full text of which reads as follows:
President is the sole organ of the nation in its external relations, and its sole representative with
foreign relations." SECTION 25. After the expiration in 1991 of the Agreement between the Republic of the
Philippines and the United States of America concerning Military Bases, foreign military bases,
It is quite apparent that if, in the maintenance of our international relations, troops, or facilities shall not be allowed in the Philippines except under a treaty duly concurred in
embarrassment - perhaps serious embarrassment - is to be avoided and by the Senate and, when the Congress so requires, ratified by a majority of the votes cast by the
success for our aims achieved, congressional legislation which is to be made people in a national referendum held for that purpose, and recognized as a treaty by the other
effective through negotiation and inquiry within the international field must contracting State.
often accord to the President a degree of discretion and freedom from
statutory restriction which would not be admissible where domestic affairs It is quite plain that the Transitory Provisions of the 1987 Constitution intended to add to the basic
alone involved. Moreover, he, not Congress, has the better opportunity of requirements of a treaty under Section 21 of Article VII. This means that both provisions must be
knowing the conditions which prevail in foreign countries, and especially is read as additional limitations to the President's overarching executive function in matters of
this true in time of war. He has his confidential sources of information. He has defense and foreign relations.
his agents in the form of diplomatic, consular and other officials ....
3. The President, however, may enter into an executive agreement on foreign military bases,
This ruling has been incorporated in our jurisprudence through Bavan v. Executive troops, or facilities, if (a) it is not the instrument that allows the presence of foreign military
Secretary and Pimentel v. Executive Secretary; its overreaching principle was, perhaps, best bases, troops, or facilities; or (b) it merely aims to implement an existing law or treaty.
articulated in (now Chief) Justice Puno's dissent in Secretary of Justice v. Lantion:
Again we refer to Section 25, Article XVIII of the Constitution:
. . . The conduct of foreign relations is full of complexities and consequences,
sometimes with life and death significance to the nation especially in times of SECTION 25. After the expiration in 1991 of the Agreement between the Republic of the
war. It can only be entrusted to that department of government which can act Philippines and the United States of America concerning Military Bases, foreign military bases,
troops, or facilities shall not be allowed in the Philippines except under a treaty duly concurred Note that the provision "shall not be allowed" is a negative injunction. This wording signifies that
in by the Senate and, when the Congress so requires, ratified by a majority of the votes cast by the the President is not authorized by law to allow foreign military bases, troops, or facilities to enter
people in a national referendum held for that purpose, and recognized as a treaty by the other the Philippines, except under a treaty concurred in by the Senate. Hence, the constitutionally
contracting State. (Emphases supplied) restricted authority pertains to the entry of the bases, troops, or facilities, and not to the activities
to be done after entry.
In view of this provision, petitioners argue170 that EDCA must be in the form of a "treaty" duly
concurred in by the Senate. They stress that the Constitution is unambigous in mandating the Under the principles of constitutional construction, of paramount consideration is the plain
transmission to the Senate of all international agreements concluded after the expiration of the meaning of the language expressed in the Constitution, or the verba legis rule.175 It is presumed
MBA in 1991 - agreements that concern the presence of foreign military bases, troops, or facilities that the provisions have been carefully crafted in order to express the objective it seeks to
in the country. Accordingly, petitioners maintain that the Executive Department is not given the attain.176 It is incumbent upon the Court to refrain from going beyond the plain meaning of the
choice to conclude agreements like EDCA in the form of an executive agreement. words used in the Constitution. It is presumed that the framers and the people meant what they
said when they said it, and that this understanding was reflected in the Constitution and
This is also the view of the Senate, which, through a majority vote of 15 of its members - with 1 understood by the people in the way it was meant to be understood when the fundamental law
against and 2 abstaining - says in SR 105171 that EDCA must be submitted to the Senate in the form was ordained and promulgated.177 As this Court has often said:
of a treaty for concurrence by at least two-thirds of all its members.
We look to the language of the document itself in our search for its meaning. We do not of course
The Senate cites two constitutional provisions (Article VI, Section 21 and Article XVIII, Section 25) stop there, but that is where we begin. It is to be assumed that the words in which constitutional
to support its position. Compared with the lone constitutional provision that the Office of the provisions are couched express the objective sought to be attained. They are to be given their
Solicitor General (OSG) cites, which is Article XVIII, Section 4(2), which includes the ordinary meaning except where technical terms are employed in which case the significance thus
constitutionality of "executive agreement(s)" among the cases subject to the Supreme Court's attached to them prevails. As the Constitution is not primarily a lawyer's document, it being
power of judicial review, the Constitution clearly requires submission of EDCA to the Senate. Two essential for the rule of law to obtain that it should ever be present in the people's
specific provisions versus one general provision means that the specific provisions prevail. The consciousness, its language as much as possible should be understood in the sense they have in
term "executive agreement" is "a term wandering alone in the Constitution, bereft of provenance common use. What it says according to the text of the provision to be construed compels
and an unidentified constitutional mystery." acceptance and negates the power of the courts to alter it, based on the postulate that the
framers and the people mean what they say. Thus, these are the cases where the need for
construction is reduced to a minimum.178(Emphases supplied)
The author of SR 105, Senator Miriam Defensor Santiago, upon interpellation even added that the
MDT, which the Executive claims to be partly implemented through EDCA, is already obsolete.
It is only in those instances in which the constitutional provision is unclear, ambiguous, or silent
that further construction must be done to elicit its meaning.179 In Ang Bagong Bayani-OFW v.
There are two insurmountable obstacles to this Court's agreement with SR 105, as well as with the Commission on Elections,180 we reiterated this guiding principle:
comment on interpellation made by Senator Santiago.

it [is] safer to construe the Constitution from what appears upon its face. The proper
First, the concept of "executive agreement" is so well-entrenched in this Court's pronouncements interpretation therefore depends more on how it was understood by the people adopting it than
on the powers of the President. When the Court validated the concept of "executive agreement," in the framers' understanding thereof. (Emphases supplied)
it did so with full knowledge of the Senate's role in concurring in treaties. It was aware of the
problematique of distinguishing when an international agreement needed Senate concurrence for
validity, and when it did not; and the Court continued to validate the existence of "executive The effect of this statement is surprisingly profound, for, if taken literally, the phrase "shall not be
agreements" even after the 1987 Constitution.172 This follows a long line of similar decisions allowed in the Philippines" plainly refers to the entry of bases, troops, or facilities in the country.
upholding the power of the President to enter into an executive agreement. 173 The Oxford English Dictionary defines the word "allow" as a transitive verb that means "to permit,
enable"; "to give consent to the occurrence of or relax restraint on (an action, event, or activity)";
"to consent to the presence or attendance of (a person)"; and, when with an adverbial of place,
Second, the MDT has not been rendered obsolescent, considering that as late as 2009, 174 this "to permit (a person or animal) to go, come, or be in, out, near, etc."181 Black's Law
Court continued to recognize its validity. Dictionary defines the term as one that means "[t]o grant, approve, or permit."182

Third, to this Court, a plain textual reading of Article XIII, Section 25, inevitably leads to the The verb "allow" is followed by the word "in," which is a preposition used to indicate "place or
conclusion that it applies only to a proposed agreement between our government and a foreign position in space or anything having material extension: Within the limits or bounds of, within (any
government, whereby military bases, troops, or facilities of such foreign government would be place or thing)."183 That something is the Philippines, which is the noun that follows.
"allowed" or would "gain entry" Philippine territory.
It is evident that the constitutional restriction refers solely to the initial entry of the foreign
military bases, troops, or facilities. Once entry is authorized, the subsequent acts are thereafter
subject only to the limitations provided by the rest of the Constitution and Philippine law, and not attached to them prevails. As the Constitution is not primarily a lawyer's document, it being
to the Section 25 requirement of validity through a treaty. essential for the rule of law to obtain that it should ever be present in the people's
consciousness, its language as much as possible should be understood in the sense they have in
The VFA has already allowed the entry of troops in the Philippines. This Court stated in Lim v. common use. What it says according to the text of the provision to be construed compels
Executive Secretary: acceptance and negates the power of the courts to alter it, based on the postulate that the
framers and the people mean what they say. Thus, these are the cases where the need for
construction is reduced to a minimum.190(Emphases supplied)
After studied reflection, it appeared farfetched that the ambiguity surrounding the meaning of the
word "activities" arose from accident. In our view, it was deliberately made that way to give both
parties a certain leeway in negotiation. In this manner, visiting US forces may sojourn in As applied, verba legis aids in construing the ordinary meaning of terms. In this case, the phrase
Philippine territory for purposes other than military. As conceived, the joint exercises may include being construed is "shall not be allowed in the Philippines" and not the preceding one referring to
training on new techniques of patrol and surveillance to protect the nation's marine resources, sea "the expiration in 1991 of the Agreement between the Republic of the Philippines and the United
search-and-rescue operations to assist vessels in distress, disaster relief operations, civic action States of America concerning Military Bases, foreign military bases, troops, or facilities." It is
projects such as the building of school houses, medical and humanitarian missions, and the like. explicit in the wording of the provision itself that any interpretation goes beyond the text itself and
into the discussion of the framers, the context of the Constitutional Commission's time of drafting,
and the history of the 1947 MBA. Without reference to these factors, a reader would not
Under these auspices, the VFA gives legitimacy to the current Balikatan exercises. It is only logical understand those terms. However, for the phrase "shall not be allowed in the Philippines," there is
to assume that "Balikatan 02-1," a "mutual anti- terrorism advising, assisting and training no need for such reference. The law is clear. No less than the Senate understood this when it
exercise," falls under the umbrella of sanctioned or allowable activities in the context of the ratified the VFA.
agreement. Both the history and intent of the Mutual Defense Treaty and the VFA support the
conclusion that combat-related activities -as opposed to combat itself-such as the one subject of
the instant petition, are indeed authorized.184 (Emphasis supplied) 4. The President may generally enter into executive agreements subject to limitations defined by
the Constitution and may be in furtherance of a treaty already concurred in by the Senate.

Moreover, the Court indicated that the Constitution continues to govern the conduct of foreign
military troops in the Philippines,185 readily implying the legality of their initial entry into the We discuss in this section why the President can enter into executive agreements.
country.
It would be helpful to put into context the contested language found in Article XVIII, Section 25. Its
The OSG emphasizes that EDCA can be in the form of an executive agreement, since it merely more exacting requirement was introduced because of the previous experience of the country
involves "adjustments in detail" in the implementation of the MDT and the VFA.186 It points out when its representatives felt compelled to consent to the old MBA.191 They felt constrained to
that there are existing treaties between the Philippines and the U.S. that have already been agree to the MBA in fulfilment of one of the major conditions for the country to gain
concurred in by the Philippine Senate and have thereby met the requirements of the Constitution independence from the U.S.192 As a result of that experience, a second layer of consent for
under Section 25. Because of the status of these prior agreements, respondent emphasizes that agreements that allow military bases, troops and facilities in the country is now articulated in
EDCA need not be transmitted to the Senate. Article XVIII of our present Constitution.

The aforecited Dissenting Opinion of Justice Brion disagrees with the ponencia's application This second layer of consent, however, cannot be interpreted in such a way that we completely
of verba legis construction to the words of Article XVIII, Section 25.187 It claims that the provision is ignore the intent of our constitutional framers when they provided for that additional layer, nor
"neither plain, nor that simple."188 To buttress its disagreement, the dissent states that the the vigorous statements of this Court that affirm the continued existence of that class of
provision refers to a historical incident, which is the expiration of the 1947 MBA.189 Accordingly, international agreements called "executive agreements."
this position requires questioning the circumstances that led to the historical event, and the
meaning of the terms under Article XVIII, Section 25. The power of the President to enter into binding executive agreements without Senate
concurrence is already well-established in this jurisdiction.193 That power has been alluded to in
This objection is quite strange. The construction technique of verba legis is not inapplicable just our present and past Constitutions,194 in various statutes,195 in Supreme Court decisions,196 and
because a provision has a specific historical context. In fact, every provision of the Constitution has during the deliberations of the Constitutional Commission.197 They cover a wide array of subjects
a specific historical context. The purpose of constitutional and statutory construction is to set tiers with varying scopes and purposes,198 including those that involve the presence of foreign military
of interpretation to guide the Court as to how a particular provision functions. Verba legis is of forces in the country.199
paramount consideration, but it is not the only consideration. As this Court has often said:
As the sole organ of our foreign relations200 and the constitutionally assigned chief architect of our
We look to the language of the document itself in our search for its meaning. We do not of course foreign policy,201the President is vested with the exclusive power to conduct and manage the
stop there, but that is where we begin. It is to be assumed that the words in which constitutional country's interface with other states and governments. Being the principal representative of the
provisions are couched express the objective sought to be attained. They are to be given their Philippines, the Chief Executive speaks and listens for the nation; initiates, maintains, and develops
ordinary meaning except where technical terms are employed in which case the significance thus diplomatic relations with other states and governments; negotiates and enters into international
agreements; promotes trade, investments, tourism and other economic relations; and settles One of the distinguishing features of executive agreements is that their validity and effectivity are
international disputes with other states.202 not affected by a lack of Senate concurrence.206 This distinctive feature was recognized as early as
in Eastern Sea Trading (1961), viz:
As previously discussed, this constitutional mandate emanates from the inherent power of the
President to enter into agreements with other states, including the prerogative to Treaties are formal documents which require ratification with the approval of two-thirds of the
conclude binding executive agreements that do not require further Senate concurrence. The Senate. Executive agreements become binding through executive action without the need of a
existence of this presidential power203 is so well-entrenched that Section 5(2)(a), Article VIII of the vote by the Senate or by Congress.
Constitution, even provides for a check on its exercise. As expressed below, executive agreements
are among those official governmental acts that can be the subject of this Court's power of judicial xxxx
review:

[T]he right of the Executive to enter into binding agreements without the necessity of
(2) Review, revise, reverse, modify, or affirm on appeal or certiorari, as the law or the subsequent Congressional approval has been confirmed by long usage. From the earliest days of
Rules of Court may provide, final judgments and orders of lower courts in: our history we have entered into executive agreements covering such subjects as commercial and
consular relations, most-favored-nation rights, patent rights, trademark and copyright protection,
(a) All cases in which the constitutionality or postal and navigation arrangements and the settlement of claims. The validity of these has never
validity of any treaty, international or executive agreement, law, presidential been seriously questioned by our courts. (Emphases Supplied)
decree, proclamation, order, instruction, ordinance, or regulation is in
question. (Emphases supplied) That notion was carried over to the present Constitution. In fact, the framers specifically
deliberated on whether the general term "international agreement" included executive
In Commissioner of Customs v. Eastern Sea Trading, executive agreements are defined as agreements, and whether it was necessary to include an express proviso that would exclude
"international agreements embodying adjustments of detail carrying out well-established national executive agreements from the requirement of Senate concurrence. After noted constitutionalist
policies and traditions and those involving arrangements of a more or less temporary Fr. Joaquin Bernas quoted the Court's ruling in Eastern Sea Trading, the Constitutional Commission
nature."204 In Bayan Muna v. Romulo, this Court further clarified that executive agreements can members ultimately decided that the term "international agreements" as contemplated in Section
cover a wide array of subjects that have various scopes and purposes.205 They are no longer limited 21, Article VII, does not include executive agreements, and that a proviso is no longer needed.
to the traditional subjects that are usually covered by executive agreements as identified Their discussion is reproduced below:207
in Eastern Sea Trading. The Court thoroughly discussed this matter in the following manner:
MS. AQUINO: Madam President, first I would like a clarification from the Committee. We have
The categorization of subject matters that may be covered by international retained the words "international agreement" which I think is the correct judgment on the matter
agreementsmentioned in Eastern Sea Trading is not cast in stone. x x x. because an international agreement is different from a treaty. A treaty is a contract between
parties which is in the nature of international agreement and also a municipal law in the sense that
As may be noted, almost half a century has elapsed since the Court rendered its decision the people are bound. So there is a conceptual difference. However, I would like to be clarified if
in Eastern Sea Trading. Since then, the conduct of foreign affairs has become more complex and the international agreements include executive agreements.
the domain of international law wider, as to include such subjects as human rights, the
environment, and the sea. In fact, in the US alone, the executive agreements executed by its MR. CONCEPCION: That depends upon the parties. All parties to these international negotiations
President from 1980 to 2000 covered subjects such as defense, trade, scientific cooperation, stipulate the conditions which are necessary for the agreement or whatever it may be to become
aviation, atomic energy, environmental cooperation, peace corps, arms limitation, and nuclear valid or effective as regards the parties.
safety, among others. Surely, the enumeration in Eastern Sea Trading cannot circumscribe the
option of each state on the matter of which the international agreement format would be MS. AQUINO: Would that depend on the parties or would that depend on the nature of the
convenient to serve its best interest. As Francis Sayre said in his work referred to earlier: executive agreement? According to common usage, there are two types of executive
agreement: one is purely proceeding from an executive act which affects external relations
. . . It would be useless to undertake to discuss here the large variety of executive agreements as independent of the legislative and the other is an executive act in pursuance of legislative
such concluded from time to time. Hundreds of executive agreements, other than those entered authorization. The first kind might take the form of just conventions or exchanges of notes or
into under the trade-agreement act, have been negotiated with foreign governments. . . . They protocol while the other, which would be pursuant to the legislative authorization, may be in
cover such subjects as the inspection of vessels, navigation dues, income tax on shipping profits, the nature of commercial agreements.
the admission of civil air craft, custom matters and commercial relations generally, international
claims, postal matters, the registration of trademarks and copyrights, etc .... (Emphases Supplied) MR. CONCEPCION: Executive agreements are generally made to implement a treaty already
enforced or to determine the details for the implementation of the treaty. We are speaking of
executive agreements, not international agreements.
MS. AQUINO: I am in full agreement with that, except that it does not cover the first kind of THE PRESIDENT: Is Commissioner Aquino satisfied?
executive agreement which is just protocol or an exchange of notes and this would be in the
nature of reinforcement of claims of a citizen against a country, for example. MS. AQUINO: Yes. There is already an agreement among us on the definition of "executive
agreements" and that would make unnecessary any explicit proviso on the matter.
MR. CONCEPCION: The Commissioner is free to require ratification for validity insofar as the
Philippines is concerned. xxx

MS. AQUINO: It is my humble submission that we should provide, unless the Committee explains MR. GUINGONA: I am not clear as to the meaning of "executive agreements" because I heard that
to us otherwise, an explicit proviso which would except executive agreements from these executive agreements must rely on treaties. In other words, there must first be treaties.
the requirement of concurrence of two-thirds of the Members of the Senate. Unless I am
enlightened by the Committee I propose that tentatively, the sentence should read. "No treaty or
international agreement EXCEPT EXECUTIVE AGREEMENTS shall be valid and effective." MR. CONCEPCION: No, I was speaking about the common use, as executive agreements being the
implementation of treaties, details of which do not affect the sovereignty of the State.

FR. BERNAS: I wonder if a quotation from the Supreme Court decision [in Eastern Sea Trading]
might help clarify this: MR. GUINGONA: But what about the matter of permanence, Madam President? Would 99 years
be considered permanent? What would be the measure of permanency? I do not conceive of a
treaty that is going to be forever, so there must be some kind of a time limit.
The right of the executive to enter into binding agreements without the necessity of subsequent
Congressional approval has been confirmed by long usage. From the earliest days of our history,
we have entered into executive agreements covering such subjects as commercial and consular MR. CONCEPCION: I suppose the Commissioner's question is whether this type of agreement
relations, most favored nation rights, patent rights, trademark and copyright protection, postal should be included in a provision of the Constitution requiring the concurrence of Congress.
and navigation arrangements and the settlement of claims. The validity of this has never been
seriously questioned by our Courts. MR. GUINGONA: It depends on the concept of the executive agreement of which I am not clear. If
the executive agreement partakes of the nature of a treaty, then it should also be included.
Agreements with respect to the registration of trademarks have been concluded by the executive
of various countries under the Act of Congress of March 3, 1881 (21 Stat. 502) . . . International MR. CONCEPCION: Whether it partakes or not of the nature of a treaty, it is within the power of
agreements involving political issues or changes of national policy and those involving the Constitutional Commission to require that.
international agreements of a permanent character usually take the form of treaties. But
international agreements embodying adjustments of detail, carrying out well established national MR. GUINGONA: Yes. That is why I am trying to clarify whether the words "international
policies and traditions and those involving arrangements of a more or less temporary agreements" would include executive agreements.
nature usually take the form of executive agreements.

MR. CONCEPCION: No, not necessarily; generally no.


MR. ROMULO: Is the Commissioner, therefore, excluding the executive agreements?

xxx
FR. BERNAS: What we are referring to, therefore, when we say international agreements which
need concurrence by at least two-thirds are those which are permanent in nature.
MR. ROMULO: I wish to be recognized first. I have only one question. Do we take it, therefore,
that as far as the Committee is concerned, the term "international agreements" does not include
MS. AQUINO: And it may include commercial agreements which are executive agreements the term "executive agreements" as read by the Commissioner in that text?
essentially but which are proceeding from the authorization of Congress. If that is our
understanding, then I am willing to withdraw that amendment.
FR. BERNAS: Yes. (Emphases Supplied)
FR. BERNAS: If it is with prior authorization of Congress, then it does not need subsequent
concurrence by Congress. The inapplicability to executive agreements of the requirements under Section 21 was again
recognized in Bayan v. Zamora and in Bayan Muna v. Romulo. These cases, both decided under the
aegis of the present Constitution, quoted Eastern Sea Trading in reiterating that executive
MS. AQUINO: In that case, I am withdrawing my amendment. agreements are valid and binding even without the concurrence of the Senate.

MR. TINGSON: Madam President. Executive agreements may dispense with the requirement of Senate concurrence because of the
legal mandate with which they are concluded. As culled from the afore-quoted deliberations of the
Constitutional Commission, past Supreme Court Decisions, and works of noted b. The fixing of tariff rates, import and export quotas, tonnage and wharfage dues, and
scholars,208 executive agreements merely involve arrangements on the implementation other duties or imposts, which must be pursuant to the authority granted by Congress 222
of existing policies, rules, laws, or agreements. They are concluded (1) to adjust the details of a
treaty;209 (2) pursuant to or upon confirmation by an act of the Legislature;210 or (3) in the exercise c. The grant of any tax exemption, which must be pursuant to a law concurred in by a
of the President's independent powers under the Constitution. 211 The raison d'etre of executive majority of all the Members of Congress223
agreements hinges on prior constitutional or legislative authorizations.

d. The contracting or guaranteeing, on behalf of the Philippines, of foreign loans that


The special nature of an executive agreement is not just a domestic variation in international must be previously concurred in by the Monetary Board224
agreements. International practice has accepted the use of various forms and designations of
international agreements, ranging from the traditional notion of a treaty - which connotes a
formal, solemn instrument - to engagements concluded in modem, simplified forms that no longer e. The authorization of the presence of foreign military bases, troops, or facilities in the
necessitate ratification.212 An international agreement may take different forms: treaty, act, country must be in the form of a treaty duly concurred in by the Senate.225
protocol, agreement, concordat, compromis d'arbitrage, convention, covenant, declaration,
exchange of notes, statute, pact, charter, agreed minute, memorandum of agreement, modus f. For agreements that do not fall under paragraph 5, the concurrence of the Senate is
vivendi, or some other form.213 Consequently, under international law, the distinction between a required, should the form of the government chosen be a treaty.
treaty and an international agreement or even an executive agreement is irrelevant for purposes
of determining international rights and obligations. 5. The President had the choice to enter into EDCA by way of an executive agreement or a
treaty.
However, this principle does not mean that the domestic law distinguishing treaties, international
agreements, and executive agreements is relegated to a mere variation in form, or that the No court can tell the President to desist from choosing an executive agreement over a treaty to
constitutional requirement of Senate concurrence is demoted to an optional constitutional embody an international agreement, unless the case falls squarely within Article VIII, Section 25.
directive. There remain two very important features that distinguish treaties from executive
agreements and translate them into terms of art in the domestic setting.
As can be gleaned from the debates among the members of the Constitutional Commission, they
were aware that legally binding international agreements were being entered into by countries in
First, executive agreements must remain traceable to an express or implied authorization under forms other than a treaty. At the same time, it is clear that they were also keen to preserve the
the Constitution, statutes, or treaties. The absence of these precedents puts the validity and concept of "executive agreements" and the right of the President to enter into such agreements.
effectivity of executive agreements under serious question for the main function of the Executive
is to enforce the Constitution and the laws enacted by the Legislature, not to defeat or interfere in
the performance of these rules.214 In turn, executive agreements cannot create new international What we can glean from the discussions of the Constitutional Commissioners is that they
obligations that are not expressly allowed or reasonably implied in the law they purport to understood the following realities:
implement.
1. Treaties, international agreements, and executive agreements are all constitutional
Second, treaties are, by their very nature, considered superior to executive agreements. Treaties manifestations of the conduct of foreign affairs with their distinct legal characteristics.
are products of the acts of the Executive and the Senate215 unlike executive agreements, which are
solely executive actions.216Because of legislative participation through the Senate, a treaty is a. Treaties are formal contracts between the Philippines and other States-
regarded as being on the same level as a statute.217 If there is an irreconcilable conflict, a later law parties, which are in the nature of international agreements, and also of
or treaty takes precedence over one that is prior.218 An executive agreement is treated differently. municipal laws in the sense of their binding nature.226
Executive agreements that are inconsistent with either a law or a treaty are considered
ineffective.219 Both types of international agreement are nevertheless subject to the supremacy of
b. International agreements are similar instruments, the provisions of which
the Constitution.220
may require the ratification of a designated number of parties thereto. These
agreements involving political issues or changes in national policy, as well as
This rule does not imply, though, that the President is given carte blanche to exercise this those involving international agreements of a permanent character, usually
discretion. Although the Chief Executive wields the exclusive authority to conduct our foreign take the form of treaties. They may also include commercial agreements,
relations, this power must still be exercised within the context and the parameters set by the which are executive agreements essentially, but which proceed from previous
Constitution, as well as by existing domestic and international laws. There are constitutional authorization by Congress, thus dispensing with the requirement of
provisions that restrict or limit the President's prerogative in concluding international agreements, concurrence by the Senate.227
such as those that involve the following:

a. The policy of freedom from nuclear weapons within Philippine territory221


c. Executive agreements are generally intended to implement a treaty already x x x. As the President wields vast powers and influence, her conduct in the external affairs of the
enforced or to determine the details of the implementation thereof that do nation is, as Bayan would put it, "executive altogether." The right of the President to enter into or
not affect the sovereignty of the State.228 ratify binding executive agreements has been confirmed by long practice.

2. Treaties and international agreements that cannot be mere executive agreements In thus agreeing to conclude the Agreement thru E/N BF0-028-03, then President Gloria
must, by constitutional decree, be concurred in by at least two-thirds of the Senate. Macapagal-Arroyo, represented by the Secretary of Foreign Affairs, acted within the scope of the
authority and discretion vested in her by the Constitution. At the end of the day, the President -
3. However, an agreement - the subject of which is the entry of foreign military troops, by ratifying, thru her deputies, the non-surrender agreement - did nothing more than discharge
bases, or facilities - is particularly restricted. The requirements are that it be in the form a constitutional duty and exercise a prerogative that pertains to her office. (Emphases supplied)
of a treaty concurred in by the Senate; that when Congress so requires, it be ratified by
a majority of the votes cast by the people in a national referendum held for that Indeed, in the field of external affairs, the President must be given a larger measure of authority
purpose; and that it be recognized as a treaty by the other contracting State. and wider discretion, subject only to the least amount of checks and restrictions under the
Constitution.229 The rationale behind this power and discretion was recognized by the Court
4. Thus, executive agreements can continue to exist as a species of international in Vinuya v. Executive Secretary, cited earlier.230
agreements.
Section 9 of Executive Order No. 459, or the Guidelines in the Negotiation of International
That is why our Court has ruled the way it has in several cases. Agreements and its Ratification, thus, correctly reflected the inherent powers of the President
when it stated that the DFA "shall determine whether an agreement is an executive agreement or
a treaty."
In Bayan Muna v. Romulo, we ruled that the President acted within the scope of her constitutional
authority and discretion when she chose to enter into the RP-U.S. Non-Surrender Agreement in
the form of an executive agreement, instead of a treaty, and in ratifying the agreement without Accordingly, in the exercise of its power of judicial review, the Court does not look into whether an
Senate concurrence. The Court en banc discussed this intrinsic presidential prerogative as follows: international agreement should be in the form of a treaty or an executive agreement, save in cases
in which the Constitution or a statute requires otherwise. Rather, in view of the vast constitutional
powers and prerogatives granted to the President in the field of foreign affairs, the task of the
Petitioner parlays the notion that the Agreement is of dubious validity, partaking as it does of the Court is to determine whether the international agreement is consistent with the applicable
nature of a treaty; hence, it must be duly concurred in by the Senate. x x x x. Pressing its point, limitations.
petitioner submits that the subject of the Agreement does not fall under any of the subject-
categories that xx x may be covered by an executive agreement, such as commercial/consular
relations, most-favored nation rights, patent rights, trademark and copyright protection, postal 6. Executive agreements may cover the matter of foreign military forces if it merely involves
and navigation arrangements and settlement of claims. detail adjustments.

The categorization of subject matters that may be covered by international agreements mentioned The practice of resorting to executive agreements in adjusting the details of a law or a treaty that
in Eastern Sea Trading is not cast in stone. There are no hard and fast rules on the propriety of already deals with the presence of foreign military forces is not at all unusual in this jurisdiction. In
entering, on a given subject, into a treaty or an executive agreement as an instrument of fact, the Court has already implicitly acknowledged this practice in Lim v. Executive Secretary.231 In
international relations. The primary consideration in the choice of the form of agreement is that case, the Court was asked to scrutinize the constitutionality of the Terms of Reference of
the parties' intent and desire to craft an international agreement in the form they so wish to the Balikatan 02-1 joint military exercises, which sought to implement the VFA. Concluded in the
further their respective interests. Verily, the matter of form takes a back seat when it comes to form of an executive agreement, the Terms of Reference detailed the coverage of the term
effectiveness and binding effect of the enforcement of a treaty or an executive agreement, as the "activities" mentioned in the treaty and settled the matters pertaining to the construction of
parties in either international agreement each labor under the pacta sunt servanda principle. temporary structures for the U.S. troops during the activities; the duration and location of the
exercises; the number of participants; and the extent of and limitations on the activities of the U.S.
forces. The Court upheld the Terms of Reference as being consistent with the VFA. It no longer
xxxx took issue with the fact that the Balikatan Terms of Reference was not in the form of a treaty
concurred in by the Senate, even if it dealt with the regulation of the activities of foreign military
But over and above the foregoing considerations is the fact that - save for the situation and forces on Philippine territory.
matters contemplated in Sec. 25, Art. XVIII of the Constitution - when a treaty is required, the
Constitution does not classify any subject, like that involving political issues, to be in the form of, In Nicolas v. Romulo,232 the Court again impliedly affirmed the use of an executive agreement in an
and ratified as, a treaty. What the Constitution merely prescribes is that treaties need the attempt to adjust the details of a provision of the VFA. The Philippines and the U.S. entered into
concurrence of the Senate by a vote defined therein to complete the ratification process. the Romulo-Kenney Agreement, which undertook to clarify the detention of a U.S. Armed Forces
member, whose case was pending appeal after his conviction by a trial court for the crime of rape.
xxxx In testing the validity of the latter agreement, the Court precisely alluded to one of the inherent
limitations of an executive agreement: it cannot go beyond the terms of the treaty it purports to negotiation."239 The Court eventually ruled that the Terms of Reference fell within the sanctioned
implement. It was eventually ruled that the Romulo-Kenney Agreement was "not in accord" with or allowable activities, especially in the context of the VFA and the MDT.
the VFA, since the former was squarely inconsistent with a provision in the treaty requiring that
the detention be "by Philippine authorities." Consequently, the Court ordered the Secretary of The Court applied the same approach to Nicolas v. Romulo. It studied the provisions of the VFA on
Foreign Affairs to comply with the VFA and "forthwith negotiate with the United States custody and detention to ascertain the validity of the Romulo-Kenney Agreement.240 It eventually
representatives for the appropriate agreement on detention facilities under Philippine authorities found that the two international agreements were not in accord, since the Romulo-Kenney
as provided in Art. V, Sec. 10 of the VFA. "233 Agreement had stipulated that U.S. military personnel shall be detained at the U.S. Embassy
Compound and guarded by U.S. military personnel, instead of by Philippine authorities. According
Culling from the foregoing discussions, we reiterate the following pronouncements to guide us in to the Court, the parties "recognized the difference between custody during the trial and
resolving the present controversy: detention after conviction."241 Pursuant to Article V(6) of the VFA, the custody of a U.S. military
personnel resides with U.S. military authorities during trial. Once there is a finding of guilt, Article
1. Section 25, Article XVIII of the Constitution, contains stringent requirements that must V(l0) requires that the confinement or detention be "by Philippine authorities."
be fulfilled by the international agreement allowing the presence of foreign military
bases, troops, or facilities in the Philippines: (a) the agreement must be in the form of a Justice Marvic M.V.F. Leonen's Dissenting Opinion posits that EDCA "substantially modifies or
treaty, and (b) it must be duly concurred in by the Senate. amends the VFA"242and follows with an enumeration of the differences between EDCA and the
VFA. While these arguments will be rebutted more fully further on, an initial answer can already be
2. If the agreement is not covered by the above situation, then the President may given to each of the concerns raised by his dissent.
choose the form of the agreement (i.e., either an executive agreement or a treaty),
provided that the agreement dealing with foreign military bases, troops, or facilities is The first difference emphasized is that EDCA does not only regulate visits as the VFA does, but
not the principal agreement that first allows their entry or presence in the Philippines. allows temporary stationing on a rotational basis of U.S. military personnel and their contractors in
physical locations with permanent facilities and pre-positioned military materiel.
3. The executive agreement must not go beyond the parameters, limitations, and
standards set by the law and/or treaty that the former purports to implement; and must This argument does not take into account that these permanent facilities, while built by U.S.
not unduly expand the international obligation expressly mentioned or necessarily forces, are to be owned by the Philippines once constructed.243 Even the VFA allowed construction
implied in the law or treaty. for the benefit of U.S. forces during their temporary visits.

4. The executive agreement must be consistent with the Constitution, as well as with The second difference stated by the dissent is that EDCA allows the prepositioning of military
existing laws and treaties. materiel, which can include various types of warships, fighter planes, bombers, and vessels, as well
as land and amphibious vehicles and their corresponding ammunition. 244
In light of the President's choice to enter into EDCA in the form of an executive agreement,
respondents carry the burden of proving that it is a mere implementation of existing laws and However, the VFA clearly allows the same kind of equipment, vehicles, vessels, and aircraft to be
treaties concurred in by the Senate. EDCA must thus be carefully dissected to ascertain if it brought into the country. Articles VII and VIII of the VFA contemplates that U.S. equipment,
remains within the legal parameters of a valid executive agreement. materials, supplies, and other property are imported into or acquired in the Philippines by or on
behalf of the U.S. Armed Forces; as are vehicles, vessels, and aircraft operated by or for U.S. forces
7. EDCA is consistent with the content, purpose, and framework of the MDT and the VFA in connection with activities under the VFA. These provisions likewise provide for the waiver of the
specific duties, taxes, charges, and fees that correspond to these equipment.

The starting point of our analysis is the rule that "an executive agreement xx x may not be used to
amend a treaty."234 In Lim v. Executive Secretary and in Nicolas v. Romulo, the Court approached The third difference adverted to by the Justice Leonen's dissent is that the VFA contemplates the
the question of the validity of executive agreements by comparing them with the general entry of troops for training exercises, whereas EDCA allows the use of territory for launching
framework and the specific provisions of the treaties they seek to implement. military and paramilitary operations conducted in other states.245 The dissent of Justice Teresita J.
Leonardo-De Castro also notes that VFA was intended for non-combat activides only, whereas the
entry and activities of U.S. forces into Agreed Locations were borne of military necessity or had a
In Lim, the Terms of Reference of the joint military exercises was scrutinized by studying "the martial character, and were therefore not contemplated by the VFA.246
framework of the treaty antecedents to which the Philippines bound itself,"235 i.e., the MDT and
the VFA. The Court proceeded to examine the extent of the term "activities" as contemplated in
Articles 1236 and II237 of the VFA. It later on found that the term "activities" was deliberately left This Court's jurisprudence however established in no uncertain terms that combat-related
undefined and ambiguous in order to permit "a wide scope of undertakings subject only to the activities, as opposed to actual combat, were allowed under the MDT and VFA, viz:
approval of the Philippine government"238 and thereby allow the parties "a certain leeway in
Both the history and intent of the Mutual Defense Treaty and the VFA support the conclusion that VFA, which already allows the entry of U.S. troops and personnel into the country. Respondent
combat-related activities as opposed to combat itself such as the one subject of the instant stresses this Court's recognition in Lim v. Executive Secretary that U.S. troops and personnel are
petition, are indeed authorized.247 authorized to conduct activities that promote the goal of maintaining and developing their defense
capability.
Hence, even if EDCA was borne of military necessity, it cannot be said to have strayed from the
intent of the VFA since EDCA's combat-related components are allowed under the treaty. Petitioners contest251 the assertion that the provisions of EDCA merely implement the MDT.
According to them, the treaty does not specifically authorize the entry of U.S. troops in the country
Moreover, both the VFA and EDCA are silent on what these activities actually are. Both the VFA in order to maintain and develop the individual and collective capacities of both the Philippines
and EDCA deal with the presence of U.S. forces within the Philippines, but make no mention of and the U.S. to resist an armed attack. They emphasize that the treaty was concluded at a time
being platforms for activity beyond Philippine territory. While it may be that, as applied, military when there was as yet no specific constitutional prohibition on the presence of foreign military
operations under either the VFA or EDCA would be carried out in the future the scope of judicial forces in the country.
review does not cover potential breaches of discretion but only actual occurrences or blatantly
illegal provisions. Hence, we cannot invalidate EDCA on the basis of the potentially abusive use of Petitioners also challenge the argument that EDCA simply implements the VFA. They assert that
its provisions. the agreement covers only short-term or temporary visits of U.S. troops "from time to time" for
the specific purpose of combined military exercises with their Filipino counterparts. They stress
The fourth difference is that EDCA supposedly introduces a new concept not contemplated in the that, in contrast, U.S. troops are allowed under EDCA to perform activities beyond combined
VFA or the MDT: Agreed Locations, Contractors, Pre-positioning, and Operational Control.248 military exercises, such as those enumerated in Articles 111(1) and IV(4) thereof. Furthermore,
there is some degree of permanence in the presence of U.S. troops in the country, since the
effectivity of EDCA is continuous until terminated. They proceed to argue that while troops have a
As previously mentioned, these points shall be addressed fully and individually in the latter "rotational" presence, this scheme in fact fosters their permanent presence.
analysis of EDCA's provisions. However, it must already be clarified that the terms and details used
by an implementing agreement need not be found in the mother treaty. They must be sourced
from the authority derived from the treaty, but are not necessarily expressed word-for-word in the a. Admission of U.S. military and civilian personnel into Philippine territory is already allowed under
mother treaty. This concern shall be further elucidated in this Decision. the VFA

The fifth difference highlighted by the Dissenting Opinion is that the VFA does not have provisions We shall first deal with the recognition under EDCA of the presence in the country of three distinct
that may be construed as a restriction on or modification of obligations found in existing statues, classes of individuals who will be conducting different types of activities within the Agreed
including the jurisdiction of courts, local autonomy, and taxation. Implied in this argument is that Locations: (1) U.S. military personnel; (2) U.S. civilian personnel; and (3) U.S. contractors. The
EDCA contains such restrictions or modifications.249 agreement refers to them as follows:

This last argument cannot be accepted in view of the clear provisions of EDCA. Both the VFA and "United States personnel" means United States military and civilian personnel temporarily in the
EDCA ensure Philippine jurisdiction in all instances contemplated by both agreements, with the territory of the Philippines in connection with activities approved by the Philippines, as those
exception of those outlined by the VFA in Articles III-VI. In the VFA, taxes are clearly waived terms are defined in the VFA.252
whereas in EDCA, taxes are assumed by the government as will be discussed later on. This fact
does not, therefore, produce a diminution of jurisdiction on the part of the Philippines, but rather "United States forces" means the entity comprising United States personnel and all property,
a recognition of sovereignty and the rights that attend it, some of which may be waived as in the equipment, and materiel of the United States Armed Forces present in the territory of the
cases under Articles III-VI of the VFA. Philippines.253

Taking off from these concerns, the provisions of EDCA must be compared with those of the MDT "United States contractors" means companies and firms, and their employees, under contract or
and the VFA, which are the two treaties from which EDCA allegedly draws its validity. subcontract to or on behalf of the United States Department of Defense. United States contractors
are not included as part of the definition of United States personnel in this Agreement, including
"Authorized presence" under the VFA versus "authorized activities" under EDCA: (1) U.S. within the context of the VFA.254
personnel and (2) U.S. contractors
United States forces may contract for any materiel, supplies, equipment, and services (including
The OSG argues250 that EDCA merely details existing policies under the MDT and the VFA. It construction) to be furnished or undertaken in the territory of the Philippines without restriction
explains that EDCA articulates the principle of defensive preparation embodied in Article II of the as to choice of contractor, supplier, or person who provides such materiel, supplies, equipment,
MDT; and seeks to enhance the defensive, strategic, and technological capabilities of both parties or services. Such contracts shall be solicited, awarded, and administered in accordance with the
pursuant to the objective of the treaty to strengthen those capabilities to prevent or resist a laws and regulations of the United States.255 (Emphases Supplied)
possible armed attack. Respondent also points out that EDCA simply implements Article I of the
A thorough evaluation of how EDCA is phrased clarities that the agreement does not deal with VFA. As the implementing agreement, it regulates and limits the presence of U.S. personnel in the
the entry into the country of U.S. personnel and contractors per se. While Articles I(l)(b)256 and country.
II(4)257 speak of "the right to access and use" the Agreed Locations, their wordings indicate the
presumption that these groups have already been allowed entry into Philippine territory, for b. EDCA does not provide the legal basis for admission of U.S. contractors into Philippine territory;
which, unlike the VFA, EDCA has no specific provision. Instead, Article II of the latter simply alludes their entry must be sourced from extraneous Philippine statutes and regulations for the admission
to the VFA in describing U.S. personnel, a term defined under Article I of the treaty as follows: of alien employees or business persons.

As used in this Agreement, "United States personnel" means United States military and civilian Of the three aforementioned classes of individuals who will be conducting certain activities within
personnel temporarily in the Philippines in connection with activities approved by the Philippine the Agreed Locations, we note that only U.S. contractors are not explicitly mentioned in the VFA.
Government. Within this definition: This does not mean, though, that the recognition of their presence under EDCA is ipso facto an
amendment of the treaty, and that there must be Senate concurrence before they are allowed to
1. The term "military personnel" refers to military members of the United enter the country.
States Army, Navy, Marine Corps, Air Force, and Coast Guard.
Nowhere in EDCA are U.S. contractors guaranteed immediate admission into the Philippines.
2. The term "civilian personnel" refers to individuals who are neither Articles III and IV, in fact, merely grant them the right of access to, and the authority to conduct
nationals of nor ordinarily resident in the Philippines and who are employed certain activities within the Agreed Locations. Since Article II(3) of EDCA specifically leaves out U.S.
by the United States armed forces or who are accompanying the United contractors from the coverage of the VFA, they shall not be granted the same entry
States armed forces, such as employees of the American Red Cross and accommodations and privileges as those enjoyed by U.S. military and civilian personnel under the
the United Services Organization.258 VFA.

Article II of EDCA must then be read with Article III of the VFA, which provides for the entry Consequently, it is neither mandatory nor obligatory on the part of the Philippines to admit U.S.
accommodations to be accorded to U.S. military and civilian personnel: contractors into the country.259 We emphasize that the admission of aliens into Philippine territory
is "a matter of pure permission and simple tolerance which creates no obligation on the part of the
1. The Government of the Philippines shall facilitate the admission of United States government to permit them to stay."260 Unlike U.S. personnel who are accorded entry
personnel and their departure from the Philippines in connection with activities covered accommodations, U.S. contractors are subject to Philippine immigration laws.261The latter must
by this agreement. comply with our visa and passport regulations262 and prove that they are not subject to exclusion
under any provision of Philippine immigration laws.263 The President may also deny them entry
pursuant to his absolute and unqualified power to prohibit or prevent the admission of aliens
2. United States military personnel shall be exempt from passport and visa regulations whose presence in the country would be inimical to public interest.264
upon enteringand departing the Philippines.

In the same vein, the President may exercise the plenary power to expel or deport U.S.
3. The following documents only, which shall be required in respect of United States contractors265 as may be necessitated by national security, public safety, public health, public
military personnel who enter the Philippines; xx xx. morals, and national interest.266 They may also be deported if they are found to be illegal or
undesirable aliens pursuant to the Philippine Immigration Act267 and the Data Privacy Act.268 In
4. United States civilian personnel shall be exempt from visa requirements but shall contrast, Article 111(5) of the VFA requires a request for removal from the Philippine government
present, upon demand, valid passports upon entry and departure of the Philippines. before a member of the U.S. personnel may be "dispos[ed] xx x outside of the Philippines."
(Emphases Supplied)
c. Authorized activities of U.S. military and civilian personnel within Philippine territory are in
By virtue of Articles I and III of the VFA, the Philippines already allows U.S. military and civilian furtherance of the MDT and the VFA
personnel to be "temporarily in the Philippines," so long as their presence is "in connection with
activities approved by the Philippine Government." The Philippines, through Article III, even We begin our analysis by quoting the relevant sections of the MDT and the VFA that pertain to the
guarantees that it shall facilitate the admission of U.S. personnel into the country and grant activities in which U.S. military and civilian personnel may engage:
exemptions from passport and visa regulations. The VFA does not even limit their temporary
presence to specific locations.
MUTUAL DEFENSE TREATY

Based on the above provisions, the admission and presence of U.S. military and civilian personnel
in Philippine territory are already allowed under the VFA, the treaty supposedly being Article II
implemented by EDCA. What EDCA has effectively done, in fact, is merely provide the mechanism
to identify the locations in which U.S. personnel may perform allowed activities pursuant to the
In order more effectively to achieve the objective of this Treaty, the Parties separately and jointly property from the Philippines at any time, free from export duties, taxes, and other similar
byself-help and mutual aid will maintain and develop their individual and collective capacity to charges. x x x.
resist armed attack.
Article VIII - Movement of Vessels and Aircraft
Article III
1. Aircraft operated by or for the United States armed forces may enter the Philippines upon
The Parties, through their Foreign Ministers or their deputies, will consult together from time to approval of the Government of the Philippines in accordance with procedures stipulated
time regarding the implementation of this Treaty and whenever in the opinion of either of them in implementing arrangements.
the territorial integrity, political independence or security of either of the Parties is threatened by
external armed attack in the Pacific. 2. Vessels operated by or for the United States armed forces may enter the Philippines upon
approval of the Government of the Philippines. The movement of vessels shall be in accordance
VISITING FORCES AGREEMENT with international custom and practice governing such vessels, and such agreed implementing
arrangements as necessary. x x x (Emphases Supplied)
Preamble
Manifest in these provisions is the abundance of references to the creation of further
xxx "implementing arrangements" including the identification of "activities [to be] approved by the
Philippine Government." To determine the parameters of these implementing arrangements and
activities, we referred to the content, purpose, and framework of the MDT and the VFA.
Reaffirming their obligations under the Mutual Defense Treaty of August 30, 1951;

By its very language, the MDT contemplates a situation in which both countries shall engage
Noting that from time to time elements of the United States armed forces may visit the Republic of in joint activities, so that they can maintain and develop their defense capabilities. The wording
the Philippines; itself evidently invites a reasonable construction that the joint activities shall involve joint military
trainings, maneuvers, and exercises. Both the interpretation269 and the subsequent practice270 of
Considering that cooperation between the United States and the Republic of the the parties show that the MDT independently allows joint military exercises in the country. Lim v.
Philippines promotes their common security interests; Executive Secretary271 and Nicolas v. Romulo272 recognized that Balikatan exercises, which are
activities that seek to enhance and develop the strategic and technological capabilities of the
xxx parties to resist an armed attack, "fall squarely under the provisions of the RP-US
MDT."273 In Lim, the Court especially noted that the Philippines and the U.S. continued to conduct
joint military exercises even after the expiration of the MBA and even before the conclusion of the
Article I - Definitions VFA.274 These activities presumably related to the Status of Forces Agreement, in which the parties
agreed on the status to be accorded to U.S. military and civilian personnel while conducting
As used in this Agreement, "United States personnel" means United States military and civilian activities in the Philippines in relation to the MDT.275
personnel temporarily in the Philippines in connection with activities approved by the Philippine
Government. Within this definition: xx x Further, it can be logically inferred from Article V of the MDT that these joint activities may be
conducted on Philippine or on U.S. soil. The article expressly provides that the term armed
Article II - Respect for Law attack includes "an armed attack on the metropolitan territory of either of the Parties, or on
the island territories under its jurisdiction in the Pacific or on its armed forces, public vessels or
aircraft in the Pacific." Surely, in maintaining and developing our defense capabilities, an
It is the duty of United States personnel to respect the laws of the Republic of the
assessment or training will need to be performed, separately and jointly by self-help and mutual
Philippines and to abstain from any activity inconsistent with the spirit of this agreement, and, in
aid, in the territories of the contracting parties. It is reasonable to conclude that the assessment of
particular, from any political activity in the Philippines. The Government of the United States shall
defense capabilities would entail understanding the terrain, wind flow patterns, and other
take all measures within its authority to ensure that this is done.
environmental factors unique to the Philippines.

Article VII - Importation and Exportation


It would also be reasonable to conclude that a simulation of how to respond to attacks in
vulnerable areas would be part of the training of the parties to maintain and develop their capacity
1. United States Government equipment, materials, supplies, and other property imported into to resist an actual armed attack and to test and validate the defense plan of the Philippines. It is
or acquired in the Philippines by or on behalf of the United States armed forces in connection with likewise reasonable to imagine that part of the training would involve an analysis of the effect of
activities to which this agreement applies, shall be free of all Philippine duties, taxes and other the weapons that may be used and how to be prepared for the eventuality. This Court recognizes
similar charges. Title to such property shall remain with the United States, which may remove such
that all of this may require training in the area where an armed attack might be directed at the the Chief of Staff of the AFP and the Commander in Chief of the U.S. Pacific Command-that the
Philippine territory. VFA exercises are planned. Final approval of any activity involving U.S. forces is,
however, invariably given by the Philippine Government.
The provisions of the MDT must then be read in conjunction with those of the VFA.
xxxx
Article I of the VFA indicates that the presence of U.S. military and civilian personnel in the
Philippines is "in connection with activities approved by the Philippine Government." While the Siazon clarified that it is not the VFA by itself that determines what activities will be
treaty does not expressly enumerate or detail the nature of activities of U.S. troops in the country, conductedbetween the armed forces of the U.S. and the Philippines. The VFA regulates and
its Preamble makes explicit references to the reaffirmation of the obligations of both countries provides the legal framework for the presence, conduct and legal status of U.S. personnel while
under the MDT. These obligations include the strengthening of international and regional security they are in the country for visits, joint exercises and other related activities. (Emphases Supplied)
in the Pacific area and the promotion of common security interests.
What can be gleaned from the provisions of the VFA, the joint report of the Senate committees
The Court has already settled in Lim v. Executive Secretary that the phrase "activities approved by on foreign relations and on national defense and security, and the ruling of this Court in Lim is
the Philippine Government" under Article I of the VFA was intended to be ambiguous in order to that the "activities" referred to in the treaty are meant to be specified and
afford the parties flexibility to adjust the details of the purpose of the visit of U.S. personnel. 276 In identified infurther agreements. EDCA is one such agreement.
ruling that the Terms of Reference for the Balikatan Exercises in 2002 fell within the context of the
treaty, this Court explained: EDCA seeks to be an instrument that enumerates the Philippine-approved activities of U.S.
personnel referred to in the VFA. EDCA allows U.S. military and civilian personnel to perform
After studied reflection, it appeared farfetched that the ambiguity surrounding the meaning of "activities approved by the Philippines, as those terms are defined in the VFA"278 and clarifies that
the word "activities" arose from accident. In our view, it was deliberately made that way to give these activities include those conducted within the Agreed Locations:
both parties a certain leeway in negotiation. In this manner, visiting US forces may sojourn in
Philippine territory for purposes other than military. As conceived, the joint exercises may include 1. Security cooperation exercises; joint and combined training activities; humanitarian assistance
training on new techniques of patrol and surveillance to protect the nation's marine resources, sea and disaster relief activities; and such other activities as may be agreed upon by the Parties279
search-and-rescue operations to assist vessels in distress, disaster relief operations, civic action
projects such as the building of school houses, medical and humanitarian missions, and the like.
2. Training; transit; support and related activities; refueling of aircraft; bunkering of vessels;
temporary maintenance of vehicles, vessels, and aircraft; temporary accommodation of personnel;
Under these auspices, the VFA gives legitimacy to the current Balikatan exercises. It is only logical communications; prepositioning of equipment, supplies, and materiel; deployment of forces and
to assume that "Balikatan 02-1," a "mutual anti-terrorism advising, assisting and training materiel; and such other activities as the Parties may agree280
exercise," falls under the umbrella of sanctioned or allowable activities in the context of the
agreement. Both the history and intent of the Mutual Defense Treaty and the VFA support the
conclusion that combat-related activities - as opposed to combat itself- such as the one subject of 3. Exercise of operational control over the Agreed Locations for construction activities and other
the instant petition, are indeed authorized. (Emphases Supplied) types of activity, including alterations and improvements thereof281

The joint report of the Senate committees on foreign relations and on national defense and 4. Exercise of all rights and authorities within the Agreed Locations that are necessary for their
security further explains the wide range and variety of activities contemplated in the VFA, and how operational control or defense, including the adoption of apfropriate measures to protect U.S.
these activities shall be identified:277 forces and contractors282

These joint exercises envisioned in the VFA are not limited to combat-related activities; they have 5. Use of water, electricity, and other public utilities283
a wide range and variety. They include exercises that will reinforce the AFP's ability to acquire
new techniques of patrol and surveillance to protect the country's maritime resources; sea-search 6. Operation of their own telecommunication systems, including the utilization of such means and
and rescue operations to assist ships in distress; and disaster-relief operations to aid the civilian services as are required to ensure the full ability to operate telecommunication systems, as well as
victims of natural calamities, such as earthquakes, typhoons and tidal waves. the use of the necessary radio spectrum allocated for this purpose284

xxxx According to Article I of EDCA, one of the purposes of these activities is to maintain and develop,
jointly and by mutual aid, the individual and collective capacities of both countries to resist an
Joint activities under the VFA will include combat maneuvers; training in aircraft maintenance and armed attack. It further states that the activities are in furtherance of the MDT and within the
equipment repair; civic-action projects; and consultations and meetings of the Philippine-U.S. context of the VFA.
Mutual Defense Board. It is at the level of the Mutual Defense Board-which is headed jointly by
We note that these planned activities are very similar to those under the Terms of b. At no time shall US Forces operate independently within RP territory.
Reference285 mentioned in Lim. Both EDCA and the Terms of Reference authorize the U.S. to
perform the following: (a) participate in training exercises; (b) retain command over their forces; c. Flight plans of all aircraft involved in the exercise will comply with the local
(c) establish temporary structures in the country; (d) share in the use of their respective resources, air traffic regulations.
equipment and other assets; and (e) exercise their right to self-defense. We quote the relevant
portion of the Terms and Conditions as follows:286
2. ADMINISTRATION & LOGISTICS
I. POLICY LEVEL
xxxx

xxxx
a. RP and US participating forces may share, in accordance with their respective laws and
regulations, in the use of their resources, equipment and other assets. They will use their
No permanent US basing and support facilities shall be established. Temporary structures such as respective logistics channels. x x x. (Emphases Supplied)
those for troop billeting, classroom instruction and messing may be set up for use by RP and US
Forces during the Exercise.
After a thorough examination of the content, purpose, and framework of the MDT and the VFA,
we find that EDCA has remained within the parameters set in these two treaties. Just like the
The Exercise shall be implemented jointly by RP and US Exercise Co-Directors under the authority Terms of Reference mentioned in Lim, mere adjustments in detail to implement the MDT and the
of the Chief of Staff, AFP. In no instance will US Forces operate independently during field training VFA can be in the form of executive agreements.
exercises (FTX). AFP and US Unit Commanders will retain command over their respective forces
under the overall authority of the Exercise Co-Directors. RP and US participants shall comply with
operational instructions of the AFP during the FTX. Petitioners assert287 that the duration of the activities mentioned in EDCA is no longer consistent
with the temporary nature of the visits as contemplated in the VFA. They point out that Article
XII(4) of EDCA has an initial term of 10 years, a term automatically renewed unless the Philippines
The exercise shall be conducted and completed within a period of not more than six months, with or the U.S. terminates the agreement. According to petitioners, such length of time already has a
the projected participation of 660 US personnel and 3,800 RP Forces. The Chief of Staff, AFP shall badge of permanency.
direct the Exercise Co-Directors to wind up and terminate the Exercise and other activities within
the six month Exercise period.
In connection with this, Justice Teresita J. Leonardo-De Castro likewise argues in her Concurring
and Dissenting Opinion that the VFA contemplated mere temporary visits from U.S. forces,
The Exercise is a mutual counter-terrorism advising, assisting and training Exercise relative to whereas EDCA allows an unlimited period for U.S. forces to stay in the Philippines. 288
Philippine efforts against the ASG, and will be conducted on the Island of Basilan. Further advising,
assisting and training exercises shall be conducted in Malagutay and the Zamboanga area. Related
activities in Cebu will be for support of the Exercise. However, the provisions of EDCA directly contradict this argument by limiting itself to 10 years of
effectivity. Although this term is automatically renewed, the process for terminating the
agreement is unilateral and the right to do so automatically accrues at the end of the 10 year
xx xx. period. Clearly, this method does not create a permanent obligation.

US exercise participants shall not engage in combat, without prejudice to their right of self- Drawing on the reasoning in Lim, we also believe that it could not have been by chance that the
defense. VFA does not include a maximum time limit with respect to the presence of U.S. personnel in the
country. We construe this lack of specificity as a deliberate effort on the part of the Philippine and
These terms of Reference are for purposes of this Exercise only and do not create additional legal the U.S. governments to leave out this aspect and reserve it for the "adjustment in detail" stage of
obligations between the US Government and the Republic of the Philippines. the implementation of the treaty. We interpret the subsequent, unconditional concurrence of the
Senate in the entire text of the VFA as an implicit grant to the President of a margin of
II. EXERCISE LEVEL appreciation in determining the duration of the "temporary" presence of U.S. personnel in the
country.

1. TRAINING
Justice Brion's dissent argues that the presence of U.S. forces under EDCA is "more permanent" in
nature.289However, this argument has not taken root by virtue of a simple glance at its provisions
a. The Exercise shall involve the conduct of mutual military assisting, advising on the effectivity period. EDCA does not grant permanent bases, but rather temporary rotational
and trainingof RP and US Forces with the primary objective of enhancing the access to facilities for efficiency. As Professor Aileen S.P. Baviera notes:
operational capabilities of both forces to combat terrorism.
The new EDCA would grant American troops, ships and planes rotational access to facilities of the engendered that a much longer period of military training is needed, the President must be given
Armed Forces of the Philippines but not permanent bases which are prohibited under the ample discretion to adopt necessary measures including the flexibility to set an extended
Philippine Constitution - with the result of reducing response time should an external threat from a timetable.
common adversary crystallize.290
Due to the sensitivity and often strict confidentiality of these concerns, we acknowledge that the
EDCA is far from being permanent in nature compared to the practice of states as shown in other President may not always be able to candidly and openly discuss the complete situation being
defense cooperation agreements. For example, Article XIV(l) of the U.S.-Romania defense faced by the nation. The Chief Executive's hands must not be unduly tied, especially if the situation
agreement provides the following: calls for crafting programs and setting timelines for approved activities. These activities may be
necessary for maintaining and developing our capacity to resist an armed attack, ensuring our
This Agreement is concluded for an indefinite period and shall enter into force in accordance with national sovereignty and territorial integrity, and securing our national interests. If the Senate
the internal laws of each Party x x x. (emphasis supplied) decides that the President is in the best position to define in operational terms the meaning
of temporary in relation to the visits, considered individually or in their totality, the Court must
respect that policy decision. If the Senate feels that there is no need to set a time limit to these
Likewise, Article 36(2) of the US-Poland Status of Forces Agreement reads: visits, neither should we.

This Agreement has been concluded for an indefinite period of time. It may be terminated by Evidently, the fact that the VFA does not provide specificity in regard to the extent of the
written notification by either Party and in that event it terminates 2 years after the receipt of the "temporary" nature of the visits of U.S. personnel does not suggest that the duration to which the
notification. (emphasis supplied) President may agree is unlimited. Instead, the boundaries of the meaning of the term temporary in
Article I of the treaty must be measured depending on the purpose of each visit or activity.295 That
Section VIII of US.-Denmark Mutual Support Agreement similarly provides: purpose must be analyzed on a case-by-case basis depending on the factual circumstances
surrounding the conclusion of the implementing agreement. While the validity of the President's
8.1 This Agreement, which consists of a Preamble, SECTIONs I-VIII, and Annexes A and B, shall actions will be judged under less stringent standards, the power of this Court to determine
become effective on the date of the last signature affixed below and shall remain in force until whether there was grave abuse of discretion remains unimpaired.
terminated by the Parties, provided that it may be terminated by either Party upon 180 days
written notice of its intention to do so to the other Party. (emphasis supplied) d. Authorized activities performed by US. contractors within Philippine territory - who were
legitimately permitted to enter the country independent of EDCA - are subject to relevant Philippine
On the other hand, Article XXI(3) of the US.-Australia Force Posture Agreement provides a longer statutes and regulations and must be consistent with the MDT and the VFA
initial term:
Petitioners also raise296 concerns about the U.S. government's purported practice of hiring private
3. This Agreement shall have an initial term of 25 years and thereafter shall continue in force, but security contractors in other countries. They claim that these contractors - one of which has
may be terminated by either Party at any time upon one year's written notice to the other Party already been operating in Mindanao since 2004 - have been implicated in incidents or scandals in
through diplomatic channels. (emphasis supplied) other parts of the globe involving rendition, torture and other human rights violations. They also
assert that these contractors employ paramilitary forces in other countries where they are
operating.
The phrasing in EDCA is similar to that in the U.S.-Australia treaty but with a term less than half of
that is provided in the latter agreement. This means that EDCA merely follows the practice of other
states in not specifying a non-extendible maximum term. This practice, however, does not Under Articles III and IV of EDCA, U.S. contractors are authorized to perform only the following
automatically grant a badge of permanency to its terms. Article XII(4) of EDCA provides very activities:
clearly, in fact, that its effectivity is for an initial term of 10 years, which is far shorter than the
terms of effectivity between the U.S. and other states. It is simply illogical to conclude that the 1. Training; transit; support and related activities; refueling of aircraft; bunkering of
initial, extendible term of 10 years somehow gives EDCA provisions a permanent character. vessels; temporary maintenance of vehicles, vessels, and aircraft; temporary
accommodation of personnel; communications; prepositioning of equipment, supplies,
The reasoning behind this interpretation is rooted in the constitutional role of the President who, and materiel; deployment of forces and materiel; and such other activities as the Parties
as Commander-in-Chief of our armed forces, is the principal strategist of the nation and, as such, may agree297
duty-bound to defend our national sovereignty and territorial integrity;291 who, as chief architect
of our foreign relations, is the head policymaker tasked to assess, ensure, and protect our national 2. Prepositioning and storage of defense equipment, supplies, and materiel, including
security and interests;292 who holds the most comprehensive and most confidential information delivery, management, inspection, use, maintenance, and removal of such equipment,
about foreign countries293 that may affect how we conduct our external affairs; and who has supplies and materiel298
unrestricted access to highly classified military intelligence data 294 that may threaten the life of the
nation. Thus, if after a geopolitical prognosis of situations affecting the country, a belief is
3. Carrying out of matters in accordance with, and to the extent permissible under, U.S. Nevertheless, we emphasize that U.S. contractors are explicitly excluded from the coverage of the
laws, regulations, and policies299 VFA. As visiting aliens, their entry, presence, and activities are subject to all laws and treaties
applicable within the Philippine territory. They may be refused entry or expelled from the country
EDCA requires that all activities within Philippine territory be in accordance with Philippine law. if they engage in illegal or undesirable activities. There is nothing that prevents them from being
This means that certain privileges denied to aliens are likewise denied to foreign military detained in the country or being subject to the jurisdiction of our courts. Our penal laws, 309 labor
contractors. Relevantly, providing security300and carrying, owning, and possessing firearms301 are laws,310 and immigrations laws311 apply to them and therefore limit their activities here. Until and
illegal for foreign civilians. unless there is another law or treaty that specifically deals with their entry and activities, their
presence in the country is subject to unqualified Philippine jurisdiction.

The laws in place already address issues regarding the regulation of contractors. In the 2015
Foreign Investment Negative list,302 the Executive Department has already identified corporations EDCA does not allow the presence of U.S.-owned or -controlled military facilities and bases in the
that have equity restrictions in Philippine jurisdiction. Of note is No. 5 on the list - private security Philippines
agencies that cannot have any foreign equity by virtue of Section 4 of Republic Act No.
5487;303 and No. 15, which regulates contracts for the construction of defense-related structures Petitioners Saguisag et al. claim that EDCA permits the establishment of U.S. military bases
based on Commonwealth Act No. 541. through the "euphemistically" termed "Agreed Locations. "312 Alluding to the definition of this
term in Article II(4) of EDCA, they point out that these locations are actually military bases, as the
Hence, any other entity brought into the Philippines by virtue of EDCA must subscribe to corporate definition refers to facilities and areas to which U.S. military forces have access for a variety of
and civil requirements imposed by the law, depending on the entity's corporate structure and the purposes. Petitioners claim that there are several badges of exclusivity in the use of the Agreed
nature of its business. Locations by U.S. forces. First, Article V(2) of EDCA alludes to a "return" of these areas once they
are no longer needed by U.S. forces, indicating that there would be some transfer of
use. Second, Article IV(4) ofEDCA talks about American forces' unimpeded access to the Agreed
That Philippine laws extraneous to EDCA shall govern the regulation of the activities of U.S. Locations for all matters relating to the prepositioning and storage of U.S. military equipment,
contractors has been clear even to some of the present members of the Senate. supplies, and materiel. Third, Article VII of EDCA authorizes U.S. forces to use public utilities and to
operate their own telecommunications system.
For instance, in 2012, a U.S. Navy contractor, the Glenn Marine, was accused of spilling fuel in the
waters off Manila Bay.304 The Senate Committee on Foreign Relations and the Senate Committee a. Preliminary point on badges of exclusivity
on Environment and Natural Resources chairperson claimed environmental and procedural
violations by the contractor.305 The U.S. Navy investigated the contractor and promised stricter
guidelines to be imposed upon its contractors.306 The statement attributed to Commander Ron As a preliminary observation, petitioners have cherry-picked provisions of EDCA by presenting so-
Steiner of the public affairs office of the U.S. Navy's 7th Fleet - that U.S. Navy contractors are called "badges of exclusivity," despite the presence of contrary provisions within the text of the
bound by Philippine laws - is of particular relevance. The statement acknowledges not just the agreement itself.
presence of the contractors, but also the U.S. position that these contractors are bound by the
local laws of their host state. This stance was echoed by other U.S. Navy representatives.307 First, they clarify the word "return" in Article V(2) of EDCA. However, the use of the word "return"
is within the context of a lengthy provision. The provision as a whole reads as follows:
This incident simply shows that the Senate was well aware of the presence of U.S. contractors for
the purpose of fulfilling the terms of the VFA. That they are bound by Philippine law is clear to all, The United States shall return to the Philippines any Agreed Locations, or any portion thereof,
even to the U.S. including non-relocatable structures and assemblies constructed, modified, or improved by the
United States, once no longer required by United States forces for activities under this Agreement.
As applied to EDCA, even when U.S. contractors are granted access to the Agreed Locations, all The Parties or the Designated Authorities shall consult regarding the terms of return of any Agreed
their activities must be consistent with Philippine laws and regulations and pursuant to the MDT Locations, including possible compensation for improvements or construction.
and the VFA.
The context of use is "required by United States forces for activities under this Agreement."
While we recognize the concerns of petitioners, they do not give the Court enough justification to Therefore, the return of an Agreed Location would be within the parameters of an activity that the
strike down EDCA. In Lim v. Executive Secretary, we have already explained that we cannot take Mutual Defense Board (MDB) and the Security Engagement Board (SEB) would authorize. Thus,
judicial notice of claims aired in news reports, "not because of any issue as to their truth, accuracy, possession by the U.S. prior to its return of the Agreed Location would be based on the authority
or impartiality, but for the simple reason that facts must be established in accordance with the given to it by a joint body co-chaired by the "AFP Chief of Staff and Commander, U.S. PACOM with
rules of evidence."308 What is more, we cannot move one step ahead and speculate that the representatives from the Philippines' Department of National Defense and Department of Foreign
alleged illegal activities of these contractors in other countries would take place in the Philippines Affairs sitting as members."313 The terms shall be negotiated by both the Philippines and the U.S.,
with certainty. As can be seen from the above discussion, making sure that U.S. contractors comply or through their Designated Authorities. This provision, seen as a whole, contradicts petitioners'
with Philippine laws is a function of law enforcement. EDCA does not stand in the way of law interpretation of the return as a "badge of exclusivity." In fact, it shows the cooperation and
enforcement. partnership aspect of EDCA in full bloom.
Second, the term "unimpeded access" must likewise be viewed from a contextual perspective. respect of every activity, including construction, by giving the MDB and the SEB the power to
Article IV(4) states that U.S. forces and U.S. contractors shall have "unimpeded access to Agreed determine the details of all activities such as, but not limited to, operation, maintenance, utility,
Locations for all matters relating to the prepositioning and storage of defense equipment, supplies, occupancy, garrisoning, and control.322
and materiel, including delivery, management, inspection, use, maintenance, and removal of such
equipment, supplies and materiel." The "species of ownership" on the other hand, is distinguished by the nature of the property. For
immovable property constructed or developed by the U.S., EDCA expresses that ownership will
At the beginning of Article IV, EDCA states that the Philippines gives the U.S. the authority to bring automatically be vested to the Philippines.323 On the other hand, for movable properties brought
in these equipment, supplies, and materiel through the MDB and SEB security mechanism. These into the Philippines by the U.S., EDCA provides that ownership is retained by the latter. In contrast,
items are owned by the U.S.,314 are exclusively for the use of the U.S.315 and, after going through the MBA dictates that the U.S. retains ownership over immovable and movable properties.
the joint consent mechanisms of the MDB and the SEB, are within the control of the U.S.316 More
importantly, before these items are considered prepositioned, they must have gone through the To our mind, both EDCA and the MBA simply incorporate what is already the law of the land in the
process of prior authorization by the MDB and the SEB and given proper notification to the AFP.317 Philippines. The Civil Code's provisions on ownership, as applied, grant the owner of a movable
property full rights over that property, even if located in another person's property.324
Therefore, this "unimpeded access" to the Agreed Locations is a necessary adjunct to the
ownership, use, and control of the U.S. over its own equipment, supplies, and materiel and must The parallelism, however, ends when the situation involves facilities that can be considered
have first been allowed by the joint mechanisms in play between the two states since the time of immovable. Under the MBA, the U.S. retains ownership if it paid for the facility.325 Under EDCA, an
the MDT and the VFA. It is not the use of the Agreed Locations that is exclusive per se; it is mere immovable is owned by the Philippines, even if built completely on the back of U.S. funding. 326 This
access to items in order to exercise the rights of ownership granted by virtue of the Philippine Civil is consistent with the constitutional prohibition on foreign land ownership. 327
Code.318

Despite the apparent similarity, the ownership of property is but a part of a larger whole that must
As for the view that EDCA authorizes U.S. forces to use public utilities and to operate their own be considered before the constitutional restriction is violated. Thus, petitioners' points on
telecommunications system, it will be met and answered in part D, infra. operational control will be given more attention in the discussion below. The arguments on policy
are, however, outside the scope of judicial review and will not be discussed
Petitioners also point out319 that EDCA is strongly reminiscent of and in fact bears a one-to-one
correspondence with the provisions of the 1947 MBA. They assert that both agreements (a) allow Moreover, a direct comparison of the MBA and EDCA will result in several important distinctions
similar activities within the area; (b) provide for the same "species of ownership" over facilities; that would allay suspicion that EDCA is but a disguised version of the MBA.
and (c) grant operational control over the entire area. Finally, they argue320 that EDCA is in fact an
implementation of the new defense policy of the U.S. According to them, this policy was not what
was originally intended either by the MDT or by the VFA. b. There are substantial matters that the US. cannot do under EDCA, but which it was authorized to
do under the 1947 MBA

On these points, the Court is not persuaded.


The Philippine experience with U.S. military bases under the 1947 MBA is simply not possible
under EDCA for a number of important reasons.
petitioners321
The similar activities cited by simply show that under the MBA, the U.S. had the right
to construct, operate, maintain, utilize, occupy, garrison, and control the bases. The so-called
parallel provisions of EDCA allow only operational control over the Agreed Locations specifically for First, in the 1947 MBA, the U.S. retained all rights of jurisdiction in and over Philippine territory
construction activities. They do not allow the overarching power to operate, maintain, utilize, occupied by American bases. In contrast, the U.S. under EDCA does not enjoy any such right over
occupy, garrison, and control a base with full discretion. EDCA in fact limits the rights of the U.S. in any part of the Philippines in which its forces or equipment may be found. Below is a comparative
table between the old treaty and EDCA:

1947 MBA/ 1946 Treaty of General Relations EDCA

1947 MBA, Art. I(1): EDCA, preamble:

The Government of the Republic of the Philippines (hereinafter referred to as the


Philippines) grants to the Government of the United States of America (hereinafter
referred to as the United States) the right to retain the use of the bases in the Affirming that the Parties share an understanding for the United States not to
Philippines listed in Annex A attached hereto. establish a permanent military presence or base in the territory of the
Philippines;
1947 MBA, Art. XVII(2):
xxxx
All buildings and structures which are erected by the United States in the bases
shall be the property of the United States and may be removed by it before the Recognizing that all United States access to and use of facilities and areas will be at
expiration of this Agreement or the earlier relinquishment of the base on which the the invitation of the Philippines and with full respect for the Philippine
structures are situated. There shall be no obligation on the part of the Philippines Constitution and Philippine laws;
or of the United States to rebuild or repair any destruction or damage inflicted
from any cause whatsoever on any of the said buildings or structures owned or xxxx
used by the United States in the bases. x x x x.

EDCA, Art. II(4):


1946 Treaty of Gen. Relations, Art. I:

"Agreed Locations" means facilities and areas that are provided by the
The United States of America agrees to withdraw and surrender, and does hereby Government of the Philippines through the AFP and that United States forces,
withdraw and surrender, all rights of possession, supervision, jurisdiction, control United States contractors, and others as mutually agreed, shall have the right to
or sovereignty existing and exercised by the United States of America in and over access and use pursuant to this Agreement. Such Agreed Locations may be listed in
the territory and the people of the Philippine Islands, except the use of such an annex to be appended to this Agreement, and may be further described in
bases, necessary appurtenances to such bases, and the rights incident thereto, as implementing arrangements.
the United States of America, by agreement with the Republic of the Philippines
may deem necessary to retain for the mutual protection of the Republic of the
Philippines and of the United States of America. x x x. EDCA, Art. V:

1. The Philippines shall retain ownership of and title to Agreed Locations.

xxxx

4. All buildings, non-relocatable structures, and assemblies affixed to the land in


the Agreed Locations, including ones altered or improved by United States
forces, remain the property of the Philippines.Permanent buildings constructed by
United States forces become the property of the Philippines, once constructed, but
shall be used by United States forces until no longer required by United States
forces.

Second, in the bases agreement, the U.S. and the Philippines were visibly not on equal footing when it came to deciding whether to expand or to increase the number of bases, as the Philippines may be
compelled to negotiate with the U.S. the moment the latter requested an expansion of the existing bases or to acquire additional bases. In EDCA, U.S. access is purely at the invitation of the Philippines.

1947 MBA/ 1946 Treaty of General Relations EDCA

1947 MBA, Art.I(3): EDCA, preamble:


The Philippines agree to enter into negotiations with the United States at the Recognizing that all United States access to and use of facilities and areas will be at
latter's request, to permit the United States to expand such bases, to exchange the invitation of the Philippines and with full respect for the Philippine
such bases for other bases, to acquire additional bases, or relinquish rights to Constitution and Philippine laws;
bases, as any of such exigencies may be required by military necessity.
xxxx
1946 Treaty of Gen. Relations, Art. I:
EDCA. Art. II(4):
The United States of America agrees to withdraw and surrender, and does hereby
withdraw and surrender, all rights of possession, supervision, jurisdiction, control "Agreed Locations" means facilities and areas that are provided by the
or sovereignty existing and exercised by the United States of America in and over Government of the Philippines through the AFP and that United States forces,
the territory and the people of the Philippine Islands, except the use of such United States contractors, and others as mutually agreed, shall have the right to
bases, necessary appurtenances to such bases, and the rights incident thereto, as access and use pursuant to this Agreement. Such Agreed Locations may be listed in
the United States of America, by agreement with the Republic of the an annex to be appended to this Agreement, and may be further described in
Philippines may deem necessary to retain for the mutual protection of the implementing arrangements.
Republic of the Philippines and of the United States of America. x x x.

Third, in EDCA, the Philippines is- guaranteed access over the entire area of the Agreed Locations. On the other hand, given that the U.S. had complete control over its military bases under the 1947 MBA, the
treaty did not provide for any express recognition of the right of access of Philippine authorities. Without that provision and in light of the retention of U.S. sovereignty over the old military bases, the U.S.
could effectively prevent Philippine authorities from entering those bases.

1947 MBA EDCA

No equivalent provision. EDCA, Art. III(5):

The Philippine Designated Authority and its authorized representative shall have
access to the entire area of the Agreed Locations. Such access shall be provided
promptly consistent with operational safety and security requirements in
accordance with agreed procedures developed by the Parties.

Fourth, in the bases agreement, the U.S. retained the right, power, and authority over the establishment, use, operation, defense, and control of military bases, including the limits of territorial waters and air
space adjacent to or in the vicinity of those bases. The only standard used in determining the extent of its control was military necessity. On the other hand, there is no such grant of power or authority under
EDCA. It merely allows the U.S. to exercise operational control over the construction of Philippine-owned structures and facilities:

1947 MBA EDCA

1947 MBA, Art.I(2): EDCA, Art. III(4):

The Philippines agrees to permit the United States, upon notice to the Philippines, The Philippines hereby grants to the United States, through bilateral security
to use such of those bases listed in Annex B as the United States determines to be mechanisms, such as the MDB and SEB, operational control of Agreed
required by military necessity. Locations for construction activities and authority to undertake such activities on,
and make alterations and improvements to, Agreed Locations. United States forces
1947 MBA, Art. III(1): shall consult on issues regarding such construction, alterations, and
improvements based on the Parties' shared intent that the technical requirements
and construction standards of any such projects undertaken by or on behalf of
It is mutually agreed that the United Statesshall have the rights, power and United States forces should be consistent with the requirements and standards of
authority within the bases which are necessary for the establishment, use, both Parties.
operation and defense thereof or appropriate for the control thereof and all the
rights, power and authority within the limits of territorial waters and air space
adjacent to, or in the vicinity of, the bases which are necessary to provide access
to them, or appropriate for their control.

Fifth, the U.S. under the bases agreement was given the authority to use Philippine territory for additional staging areas, bombing and gunnery ranges. No such right is given under EDCA, as seen below:

1947 MBA EDCA

1947 MBA, Art. VI: EDCA, Art. III(1):

The United States shall, subject to previous agreement with the Philippines, have With consideration of the views of the Parties,
the right to use land and coastal sea areas of appropriate size and location for the Philippines hereby authorizes and agrees that United States forces, United
periodic maneuvers, for additional staging areas, bombing and gunnery ranges, States contractors, and vehicles, vessels, and aircraft operated by or for United
and for such intermediate airfields as may be required for safe and efficient air States forces may conduct the following activities with respect to Agreed Locations:
operations. Operations in such areas shall be carried on with due regard and training; transit; support and related activities; refueling of aircraft; bunkering of
safeguards for the public safety. vessels; temporary maintenance of vehicles, vessels, and aircraft; temporary
accommodation of personnel; communications; prepositioning of equipment,
1947 MBA, Art.I(2): supplies, and materiel; deploying forces and materiel; and such other activities as
the Parties may agree.

The Philippines agrees to permit the United States, upon notice to the Philippines,
to use such of those bases listed in Annex B as the United States determines to be
required by military necessity.

Sixth, under the MBA, the U.S. was given the right, power, and authority to control and prohibit the movement and operation of all types of vehicles within the vicinity of the bases. The U.S. does not have any
right, power, or authority to do so under EDCA.

1947 MBA EDCA

1947 MBA, Art. 111(2)(c) No equivalent provision.

Such rights, power and authority shall include, inter alia, the right, power and
authority: x x x x to control (including the right to prohibit) in so far as may be
required for the efficient operation and safety of the bases, and within the limits of
military necessity, anchorages, moorings, landings, takeoffs, movements and
operation of ships and water-borne craft, aircraft and other vehicles on water, in
the air or on land comprising

Seventh, under EDCA, the U.S. is merely given temporary access to land and facilities (including roads, ports, and airfields). On the other hand, the old treaty gave the U.S. the right to improve and deepen the
harbors, channels, entrances, and anchorages; and to construct or maintain necessary roads and bridges that would afford it access to its military bases.
1947 MBA EDCA

1947 MBA, Art. III(2)(b): EDCA, Art. III(2):

Such rights, power and authority shall include, inter alia, the right, power and When requested, the Designated Authority of the Philippines shall assist
authority: x x x x to improve and deepen the harbors, channels, entrances and in facilitating transit or temporary access by United States forces to public land
anchorages, and to construct or maintain necessary roadsand bridges affording and facilities (including roads, ports, and airfields), including those owned or
access to the bases. controlled by local governments, and to other land and facilities (including roads,
ports, and airfields).

Eighth, in the 1947 MBA, the U.S. was granted the automatic right to use any and all public utilities, services and facilities, airfields, ports, harbors, roads, highways, railroads, bridges, viaducts, canals, lakes,
rivers, and streams in the Philippines in the same manner that Philippine military forces enjoyed that right. No such arrangement appears in EDCA. In fact, it merely extends to U.S. forces temporary access to
public land and facilities when requested:

1947 MBA EDCA

1947 MBA, Art. VII: EDCA, Art. III(2):

It is mutually agreed that the United States may employ and use for United States When requested, the Designated Authority of the Philippines shall assist
military forces any and all public utilities, other services and facilities, airfields, in facilitating transit or temporary access by United States forces to public land
ports, harbors, roads, highways, railroads, bridges, viaducts, canals, lakes, rivers and facilities (including roads, ports, and airfields), including those owned or
and streams in the Philippines under conditions no less favorable than those that controlled by local governments, and to other land and facilities (including roads,
may be applicablefrom time to time to the military forces of the Philippines. ports, and airfields).

Ninth, under EDCA, the U.S. no longer has the right, power, and authority to construct, install, maintain, and employ any type of facility, weapon, substance, device, vessel or vehicle, or system unlike in the old
treaty. EDCA merely grants the U.S., through bilateral security mechanisms, the authority to undertake construction, alteration, or improvements on the Philippine-owned Agreed Locations.

1947 MBA EDCA

1947 MBA, Art. III(2)(e): EDCA, Art. III(4):

Such rights, power and authority shall include, inter alia, the right, power and The Philippines hereby grants to the United States, through bilateral security
authority: x x x x to construct, install, maintain, and employ on any base any mechanisms, such as the MDB and SEB, operational control of Agreed Locations for
type of facilities, weapons, substance, device, vessel or vehicle on or under the construction activities and authority to undertake such activities on, and make
ground, in the air or on or under the water that may be requisite or appropriate, alterations and improvements to, Agreed Locations. United States forces shall
including meteorological systems, aerial and water navigation lights, radio and consult on issues regarding such construction, alterations, and improvements
radar apparatus and electronic devices, of any desired power, type of emission and based on the Parties' shared intent that the technical requirements and
frequency. construction standards of any such projects undertaken by or on behalf of United
States forces should be consistent with the requirements and standards of both
Parties.

Tenth, EDCA does not allow the U.S. to acquire, by condemnation or expropriation proceedings, real property belonging to any private person. The old military bases agreement gave this right to the U.S. as
seen below:
1947 MBA EDCA

1947 MBA, Art. XXII(l): No equivalent provision.

Whenever it is necessary to acquire by

condemnation or expropriation proceedings real property belonging to any


private persons, associations or corporations located in bases named in Annex A
and Annex B in order to carry out the purposes of this Agreement, the Philippines
will institute and prosecute such condemnation or expropriation proceedings in
accordance with the laws of the Philippines. The United States agrees to reimburse
the Philippines for all the reasonable expenses, damages and costs therebv
incurred, including the value of the property as determined by the Court. In
addition, subject to the mutual agreement of the two Governments, the United
States will reimburse the Philippines for the reasonable costs of transportation and
removal of any occupants displaced or ejected by reason of the condemnation or
expropriation.

Eleventh, EDCA does not allow the U.S. to unilaterally bring into the country non-Philippine nationals who are under its employ, together with their families, in connection with the construction, maintenance,
or operation of the bases. EDCA strictly adheres to the limits under the VFA.

1947 MBA EDCA

1947 MBA, Art. XI(l): EDCA, Art. II:

It is mutually agreed that the United States shall have the right to bring into the 1. "United States personnel" means United States military and civilian
Philippines members of the United States military forces and the United States personneltemporarily in the territory of the Philippines in connection with
nationals employed by or under a contract with the United States together with activities approved by the Philippines, as those terms are defined in the VFA.
their families, and technical personnel of other nationalities (not being persons
excluded by the laws of the Philippines) in connection with the construction, x xx x
maintenance, or operation of the bases. The United States shall make suitable
arrangements so that such persons may be readily identified and their status
established when necessary by the Philippine authorities. Such persons, other than 3. "United States contractors" means companies and firms, and their employees,
members of the United States armed forces in uniform, shall present their travel under contract or subcontract to or on behalf of the United States Department of
documents to the appropriate Philippine authorities for visas, it being understood Defense. United States contractors are not includedas part of
that no objection will be made to their travel to the Philippines as non- the definition of United States personnel in this Agreement, including within the
immigrants. context of the VFA.

Twelfth, EDCA does not allow the U.S. to exercise jurisdiction over any offense committed by any person within the Agreed Locations, unlike in the former military bases:

1947 MBA EDCA


1947 MBA, Art. XIII(l)(a): No equivalent provision.

The Philippines consents that the United

States shall have the right to exercise jurisdiction over the following offenses:
(a) Any offense committed by any person within any base except where the
offender and offended parties are both Philippine citizens (not members of the
armed forces of the United States on active duty) or the offense is against the
security of the Philippines.

Thirteenth, EDCA does not allow the U.S. to operate military post exchange (PX) facilities, which is free of customs duties and taxes, unlike what the expired MBA expressly allowed. Parenthetically, the PX
store has become the cultural icon of U.S. military presence in the country.

1947 MBA EDCA

1947 MBA, Art. XVIII(l): No equivalent provision.

It is mutually agreed that the United States

shall have the right to establish on bases, free of all licenses; fees; sales, excise or
other taxes, or imposts; Government agencies, including concessions, such
as sales commissaries and post exchanges; messes and social clubs, for the
exclusive use of the United States military forces and authorized civilian
personnel and their families. The merchandise or services sold or dispensed by
such agencies shall be free of all taxes, duties and inspection by the Philippine
authorities. Administrative measures shall be taken by the appropriate authorities
of the United States to prevent the resale of goods which are sold under the
provisions of this Article to persons not entitled to buy goods at such agencies and,
generally, to prevent abuse of the privileges granted under this Article. There shall
be cooperation between such authorities and the Philippines to this end.
In sum, EDCA is a far cry from a basing agreement as was understood by the people at the time reservations and their extensions (John Hay Station, Wallace Air Station, O'Donnell Transmitter
that the 1987 Constitution was adopted. Station, San Miguel Naval Communications Station and Capas Relay Station), to raise funds by the
sale of portions of Metro Manila military camps, and to apply said funds as provided herein for the
Nevertheless, a comprehensive review of what the Constitution means by "foreign military bases" development and conversion to productive civilian use of the lands covered under the 194 7
and "facilities" is required before EDCA can be deemed to have passed judicial scrutiny. Military Bases Agreement between the Philippines and the United States of America, as
amended.330

c. The meaning of military facilities and bases


The result of the debates and subsequent voting is Section 25, Article XVIII of the Constitution,
which specifically restricts, among others, foreign military facilities or bases. At the time of its
An appreciation of what a military base is, as understood by the Filipino people in 1987, would be crafting of the Constitution, the 1986 Constitutional Commission had a clear idea of what exactly it
vital in determining whether EDCA breached the constitutional restriction. was restricting. While the term "facilities and bases" was left undefined, its point of reference was
clearly those areas covered by the 1947 MBA as amended.
Prior to the drafting of the 1987 Constitution, the last definition of "military base" was provided
under Presidential Decree No. (PD) 1227.328 Unlawful entry into a military base is punishable under Notably, nearly 30 years have passed since then, and the ever-evolving world of military
the decree as supported by Article 281 of the Revised Penal Code, which itself prohibits the act of technology and geopolitics has surpassed the understanding of the Philippine people in 1986. The
trespass. last direct military action of the U.S. in the region was the use of Subic base as the staging ground
for Desert Shield and Desert Storm during the Gulf War.331In 1991, the Philippine Senate rejected
Section 2 of the law defines the term in this manner: "'[M]ilitary base' as used in this decree means the successor treaty of the 1947 MBA that would have allowed the continuation of U.S. bases in
any military, air, naval, or coast guard reservation, base, fort, camp, arsenal, yard, station, or the Philippines.
installation in the Philippines."
Henceforth, any proposed entry of U.S. forces into the Philippines had to evolve likewise, taking
Commissioner Tadeo, in presenting his objections to U.S. presence in the Philippines before the into consideration the subsisting agreements between both parties, the rejection of the 1991
1986 Constitutional Commission, listed the areas that he considered as military bases: proposal, and a concrete understanding of what was constitutionally restricted. This trend birthed
the VFA which, as discussed, has already been upheld by this Court.
1,000 hectares Camp O'Donnel
The latest agreement is EDCA, which proposes a novel concept termed "Agreed Locations."
20,000 hectares Crow Valley Weapon's Range
By definition, Agreed Locations are
55,000 hectares Clark Air Base
facilities and areas that are provided by the Government of the Philippines through the AFP and
150 hectares Wallace Air Station that United States forces, United States contractors, and others as mutually agreed, shall have the
right to access and use pursuant to this Agreement. Such Agreed Locations may be listed in an
annex to be appended to this Agreement, and may be further described in implementing
400 hectares John Hay Air Station arrangements.332

15,000 hectares Subic Naval Base Preliminarily, respondent already claims that the proviso that the Philippines shall retain
ownership of and title to the Agreed Locations means that EDCA is "consistent with Article II of the
1,000 hectares San Miguel Naval Communication VFA which recognizes Philippine sovereignty and jurisdiction over locations within Philippine
territory.333
750 hectares Radio Transmitter in Capas, Tarlac
By this interpretation, respondent acknowledges that the contention of petitioners springs from an
understanding that the Agreed Locations merely circumvent the constitutional restrictions.
900 hectares Radio Bigot Annex at Bamban, Tarlac329
Framed differently, the bone of contention is whether the Agreed Locations are, from a legal
perspective, foreign military facilities or bases. This legal framework triggers Section 25, Article
The Bases Conversion and Development Act of 1992 described its coverage in its Declaration of XVIII, and makes Senate concurrence a sine qua non.
Policies:
Article III of EDCA provides for Agreed Locations, in which the U.S. is authorized by the Philippines
Sec. 2. Declaration of Policies. - It is hereby declared the policy of the Government to accelerate to "conduct the following activities: "training; transit; support and related activities; refueling of
the sound and balanced conversion into alternative productive uses of the Clark and Subic military
aircraft; bunkering of vessels; temporary maintenance of vehicles, vessels and aircraft; temporary deterrents to such attack; 2) whether an alliance or mutual defense treaty is a derogation of our
accommodation of personnel; communications; prepositioning of equipment, supplies and national sovereignty; 3) whether criticism of us by Russia, Vietnam and North Korea is outweighed
materiel; deploying forces and materiel; and such other activities as the Parties may agree." by the support for us of the ASEAN countries, the United States, South Korea, Taiwan, Australia
and New Zealand; and 4) whether the social, moral and legal problems spawned by the military
This creation of EDCA must then be tested against a proper interpretation of the Section 25 bases and their operations can be compensated by the economic benefits outlined in papers which
restriction. have been furnished recently to all of us.335

d. Reasons for the constitutional requirements and legal standards for constitutionally compatible xxxx
military bases and facilities
Of course, one side of persuasion has submitted categorical, unequivocal and forceful assertions of
Section 25 does not define what is meant by a "foreign military facility or base." While it their positions. They are entitled to the luxury of the absolutes. We are urged now to adopt the
specifically alludes to U.S. military facilities and bases that existed during the framing of the proposed declaration as a "golden," "unique" and "last" opportunity for Filipinos to assert their
Constitution, the provision was clearly meant to apply to those bases existing at the time and to sovereign rights. Unfortunately, I have never been enchanted by superlatives, much less for the
any future facility or base. The basis for the restriction must first be deduced from the spirit of the applause of the moment or the ovation of the hour. Nor do I look forward to any glorious summer
law, in order to set a standard for the application of its text, given the particular historical events after a winter of political discontent. Hence, if I may join Commissioner Laurel, I also invoke a
preceding the agreement. caveat not only against the tyranny of labels but also the tyranny of slogans.336

Once more, we must look to the 1986 Constitutional Commissioners to glean, from their collective xxxx
wisdom, the intent of Section 25. Their speeches are rich with history and wisdom and present a
clear picture of what they considered in the crafting the provision. SPEECH OF COMMISSIONER SUAREZ337

SPEECH OF COMMISSIONER REGALADO334 MR. SUAREZ: Thank you, Madam President.

xxxx I am quite satisfied that the crucial issues involved in the resolution of the problem of the removal
of foreign bases from the Philippines have been adequately treated by previous speakers. Let me,
We have been regaled here by those who favor the adoption of the anti-bases provisions with therefore, just recapitulate the arguments adduced in favor of a foreign bases-free Philippines:
what purports to be an objective presentation of the historical background of the military bases in
the Philippines. Care appears, however, to have been taken to underscore the inequity in their 1. That every nation should be free to shape its own destiny without outside
inception as well as their implementation, as to seriously reflect on the supposed objectivity of interference;
the report. Pronouncements of military and civilian officials shortly after World War II are quoted
in support of the proposition on neutrality; regrettably, the implication is that the same remains 2. That no lasting peace and no true sovereignty would ever be achieved so long as
valid today, as if the world and international activity stood still for the last 40 years. there are foreign military forces in our country;

We have been given inspired lectures on the effect of the presence of the military bases on our 3. That the presence of foreign military bases deprives us of the very substance of
sovereignty - whether in its legal or political sense is not clear - and the theory that any country national sovereigntyand this is a constant source of national embarrassment and an
with foreign bases in its territory cannot claim to be fully sovereign or completely independent. I insult to our national dignity and selfrespect as a nation;
was not aware that the concepts of sovereignty and independence have now assumed the totality
principle, such that a willing assumption of some delimitations in the exercise of some aspects
thereof would put that State in a lower bracket of nationhood. 4. That these foreign military bases unnecessarily expose our country to devastating
nuclear attacks;

xxxx
5. That these foreign military bases create social problems and are designed to
perpetuate the strangle-hold of United States interests in our national economy and
We have been receiving a continuous influx of materials on the pros and cons on the advisability of development;
having military bases within our shores. Most of us who, only about three months ago, were just
mulling the prospects of these varying contentions are now expected, like armchair generals, to
decide not only on the geopolitical aspects and contingent implications of the military bases but 6. That the extraterritorial rights enjoyed by these foreign bases operate to deprive our
also on their political, social, economic and cultural impact on our national life. We are asked to country of jurisdiction over civil and criminal offenses committed within our own
answer a plethora of questions, such as: 1) whether the bases are magnets of nuclear attack or are national territory and against Filipinos;
7. That the bases agreements are colonial impositions and dictations upon our helpless use as naval and air bases. It was done in consideration of 50 overaged destroyers which the
country; and United States gave to England for its use in the Battle of the Atlantic.

8. That on the legal viewpoint and in the ultimate analysis, all the bases agreements are A few years ago, England gave the Island of Diego Garcia to the United States for the latter's use as
null and void ab initio, especially because they did not count the sovereign consent and a naval base in the Indian Ocean. About the same time, the United States obtained bases in Spain,
will of the Filipino people.338 Egypt and Israel. In doing so, these countries, in effect, contributed to the launching of a
preventive defense posture against possible trouble in the Middle East and in the Indian Ocean for
xxxx their own protection.345

In the real sense, Madam President, if we in the Commission could accommodate the provisions I SPEECH OF COMMISSIONER TINGSON346
have cited, what is our objection to include in our Constitution a matter as priceless as the
nationalist values we cherish? A matter of the gravest concern for the safety and survival of this xxxx
nation indeed deserves a place in our Constitution.
In the case of the Philippines and the other Southeast Asian nations, the presence of American
xxxx troops in the country is a projection of America's security interest. Enrile said that nonetheless,
they also serve, although in an incidental and secondary way, the security interest of the Republic
x x x Why should we bargain away our dignity and our self-respect as a nation and the future of of the Philippines and the region. Yes, of course, Mr. Enrile also echoes the sentiments of most of
generations to come with thirty pieces of silver?339 us in this Commission, namely: It is ideal for us as an independent and sovereign nation to
ultimately abrogate the RP-US military treaty and, at the right time, build our own air and naval
might.347
SPEECH OF COMMISSIONER BENNAGEN340

xxxx
xxxx

Allow me to say in summation that I am for the retention of American military bases in the
The underlying principle of military bases and nuclear weapons wherever they are found and Philippines provided that such an extension from one period to another shall be concluded upon
whoever owns them is that those are for killing people or for terrorizing humanity. This objective concurrence of the parties, and such extension shall be based on justice, the historical amity of
by itself at any point in history is morally repugnant. This alone is reason enough for us to the people of the Philippines and the United States and their common defense interest.348
constitutionalize the ban on foreign military bases and on nuclear weapons.341
SPEECH OF COMMISSIONER ALONTO349
SPEECH OF COMMISSIONER BACANI342

xxxx
xxxx

Madam President, sometime ago after this Commission started with this task of framing a
x x x Hence, the remedy to prostitution does not seem to be primarily to remove the constitution, I read a statement of President Aquino to the effect that she is for the removal of the
bases because even if the bases are removed, the girls mired in poverty will look for their clientele U.S. military bases in this country but that the removal of the U.S. military bases should not be
elsewhere. The remedy to the problem of prostitution lies primarily elsewhere - in an alert and done just to give way to other foreign bases. Today, there are two world superpowers, both vying
concerned citizenry, a healthy economy and a sound education in values.343 to control any and all countries which have importance to their strategy for world domination. The
Philippines is one such country.
SPEECH OF COMMISSIONER JAMIR344
Madam President, I submit that I am one of those ready to completely remove any vestiges of
xxxx the days of enslavement, but not prepared to erase them if to do so would merely leave a vacuum
to be occupied by a far worse type.350
One of the reasons advanced against the maintenance of foreign military bases here is that they
impair portions of our sovereignty. While I agree that our country's sovereignty should not be SPEECH OF COMMISSIONER GASCON351
impaired, I also hold the view that there are times when it is necessary to do so according to the
imperatives of national interest. There are precedents to this effect. Thus, during World War II, xxxx
England leased its bases in the West Indies and in Bermuda for 99 years to the United States for its
Let us consider the situation of peace in our world today. Consider our brethren in the Middle East, So for the record, Mr. Presiding Officer, I would like to declare my support for the committee's
in Indo-China, Central America, in South Africa - there has been escalation of war in some of these position to enshrine in the Constitution a fundamental principle forbidding foreign military bases,
areas because of foreign intervention which views these conflicts through the narrow prism of the troops or facilities in any part of the Philippine territory as a clear and concrete manifestation of
East-West conflict. The United States bases have been used as springboards for intervention in our inherent right to national self-determination, independence and sovereignty.
some of these conflicts. We should not allow ourselves to be party to the warlike mentality of
these foreign interventionists. We must always be on the side of peace this means that we Mr. Presiding Officer, I would like to relate now these attributes of genuine nationhood to the
should not always rely on military solution. 352 social cost of allowing foreign countries to maintain military bases in our country. Previous
speakers have dwelt on this subject, either to highlight its importance in relation to the other
xxxx issues or to gloss over its significance and !llake this a part of future negotiations. 357

x x x The United States bases, therefore, are springboards for intervention in our own internal xxxx
affairs and in the affairs of other nations in this region.
Mr. Presiding Officer, I feel that banning foreign military bases is one of the solutions and is the
xxxx response of the Filipino people against this condition and other conditions that have already been
clearly and emphatically discussed in past deliberations. The deletion, therefore, of Section 3 in the
Thus, I firmly believe that a self-respecting nation should safeguard its fundamental freedoms Constitution we are drafting will have the following implications:
which should logically be declared in black and white in our fundamental law of the land - the
Constitution. Let us express our desire for national sovereignty so we may be able to achieve First, the failure of the Constitutional Commission to decisively respond to the continuing violation
national self-determination. Let us express our desire for neutrality so that we may be able to of our territorial integrity via the military bases agreement which permits the retention of U.S.
follow active nonaligned independent foreign policies. Let us express our desire for peace and a facilities within the Philippine soil over which our authorities have no exclusive jurisdiction
nuclear-free zone so we may be able to pursue a healthy and tranquil existence, to have peace contrary to the accepted definition of the exercise of sovereignty.
that is autonomous and not imposed. 353
Second, consent by this forum, this Constitutional Commission, to an exception in the application
xxxx of a provision in the Bill of Rights that we have just drafted regarding equal application of the laws
of the land to all inhabitants, permanent or otherwise, within its territorial boundaries.
SPEECH OF COMMISSIONER TADEO354
Third, the continued exercise by the United States of extraterritoriality despite the
Para sa magbubukid, ano ha ang kahulugan ng U.S. military bases? Para sa magbubukid, ang condemnations of such practice by the world community of nations in the light of overwhelming
kahulugan nito ay pagkaalipin. Para sa magbubukid, ang pananatili ng U.S. military bases ay tinik international approval of eradicating all vestiges of colonialism.358
sa dibdib ng sambayanang Pilipinong patuloy na nakabaon. Para sa sambayanang magbubukid,
ang ibig sabihin ng U.S. military bases ay batong pabigat na patuloy na pinapasan ng xxxx
sambayanang Pilipino. Para sa sambayanang magbubukid, ang pananatili ng U.S. military
bases ay isang nagdudumilat na katotohanan ng patuloy na paggahasa ng imperyalistang Sixth, the deification of a new concept called pragmatic sovereignty, in the hope that such can be
Estados Unidos sa ating Inang Bayan - economically, politically and culturally. Para sa wielded to force the United States government to concede to better terms and conditions
sambayanang magbubukid ang U.S. military bases ay kasingkahulugan ng nuclear weapon - ang concerning the military bases agreement, including the transfer of complete control to the
kahulugan ay magneto ng isang nuclear war. Para sa sambayanang magbubukid, ang kahulugan Philippine government of the U.S. facilities, while in the meantime we have to suffer all existing
ng U.S. military bases ay isang salot.355 indignities and disrespect towards our rights as a sovereign nation.

SPEECH OF COMMISSIONER QUESADA356 xxxx

xxxx Eighth, the utter failure of this forum to view the issue of foreign military bases as essentially a
question of sovereignty which does not require in-depth studies or analyses and which this forum
The drift in the voting on issues related to freeing ourselves from the instruments of domination has, as a constituent assembly drafting a constitution, the expertise and capacity to decide on
and subservience has clearly been defined these past weeks. except that it lacks the political will that brought it to existence and now engages in an elaborate
scheme of buck-passing.
xxxx
xxxx
Without any doubt we can establish a new social order in our country, if we reclaim, restore, Property is classified as private or public.365 It is public if "intended for public use, such as roads,
uphold and defend our national sovereignty. National sovereignty is what the military bases issue canals, rivers, torrents, ports and bridges constructed by the State, banks, shores, roadsteads, and
is all about. It is only the sovereign people exercising their national sovereignty who can design an others of similar character[,]" or "[t]hose which belong to the State, without being for public use,
independent course and take full control of their national destiny.359 and are intended for some public service or for the development of the national wealth. " 366

SPEECH OF COMMISSIONER P ADILLA360 Quite clearly, the Agreed Locations are contained within a property for public use, be it within a
government military camp or property that belongs to the Philippines.1avvphi1
xxxx
Once ownership is established, then the rights of ownership flow freely. Article 428 of the Civil
Mr. Presiding Officer, in advocating the majority committee report, specifically Sections 3 and 4 on Code provides that "[t]he owner has the right to enjoy and dispose of a thing, without other
neutrality, nuclear and bases-free country, some views stress sovereignty of the Republic and limitations than those established by law." Moreover, the owner "has also a right of action against
even invoke survival of the Filipino nation and people.361 the holder and possessor of the thing in order to recover it."

REBUTTAL OF COMMISSIONER NOLLEDO362 Philippine civil law therefore accords very strong rights to the owner of property, even against
those who hold the property. Possession, after all, merely raises a disputable presumption of
ownership, which can be contested through normal judicial processes.367
xxxx

In this case, EDCA explicitly provides that ownership of the Agreed Locations remains with the
The anachronistic and ephemeral arguments against the provisions of the committee report to Philippine govemment.368 What U.S. personnel have a right to, pending mutual agreement, is
dismantle the American bases after 1991 only show the urgent need to free our country from the access to and use of these locations.369
entangling alliance with any power bloc.363

The right of the owner of the property to allow access and use is consistent with the Civil Code,
xxxx since the owner may dispose of the property in whatever way deemed fit, subject to the limits of
the law. So long as the right of ownership itself is not transferred, then whatever rights are
xx x Mr. Presiding Officer, it is not necessary for us to possess expertise to know that the so-called transmitted by agreement does not completely divest the owner of the rights over the property,
RP-US Bases Agreement will expire in 1991, that it infringes on our sovereignty and jurisdiction but may only limit them in accordance with law.
as well as national dignity and honor, that it goes against the UN policy of disarmament and that it
constitutes unjust intervention in our internal affairs.364 (Emphases Supplied) Hence, even control over the property is something that an owner may transmit freely. This act
does not translate into the full transfer of ownership, but only of certain rights. In Roman Catholic
The Constitutional Commission eventually agreed to allow foreign military bases, troops, or Apostolic Administrator of Davao, Inc. v. Land Registration Commission, we stated that the
facilities, subject to the provisions of Section 25. It is thus important to read its discussions constitutional proscription on property ownership is not violated despite the foreign national's
carefully. From these discussions, we can deduce three legal standards that were articulated by control over the property.370
the Constitutional Commission Members. These are characteristics of any agreement that the
country, and by extension this Court, must ensure are observed. We can thereby determine EDCA, in respect of its provisions on Agreed Locations, is essentially a contract of use and access.
whether a military base or facility in the Philippines, which houses or is accessed by foreign military Under its pertinent provisions, it is the Designated Authority of the Philippines that shall, when
troops, is foreign or remains a Philippine military base or facility. The legal standards we find requested, assist in facilitating transit or access to public land and facilities. 371 The activities carried
applicable are: independence from foreign control, sovereignty and applicable law, and national out within these locations are subject to agreement as authorized by the Philippine
security and territorial integrity. govemment.372 Granting the U.S. operational control over these locations is likewise subject to
EDCA' s security mechanisms, which are bilateral procedures involving Philippine consent and
i. First standard: independence from foreign control cooperation.373 Finally, the Philippine Designated Authority or a duly designated representative is
given access to the Agreed Locations.374
Very clearly, much of the opposition to the U.S. bases at the time of the Constitution's drafting was
aimed at asserting Philippine independence from the U.S., as well as control over our country's To our mind, these provisions do not raise the spectre of U.S. control, which was so feared by the
territory and military. Constitutional Commission. In fact, they seem to have been the product of deliberate negotiation
from the point of view of the Philippine government, which balanced constitutional restrictions on
Under the Civil Code, there are several aspects of control exercised over property. foreign military bases and facilities against the security needs of the country. In the 1947 MBA, the
U.S. forces had "the right, power and authority x x x to construct (including dredging and filling),
operate, maintain, utilize, occupy, garrison and control the bases."375 No similarly explicit provision
is present in EDCA.
Nevertheless, the threshold for allowing the presence of foreign military facilities and bases has the United States to furnish us the necessary hardware in connection with the military bases
been raised by the present Constitution. Section 25 is explicit that foreign military bases, troops, or agreement. Please be informed that there are three (3) treaties connected with the military bases
facilities shall not be allowed in the Philippines, except under a treaty duly concurred in by the agreement; namely: the RP-US Military Bases Agreement, the Mutual Defense Treaty and the
Senate. Merely stating that the Philippines would retain ownership would do violence to the Military Assistance Program.
constitutional requirement if the Agreed Locations were simply to become a less obvious
manifestation of the U.S. bases that were rejected in 1991. My dear Commissioner, when we enter into a treaty and we are furnished the military hardware
pursuant to that treaty, it is not in control of our armed forces nor control of our
When debates took place over the military provisions of the Constitution, the committee rejected government. True indeed, we have military officers trained in the U.S. armed forces school. This is
a specific provision proposed by Commissioner Sarmiento. The discussion illuminates and provides part of our Military Assistance Program, but it does not mean that the minds of our military
context to the 1986 Constitutional Commission's vision of control and independence from the U.S., officers are for the U.S. government, no. I am one of those who took four courses in the United
to wit: States schools, but I assure you, my mind is for the Filipino people. Also, while we are sending
military officers to train or to study in U.S. military schools, we are also sending our officers to
MR. SARMIENTO: Madam President, my proposed amendment reads as follows: "THE STATE SHALL study in other military schools such as in Australia, England and in Paris. So, it does not mean that
ESTABLISH AND MAINTAIN AN INDEPENDENT AND SELF-RELIANT ARMED FORCES OF THE when we send military officers to United States schools or to other military schools, we will be
PHILIPPINES." Allow me to briefly explain, Madam President. The Armed Forces of the Philippines under the control of that country. We also have foreign officers in our schools, we in the Command
is a vital component of Philippine society depending upon its training, orientation and support. It and General Staff College in Fort Bonifacio and in our National Defense College, also in Fort
will either be the people's protector or a staunch supporter of a usurper or tyrant, local and Bonifacio.377 (Emphases supplied)
foreign interest. The Armed Forces of the Philippines' past and recent experience shows it has
never been independent and self-reliant. Facts, data and statistics will show that it has been This logic was accepted in Taada v. Angara, in which the Court ruled that independence does not
substantially dependent upon a foreign power. In March 1968, Congressman Barbero, himself a mean the absence of foreign participation:
member of the Armed Forces of the Philippines, revealed top secret documents showing what he
described as U.S. dictation over the affairs of the Armed Forces of the Philippines. He showed that Furthermore, the constitutional policy of a "self-reliant and independent national economy" does
under existing arrangements, the United States unilaterally determines not only the types and not necessarily rule out the entry of foreign investments, goods and services. It contemplates
quantity of arms and equipments that our armed forces would have, but also the time when neither "economic seclusion" nor "mendicancy in the international community." As explained by
these items are to be made available to us. It is clear, as he pointed out, that the composition, Constitutional Commissioner Bernardo Villegas, sponsor of this constitutional policy:
capability and schedule of development of the Armed Forces of the Philippines is under the
effective control of the U.S. government.376 (Emphases supplied)
Economic self reliance is a primary objective of a developing country that is keenly aware of
overdependence on external assistance for even its most basic needs. It does not mean autarky or
Commissioner Sarmiento proposed a motherhood statement in the 1987 Constitution that would economic seclusion; rather, it means avoiding mendicancy in the international
assert "independent" and "self-reliant" armed forces. This proposal was rejected by the community. Independence refers to the freedom from undue foreign control of the national
committee, however. As Commissioner De Castro asserted, the involvement of the Philippine economy, especially in such strategic industries as in the development of natural resources and
military with the U.S. did not, by itself, rob the Philippines of its real independence. He made public utilities.378 (Emphases supplied)
reference to the context of the times: that the limited resources of the Philippines and the current
insurgency at that time necessitated a strong military relationship with the U.S. He said that the
U.S. would not in any way control the Philippine military despite this relationship and the fact that The heart of the constitutional restriction on foreign military facilities and bases is therefore the
the former would furnish military hardware or extend military assistance and training to our assertion of independence from the U.S. and other foreign powers, as independence is exhibited
military. Rather, he claimed that the proposal was in compliance with the treaties between the by the degree of foreign control exerted over these areas.1wphi1 The essence of that
two states. independence is self-governance and self-control.379 Independence itself is "[t]he state or
condition of being free from dependence, subjection, or control. "380

MR. DE CASTRO: If the Commissioner will take note of my speech on U.S. military bases on 12
September 1986, I spoke on the selfreliance policy of the armed forces. However, due to very Petitioners assert that EDCA provides the U.S. extensive control and authority over Philippine
limited resources, the only thing we could do is manufacture small arms ammunition. We cannot facilities and locations, such that the agreement effectively violates Section 25 of the 1987
blame the armed forces. We have to blame the whole Republic of the Philippines for failure to Constitution.381
provide the necessary funds to make the Philippine Armed Forces self-reliant. Indeed that is a
beautiful dream. And I would like it that way. But as of this time, fighting an insurgency case, a Under Article VI(3) of EDCA, U.S. forces are authorized to act as necessary for "operational control
rebellion in our country - insurgency - and with very limited funds and very limited number of men, and defense." The term "operational control" has led petitioners to regard U.S. control over the
it will be quite impossible for the Philippines to appropriate the necessary funds Agreed Locations as unqualified and, therefore, total.382 Petitioners contend that the word "their"
therefor. However, if we say that the U.S. government is furnishing us the military hardware, it is refers to the subject "Agreed Locations."
not control of our armed forces or of our government. It is in compliance with the Mutual
Defense Treaty. It is under the military assistance program that it becomes the responsibility of This argument misreads the text, which is quoted below:
United States forces are authorized to exercise all rights and authorities within Agreed Locations b. Operational control vis--vis effective command and control
that are necessary for their operational control or defense, including taking appropriate measure
to protect United States forces and United States contractors. The United States should coordinate Petitioners assert that beyond the concept of operational control over personnel, qualifying access
such measures with appropriate authorities of the Philippines. to the Agreed Locations by the Philippine Designated Authority with the phrase "consistent with
operational safety and security requirements in accordance with agreed procedures developed by
A basic textual construction would show that the word "their," as understood above, is a the Parties" leads to the conclusion that the U.S. exercises effective control over the Agreed
possessive pronoun for the subject "they," a third-person personal pronoun in plural form. Thus, Locations.389 They claim that if the Philippines exercises possession of and control over a given
"their" cannot be used for a non-personal subject such as "Agreed Locations." The simple area, its representative should not have to be authorized by a special provision.390
grammatical conclusion is that "their" refers to the previous third-person plural noun, which is
"United States forces." This conclusion is in line with the definition of operational control. For these reasons, petitioners argue that the "operational control" in EDCA is the "effective
command and control" in the 1947 MBA.391 In their Memorandum, they distinguish effective
a. U.S. operational control as the exercise of authority over U.S. personnel, and not over the command and control from operational control in U.S. parlance.392 Citing the Doctrine for the
Agreed Locations Armed Forces of the United States, Joint Publication 1, "command and control (C2)" is defined as
"the exercise of authority and direction by a properly designated commander over assigned and
Operational control, as cited by both petitioner and respondents, is a military term referring to attached forces in the accomplishment of the mission x x x."393 Operational control, on the other
hand, refers to "[t]hose functions of command over assigned forces involving the composition of
subordinate forces, the assignment of tasks, the designation of objectives, the overall control of
[t]he authority to perform those functions of command over subordinate forces involving assigned resources, and the full authoritative direction necessary to accomplish the mission."394
organizing and employing commands and forces, assigning tasks, designating objective, and giving
authoritative direction necessary to accomplish the mission.383
Two things demonstrate the errors in petitioners' line of argument.

At times, though, operational control can mean something slightly different. In JUSMAG Philippines
v. National Labor Relations Commission, the Memorandum of Agreement between the AFP and Firstly, the phrase "consistent with operational safety and security requirements in accordance
JUSMAG Philippines defined the term as follows:384 with agreed procedures developed by the Parties" does not add any qualification beyond that
which is already imposed by existing treaties. To recall, EDCA is based upon prior treaties, namely
the VFA and the MDT.395 Treaties are in themselves contracts from which rights and obligations
The term "Operational Control" includes, but is not limited to, all personnel administrative actions, may be claimed or waived.396 In this particular case, the Philippines has already agreed to abide by
such as: hiring recommendations; firing recommendations; position classification; discipline; the security mechanisms that have long been in place between the U.S. and the Philippines based
nomination and approval of incentive awards; and payroll computation. on the implementation of their treaty relations.397

Clearly, traditional standards define "operational control" as personnel control. Philippine law, for Secondly, the full document cited by petitioners contradicts the equation of "operational control"
instance, deems operational control as one exercised by police officers and civilian authorities over with "effective command and control," since it defines the terms quite differently, viz:398
their subordinates and is distinct from the administrative control that they also exercise over
police subordinates.385 Similarly, a municipal mayor exercises operational control over the police
within the municipal government,386 just as city mayor possesses the same power over the police Command and control encompasses the exercise of authority, responsibility, and direction by a
within the city government.387 commander over assigned and attached forces to accomplish the mission. Command at all levels is
the art of motivating and directing people and organizations into action to accomplish missions.
Control is inherent in command. To control is to manage and direct forces and functions consistent
Thus, the legal concept of operational control involves authority over personnel in a commander- with a commander's command authority. Control of forces and functions helps commanders and
subordinate relationship and does not include control over the Agreed Locations in this particular staffs compute requirements, allocate means, and integrate efforts. Mission command is the
case. Though not necessarily stated in EDCA provisions, this interpretation is readily implied by the preferred method of exercising C2. A complete discussion of tenets, organization, and processes
reference to the taking of "appropriate measures to protect United States forces and United States for effective C2 is provided in Section B, "Command and Control of Joint Forces," of Chapter V
contractors." "Joint Command and Control."

It is but logical, even necessary, for the U.S. to have operational control over its own forces, in Operational control is defined thus:399
much the same way that the Philippines exercises operational control over its own units.

OPCON is able to be delegated from a lesser authority than COCOM. It is the authority to perform
For actual operations, EDCA is clear that any activity must be planned and pre-approved by the those functions of command over subordinate forces involving organizing and employing
MDB-SEB.388 This provision evinces the partnership aspect of EDCA, such that both stakeholders commands and forces, assigning tasks, designating objectives, and giving authoritative direction
have a say on how its provisions should be put into effect. over all aspects of military operations and joint training necessary to accomplish the mission. It
should be delegated to and exercised by the commanders of subordinate organizations; normally,
this authority is exercised through subordinate JFCs, Service, and/or functional component This point leads us to the second standard envisioned by the framers of the Constitution: that the
commanders. OPCON provides authority to organize and employ commands and forces as the Philippines must retain sovereignty and jurisdiction over its territory.
commander considers necessary to accomplish assigned missions. It does not include authoritative
direction for logistics or matters of administration, discipline, internal organization, or unit ii. Second standard: Philippine sovereignty and applicable law
training. These elements of COCOM must be specifically delegated by the CCDR. OPCON does
include the authority to delineate functional responsibilities and operational areas of subordinate
JFCs. EDCA states in its Preamble the "understanding for the United States not to establish a permanent
military presence or base in the territory of the Philippines." Further on, it likewise states the
recognition that "all United States access to and use of facilities and areas will be at the invitation
Operational control is therefore the delegable aspect of combatant command, while command of the Philippines and with full respect for the Philippine Constitution and Philippine laws."
and control is the overall power and responsibility exercised by the commander with reference to
a mission. Operational control is a narrower power and must be given, while command and control
is plenary and vested in a commander. Operational control does not include the planning, The sensitivity of EDCA provisions to the laws of the Philippines must be seen in light of Philippine
programming, budgeting, and execution process input; the assignment of subordinate sovereignty and jurisdiction over the Agreed Locations.
commanders; the building of relationships with Department of Defense agencies; or the directive
authority for logistics, whereas these factors are included in the concept of command and Sovereignty is the possession of sovereign power,406 while jurisdiction is the conferment by law of
control.400 power and authority to apply the law.407 Article I of the 1987 Constitution states:

This distinction, found in the same document cited by petitioners, destroys the very foundation of The national territory comprises the Philippine archipelago, with all the islands and waters
the arguments they have built: that EDCA is the same as the MBA. embraced therein, and all other territories over which the Philippines has sovereignty or
jurisdiction, consisting of its terrestrial, fluvial, and aerial domains, including its territorial sea, the
c. Limited operational control over the Agreed Locations only for construction activitites seabed, the subsoil, the insular shelves, and other submarine areas. The waters around, between,
and connecting the islands of the archipelago, regardless of their breadth and dimensions, form
part of the internal waters of the Philippines. (Emphasis supplied)
As petitioners assert, EDCA indeed contains a specific provision that gives to the U.S. operational
control within the Agreed Locations during construction activities.401 This exercise of operational
control is premised upon the approval by the MDB and the SEB of the construction activity through From the text of EDCA itself, Agreed Locations are territories of the Philippines that the U.S. forces
consultation and mutual agreement on the requirements and standards of the construction, are allowed to access and use.408 By withholding ownership of these areas and retaining
alteration, or improvement.402 unrestricted access to them, the government asserts sovereignty over its territory. That
sovereignty exists so long as the Filipino people exist.409

Despite this grant of operational control to the U.S., it must be emphasized that the grant is only
for construction activities. The narrow and limited instance wherein the U.S. is given operational Significantly, the Philippines retains primary responsibility for security with respect to the Agreed
control within an Agreed Location cannot be equated with foreign military control, which is so Locations.410Hence, Philippine law remains in force therein, and it cannot be said that jurisdiction
abhorred by the Constitution. has been transferred to the U.S. Even the previously discussed necessary measures for operational
control and defense over U.S. forces must be coordinated with Philippine authorities. 411

The clear import of the provision is that in the absence of construction activities, operational
control over the Agreed Location is vested in the Philippine authorities. This meaning is implicit in Jurisprudence bears out the fact that even under the former legal regime of the MBA, Philippine
the specific grant of operational control only during construction activities. The principle of laws continue to be in force within the bases.412 The difference between then and now is that
constitutional construction, "expressio unius est exclusio alterius," means the failure to mention EDCA retains the primary jurisdiction of the Philippines over the security of the Agreed Locations,
the thing becomes the ground for inferring that it was deliberately excluded.403Following this an important provision that gives it actual control over those locations. Previously, it was the
construction, since EDCA mentions the existence of U.S. operational control over the Agreed provost marshal of the U.S. who kept the peace and enforced Philippine law in the bases. In this
Locations for construction activities, then it is quite logical to conclude that it is not exercised over instance, Philippine forces act as peace officers, in stark contrast to the 1947 MBA provisions on
other activities. jurisdiction.413

Limited control does not violate the Constitution. The fear of the commissioners was total control, iii. Third standard: must respect national security and territorial integrity
to the point that the foreign military forces might dictate the terms of their acts within the
Philippines.404 More important, limited control does not mean an abdication or derogation of The last standard this Court must set is that the EDCA provisions on the Agreed Locations must not
Philippine sovereignty and legal jurisdiction over the Agreed Locations. It is more akin to the impair or threaten the national security and territorial integrity of the Philippines.
extension of diplomatic courtesies and rights to diplomatic agents, 405 which is a waiver of control
on a limited scale and subject to the terms of the treaty. This Court acknowledged in Bayan v. Zamora that the evolution of technology has essentially
rendered the prior notion of permanent military bases obsolete.
Moreover, military bases established within the territory of another state is no longer viable exercises.424 Another example given is an advanced communications technology installation for
because of the alternatives offered by new means and weapons of warfare such as nuclear purposes of information gathering and communication.425 Unsurprisingly, he deems these non-
weapons, guided missiles as well as huge sea vessels that can stay afloat in the sea even for combat uses as borderline situations that would be excluded from the functional understanding of
months and years without returning to their home country. These military warships are actually military bases and installations.426
used as substitutes for a land-home base not only of military aircraft but also of military personnel
and facilities. Besides, vessels are mobile as compared to a land-based military headquarters.414 By virtue of this ambiguity, the laws of war dictate that the status of a building or person is
presumed to be protected, unless proven otherwise.427 Moreover, the principle of distinction
The VFA serves as the basis for the entry of U.S. troops in a limited scope. It does not allow, for requires combatants in an armed conflict to distinguish between lawful targets 428 and protected
instance, the re-establishment of the Subic military base or the Clark Air Field as U.S. military targets.429 In an actual armed conflict between the U.S. and a third state, the Agreed Locations
reservations. In this context, therefore, this Court has interpreted the restrictions on foreign bases, cannot be considered U.S. territory, since ownership of territory even in times of armed conflict
troops, or facilities as three independent restrictions. In accord with this interpretation, each does not change.430
restriction must have its own qualification.
Hence, any armed attack by forces of a third state against an Agreed Location can only be
Petitioners quote from the website http://en.wikipedia.org to define what a military base legitimate under international humanitarian law if it is against a bona fide U.S. military base,
is.415 While the source is not authoritative, petitioners make the point that the Agreed Locations, facility, or installation that directly contributes to the military effort of the U.S. Moreover, the third
by granting access and use to U.S. forces and contractors, are U.S. bases under a different state's forces must take all measures to ensure that they have complied with the principle of
name.416 More important, they claim that the Agreed Locations invite instances of attack on the distinction (between combatants and non-combatants).
Philippines from enemies of the U.S.417
There is, then, ample legal protection for the Philippines under international law that would
We believe that the raised fear of an attack on the Philippines is not in the realm of law, but of ensure its territorial integrity and national security in the event an Agreed Location is subjected to
politics and policy. At the very least, we can say that under international law, EDCA does not attack. As EDCA stands, it does not create the situation so feared by petitioners - one in which the
provide a legal basis for a justified attack on the Philippines. Philippines, while not participating in an armed conflict, would be legitimately targeted by an
enemy of the U.S.431
In the first place, international law disallows any attack on the Agreed Locations simply because of
the presence of U.S. personnel. Article 2(4) of the United Nations Charter states that "All Members In the second place, this is a policy question about the wisdom of allowing the presence of U.S.
shall refrain in their international relations from the threat or use of force against the territorial personnel within our territory and is therefore outside the scope of judicial review.
integrity or political independence of any state, or in any other manner inconsistent with the
Purposes of the United Nations."418 Any unlawful attack on the Philippines breaches the treaty, Evidently, the concept of giving foreign troops access to "agreed" locations, areas, or facilities
and triggers Article 51 of the same charter, which guarantees the inherent right of individual or within the military base of another sovereign state is nothing new on the international plane. In
collective self-defence. fact, this arrangement has been used as the framework for several defense cooperation
agreements, such as in the following:
Moreover, even if the lawfulness of the attack were not in question, international humanitarian
law standards prevent participants in an armed conflict from targeting non-participants. 1. 2006 U.S.-Bulgaria Defense Cooperation Agreement432
International humanitarian law, which is the branch of international law applicable to armed
conflict, expressly limits allowable military conduct exhibited by forces of a participant in an armed
conflict.419 Under this legal regime, participants to an armed conflict are held to specific standards 2. 2009 U.S.-Colombia Defense Cooperation Agreement433
of conduct that require them to distinguish between combatants and non-combatants,420 as
embodied by the Geneva Conventions and their Additional Protocols.421 3. 2009 U.S.-Poland Status of Forces Agreement434

Corollary to this point, Professor John Woodcliffe, professor of international law at the University 4. 2014 U.S.-Australia Force Posture Agreement435
of Leicester, noted that there is no legal consensus for what constitutes a base, as opposed to
other terms such as "facilities" or "installation."422 In strategic literature, "base" is defined as an 5. 2014 U.S.-Afghanistan Security and Defense Cooperation Agreement436
installation "over which the user State has a right to exclusive control in an extraterritorial
sense."423 Since this definition would exclude most foreign military installations, a more important
distinction must be made. In all of these arrangements, the host state grants U.S. forces access to their military bases. 437 That
access is without rental or similar costs to the U.S.438 Further, U.S. forces are allowed to undertake
construction activities in, and make alterations and improvements to, the agreed locations,
For Woodcliffe, a type of installation excluded from the definition of "base" is one that does not facilities, or areas.439 As in EDCA, the host states retain ownership and jurisdiction over the said
fulfill a combat role. He cites an example of the use of the territory of a state for training purposes, bases.440
such as to obtain experience in local geography and climactic conditions or to carry out joint
In fact, some of the host states in these agreements give specific military-related rights to the U.S. equipment on the U.S. Munitions List, during an approved activity."450Those activities include
For example, under Article IV(l) of the US.-Bulgaria Defense Cooperation Agreement, "the United "combined exercises and training, operations and other deployments" and "cooperative efforts,
States forces x x x are authorized access to and may use agreed facilities and areas x x x for staging such as humanitarian assistance, disaster relief and rescue operations, and maritime anti-pollution
and deploying of forces and materiel, with the purpose of conducting x x x contingency operations operations" within or outside Philippine territory.451 Under EDCA, the equipment, supplies, and
and other missions, including those undertaken in the framework of the North Atlantic Treaty." In materiel that will be prepositioned at Agreed Locations include "humanitarian assistance and
some of these agreements, host countries allow U.S. forces to construct facilities for the latters disaster relief equipment, supplies, and materiel. "452 Nuclear weapons are specifically excluded
exclusive use.441 from the materiel that will be prepositioned.

Troop billeting, including construction of temporary structures, is nothing new. In Lim v. Executive Therefore, there is no basis to invalidate EDCA on fears that it increases the threat to our national
Secretary, the Court already upheld the Terms of Reference of Balikatan 02-1, which authorized security. If anything, EDCA increases the likelihood that, in an event requiring a defensive
U.S. forces to set up "[t]emporary structures such as those for troop billeting, classroom response, the Philippines will be prepared alongside the U.S. to defend its islands and insure its
instruction and messing x x x during the Exercise." Similar provisions are also in the Mutual territorial integrity pursuant to a relationship built on the MDT and VFA.
Logistics Support Agreement of 2002 and 2007, which are essentially executive agreements that
implement the VFA, the MDT, and the 1953 Military Assistance Agreement. These executive 8. Others issues and concerns raised
agreements similarly tackle the "reciprocal provision of logistic support, supplies, and
services,"442 which include "[b ]illeting, x x x operations support (and construction and use of
temporary structures incident to operations support), training services, x x x storage services, x x x A point was raised during the oral arguments that the language of the MDT only refers to mutual
during an approved activity."443 These logistic supplies, support, and services include temporary help and defense in the Pacific area.453 We believe that any discussion of the activities to be
use of "nonlethal items of military equipment which are not designated as significant military undertaken under EDCA vis-a-vis the defense of areas beyond the Pacific is premature. We note
equipment on the U.S. Munitions List, during an approved activity."444 The first Mutual Logistics that a proper petition on that issue must be filed before we rule thereon. We also note that none
Support Agreement has lapsed, while the second one has been extended until 2017 without any of the petitions or memoranda has attempted to discuss this issue, except only to theorize that the
formal objection before this Court from the Senate or any of its members. U.S. will not come to our aid in the event of an attack outside of the Pacific. This is a matter of
policy and is beyond the scope of this judicial review.

The provisions in EDCA dealing with Agreed Locations are analogous to those in the
aforementioned executive agreements. Instead of authorizing the building of temporary structures In reference to the issue on telecommunications, suffice it to say that the initial impression of the
as previous agreements have done, EDCA authorizes the U.S. to build permanent structures or facility adverted to does appear to be one of those that require a public franchise by way of
alter or improve existing ones for, and to be owned by, the Philippines. 445 EDCA is clear that the congressional action under Section 11, Article XII of the Constitution. As respondents submit,
Philippines retains ownership of altered or improved facilities and newly constructed permanent however, the system referred to in the agreement does not provide telecommunications services
or non-relocatable structures.446 Under EDCA, U.S. forces will also be allowed to use facilities and to the public for compensation.454 It is clear from Article VIl(2) of EDCA that the telecommunication
areas for "training; x x x; support and related activities; x x x; temporary accommodation of system is solely for the use of the U.S. and not the public in general, and that this system will not
personnel; communications" and agreed activities.447 interfere with that which local operators use. Consequently, a public franchise is no longer
necessary.

Concerns on national security problems that arise from foreign military equipment being present
in the Philippines must likewise be contextualized. Most significantly, the VFA already authorizes Additionally, the charge that EDCA allows nuclear weapons within Philippine territory is entirely
the presence of U.S. military equipment in the country. Article VII of the VFA already authorizes speculative. It is noteworthy that the agreement in fact specifies that the prepositioned materiel
the U.S. to import into or acquire in the Philippines "equipment, materials, supplies, and other shall not include nuclear weapons.455Petitioners argue that only prepositioned nuclear weapons
property" that will be used "in connection with activities" contemplated therein. The same section are prohibited by EDCA; and that, therefore, the U.S. would insidiously bring nuclear weapons to
also recognizes that "[t]itle to such property shall remain" with the US and that they have the Philippine territory.456 The general prohibition on nuclear weapons, whether prepositioned or not,
discretion to "remove such property from the Philippines at any time." is already expressed in the 1987 Constitution.457 It would be unnecessary or superfluous to include
all prohibitions already in the Constitution or in the law through a document like EDCA.

There is nothing novel, either, in the EDCA provision on the prepositioning and storing of "defense
equipment, supplies, and materiel,"448 since these are sanctioned in the VFA. In fact, the two Finally, petitioners allege that EDCA creates a tax exemption, which under the law must originate
countries have already entered into various implementing agreements in the past that are from Congress. This allegation ignores jurisprudence on the government's assumption of tax
comparable to the present one. The Balikatan 02-1 Terms of Reference mentioned in Lim v. liability. EDCA simply states that the taxes on the use of water, electricity, and public utilities are
Executive Secretary specifically recognizes that Philippine and U.S. forces "may share x x x in the for the account of the Philippine Government.458 This provision creates a situation in which a
use of their resources, equipment and other assets." Both the 2002 and 2007 Mutual Logistics contracting party assumes the tax liability of the other.459 In National Power Corporation v.
Support Agreements speak of the provision of support and services, including the "construction Province of Quezon, we distinguished between enforceable and unenforceable stipulations on the
and use of temporary structures incident to operations support" and "storage services" during assumption of tax liability. Afterwards, we concluded that an enforceable assumption of tax
approved activities.449 These logistic supplies, support, and services include the "temporary use of liability requires the party assuming the liability to have actual interest in the property
x x x nonlethal items of military equipment which are not designated as significant military taxed.460 This rule applies to EDCA, since the Philippine Government stands to benefit not only
from the structures to be built thereon or improved, but also from the joint training with U.S. As it is, EDCA is not constitutionally infirm. As an executive agreement, it remains consistent with
forces, disaster preparation, and the preferential use of Philippine suppliers. 461 Hence, the existing laws and treaties that it purports to implement.
provision on the assumption of tax liability does not constitute a tax exemption as petitioners have
posited. WHEREFORE, we hereby DISMISS the petitions.

Additional issues were raised by petitioners, all relating principally to provisions already sufficiently SO ORDERED.
addressed above. This Court takes this occasion to emphasize that the agreement has been
construed herein as to absolutely disauthorize the violation of the Constitution or any applicable
statute. On the contrary, the applicability of Philippine law is explicit in EDCA.

EPILOGUE

The fear that EDCA is a reincarnation of the U.S. bases so zealously protested by noted
personalities in Philippine history arises not so much from xenophobia, but from a genuine desire
for self-determination, nationalism, and above all a commitment to ensure the independence of
the Philippine Republic from any foreign domination.

Mere fears, however, cannot curtail the exercise by the President of the Philippines of his
Constitutional prerogatives in respect of foreign affairs. They cannot cripple him when he deems
that additional security measures are made necessary by the times. As it stands, the Philippines
through the Department of Foreign Affairs has filed several diplomatic protests against the actions
of the People's Republic of China in the West Philippine Sea;462 initiated arbitration against that
country under the United Nations Convention on the Law of the Sea;463 is in the process of
negotiations with the Moro Islamic Liberation Front for peace in Southern Philippines, 464 which is
the subject of a current case before this Court; and faces increasing incidents of kidnappings of
Filipinos and foreigners allegedly by the Abu Sayyaf or the New People's Army. 465 The Philippine
military is conducting reforms that seek to ensure the security and safety of the nation in the years
to come.466 In the future, the Philippines must navigate a world in which armed forces fight with
increasing sophistication in both strategy and technology, while employing asymmetric warfare
and remote weapons.

Additionally, our country is fighting a most terrifying enemy: the backlash of Mother Nature. The
Philippines is one of the countries most directly affected and damaged by climate change. It is no
coincidence that the record-setting tropical cyclone Yolanda (internationally named Haiyan), one
of the most devastating forces of nature the world has ever seen hit the Philippines on 8
November 2013 and killed at least 6,000 people.467 This necessitated a massive rehabilitation
project.468 In the aftermath, the U.S. military was among the first to extend help and support to the
Philippines.

That calamity brought out the best in the Filipinos as thousands upon thousands volunteered their
help, their wealth, and their prayers to those affected. It also brought to the fore the value of
having friends in the international community.

In order to keep the peace in its archipelago in this region of the world, and to sustain itself at the
same time against the destructive forces of nature, the Philippines will need friends. Who they are,
and what form the friendships will take, are for the President to decide. The only restriction is
what the Constitution itself expressly prohibits. It appears that this overarching concern for
balancing constitutional requirements against the dictates of necessity was what led to EDCA.

You might also like